Culegere 1 [PDF]

  • 0 0 0
  • Gefällt Ihnen dieses papier und der download? Sie können Ihre eigene PDF-Datei in wenigen Minuten kostenlos online veröffentlichen! Anmelden
Datei wird geladen, bitte warten...
Zitiervorschau

˘ MATEMATICA ˘ ANALIZA PROBLEME 1

Profesorilor no¸stri

Cuprins

Introducere

viii

1 MULT ¸ IMI. RELAT ¸ II. FUNCT ¸ II (A. Croitoru) Probleme . . . . . . . . . . . . . . . . . . . . . . . . . . . . . Solut¸ii . . . . . . . . . . . . . . . . . . . . . . . . . . . . . . .

1 17 31

2 S ¸ IRURI DE NUMERE REALE (M. Durea) 62 Probleme . . . . . . . . . . . . . . . . . . . . . . . . . . . . . 68 Solut¸ii . . . . . . . . . . . . . . . . . . . . . . . . . . . . . . . 77 3 SERII NUMERICE (A. Croitoru) 115 Probleme . . . . . . . . . . . . . . . . . . . . . . . . . . . . . 126 Solut¸ii . . . . . . . . . . . . . . . . . . . . . . . . . . . . . . . 140 4 NOT ¸ IUNI DE TOPOLOGIE ˆ IN IRp (M. Durea) 173 Probleme . . . . . . . . . . . . . . . . . . . . . . . . . . . . . 176 Solut¸ii . . . . . . . . . . . . . . . . . . . . . . . . . . . . . . . 179 5 LIMITE DE FUNCT ¸ II. CONTINUITATE (C. V˘ aideanu) 185 Probleme . . . . . . . . . . . . . . . . . . . . . . . . . . . . . 194 Solut¸ii . . . . . . . . . . . . . . . . . . . . . . . . . . . . . . . 199 6 FUNCT ¸ II DERIVABILE (C. V˘ aideanu) 211 Probleme . . . . . . . . . . . . . . . . . . . . . . . . . . . . . 217 Solut¸ii . . . . . . . . . . . . . . . . . . . . . . . . . . . . . . . 226

vi

vii 7 S ¸ IRURI DE FUNCT ¸ II (C. V˘ aideanu) 257 Probleme . . . . . . . . . . . . . . . . . . . . . . . . . . . . . 260 Solut¸ii . . . . . . . . . . . . . . . . . . . . . . . . . . . . . . . 261 8 SERII DE FUNCT ¸ II. SERII DE PUTERI (C. V˘ aideanu) 262 Probleme . . . . . . . . . . . . . . . . . . . . . . . . . . . . . 265 Note istorice

270

Teste tip examen

277

Bibliografie

280

Index

282

Introducere

Acest volum cuprinde 301 probleme ¸si urm˘are¸ste programa cursului de Analiz˘ a matematic˘ a din primul semestru al anului I de la Facultatea de Matematic˘a din Universitatea ”Al.I. Cuza” Ia¸si. Culegerea se adreseaz˘a ˆın primul rˆand student¸ilor de la Facult˘ a¸tile de Matematic˘a, dar poate fi folosit˘a ¸si de student¸ii din primii ani ai Facult˘ a¸tilor de Informatic˘a sau Politehnice care ofer˘a cursuri de Analiz˘ a matematic˘ a. Problemele au fost culese dintr-o variat˘ a bibliografie sau propuse de autori. Unele probleme au fost propuse la diverse concursuri ¸scolare sau student¸e¸sti, la concursurile de definitivat, gradul II sau titularizare din ˆınv˘a¸t˘amˆantul preuniversitar, aceast˘a carte adresˆandu-se ˆın egal˘a m˘asur˘ a elevilor de liceu ¸si cadrelor didactice. Sunt de asemenea prezentate probleme date la examenele pentru cursul de Analiz˘ a matematic˘ a ¸tinute de profesor dr. Anca-Maria Precupanu ¸si profesor dr. Constantin Z˘alinescu cu student¸ii din Anul I ai Facult˘a¸tii de Matematic˘a. Culegerea cuprinde ¸sase capitole: Mult¸imi. Relat¸ii. Funct¸ii, S ¸ iruri de p numere reale, Serii numerice, Not¸iuni de topologie ˆın IR , Limite de funct¸ii. Continuitate ¸si Funct¸ii derivabile. Fiecare capitol este structurat ˆın trei p˘art¸i. La ˆınceput sunt prezentate doar enunt¸uri ale definit¸iilor, teoremelor ¸si propozit¸iilor ce se refer˘a la subiectul respectiv. Teoria prezentat˘ a nu este exhaustiv˘a, autorii recomandˆand pentru completare ¸si demonstrat¸ii cartea doamnei profesor dr. Anca Maria Precupanu, Bazele Analizei Matematice [14] ¸si, nu ˆın ultimul rˆand, manualele de liceu. Partea a doua cont¸ine diverse exercit¸ii ¸si probleme iar ˆın ultima parte sunt oferite solut¸ii de rezolvare. Unde este posibil, sunt oferite mai multe variante de rezolvare. La unele probleme se dau doar indicat¸ii pentru demonstrat¸ie, iar altele sunt l˘asate ca tem˘ a ˆın seama cititorului. Cartea mai cuprinde cˆateva note istorice ¸si ¸sase teste tip (pentru examenul de Analiz˘ a matematic˘ a din anul I al Facult˘ a¸tii de Matematic˘a de la Universitatea ”Al.I. Cuza” Ia¸si.) viii

ix Dorim s˘a mult¸umim doamnei profesor dr. Anca Maria Precupanu ¸si domnului profesor dr. Constantin Z˘alinescu pentru r˘abdarea ¸si atent¸ia deosebit˘a cu care au citit aceast˘a carte, pentru observat¸iile pertinente ¸si discut¸iile utile purtate pe marginea cont¸inutului, precum ¸si pentru variantele de rezolvare oferite la unele probleme. De asemenea, mult¸umim doamnei Cristina Stamate pentru sugestiile interesante f˘acute la rezolvarea unor probleme. ˆIn sfˆar¸sit, mult¸umim doamnelor Luminit¸a Teodorescu ¸si Carmen Savin, prin grija c˘arora cartea are forma de fat¸˘ a, familiilor ¸si prietenilor pentru ˆıncuraj˘ari ¸si sprijin moral.

Ia¸si, iulie 2005

Autorii

x

Capitolul 1 MULT ¸ IMI. RELAT ¸ II. FUNCT ¸ II

O mult¸ime este o colect¸ie de obiecte bine definite ¸si se va nota cu litere mari: A, B, .... Obiectele din mult¸ime se numesc elemente ¸si se vor nota cu litere mici: a, b, .... Mult¸imea vid˘a (care nu cont¸ine nici un element) se noteaz˘a ∅. Familiile de mult¸imi se vor nota cu litere mari ronde: A, B, .... ˆIn continuare, vom presupune cunoscute urm˘atoarele simboluri: ∨ (sau), ∧ (¸si), ⇒ (implic˘a), ⇔ (echivalent logic), ∀ (oricare), ∃ (exist˘ a), ∃! (exist˘a ¸si este unic). Dac˘a A este o mult¸ime ¸si x este un element al mult¸imii A, atunci vom nota: x ∈ A. ˆIn caz contrar not˘am: x ∈ / A. Dou˘a mult¸imi A ¸si B se numesc egale dac˘a au acelea¸si elemente; vom nota prin A = B. A ¸si B se numesc diferite dac˘a nu sunt egale; vom nota A 6= B. O mult¸ime A se nume¸ste submult¸ime sau parte a unei mult¸imi B dac˘ a orice element al lui A apart¸ine lui B. Se mai spune c˘a A este inclus˘a ˆın B sau B este supramult¸ime a lui A ¸si not˘am: A ⊂ B. Spunem c˘a mult¸imea A este submult¸ime strict˘a a lui B sau c˘a A este inclus˘a strict ˆın B dac˘a A ⊂ B ¸si A 6= B. Se observ˘a c˘a A = B ⇔ A ⊂ B ¸si B ⊂ A. Dac˘a X este o mult¸ime, vom nota cu P(X) mult¸imea tuturor p˘art¸ilor lui X. Mult¸imea tuturor elementelor x din X care au proprietatea P se noteaz˘a prin {x ∈ X|x verific˘a P }. Vom nota: IN = mult¸imea numerelor naturale, IN = {0, 1, 2, 3, · · · }, IN∗ = mult¸imea numerelor naturale diferite de zero, Z = mult¸imea numerelor ˆıntregi, Z = {· · · , −3, −2, −1, 0, 1, 2, 3, · · · } Z∗ = mult¸imea numerelor ˆıntregi diferitende zero, o a Q = mult¸imea numerelor rat¸ionale, Q = |a ∈ Z, b ∈ IN∗ , b Q∗ = mult¸imea numerelor rat¸ionale diferite de zero. 1

2 Definit¸ia 1.1. Fie X, Y mult¸imi nevide ¸si A, B ∈ P(X). Se definesc urm˘atoarele mult¸imi: a) Reuniunea mult¸imilor A ¸si B: A ∪ B = {x ∈ X|x ∈ A sau x ∈ B}. b) Intersect¸ia mult¸imilor A ¸si B: A ∩ B = {x ∈ X|x ∈ A ¸si x ∈ B}. Dac˘a A ∩ B = ∅, mult¸imile A ¸si B se numesc disjuncte. c) Diferent¸a mult¸imilor A ¸si B: A \ B = {x ∈ X|x ∈ A ¸si x ∈ / B}. Diferent¸a X \ A se nume¸ste complementara mult¸imii A fat¸˘ a de X ¸si se noteaz˘a cX A sau cA dac˘ a nu exist˘a nici o posibilitate de confuzie. d) Diferent¸a simetric˘a a mult¸imilor A ¸si B: A4B = (A \ B) ∪ (B \ A). e) Produsul cartezian al mult¸imilor X ¸si Y este mult¸imea tuturor perechilor ordonate (x, y) cu x ∈ X, y ∈ Y . Se noteaz˘a: X × Y = {(x, y) | x ∈ X ¸si y ∈ Y }. Dou˘ a perechi (x, y) ¸si (x0 , y 0 ) sunt egale dac˘ a ¸si numai dac˘ a x = x0 ¸si 0 y=y. Pentru X1 , X2 , ..., Xn mult¸imi nevide (n ∈ IN∗ ), produsul lor cartezian este: X1 × X2 × ... × Xn = {(x1 , x2 , ..., xn )|xi ∈ Xi , ∀i ∈ {1, 2, ..., n}}. Mult¸imea X1 × X2 × . . . × Xn se noteaz˘ a ¸si Produsul cartezian al mult¸imilor nevide Y n∈IN∗

Xn =

∞ Y

n Q

Xi . i=1 X1 , X2 , . . . , Xn , . . .

este:

Xn = {(x1 , x2 , ..., xn , ...)|xn ∈ Xn , ∀n ∈ IN∗ }.

n=1

Mult¸imile Xn se numesc factorii produsului cartezian. Dac˘ a exist˘ a un factor egal cu mult¸imea vid˘ a, produsul cartezian este mult¸imea vid˘ a. Propozit¸ia 1.2. Fie X o mult¸ime nevid˘a. Atunci pentru orice A, B, C ∈ P(X) au loc urm˘atoarele propriet˘a¸ti:

3 i) ii) iii) iv) v) vi) vii) viii) ix) x) xi) xii) xiii) xiv)

A ∪ B = B ∪ A; (A ∪ B) ∪ C = A ∪ (B ∪ C); A ∪ ∅ = A; A ∪ cA = X; A ∩ B = B ∩ A; (A ∩ B) ∩ C = A ∩ (B ∩ C); A ∩ ∅ = ∅; A ∩ X = A; A \ ∅ = A; A \ A = ∅; A4B = B4A; (A4B)4C = A4(B4C); A4∅ = A; A4A = ∅.

Definit¸ia 1.3. Fie X ¸si Y mult¸imi nevide. O submult¸ime R a produsului cartezian X × Y se nume¸ste relat¸ie de la X la Y . Dac˘a (x, y) ∈ R, atunci uneori scriem xRy ¸si spunem c˘a x este ˆın relat¸ia R cu y. Dac˘a (x, y) ∈ / R, vom mai nota aceasta prin xR /y. Dac˘a X = Y , atunci R se nume¸ste relat¸ie pe X. De exemplu, 4(X) = {(x, x)|x ∈ X} este o relat¸ie pe X, numit˘ a diagonala lui X. Definit¸ia 1.4. Pentru relat¸ia R ⊂ X × Y se definesc: a) Inversa relat¸iei R: R−1 = {(y, x) ∈ Y × X|(x, y) ∈ R}. b) Domeniul relat¸iei R: Dom R = {x ∈ X|∃y ∈ Y a.ˆı. (x, y) ∈ R}. c) Imaginea (sau codomeniul) relat¸iei R: Im R = {y ∈ Y |∃x ∈ Xa.ˆı. (x, y) ∈ R}. d) Imaginea mult¸imii A ⊂ X prin relat¸ia R: R(A) = {y ∈ Y |∃x ∈ Aa.ˆı. (x, y) ∈ R}. Observat¸ia 1.5. a) R(X) = Im R.

4 b) Dac˘a B ⊂ Y , atunci imaginea lui B prin R−1 , notat˘a R−1 (B), se nume¸ste ¸si contraimaginea lui B prin R ¸si avem: R−1 (B) = {x ∈ X|∃y ∈ B a.ˆı. (x, y) ∈ R}. Definit¸ia 1.6. Fie X, Y, T mult¸imi nevide ¸si relat¸iile R ⊂ X × Y ¸si S ⊂ Y × T . Compunerea lui S cu R, notat˘a S ◦ R, este relat¸ia de la X la T definit˘a prin: S ◦ R = {(x, t) ∈ X × T |∃y ∈ Y a.ˆı. (x, y) ∈ R, (y, t) ∈ S}. Definit¸ia 1.7. Fie X, Y mult¸imi nevide. O relat¸ie f ⊂ X ×Y se nume¸ste funct¸ie definit˘a pe X cu valori ˆın Y dac˘a ˆındepline¸ste condit¸iile: a) Dom f = X (adic˘a pentru orice x ∈ X, exist˘a y ∈ Y astfel ˆıncˆ at (x, y) ∈ f ); b) ∀x ∈ X, ∀y1 , y2 ∈ Y , ((x, y1 ) ∈ f ¸si (x, y2 ) ∈ f ) ⇒ y1 = y2 . Din definit¸ie rezult˘a c˘a pentru orice x ∈ X, exist˘a un unic y ∈ Y astfel ˆıncˆat (x, y) ∈ f . Elementul y se nume¸ste imaginea lui x prin f ¸si se noteaz˘a y = f (x). Vom nota funct¸ia prin f : X → Y , X se nume¸ste domeniul de definit¸ie a funct¸iei f , iar Y se nume¸ste codomeniul funct¸iei f sau mult¸imea ˆın care f ia valori. Graficul funct¸iei este mult¸imea Gf = {(x, y) ∈ X × Y |y = f (x)} = {(x, f (x))|x ∈ X}. Dou˘a funct¸ii f : X → Y ¸si g : A → B se numesc egale dac˘a X = A, Y = B ¸si f (x) = g(x) pentru orice x ∈ X; vom nota f = g. Funct¸iile f ¸si g se numesc diferite dac˘a nu sunt egale ¸si vom nota prin f 6= g. Observat¸ia 1.8. Fie X, Y, T mult¸imi nevide ¸si funct¸iile f : X → Y , g : Y → T. a) Im f = {y ∈ Y | ∃ x ∈ Xa.ˆı. f (x) = y} = {f (x)|x ∈ X}. b) ˆIn general, f −1 ⊂ Y × X este relat¸ie de la Y la X ¸si nu funct¸ie. c) Dac˘a A ⊂ X ¸si B ⊂ Y , atunci imaginea lui A prin f ¸si contraimaginea lui B prin f devin: f (A) = {f (x)|x ∈ A}, f −1 (B) = {x ∈ X|f (x) ∈ B}. d) g◦f este funct¸ie de la X la T ¸si pentru orice x ∈ X, (g◦f )(x) = g(f (x)). e) Funct¸ia 1X : X → X, definit˘a prin 1X (a) = a pentru orice a ∈ X, se nume¸ste funct¸ia identic˘ a.

5 f) Funct¸ia f : X → Y , definit˘a prin f (x) = y0 ∈ Y pentru orice x ∈ X, se nume¸ste funct¸ie constant˘ a. Definit¸ia 1.9. Fie X, Y mult¸imi nevide. O funct¸ie f : X → Y se nume¸ste: a) injectiv˘a (sau se spune c˘a este inject¸ie) dac˘a pentru orice x1 , x2 ∈ X cu x1 6= x2 , rezult˘a f (x1 ) 6= f (x2 ) (echivalent cu: pentru orice x1 , x2 ∈ X, din f (x1 ) = f (x2 ) rezult˘a c˘a x1 = x2 ). b) surjectiv˘a (sau este surject¸ie) dac˘a pentru orice y ∈ Y, exist˘a x ∈ X astfel ˆıncˆat f (x) = y. c) bijectiv˘a (sau este biject¸ie) dac˘a este injectiv˘a ¸si surjectiv˘a. Definit¸ia 1.10. O funct¸ie f : X → Y se nume¸ste inversabil˘ a dac˘a exist˘a o funct¸ie g : Y → X astfel ˆıncˆat f ◦ g = 1Y ¸si g ◦ f = 1X . Definit¸ia 1.11. Fie funct¸ia f : X → Y ¸si ∅ 6= A ⊂ X. Funct¸ia g : A → Y definit˘a prin g(x) = f (x) pentru orice x din A, se nume¸ste restrict¸ia lui f la mult¸imea A ¸si se noteaz˘a g = f |A . ˆIn acest caz, f se nume¸ste prelungire a lui g la X. Definit¸ia 1.12. Fie X 6= ∅. O funct¸ie f : IN → X se nume¸ste ¸sir de elemente din X. Vom nota f (n) = xn pentru orice n ∈ IN, iar f prin (xn )n∈IN sau (xn )n≥0 sau (xn )n sau (xn ). Uneori vom scrie (xn ) ⊂ X. xn se nume¸ste termenul general al ¸sirului. x0 , x1 , . . . , xn , . . . se numesc termenii ¸sirului (xn ). S¸irul (xn )n∈IN , definit prin xn = a ∈ X pentru orice n ∈ IN, se nume¸ste ¸sir constant. Definit¸ia 1.13. Fie X ¸si I mult¸imi nevide. O funct¸ie f : I → P(X) se nume¸ste familie (sau clas˘a) de mult¸imi. Dac˘a f (i) = Ai pentru orice i ∈ I, se noteaz˘a f prin (Ai )i∈I , iar I se nume¸ste mult¸ime de indici. Uneori vom scrie (Ai )i∈I ⊂ P(X). Dac˘a I este finit˘a, de exemplu I = {1, 2, · · · , n}, n ∈ IN∗ , atunci vom nota (Ai )ni=1 ¸si, ˆın acest caz, familia (Ai )i∈I se nume¸ste finit˘a. Definit¸ia 1.14. Fie X, I mult¸imi nevide ¸si (Ai )i∈I o familie de p˘art¸i ale lui X. Se definesc: a) Reuniunea mult¸imilor familiei (Ai )i∈I : [ Ai = {x ∈ X|∃i0 ∈ Ia.ˆı. x ∈ Ai0 }. i∈I

6 De exemplu, pentru I = {1, 2, ..., n} cu n ∈ IN∗ , vom scrie pentru I = IN vom scrie

S n∈IN

An sau

∞ S n=0

n S i=1

Ai , iar

An .

b) Intersect¸ia mult¸imilor familiei (Ai )i∈I : \ Ai = {x ∈ X|x ∈ Ai , ∀i ∈ I}. i∈I

Vom nota ca mai sus ˆın cazurile particulare I = {1, 2, ..., n} cu n ∈ IN∗ sau I = IN. c) Produsul cartezian: Y [ Ai = {f | f : I → Ai a.ˆı. f (i) ∈ Ai , ∀i ∈ I}. i∈I

i∈I

Dac˘a f (i) = xi ∈ Ai pentru orice i ∈ I, atunci vom mai nota f prin (xi )i∈I . Mult¸imile Ai se numesc factorii produsului cartezian. Dac˘a exist˘a un factor egal cu mult¸imea vid˘a, produsul cartezian este mult¸imea vid˘a. Definit¸ia 1.15. Fie X ¸si I mult¸imi nevide. O familie (Ai )i∈I de p˘art¸i ale a Ai 6= ∅ pentru orice i ∈ I, S lui X se nume¸ste partit¸ie a lui X dac˘ Ai = X ¸si Ai ∩ Aj = ∅ pentru orice i, j ∈ I cu i 6= j (adic˘a mult¸imile Ai

i∈I

sunt disjuncte dou˘a cˆate dou˘a sau mutual disjuncte). Definit¸ia 1.16. Fie X o mult¸ime nevid˘a ¸si R o relat¸ie pe X. R se nume¸ste: a) reflexiv˘a dac˘a ∀x ∈ X, xRx; b) simetric˘a dac˘a ∀x, y ∈ X, xRy ⇒ yRx; c) antisimetric˘a dac˘a ∀x, y ∈ X, (xRy ¸si yRx) ⇒ x = y; d) tranzitiv˘a dac˘a ∀x, y, z ∈ X, (xRy ¸si yRz) ⇒ xRz; e) total˘a dac˘a ∀x, y ∈ X, xRy sau yRx. Definit¸ia 1.17. Fie X o mult¸ime nevid˘a. O relat¸ie R pe X se nume¸ste relat¸ie de echivalent¸˘a pe X dac˘a R este reflexiv˘a, simetric˘a ¸si tranzitiv˘a. Pentru orice x din X, mult¸imea x ˆ = {y ∈ X|yRx} se nume¸ste clasa de echivalent¸˘a a lui x. Orice element din x ˆ se nume¸ste reprezentant al clasei. Mult¸imea claselor de echivalent¸˘ a, notat˘a X|R , se nume¸ste mult¸imea cˆat a lui X prin relat¸ia de echivalent¸˘ a R. Uneori vom nota o relat¸ie de echivalent¸˘ a prin “∼” iar x ˆ prin Cx . Propozit¸ia 1.18. Fie X o mult¸ime nevid˘a ¸si R o relat¸ie de echivalent¸˘ a pe X. Atunci au loc urm˘atoarele propriet˘a¸ti:

7 a) ∀x ∈ X, x ∈ x ˆ. b) Dac˘a x, y ∈ X, atunci x ˆ = yˆ ⇔ xRy ¸si x ˆ 6= yˆ ⇔ x ˆ ∩ yˆ = ∅. c) Familia {ˆ x|x ∈ X} este o partit¸ie pentru X, numit˘ a partit¸ia indus˘a pe X de relat¸ia de echivalent¸˘a R. Definit¸ia 1.19. Fie X o mult¸ime nevid˘a. O relat¸ie R pe X se nume¸ste relat¸ie de ordine (part¸ial˘a) pe X dac˘a este reflexiv˘a, antisimetric˘ a ¸si tranzitiv˘a. ˆIn acest caz, mult¸imea X ˆınzestrat˘ a cu relat¸ia de ordine R se nume¸ste mult¸ime ordonat˘a (part¸ial) ¸si se noteaz˘a (X, R). Uneori vom nota o relat¸ie de ordine prin “≤” ¸si “x ≤ y” prin “y ≥ x” (vom spune c˘a x este mai mic decˆat y sau y este mai mare decˆat x). Spunem c˘a x este strict mai mic decˆat y sau y este strict mai mare decˆat x dac˘ ax≤y ¸si x 6= y. Vom nota x < y sau y > x. Definit¸ia 1.20. Fie (X, ≤) o mult¸ime ordonat˘a ¸si ∅ 6= A ⊂ X. a x ≤ α pentru a) Un element α ∈ X se nume¸ste majorant pentru A dac˘ orice x ∈ A. b) Un element β ∈ X se nume¸ste minorant pentru A dac˘ a β ≤ x pentru orice x ∈ A. c) A se nume¸ste majorat˘a dac˘a admite un majorant. d) A se nume¸ste minorat˘a dac˘a admite un minorant. e) A se nume¸ste m˘arginit˘a dac˘a este majorat˘a ¸si minorat˘a. f) Dac˘a a ∈ A este un majorant pentru A, atunci a se nume¸ste maximum sau cel mai mare element al mult¸imii A ¸si se noteaz˘a a = max A. g) Dac˘a b ∈ A este un minorant pentru A, atunci b se nume¸ste minimum sau cel mai mic element al mult¸imii A ¸si se noteaz˘a b = min A. h) Dac˘a A este majorat˘a ¸si dac˘a exist˘a α ∈ X cel mai mic majorant pentru A, atunci α se nume¸ste margine superioar˘a sau supremum pentru A ¸si se noteaz˘a α = sup A. i) Dac˘a A este minorat˘a ¸si dac˘a exist˘a β ∈ X cel mai mare minorant pentru A, atunci β se nume¸ste margine inferioar˘a sau infimum pentru A ¸si se noteaz˘a β = inf A. j) Un element a ∈ A se nume¸ste element maximal al mult¸imii A dac˘ a din x ∈ A ¸si a ≤ x rezult˘a x = a. k) Un element b ∈ A se nume¸ste element minimal al mult¸imii A dac˘ a din x ∈ A ¸si x ≤ b rezult˘a x = b. Observat¸ia 1.21. a) Dac˘a exist˘a marginea superioar˘a (respectiv marginea inferioar˘a, maximum, minimum) pentru mult¸imea A, atunci aceasta este unic˘a.

8 b) c) d) e)

Dac˘a exist˘a α, β ∈ X astfel ˆıncˆ at α= inf A, β= sup A, atunci α ≤ β. Dac˘a α ∈ A este majorant pentru A, atunci α = max A = sup A. Dac˘a β ∈ A este minorant pentru A, atunci β = min A = inf A. Dac˘a A = {a} unde a ∈ X, atunci max A = min A = sup A = inf A = a, iar a este element maximal ¸si element minimal pentru A.

Definit¸ia 1.22. Se nume¸ste mult¸ime (sau sistem) de numere reale o mult¸ime IR cu cel put¸in dou˘a elemente ˆınzestrat˘ a cu dou˘a operat¸ii algebrice: adunarea: (x, y) 7→ x + y ∈ IR, ∀x, y ∈ IR ¸si ˆınmult¸irea: (x, y) 7→ x · y = xy ∈ IR, ∀x, y ∈ IR ¸si cu o relat¸ie pe IR, notat˘a ≤, astfel ˆıncˆ at au loc urm˘atoarele propriet˘a¸ti: I. (IR, +, ·) este corp comutativ total ordonat, adic˘a: a) b) c) d) e) f) g) h) i) j) k) l) m) n) o)

(x + y) + z = x + (y + z), ∀x, y, z ∈ IR. x + y = y + x, ∀x, y ∈ IR. ∃0 ∈ IR a.ˆı. x + 0 = x, ∀x ∈ IR. ∀x ∈ IR, ∃ − x ∈ IR a.ˆı. x + (−x) = 0. (xy)z = x(yz), ∀x, y, z ∈ IR. xy = yx, ∀x, y ∈ IR. ∃1 ∈ IR, 1 6= 0 a.ˆı. 1 · x = x, ∀x ∈ IR. 1 ∀x ∈ IR, x 6= 0, ∃x−1 ∈ IR (notat ¸si ) a.ˆı. x · x−1 = 1. x x(y + z) = xy + xz, ∀x, y, z ∈ IR. ∀x ∈ IR, x ≤ x. ∀x, y ∈ IR, (x ≤ y ¸si y ≤ x) ⇒ x = y. ∀x, y, z ∈ IR, (x ≤ y ¸si y ≤ z) ⇒ x ≤ z. ∀x, y ∈ IR, x ≤ y sau y ≤ x. ∀x, y, z ∈ IR, x ≤ y ⇒ x + z ≤ y + z. ∀x, y, z ∈ IR, (z ≥ 0 ¸si x ≤ y) ⇒ xz ≤ yz.

II. IR verific˘a axioma de completitudine (sau axioma lui Cantor - Dedekind): orice submult¸ime nevid˘a ¸si majorat˘a a lui IR admite margine superioar˘a ˆın IR. IR este unic determinat pˆan˘ a la un izomorfism de corpuri total ordonate, adic˘a pentru orice alt˘a mult¸ime (IR0 , ⊕, ¯, ° ≤ ) cu propriet˘ a¸tile de mai sus, exist˘a o biject¸ie ϕ : IR → IR0 astfel ˆıncˆ at: i) ϕ(x + y) = ϕ(x) ⊕ ϕ(y), ∀x, y ∈ IR; ii) ϕ(x · y) = ϕ(x) ¯ ϕ(y), ∀x, y ∈ IR; iii) ∀x, y ∈ IR, x ≤ y ⇒ ϕ(x)° ≤ ϕ(y).

9 Observat¸ia 1.23. i) Dac˘a x, y ∈ IR, atunci uneori vom nota “x ≤ y” prin “y ≥ x”. ii) (Q, +, ·, ≤) este un corp comutativ total ordonat care nu verific˘ a axioma de completitudine. De exemplu, mult¸imea A ⊂ Q, A = {x ∈ Q|x2 < 2}, este nevid˘a ¸si majorat˘a ˆın Q dar nu admite supremum ˆın Q (vezi problema 1.64). Definit¸ia 1.24. Fie a, b ∈ IR, a < b. Se definesc urm˘atoarele tipuri de intervale: a) intervale m˘arginite: [a, b] = {x ∈ IR|a ≤ x ≤ b} (numit interval compact), (a, b) = {x ∈ IR|a < x < b}, [a, b) = {x ∈ IR|a ≤ x < b}, (a, b] = {x ∈ IR|a < x ≤ b}; b) intervale nem˘arginite: [a, +∞) = {x ∈ IR|x ≥ a}, (a, +∞) = {x ∈ IR|x > a}, (−∞, a] = {x ∈ IR|x ≤ a}, (−∞, a) = {x ∈ IR|x < a}. Vom nota IR+ = (0, +∞), IR− = (−∞, 0), IR∗ = IR+ ∪ IR− = IR\{0}. Definit¸ia 1.25. Fie x, y ∈ IR. a) x se nume¸ste pozitiv (respectiv negativ) dac˘a x > 0 (respectiv x < 0). b) x se nume¸ste nenegativ (respectiv nepozitiv) dac˘a x ≥ 0 (respectiv x ≤ 0). c) Fie X 6= ∅ ¸si f : X → IR. Funct¸ia f se nume¸ste nenegativ˘a (respectiv nepozitiv˘a) dac˘a ia valori ˆın [0, +∞) (respectiv ˆın (−∞, 0]). Teorema 1.26.½Fie ∅ 6= A ⊂ IR ¸si α, β ∈ IR. Atunci: 1)∀x ∈ A, x ≤ α; i) sup A = α ⇔ 2)∀ε > 0, ∃xε ∈ A a.ˆı. xε > α − ε. ii) A este nemajorat˘a ⇔ ∀α ∈ IR, ∃xα ∈ A a.ˆı. xα > α ⇔ ∀ε > 0, ∃xε ∈ A a.ˆı. xε > ε. ½ 1)∀x ∈ A, x ≥ β; iii) inf A = β ⇔ 2)∀ε > 0, ∃xε ∈ A a.ˆı. xε < β + ε. iv) A este neminorat˘a ⇔ ∀α ∈ IR, ∃xα ∈ A a.ˆı. xα < α ⇔ ∀ε > 0, ∃xε ∈ A a.ˆı. xε < −ε.

10 Propozit¸ia 1.27. Fie A, B submult¸imi nevide ale lui IR astfel ˆıncˆ at A ⊂ B. i) Dac˘a B este majorat˘a, atunci A este majorat˘a ¸si sup A ≤ sup B. ii) Dac˘a B este minorat˘a, atunci A este minorat˘a ¸si inf A ≥ inf B. Propozit¸ia 1.28. Fie ∅ 6= A ⊂ IR ¸si mult¸imea −A = {−x|x ∈ A}. i) Dac˘a A este majorat˘a, atunci mult¸imea −A este minorat˘a ¸si inf(−A) = − sup A. ii) Dac˘a A este minorat˘a, atunci mult¸imea −A este majorat˘a ¸si sup(−A) = − inf A. Teorema 1.29. Orice submult¸ime nevid˘a ¸si minorat˘a a lui IR admite margine inferioar˘a ˆın IR. Teorema 1.30. (Proprietatea lui Arhimede.) Pentru orice numere reale x, y cu x > 0, exist˘a n ∈ IN astfel ˆıncˆ at nx > y. Teorema 1.31. (de existent¸˘ a ¸si unicitate a p˘ art¸ii ˆıntregi) Pentru orice x ∈ IR, exist˘a ¸si este unic k ∈ Z astfel ˆıncˆ at k ≤ x < k + 1. Se noteaz˘a k = [x] ¸si se nume¸ste partea ˆıntreag˘ a a lui x. Diferent¸a x − [x] ∈ [0, 1) se nume¸ste partea fract¸ionar˘ a a lui x. Teorema 1.32. (Densitatea lui Q ˆın IR.) Pentru orice numere reale x, y cu x < y, exist˘a r ∈ Q astfel ˆıncˆ at x < r < y. Observat¸ia 1.33. Un rezultat asem˘an˘ ator are loc ¸si pentru numere irat¸ionale: pentru orice numere reale x, y cu x < y, exist˘a α ∈ IR \ Q astfel ˆıncˆat x < α < y (vezi problema 1.65). De fapt (vezi problema 1.66), ˆıntre orice dou˘a numere reale diferite exist˘a o infinitate de numere rat¸ionale ¸si o infinitate de numere irat¸ionale. Definit¸ia 1.34. a) Vom nota prin “sgn” funct¸ia semn (sau signum) definit˘a pe IR astfel:   1, x > 0 0, x = 0 sgn x =  −1, x < 0. b) Dac˘a x, y ∈ IR, se noteaz˘a: ½ max{x, y} =

x, y,

dac˘a x ≥ y dac˘a x < y

11 ¸si

½ min{x, y} =

x, y,

dac˘ ax≤y dac˘a x > y.

c) Fie ∅ 6= A ⊂ IR ¸si f, g : A → IR. Spunem c˘a f este mai mic˘a sau egal˘a cu f (¸si not˘am f ≤ g) dac˘a f (x) ≤ g(x) pentru orice x ∈ A. Analog se definesc relat¸iile: f ≥ g, f < g ¸si f > g. d) Fie ∅ 6= A ⊂ IR ¸si f, g : A → IR. • f se nume¸ste majorat˘a dac˘a mult¸imea Imf = {f (x)|x ∈ A} este majorat˘a. In acest caz, vom nota sup(Im f ) = sup{f (x)|x ∈ A} prin sup f (x). x∈A

• f se nume¸ste minorat˘a dac˘a mult¸imea Im f este minorat˘a ¸si vom nota inf(Im f ) = inf{f (x)|x ∈ A} prin inf f (x). x∈A

a dac˘a este majorat˘a ¸si minorat˘a. • f se nume¸ste m˘arginit˘ • Dac˘a A = IN ¸si f (n) = xn , ∀n ∈ IN, atunci vom nota sup(Im f ) (respectiv inf(Im f )), dac˘a acesta exist˘a, prin sup xn sau sup xn n∈IN

n

sau sup xn (respectiv inf xn sau inf xn sau inf xn ). n∈IN

n

f (dac˘ a g(x) 6= 0, g ∀x ∈ A) ¸si f g (dac˘ a f (x) > 0, ∀x ∈ A) funct¸iile definite astfel: (f + g)(x) = f (x) + g(x), (αf )(x) = αf (x), (f g)(x) = f (x)g(x), (f g )(x) = [f (x)]g(x) . Uneori vom nota [f (x)]α prin f α (x).

• Vom nota prin f + g, αf (α ∈ IR), f · g = f g,

Definit¸ia 1.35. Fie ∅ 6= X ⊂ IR. O funct¸ie f : X → IR se nume¸ste: a) cresc˘atoare dac˘a pentru orice x1 , x2 ∈ X, din x1 < x2 rezult˘ a f (x1 ) ≤ f (x2 ); b) strict cresc˘atoare dac˘a pentru orice x1 , x2 ∈ X, din x1 < x2 rezult˘a f (x1 ) < f (x2 ); c) descresc˘atoare dac˘a pentru orice x1 , x2 ∈ X, din x1 < x2 rezult˘a f (x1 ) ≥ f (x2 ); d) strict descresc˘atoare dac˘a pentru orice x1 , x2 ∈ X, din x1 < x2 rezult˘a f (x1 ) < f (x2 ); e) strict monoton˘a dac˘a este strict cresc˘atoare sau strict descresc˘atoare; f) monoton˘a dac˘a ˆındepline¸ste una dintre condit¸iile a), b), c), d). Propozit¸ia 1.36. Fie ∅ 6= X ⊂ IR, funct¸iile f, g : X → IR ¸si α ∈ IR.

12 i) Dac˘a f, g sunt cresc˘atoare (respectiv descresc˘atoare), atunci f + g este cresc˘atoare (respectiv descresc˘atoare). ii) Dac˘a f este cresc˘atoare, atunci αf este cresc˘atoare pentru α > 0 ¸si descresc˘atoare pentru α < 0. Dac˘a f este descresc˘atoare, atunci αf este descresc˘atoare pentru α > 0 ¸si cresc˘atoare pentru α < 0. iii) Dac˘a f ≥ 0, g ≥ 0 ¸si f, g sunt cresc˘atoare (respectiv descresc˘atoare), atunci f g este cresc˘atoare (respectiv descresc˘atoare). Propozit¸ia 1.37. Fie ∅ 6= X, Y ⊂ IR ¸si f : X → Y , g : Y → IR. Dac˘a f, g sunt monotone de acela¸si sens, atunci g ◦ f este cresc˘atoare. Dac˘a f, g sunt monotone de sens contrar, atunci g ◦f este descresc˘atoare. Definit¸ia 1.38. Fie ∅ 6= A ⊂ IR ¸si funct¸ia f : A → IR. at: a) f se nume¸ste periodic˘a dac˘a exist˘a T ∈ IR∗ astfel ˆıncˆ i) x + T ∈ A, pentru orice x ∈ A, ii) f (x + T ) = f (x), ∀x ∈ A. T se nume¸ste perioad˘a a funct¸iei f . b) Presupunˆand c˘a f este periodic˘a ¸si admite cel put¸in o perioad˘a pozitiv˘a, se spune c˘a f are perioad˘a principal˘a dac˘a f admite cea mai mic˘a perioad˘a pozitiv˘a (aceasta se nume¸ste perioad˘a principal˘a). Definit¸ia 1.39. Vom nota IR = IR ∪ {−∞, +∞}, unde elementele −∞, +∞ ∈ / IR ¸si se numesc minus infinit, respectiv plus infinit. Uneori vom scrie ˆın loc de +∞ simplu ∞ ¸si vom spune, pe scurt, infinit a. ˆın loc de plus infinit. IR se nume¸ste dreapta real˘a ˆıncheiat˘ Vom prelungi ordinea uzual˘a a lui IR la IR astfel: −∞ < ∞, −∞ < x < +∞, ∀x ∈ IR. In felul acesta, mult¸imea IR devine total ordonat˘a ¸si m˘arginit˘ a ¸si avem: inf IR = min IR = −∞, sup IR = max IR = +∞. Dac˘a ∅ 6= A ⊂ IR este nemajorat˘a ˆın IR, atunci ea este majorat˘a ˆın IR ¸si sup A = +∞. Analog, dac˘a A este neminorat˘a ˆın IR atunci ea este minorat˘a ˆın IR ¸si inf A = −∞. Operat¸iile algebrice din IR se vor prelungi la IR, f˘ar˘ a a fi definite peste tot. Astfel vom avea: x + ∞ = ∞ + x = ∞, ∀x ∈ IR \ {−∞}, −∞ + x = x + (−∞) = −∞, ∀x ∈ IR \ {+∞}, ∞ · x = x · ∞ = ∞, ∀x ∈ IR, x > 0, ∞ · x = x · ∞ = −∞, ∀x ∈ IR, x < 0.

13 Urm˘atoarele operat¸ii nu au sens: ∞ + (−∞), (−∞) + ∞, 0 · ∞, 0 · (−∞), 0 ±∞ ∞ −∞ 0 0 , ,1 ,1 ,0 ,∞ . 0 ±∞ Dac˘a a ∈ IR, se definesc intervalele: [a, +∞] = {x ∈ IR | x ≥ a}, (a, +∞] = {x ∈ IR | x > a}, [−∞, a] = {x ∈ IR | x ≤ a}, [−∞, a) = {x ∈ IR | x < a}, [−∞, +∞) = {x ∈ IR | x < +∞} = IR \ {+∞}, (−∞, +∞] = {x ∈ IR | x > −∞} = IR \ {−∞}, [−∞, +∞] = IR. ˆIn continuare, consider˘am U clasa tuturor mult¸imilor. Propriet˘a¸tile mult¸imii U sunt contradictorii. ˆIntr-adev˘ ar, fie A acea submult¸ime a lui U format˘a din mult¸imile care nu se cont¸in ca element. Avem dou˘a situat¸ii posibile: I. Dac˘a A se cont¸ine ca element, atunci A ar cont¸ine o mult¸ime ce se cont¸ine ca element, ceea ce contrazice definit¸ia lui A. II. Dac˘a A nu se cont¸ine ca element, atunci din definit¸ia lui A ar rezulta c˘a A apart¸ine ca element lui A, ceea ce contrazice ipoteza din II. Aceast˘a contradict¸ie a generat numeroase discut¸ii ¸si a condus la diferite solut¸ii date de matematicieni, generˆand faimosul paradox al lui Russel, numit “mult¸imea tuturor mult¸imilor”. Pentru clarific˘ari se poate consulta Vasilache S. [19]. ˆIn cele ce urmeaz˘a, vom presupune c˘a U nu se cont¸ine pe ea ˆıns˘ a¸si ca element. Definit¸ia 1.40. Mult¸imile nevide A ¸si B se numesc echipotente (sau cardinal echivalente) dac˘a exist˘a o funct¸ie bijectiv˘a f : A → B. Vom nota aceasta prin A ∼ B. Prin convent¸ie, A ∼ ∅ dac˘a ¸si numai dac˘a A = ∅. Teorema 1.41. Relat¸ia de echipotent¸˘ a “∼” este o relat¸ie de echivalent¸˘ a pe mult¸imea U. Pentru orice A ∈ U, clasa de echivalent¸˘ a a mult¸imii A se nume¸ste cardinalul lui A, notat card A. Deci card A = {B ∈ U|B ∼ A}. Avem card A = card B ⇔ A ∼ B. Prin convent¸ie, card ∅ = 0. Definit¸ia 1.42. O mult¸ime A se nume¸ste: a) finit˘a dac˘a este echipotent˘a cu o mult¸ime de forma {1, 2, ..., n}, unde n ∈ IN∗ . ˆIn acest caz, cardinalul lui A se nume¸ste finit ¸si vom scrie card A = n. Se va considera card ∅ = 0 ca fiind un cardinal finit.

14 b) infinit˘a dac˘a nu este finit˘a. Cardinalul unei mult¸imi infinite se nume¸ste transfinit. c) num˘arabil˘a dac˘a A este echipotent˘ a cu IN. Se noteaz˘a card IN = ℵ0 (alef zero). d) cel mult num˘arabil˘ a dac˘a A este finit˘a sau num˘ arabil˘ a. Definit¸ia 1.43. Fiind date α ¸si β cardinale, se definesc urm˘atoarele operat¸ii: a) adunarea: α+β = card(A∪B), unde A ∈ α, B ∈ β astfel ˆıncˆ at A∩B = ∅. Adunarea este corect definit˘a deoarece putem alege reprezentant¸i disjunct¸i pentru orice dou˘a cardinale. b) ˆınmult¸irea: α · β = αβ = card(C × D), unde C ∈ α, D ∈ β. c) ridicarea la putere: αβ = card C D dac˘ a α 6= 0, β 6= 0, C ∈ α, D ∈ β (unde prin C D am notat mult¸imea funct¸iilor definite pe D cu valori ˆın C). Adunarea ¸si ˆınmult¸irea sunt comutative ¸si asociative, ˆınmult¸irea este distributiv˘a fat¸˘a de adunare iar ridicarea la putere are urm˘atoarele propriet˘a¸ti: αβ+γ = αβ ·αγ , (αβ)γ = αγ β γ , (αβ )γ = αβγ oricare ar fi α, β, γ 6= 0 cardinale arbitrare (vezi Tama¸s, Leoreanu [18]). Propozit¸ia 1.44. O mult¸ime A este num˘ arabil˘ a dac˘a ¸si numai dac˘a A se scrie ˆın forma A = {x1 , x2 , ..., xn , ...}, unde (xn )n∈IN∗ este un ¸sir cu termeni diferit¸i. Definit¸ia 1.45. Fie A, B mult¸imi nevide. Se spune: a) 0 ≤ card A ¸si card A ≤ card B dac˘a exist˘a o funct¸ie injectiv˘a f : A → B. a card A ≤ card B ¸si card A 6= card B (rezult˘ a c˘a b) card A < card B dac˘ 0 < card A). Propozit¸ia 1.46. Fie A ¸si B dou˘a mult¸imi astfel ˆıncˆ at A ⊂ B. Atunci card A ≤ card B. Propozit¸ia 1.47. Fie A ¸si B dou˘ a mult¸imi. arabil˘ a, atunci (A ∪ B) ∼ A. a) Dac˘a A este infinit˘a ¸si B cel mult num˘ b) Dac˘a A este infinit˘a nenum˘ arabil˘ a ¸si B ⊂ A astfel ˆıncˆ at card B ≤ ℵ0 , atunci (A \ B) ∼ A.

15 Teorema 1.48. Fie A o mult¸ime nevid˘a cu card A = α. Atunci: a) card P(A) = 2α ; b) α < 2α . Propozit¸ia 1.49. Oricare ar fi cardinalul finit n, are loc: n < ℵ0 . Teorema 1.50. Relat¸ia “≤” din definit¸ia 1.45-a) este o relat¸ie de ordine total˘a pe mult¸imea U|∼ . Propozit¸ia 1.51. Orice mult¸ime infinit˘a cont¸ine o submult¸ime num˘ arabil˘a (rezult˘a c˘a pentru orice cardinal transfinit α avem: ℵ0 ≤ α). Teorema 1.52. Au loc urm˘atoarele propriet˘a¸ti: i) Dac˘a A este num˘arabil˘a, B este nevid˘a ¸si cel mult num˘ arabil˘ a, atunci A × B este num˘arabil˘a. ii) Orice reuniune finit˘a de mult¸imi finite este finit˘a. arabile este mult¸ime nuiii) Orice reuniune nevid˘a finit˘a de mult¸imi num˘ m˘arabil˘a. iv) Orice reuniune num˘arabil˘a de mult¸imi nevide finite ¸si mutual disjuncte este mult¸ime num˘arabil˘a. v) Orice reuniune num˘arabil˘a de mult¸imi num˘ arabile este mult¸ime num˘ arabil˘a. Propozit¸ia 1.53. Urm˘atoarele mult¸imi sunt num˘ arabile: i) Z, Q; ii) mult¸imea polinoamelor cu coeficient¸i rat¸ionali; iii) mult¸imea numerelor algebrice (amintim c˘a un num˘ ar real se nume¸ste algebric dac˘a este r˘ad˘acin˘ a a unui polinom de grad mai mare sau egal ca 1 ¸si cu coeficient¸i rat¸ionali; exemple de numere algebrice: √ 3 1 √ 3, −4, , − , 3, − 2). 5 7 Teorema 1.54. Mult¸imea [0, 1] este nenum˘ arabil˘ a (infinit˘a). Definit¸ia 1.55. Cardinalul mult¸imii [0, 1] se nume¸ste puterea continuului. Se noteaz˘a card[0, 1] = c (c > ℵ0 ). Propozit¸ia 1.56. Urm˘atoarele mult¸imi au cardinalul c: i) [a, b], (a, b], [a, b), (a, b) pentru orice a, b ∈ IR cu a < b; ii) (−∞, a), (−∞, a], [a, +∞), (a, ∞) pentru orice a ∈ IR; iii) orice submult¸ime a lui IR care include un interval;

16 iv) IR \ Q; v) mult¸imea lui Cantor(vezi Precupanu A.M. [14]-propozit¸ia 1.5-29); ar real se nume¸ste tranvi) mult¸imea numerelor transcendente (un num˘ scendent dac˘a nu este algebric, de exemplu: e, π). Propozit¸ia 1.57. Fie α, β, γ cardinale α 6= 0, β 6= 0, γ 6= 0. Dac˘a α ≤ β, atunci: i) ii) iii) iv)

α + γ ≤ β + γ; α · γ ≤ β · γ; γα ≤ γβ ; αγ ≤ β γ .

Propozit¸ia 1.58. i) ii) iii) iv) v)

n + ℵ0 = ℵ0 , ∀n ∈ IN; n · ℵ0 = ℵ0 , ∀n ∈ IN∗ ; (ℵ0 )n = ℵ0 , ∀n ∈ IN∗ ; n + c = c, ∀n ∈ IN; ℵ0 + c = c.

17

Probleme 1.1 Fie X 6= ∅ ¸si A, B, C ⊂ X. Atunci au loc: a) A \ B = A ∩ cB; b) A ∩ (B \ C) = (A ∩ B) \ (A ∩ C) = (A ∩ B) \ C; c) (A ∩ B) \ C = (A \ C) ∩ (B \ C); d) (A ∪ B) \ C = (A \ C) ∪ (B \ C). 1.2 Fie X 6= ∅. Ar˘atat¸i c˘a pentru orice (Ai )i∈I ⊂ P(X) (I 6= ∅) au loc relat¸iile lui De Morgan: µ[ ¶ \ c Ai = (cAi ); i∈I

i∈I

µ\ ¶ [ c Ai = (cAi ). i∈I

i∈I

1.3 Fie X 6= ∅. Ar˘atat¸i c˘a pentru orice E ⊂ X, (Ai )i∈I ⊂ P(X)(I 6= ∅) ¸si (Bj )j∈J ⊂ P(X)(J 6= ∅) au loc formulele generale de distributivitate: ³S ´ S a) E ∩ Ai = (E ∩ Ai ); i∈I ³ i∈I T ´ T Ai = (E ∪ Ai ); b) E ∪ ³ S i∈I ´ ³ S i∈I ´ S c) Ai ∩ Bj = (Ai ∩ Bj ); i∈I j∈J (i,j)∈I×J ³T ´ ³ T ´ T d) Ai ∪ Bj = (Ai ∪ Bj ). i∈I

j∈J

(i,j)∈I×J

1.4 Fie X 6= ∅. Pentru orice A ⊂ X ¸si (Bi )i∈I ⊂ P(X)(I 6= ∅) au loc: ³S ´ T a) A \ Bi = (A \ Bi ); i∈I ³ S i∈I´ S b) Bi \ A = (Bi \ A); i∈I ³ T ´ i∈I S c) A \ Bi = (A \ Bi ); i∈I ³ T i∈I´ T Bi \ A = (Bi \ A). d) i∈I

i∈I

1.5 Fie X 6= ∅ ¸si A, B ⊂ X. Determinat¸i E ⊂ X astfel ˆıncˆ at E4A = B.

18 1.6 Fie X, Y, T 6= ∅. Atunci au loc: a) b) c) d)

X X X X

× (Y ∪ T ) = (X × Y ) ∪ (X × T ); × (Y ∩ T ) = (X × Y ) ∩ (X × T ); × (Y \ T ) = (X × Y ) \ (X × T ); × (Y 4T ) = (X × Y )4(X × T ).

1.7 Fie X, Y 6= ∅, D ⊂ X, E ⊂ Y , (Ai )i∈I ⊂ P(X)(I 6= ∅) ¸si (Bj )j∈J ⊂ P(Y )(J 6= ∅). S˘a se arate c˘a: ³T ´ ³S ´ S T a) Ai × E = (Ai × E), Ai × E = (Ai × E); i∈I ³ i∈I ³ ´ i∈I ´ i∈I T S S T b) D × Bj = (D × Bj ), D × Bj = (D × Bj ); j∈J j∈J ³ S j∈J ´ ³ S j∈J´ S c) Ai × Bj = (Ai × Bj ); i∈I j∈J (i,j)∈I×J ³T ´ ³ T ´ T d) Ai × Bj = (Ai × Bj ); i∈I j∈J (i,j)∈I×J ³T ´ ³T ´ T e) (Ai × Bi ) = Ai × Bi ; i∈I i∈I i∈I ³ ´ ³ S S S ´ f) (Ai × Bi ) ⊂ Ai × Bi ¸si precizat¸i exemple pentru inclui∈I

i∈I

i∈I

ziune strict˘a ¸si pentru egalitate. 1.8 Fie (An )n∈IN∗ ⊂ P(X). (An )n se nume¸ste ascendent dac˘a An ⊂ An+1 , ∀n ∈ IN∗ . (An )n se nume¸ste descendent dac˘a An ⊃ An+1 , ∀n ∈ IN∗ . Se definesc urm˘atoarele submult¸imi ale lui X: ∞ T ∞ S Ak (notat˘ a ¸si lim inf An sau lim inf An ) numit˘ a • lim inf An = n→∞

n=1 k=n

n

n=1 k=n

n

limita inferioar˘a a ¸sirului (An ) ¸si ∞ S ∞ T • lim sup An = Ak (notat˘ a ¸si lim sup An sau lim sup An ) numit˘ a n→∞

limita superioar˘a a ¸sirului (An ). Cele dou˘a mult¸imi definite mai sus se numesc limitele extreme ale ¸sirului (An )n . (An )n se nume¸ste convergent dac˘a lim inf An = lim sup An . Valoarea comun˘a a celor dou˘a limite se noteaz˘a lim An (sau lim An ). Mult¸imea lim An n→∞

se nume¸ste limita ¸sirului (An )n . S˘a se arate c˘a: ∞ ∞ T S An ⊂ lim inf An ⊂ lim sup An ⊂ An ; a) n=1

n=1

n

n

19 b) lim sup(cAn ) = c(lim inf An ) ¸si lim inf(cAn ) = c(lim sup An ); c) Dac˘a (An )n este ascendent, atunci el este convergent ¸si lim An = n ∞ S An ; n=1

d) Dac˘a (An )n este descendent, atunci el este convergent ¸si lim An = n ∞ T An . n=1

1.9 Fie X 6= ∅. ϕA

Pentru ½ orice mult¸ime A ⊂ X se define¸ste funct¸ia 1, x ∈ A : X → IR, ϕA (x) = numit˘ a funct¸ia caracteristic˘a a lui 0, x ∈ /A

A. Ar˘atat¸i c˘a pentru orice A, B ∈ P(X) au loc urm˘atoarele propriet˘a¸ti: a) b) c) d) e) f) g) h) i) j) k)

A = B ⇔ ϕA = ϕB ; A ⊂ B ⇔ ϕA ≤ ϕB ; ϕαA = ϕA , ∀α > 0; ϕA∩B = ϕA · ϕB ; ϕA∪B = ϕA + ϕB − ϕA ϕB ; ϕA\B = ϕA − ϕA ϕB ; ϕcA = 1 − ϕA ; ϕA4B = ϕA + ϕB − 2ϕA ϕB ; ϕ∅ = 0, ϕX = 1; ϕA = 0 ⇔ A = ∅; ϕA = 1 ⇔ A = X.

1.10 Fie X 6= ∅ ¸si A, B, C ∈ P(X). Folosind propriet˘a¸tile funct¸iei caracteristice ar˘atat¸i c˘a: a) b) c) d) e)

A \ (B ∪ C) = (A \ B) ∪ (A \ C) ⇔ A ∩ (B4C) = ∅; A \ (B ∩ C) = (A \ B) ∩ (A \ C) ⇔ A ∩ (B4C) = ∅; A ∪ (B \ C) = (A ∪ B) \ (A ∪ C) ⇔ A = ∅; (A ∩ B) ∪ (C \ A) = C ⇔ A ∩ (B4C) = ∅; (A4B)4C = A4(B4C).

1.11 Fie R = {(1, 2), (1, 3), (3, 2), (5, 4), (5, 5)} ⊂ IN × IN ¸si S = {(2, 1), (2, 2), (3, 3), (5, 3)} ⊂ IN × IN. Determinat¸i: Dom R, Im R, 4({1, 3, 9}), R−1 , R({1, 3}), R({4}), −1 R ({2, 4}), S −1 ({2}), S ◦ R, R ◦ S. 1.12 Fie S = {(a2 , a)|a ∈ IR} ⊂ IR × IR. S˘a se g˘aseasc˘ a Dom S, Im S.

20 1.13 Fie S ⊂ IR × IR, S = {(x, y) ∈ IR × IR | 0 ≤ x − y ≤ 1}. Determinat¸i ¸si S −1 ◦ S.

S −1

1.14 Fie X, Y, V, W 6= ∅ ¸si R ⊂ X × Y , S ⊂ Y × V , T ⊂ Z × W , R1 ⊂ X × Y , R2 ⊂ X × Y . Ar˘atat¸i c˘a: a) b) c) d) e) f) g) h) i) j)

R1 = R2 ⇔ R1 (A) = R2 (A), ∀A ⊂ X; R1 = R2 ⇔ R1 ({x}) = R2 ({x}), ∀x ∈ X; (R−1 )−1 = R; Im R = Dom R−1 ¸si Dom R = Im R−1 ; (S ◦ R)(A) = S(R(A)), ∀A ⊂ X; (T ◦ S) ◦ R = T ◦ (S ◦ R); (S ◦ R)−1 = R−1 ◦ S −1 ; 4(Im R) ⊂ R ◦ R−1 ; 4(Dom R) ⊂ R−1 ◦ R; ∀A, B ∈ P(X), A ⊂³B ⇒ R(A) ⊂ R(B); S ´ S k) ∀(Ai )i∈I ⊂ P(X), R Ai = R(Ai ); i∈I ³i∈I ´ T T l) ∀(Ai )i∈I ⊂ P(X), R Ai ⊂ R(Ai ) ¸si precizat¸i exemple pentru i∈I

i∈I

egalitate ¸si pentru incluziune strict˘a. 1.15 Pe IN × IN se define¸ste relat¸ia: (a, b) ∼ (c, d) ⇔ a + d = b + c, ∀(a, b), (c, d) ∈ IN × IN. Ar˘atat¸i c˘a relat¸ia “∼” este relat¸ie de echivalent¸˘ a ¸si apoi g˘asit¸i C(7,3) , clasa de echivalent¸˘a a elementului (7, 3). 1.16 Fie relat¸ia “∼” definit˘a pe IR×IR prin: (x1 , y1 ) ∼ (x2 , y2 ) ⇔ x1 = x2 , ∀(x1 , y1 ), (x2 , y2 ) ∈ IR × IR. Ar˘atat¸i c˘a relat¸ia “∼” este relat¸ie de echivalent¸˘ a ¸si determinat¸i C(a,b) , clasa de echivalent¸˘a a elementului (a, b) ∈ IR × IR. 1.17 Fie X 6= ∅ ¸si (Ai )i∈I o partit¸ie a lui X (I 6= ∅). Fie R =

S i∈I

(Ai ×Ai ) ⊂

X × X. Ar˘atat¸i c˘a R este o relat¸ie de echivalent¸˘ a pe X care determin˘a pe X aceea¸si partit¸ie ca familia (Ai )i∈I . 1.18 Pe IN∗ se define¸ste relat¸ia: x ≤ y ⇔ x|y (x divide y), ∀x, y ∈ IN∗ . Atunci: a) Relat¸ia “≤” este relat¸ie de ordine pe IN∗ . Este relat¸ia “≤” total˘a? a, ar˘atat¸i c˘a A este b) Dac˘a mult¸imea A ⊂ IN∗ este nevid˘a ¸si m˘arginit˘ finit˘a.

21 c) S˘a se arate c˘a ˆın spat¸iul (IN∗ , ≤) este satisf˘acut˘ a axioma Cantor Dedekind. d) Determinat¸i mult¸imea majorant¸ilor, mult¸imea minorant¸ilor, inf, sup, min, max, elementele maximale ¸si elementele minimale pentru mult¸imea A = {1, 2, 3, 5, 6, 7}. . 1.19 Pe IN∗ se define¸ste relat¸ia: x ≤ y ⇔ x..y (x este divizibil cu y), ∀x, y ∈ IN∗ . a) Ar˘atat¸i c˘a relat¸ia “≤” este relat¸ie de ordine pe IN∗ . Este relat¸ia “≤” total˘a? b) Determinat¸i mult¸imea majorant¸ilor, mult¸imea minorant¸ilor, inf, sup, min, max, elementele maximale ¸si elementele minimale pentru mult¸imea B = {2, 3, 4, 6, 7, 9}. 1.20 Fie X 6= ∅. Pe P(X) se define¸ste relat¸ia “≤” prin: ∀A, B ∈ P(X), A ≤ B ⇔ A ⊂ B. a) Ar˘atat¸i c˘a (P(X), ≤) este spat¸iu ordonat. Este relat¸ia “≤” total˘a? Determinat¸i min P(X) ¸si max P(X). b) Fie (Ai )i∈I ⊂ P(X) ¸si M = {Ai |i ∈ I}. Determinat¸i sup M ¸si inf M. Fie B ⊂ M. PresupunemSc˘a exist˘a A ∈ M astfel ˆıncˆ Sat A este majorant pentru B. Ar˘atat¸i c˘a E ⊂ A. Mult¸imea E este majorant E∈B

E∈B

pentru B? c) Dac˘a X = IN ¸si M ⊂ P(IN), M = {{7}, {1, 3}, {1, 3, 9}, A}, unde A = {0, 1, 2, 3, 4, 5, 6, 7, 8, 9}, determinat¸i: un minorant pentru M, un majorant pentru M, inf M, sup M, min M, max M, elementele minimale ¸si elementele maximale pentru M. 1.21 Pe mult¸imea F = {f |∅ 6= A ⊂ IR ¸si f : A → IR} se define¸ste relat¸ia R = {(f, g) | f, g ∈ F ¸si g este prelungire pentru f }. a) Verificat¸i dac˘a R este relat¸ie de ordine pe F. Este relat¸ia R total˘ a? b) Fie A = {f, g, h, t} unde: f : {1, 3} → IR,

f (x) = ½ −1, −1, g : IN → IR, g(x) = ½ 2x, −1, h : [0, ∞) → IR, h(x) =  2x,  x3 , −1, t : [−2, 4] → IR, t(x) =  2x,

x≤3 , x>3 x≤3 , x>3 −2 ≤ x < 0 0≤x≤3 3 < x ≤ 4.

22 Determinat¸i sup A ¸si inf A, dac˘a acestea exist˘a. 1.22 Pe IR × IR se define¸ste relat¸ia: (x, y) ≤ (z, t) ⇔ x < z sau (x = z ¸si y ≤ t). a) Ar˘atat¸i c˘a relat¸ia “≤” este relat¸ie de ordine total˘a pe IR × IR, numit˘ a ordinea lexicografic˘a. b) Introducet¸i simbolul corect ≤ sau ≥ ˆıntre: (2, 5) ¸si (1, 4) (3, 6) ¸si (3, 10) (6, 9) ¸si (7, 3). c) ˆIn caz de existent¸˘ a, determinat¸i max, min, sup, inf pentru urm˘atoarele mult¸imi: A = {(x, y) ∈ IR × IR|x2 + y 2 < 4}; B = {(x, y) ∈ IR × IR||x| + |y| < 2}; C = {(x, y) ∈ IR2 ||x| < 2 ¸si |y| < 2}. 1.23 Pe mult¸imea X = {(xn )n∈IN∗ |xn ∈ IR, ∀n ∈ IN∗ } se define¸ste relat¸ia: (xn )R(yn ) ⇔ xn = yn , ∀n ∈ IN∗ sau xk < yk unde k este primul indice pentru care xk 6= yk . Verificat¸i dac˘a relat¸ia R este relat¸ie de ordine pe X. Este relat¸ia R total˘a? 1.24 Fie α ∈ IR, α > 0 ¸si n ∈ IN∗ . S˘a se arate c˘a exist˘a un unic num˘ ar real pozitiv x astfel ˆıncˆ at xn = α. a pe [0, 1]}, a fixat 1.25 Fie C[0,1] = {f |f : [0, 1] → IR este funct¸ie continu˘ ˆın [0, 1] ¸si relat¸iile de la C[0,1] la IR definite astfel: S1 = {(f, f (a))|f ∈ C[0,1] }, S2 = {(f, f 0 (a)|f ∈ C[0,1] }, ¯ S3 = {(f, sup f (x)) ¯f ∈ C[0,1] }, x∈[0,1]

¯ S4 = {(f, inf f (x)) ¯f ∈ C[0,1] }, x∈[0,1]

Z S5 = {(f,

0

1

¯ f (x)dx) ¯f ∈ C[0,1] }.

Precizat¸i dac˘a acestea sunt funct¸ii ¸si, ˆın caz afirmativ, dac˘a sunt injective sau surjective.

23 Pentru rezolvarea acestei probleme recomand˘am citirea capitolelor 5,6 ¸si a manualelor de Analiz˘a matematic˘a pentru clasele a XI-a ¸si a XII-a. 1.26 Fie mult¸imile nevide X, ½ Y ¸si f ⊂ X × Y o relat¸ie de la X la Y . (i) f ◦ f −1 ⊂ 4(Y ) ¸si Ar˘atat¸i c˘a f este funct¸ie ⇔ (ii) f −1 ◦ f ⊃ 4(X). 1.27 Fie mult¸imile X, Y 6= ∅ ¸si funct¸ia f : X → Y . Ar˘atat¸i c˘a: ³T ´ T −1 f (Bi ); i) ∀(Bi )i∈I ⊂ P(Y ), f −1 Bi = i∈I

i∈I

ii) ∀B1 , B2 ∈ P(Y ), f −1 (B1 \ B2 ) = f −1 (B1 ) \ f −1 (B2 ); iii) ∀B ∈ P(Y ), f −1 (cB) = cf −1 (B); iv) ∀A ∈ P(X), A ⊂ f −1 (f (A)) (precizat¸i un exemplu pentru incluziunea strict˘a; pentru egalitate vezi problema 1.29-vi)); v) ∀B ∈ P(Y ), f (f −1 (B)) ⊂ B (dat¸i un exemplu pentru incluziunea strict˘a; pentru egalitate vezi problema 1.30-vi)); vi) ∀A ∈ P(X), f (cA) ⊃ f (X) \ f (A) (g˘asit¸i un exemplu pentru incluziunea strict˘a; pentru egalitate vezi problema 1.29-iv)). 1.28 Fie X, Y, T 6= ∅ ¸si funct¸iile f : X → Y , g : Y → T . S˘a se arate c˘a: a) dac˘a g ◦ f este injectiv˘a, atunci f este injectiv˘a; b) dac˘a g ◦ f este surjectiv˘a, atunci g este surjectiv˘a. 1.29 Fie X, Y 6= ∅ ¸si funct¸ia f : X → Y . Ar˘atat¸i c˘a urm˘atoarele afirmat¸ii sunt echivalente: i) ii) iii) iv) v) vi) vii) viii) ix) x)

f este injectiv˘a; f (A1 ∩ A2 ) = f (A1 ) ∩ f (A2 ), ∀A1 , A2 ∈ P(X); ∀A1 , A2 ∈ P(X), A1 ∩ A2 = ∅ ⇒ f (A1 ) ∩ f (A2 ) = ∅; f (cA) = f (X) \ f (A), ∀A ∈ P(X); ∀A1 , A2 ∈ P(X), f (A1 ) ⊂ f (A2 ) ⇒ A1 ⊂ A2 ; f −1 (f (A)) = A, ∀A ∈ P(X); exist˘a o surject¸ie g : Y → X astfel ˆıncˆ at g ◦ f = 1X ; −1 card f ({y}) ≤ 1 pentru orice y ∈ Y ; f −1 ◦ f = 4(X); ∀T 6= ∅ ¸si h, k : T → X, f ◦ h = f ◦ k ⇒ h = k.

1.30 Fie X ¸si Y mult¸imi nevide. Dac˘a f : X → Y este o funct¸ie oarecare, atunci urm˘atoarele afirmat¸ii sunt echivalente: i) f este surjectiv˘a;

24 ii) iii) iv) v) vi) vii) viii) ix) x)

f (X) = Y ; f −1 (B) 6= ∅ pentru orice B ∈ P(Y ), B 6= ∅; Y \ f (A) ⊂ f (cA), ∀A ∈ P(X); ∀B1 , B2 ∈ P(Y ), f −1 (B1 ) ⊂ f −1 (B2 ) ⇒ B1 ⊂ B2 ; f (f −1 (B)) = B, ∀B ∈ P(Y ); exist˘a o inject¸ie g : Y → X astfel ˆıncˆ at f ◦ g = 1Y ; −1 card f ({y}) ≥ 1 pentru orice y ∈ Y ; f ◦ f −1 = 4(Y ); ∀T 6= ∅ ¸si h, k : Y → T , h ◦ f = k ◦ f ⇒ h = k.

1.31 Fie X, Y 6= ∅. S˘a se arate c˘a funct¸ia f : X → Y este bijectiv˘a dac˘a ¸si numai dac˘a f este inversabil˘ a. 1.32 Fie X, Y 6= ∅. Pentru o funct¸ie arbitrar˘a f : X → Y fie funct¸iile f∗ : P(X) → P(Y ) ¸si f ∗ : P(Y ) → P(X) definite prin: f∗ (A) = f (A) pentru orice A ∈ P(X) ¸si f ∗ (B) = f −1 (B) pentru orice B ∈ P(Y ). Ar˘atat¸i c˘a: • urm˘atoarele afirmat¸ii sunt echivalente: a1 ) a2 ) a3 ) a4 )

f este injectiv˘a; f∗ este injectiv˘a; f ∗ ◦ f∗ = 1P(X) ; f ∗ este surjectiv˘a.

• urm˘atoarele afirmat¸ii sunt, de asemenea, echivalente: b1 ) b2 ) b3 ) b4 )

f este surjectiv˘a; f∗ este surjectiv˘a; f∗ ◦ f ∗ = 1P(Y ) ; f ∗ este injectiv˘a.

at f |Q s˘a 1.33 Dat¸i exemplu de funct¸ie neinjectiv˘a f : IR → IR astfel ˆıncˆ fie injectiv˘a. 1.34 S˘a se arate c˘a orice funct¸ie f : IR → IR se poate scrie ca suma a dou˘a funct¸ii surjective. 1.35 Fie X, Y 6= ∅ ¸si f : X → Y o funct¸ie arbitrar˘a. S˘a se arate c˘a: a) dac˘a f este injectiv˘a, atunci card X ≤ card Y ; b) dac˘a f este surjectiv˘a, atunci card Y ≤ card X; c) card(f (X)) ≤ card X.

25 1.36 Fie X o mult¸ime nevid˘a finit˘a ¸si funct¸ia f : X → X. Ar˘atat¸i c˘a urm˘atoarele afirmat¸ii sunt echivalente: a) f este injectiv˘a; b) f este surjectiv˘a; c) f este bijectiv˘a. Dac˘a X nu este finit˘a, atunci rezultatul nu se p˘astreaz˘ a. De exemplu, funct¸ia f : IN → IN, definit˘a prin f (n) = n + 3, este injectiv˘a dar nu este surjectiv˘a. 1.37 Dac˘a ∅ 6= A ⊂ IR este o mult¸ime finit˘a ¸si f : A → A este strict cresc˘atoare, atunci f este funct¸ia identic˘ a. 1.38 Fie ∅ 6= A ⊂ (0, +∞) o mult¸ime finit˘a. S˘a se determine funct¸iile f : A → A cu proprietatea: yf (x) = xf (y), pentru orice x, y ∈ A. 1.39 Fie ∅ 6= A ⊂ IR cu card A ≥ 3 ¸si funct¸iile f, g : A → (0, +∞). Ar˘atat¸i c˘a f = g dac˘a ¸si numai dac˘a f (x1 )f (x2 ) = g(x1 )g(x2 ), pentru orice x1 , x2 ∈ A, x1 6= x2 . a condit¸iile: 1.40 S˘a se g˘aseasc˘a funct¸iile f : IR → IR care verific˘ a) b) c) d)

f este injectiv˘a; f (x) = x, ∀x ∈ Q∗ \ {1}; f (x1 + x2 ) = f (x1 ) + f (x2 ), ∀x1 , x2 ∈ IR; f (x1 x2 ) = f (x1 )f (x2 ), ∀x1 , x2 ∈ IR.

1.41 Dac˘a funct¸ia f : IR → IR este monoton˘a ¸si periodic˘a, atunci ea este constant˘a. 1.42 S˘a se verifice dac˘a urm˘atoarele funct¸ii sunt periodice ¸si, ˆın caz afirmativ, dac˘a admit perioad˘a principal˘a: i) f : IR → IR, f (x) = c, unde c ∈ IR; ii) f : IR → IR, f (x) = ax ½ + b, unde a, b ∈ IR ¸si a 6= 0. 1, x ∈ Z iii) f : IR → IR, f (x) = ; 0, x ∈ IR \ Z ½ 0, x ∈ Q ; iv) f : IR → IR, f (x) = 1, x ∈ IR \ Q v) f : IR → IR, f (x) = [x] (partea ˆıntreag˘ a a lui x); vi) f : IR → IR, f (x) = x − [x] (partea fract¸ionar˘ a a lui x).

26 1.43 Fie f : IR → IR o funct¸ie periodic˘a ¸si T1 , T2 dou˘ a perioade ale lui f . ∗ Atunci T1 + T2 , −T1 , kT1 (k ∈ Z ) sunt, de asemenea, perioade pentru f . 1.44 Fie f : IR → IR o funct¸ie periodic˘a cu perioad˘a principal˘a T . Atunci singurele perioade ale lui f sunt de forma kT , k ∈ Z∗ . 1.45 Fie f : IR → IR o funct¸ie cu proprietatea c˘a orice num˘ ar irat¸ional este perioad˘a pentru f . Atunci f este constant˘ a. 1.46 Fie f1 , f2 : IR → IR funct¸ii periodice de perioade T1 ¸si respectiv T1 p T2 astfel ˆıncˆat = ∈ Q (p, q ∈ Z∗ ). Ar˘atat¸i c˘a funct¸iile αf1 + βf2 T2 q f1 (α, β ∈ IR), f1 f2 , (dac˘a f2 (x) 6= 0 pentru orice x ∈ IR) sunt periodice de f2 perioad˘a T = qT1 = pT2 . 1.47 ¾ ½ Aflat¸i¯ cardinalul2 mult¸imii ¯ 2n + 2 , n ∈ {1, 2, ..., 2004} (Examen Titularizare A = a ∈ IR ¯¯a = 2 3n + 2n + 1 Profesori - 1993). 1.48 Dac˘a B1 , B2 , ..., Bn sunt mult¸imi finite, s˘a se arate c˘a: card

n ³[

´ Bi

=

i=1

n X

card Bi −

i=1

+

X

X

card(Bi ∩ Bj )+

1≤i 3. Atunci u este majorant pentru A, iar dac˘a v este un majorant oarecare pentru A, se observ˘a c˘a u ≤ v, de unde rezult˘a c˘a u = sup A. 1.22. a) Reflexivitatea: pentru orice (x, y) ∈ IR × IR, (x, y) ≤ (x, y) deoarece x = x ¸si y ≤ y. Antisimetria: {(x, y) ≤ (z, t) ¸si (z, t) ≤ (x, y)} ⇔ {[x < z sau (x = z ¸si y ≤ t)] ¸si [z < x sau (z = x ¸si t ≤ y)]} ⇔ {x = z ¸si y = t} ⇔ (x, y) = (z, t). Tranzitivitatea se arat˘a la fel. Relat¸ia este total˘a pentru c˘a: oricare ar fi (x, y), (z, t) ∈ IR × IR, avem fie x < z, fie x = z, fie z < x. Dac˘a x < z, atunci (x, y) ≤ (z, t). Dac˘ a z < x, atunci (z, t) ≤ (x, y). Dac˘a x = z, putem avea y ≤ t sau t ≤ y. Dac˘ ax=z ¸si y ≤ t, atunci (x, y) ≤ (z, t) iar dac˘a x = z ¸si t ≤ y, atunci (z, t) ≤ (x, y). b) Avem: (2, 5) ≥ (1, 4), (3, 6) ≤ (3, 10) ¸si (6, 9) ≤ (7, 3). c) Fie (a, b) ∈ IR × IR, fixat. Dac˘a (x, y) ∈ IR × IR astfel ˆıncˆ at (a, b) ≤ (x, y), atunci a < x sau a = x ¸si b ≤ y. Reprezentˆ and grafic, (x, y) se afl˘a fie ˆın semiplanul drept determinat de dreapta x = a, fie pe semidreapta x = a deasupra punctului (a, b). ˆIn mod analog, punctele (x, y) din plan cu proprietatea (x, y) ≤ (a, b) se afl˘a fie ˆın semiplanul stˆang determinat de dreapta x = a, fie pe semidreapta x = a sub punctul (a, b). Ar˘at˘ am c˘a nu exist˘a max A. Reprezentˆ and grafic, A este mult¸imea punctelor din interiorul cercului de centru (0, 0) ¸si raz˘a 2, f˘ar˘ a punctele de pe cerc. Fie (a, b) ∈ A, ceea ce ˆınseamn˘a c˘a a2 + b2 < 4. Rezult˘a c˘a −2 < a < 2. Dac˘a ducem dreapta x = a, se constat˘a c˘a exist˘a puncte din A aflate ˆın semiplanul drept determinat de dreapta x = a (de exemplu, punctul de coordonate ( 2−a 2 , 0)). 2−a 2−a Deci exist˘a ( 2 , 0) ∈ A astfel ˆıncˆ at (a, b) ≤ ( 2 , 0), ceea ce arat˘a c˘a nu exist˘a max A. ˆIn acela¸si mod se demonstreaz˘a c˘a nu exist˘a nici min A. S˘a ar˘at˘am acum c˘a sup A = (2, 0). Mai ˆıntˆ ai observ˘am c˘a orice punct (x, y) ∈ A se afl˘a ˆın semiplanul stˆang determinat de dreapta x = 2. Cu alte cuvinte, avem (x, y) ≤ (2, 0), ∀(x, y) ∈ A. S˘a consider˘am (u, v) ∈ IR × IR astfel ˆıncˆ at (u, v) < (2, 0). Rezult˘a u < 2 sau (u = 2 ¸si v < 0). Dac˘a u < 2, atunci exist˘a ( 2−u ıncˆ at (u, v) < ( 2−u 2 , 0) ∈ A astfel ˆ 2 , 0). Dac˘a u = 2 ¸si v = 0, atunci exist˘a (0, 0) ∈ A astfel ˆıncˆ at (u, v) < (0, 0). Conform definit¸iei 1.20-h), sup A = (2, 0). Analog se arat˘a c˘a inf A = (−2, 0). Pentru mult¸imile B ¸si C se procedeaz˘a asem˘an˘ ator ¸si vom g˘asi c˘a nu exist˘a max ¸si min pentru B, C iar sup B = (2, 0), inf B = (−2, 0), sup C =

39 (2, 2), inf C = (−2, −2). 1.23. Reflexivitatea: evident (xn )R(xn ) pentru orice ¸sir (xn ) ∈ X. Antisimetria: fie (xn ), (yn ) ∈ X astfel ˆıncˆ at (xn )R(yn ) ¸si (yn )R(xn ). Din (xn )R(yn ) rezult˘a xn = yn , ∀n ∈ IN∗

(2) sau (3)

xk < yk si xi = yi , ∀i ∈ {1, 2, ..., k − 1}.

Dac˘a are loc (2), atunci (xn ) = (yn ). Presupunem c˘a are loc (3). Din (yn )R(xn ) rezult˘a (4)

yl < xl si yi = xi , ∀i ∈ {1, 2, ..., l − 1}.

Dac˘a l = k, atunci xk < yk ¸si yk < xk , absurd. Dac˘a l < k, atunci din (3) avem xl = yl ceea ce contrazice (4). Dac˘a l > k, atunci din (4) rezult˘a xk = yk , ˆın contradict¸ie cu (3). Pentru tranzitivitate, fie (xn )R(yn ) ¸si (yn )R(zn ). Atunci au loc (2) sau (3) ¸si (5)

yn = zn , ∀n ∈ IN∗

sau (6)

yl < zl ¸si yi = zi , ∀i ∈ {1, 2, ..., l − 1}.

Din (2) ¸si (5) rezult˘a xn = zn , ∀n ∈ IN∗ . Dac˘ a au loc (2) ¸si (6), atunci xl < zl ¸si xi = zi , ∀i ∈ {1, 2, ..., l − 1}. Dac˘a au loc (3) ¸si (5), atunci xk < zk ¸si xi = zi , ∀i ∈ {1, 2, ..., l − 1}. ˆIn sfˆar¸sit, din (3) ¸si (6) rezult˘a xp < zp ¸si xi = zi , ∀i ∈ {1, 2, ..., p − 1} unde p = min{k, l}. Relat¸ia este total˘a pentru c˘a relat¸ia de ordine pe IR este total˘a. ˆIntr-adev˘ar, fie (xn ), (yn ) ∈ X. Dac˘a xn = yn pentru orice n ∈ IN∗ , atunci (xn )R(yn ). Dac˘a nu, atunci exist˘a n ∈ IN∗ astfel ˆıncˆ at xn 6= yn ¸si fie k primul indice pentru care xk 6= yk . Atunci avem xk < yk sau yk < xk . Dac˘ a xk < yk , atunci (xn )R(yn ), iar din yk < xk rezult˘a (yn )R(xn ). ˆ 1.24. Fie E = {y > 0 | y n ≤ α}. Se observ˘a c˘a E este¡ nevid˘ ar, ¢ a. Intr-adev˘ α n α α dac˘a α ≥ 1, atunci 1 ∈ E. Dac˘a α < 1, atunci 2 < 1 ¸si 2 ≤ α deci 2 ∈ E. S˘a ar˘at˘am c˘a E este majorat˘a. Dac˘a α ≤ 1, atunci y n ≤ α ≤ 1 pentru orice y ∈ E. Rezult˘a y ≤ 1 ¸si astfel E este majorat˘a de 1. Dac˘a α > 1, atunci

40 y n ≤ α pentru orice y ∈ E. Dac˘ a y n < 1, atunci y < 1 < α. Dac˘a y n ≥ 1, atunci 1 ≤ y ≤ y n ≤ α deci α este un majorant pentru E. Conform axiomei de completitudine (II din definit¸ia 1.22), exist˘a x = sup E ∈ IR. Cum orice y din E este pozitiv, rezult˘a x > 0. Ar˘ at˘ am c˘a xn = α. Presupunem c˘a xn < α. Fie ε = α − xn > 0. Atunci, pentru 0 < a ≤ min{1, (1+x)εn −xn }, avem: (x + a)n = xn + Cn1 xn−1 a + Cn2 xn−2 a2 + ... + Cnn an = ¡ ¢ = xn + a Cn1 xn−1 + Cn2 xn−2 a + ... + Cnn an−1 ≤ ¡ ¢ ≤ xn + a Cn1 xn−1 + Cn2 xn−2 + ... + Cnn = = xn + a[(1 + x)n − xn ] ≤ xn + ε = α. Deci exist˘a x + a ∈ E astfel ˆıncˆ at x + a > x ceea ce contrazice faptul c˘a x = sup E. Dac˘a xn > α, se rat¸ioneaza la fel. Pentru unicitate, fie x1 , x2 ∈ (0, ∞) astfel ˆıncˆ at xn1 = xn2 = α. Dac˘ a x1 6= x2 , rezult˘a x1 < x2 sau n n x2 < x1 . Dac˘a x1 < x2 , atunci a = x1 < x2 = a ceea ce este absurd. Cazul x2 < x1 se trateaz˘a la fel. Drept consecint¸˘ a, x1 = x2 . 1.25. Pentru S1 : oricare ar fi f din C[0,1] , exist˘a ¸si este unic f (a) ∈ IR deci S1 este funct¸ie definit˘a pe C[0,1] ¸si cu valori in IR, definit˘a prin S1 (f ) = f (a). S1 nu este injectiv˘a deoarece exist˘a f, g ∈ C[0,1] , f 6= g astfel ˆıncˆ at f (a) = g(a). De exemplu, f, g ∈ C[0,1] , f (x) = 0 ¸si g(x) = x − a. Atunci S1 (f ) = f (a) = 0, S1 (g) = g(a) = 0 deci S1 (f ) = S1 (g) dar f 6= g. S1 este surjectiv˘a: ˆıntr-adev˘ ar, pentru orice b ∈ IR, exist˘ a funct¸ia constant˘ a f : [0, 1] → IR, f (x) = b astfel ˆıncˆ at S1 (f ) = f (a) = b. Pentru S2 : ¸stim c˘a exist˘a funct¸ii continue care nu sunt derivabile deci nu exist˘a f 0 (a) pentru orice f din C[0,1] ¸si prin urmare, S2 nu este funct¸ie. Pentru S3 : orice funct¸ie continu˘ a pe un compact este marginit˘a (conform teoremei lui Weierstrass 5.46). Astfel, pentru orice f din C[0,1] , exist˘a ¸si este unic sup f (x) ∈ IR deci S3 este funct¸ie de la C[0,1] la IR, definit˘a prin x∈[0,1]

S3 (f ) = sup f (x). S3 nu este injectiv˘a: fie f, g ∈ C[0,1] , f (x) = 1 ¸si g(x) = x∈[0,1]

x. Atunci f 6= g dar sup f (x) = sup g(x) = 1 ⇔ S3 (f ) = S3 (g). S3 este x∈[0,1]

x∈[0,1]

surjectiv˘a: pentru orice b ∈ IR, exist˘ a funct¸ia constant˘ a f ∈ C[0,1] , f (x) = b astfel ˆıncˆat S3 (f ) = sup f (x) = b. x∈[0,1]

Pentru S4 se procedeaz˘a ca la S3 . Pentru S5 : orice funct¸ie continu˘ a pe un interval compact [a, b] este integrabil˘a Riemann pe [a, b]. Astfel, pentru orice f din C[0,1] , exist˘ a ¸si

41 R1 este unic˘a integrala sa Riemann 0 f (x)dx ∈ IR deci S5 este funct¸ie de R1 la C[0,1] la IR, definit˘a prin S5 (f ) = 0 f (x)dx. S5 nu este injectiv˘a: fie R1 f, g ∈ C[0,1] , f (x) = 12 ¸si g(x) = 1 − x. Atunci S5 (f ) = 0 f (x)dx = 12 ¸si R1 R1 2 S5 (g) = 0 g(x)dx = 0 (1 − x)dx = x|10 − x2 |10 = 1 − 12 = 21 . Deci exist˘a f, g ∈ C[0,1] , f 6= g astfel ˆıncˆat S5 (f ) = S5 (g). S5 este surjectiv˘a: pentru orice b ∈ IR, exist˘a funct¸ia constant˘ a f ∈ C[0,1] , f (x) = b astfel ˆıncˆ at R1 S5 (f ) = 0 f (x)dx = b. 1.26. Presupunem c˘a f : X → Y este funct¸ie. Pentru a demonstra prima incluziune, fie (y1 , y2 ) ∈ f ◦ f −1 cu y1 , y2 ∈ Y. Conform definit¸iei 1.6, exist˘a x ∈ X astfel ˆıncˆat (y2 , x) ∈ f −1 ¸si (x, y1 ) ∈ f. Rezult˘a (x, y2 ) ∈ f ¸si (x, y1 ) ∈ f. Cum f este funct¸ie, avem y1 = y2 deci (y1 , y2 ) ∈ ∆(Y ). Pentru a doua incluziune, fie (x, x) ∈ ∆(X) cu x ∈ X. Notˆ and f (x) = y ∈ Y avem (x, y) ∈ f ¸si (y, x) ∈ f −1 . Rezult˘ a (x, x) ∈ f −1 ◦ f adic˘a (ii). Invers, presupunem ca au loc (i) ¸si (ii). Fie x ∈ X. Atunci (x, x) ∈ ∆(X) ¸si conform (ii), (x, x) ∈ f −1 ◦f. Rezult˘ a c˘a exist˘a y ∈ Y astfel ˆıncˆ at (x, y) ∈ f −1 ¸si (y, x) ∈ f . Deci pentru orice x ∈ X, exist˘a y ∈ Y astfel ˆıncˆ at (x, y) ∈ f ceea ce arat˘a c˘a Dom f = X. Fie acum x ∈ X ¸si y1 , y2 ∈ Y astfel ˆıncˆ at (x, y1 ) ∈ f ¸si (x, y2 ) ∈ f. Atunci ((x, y1 ) ∈ f ¸si (y2 , x) ∈ f −1 ) ⇒ ((y2 , y1 ) ∈ f ◦ f −1 ⊂ ∆(Y )) ⇒ y1 = y2 . Conform definit¸iei 1.7, f este funct¸ie. 1.27.

i) x ∈ f −1 (

T

Bi ) ⇔ f (x) ∈ i∈I T I, x ∈ f −1 (Bi )) ⇔ x ∈ f −1 (Bi ).

T i∈I

Bi ⇔ (∀ i ∈ I, f (x) ∈ Bi ) ⇔ (∀i ∈

i∈I

Analog se arat˘a ii), iii). iv) x ∈ A ⇒ f (x) ∈ f (A) ⇒ x ∈ f −1 (f (A)) deci A ⊂ f −1 (f (A)). Pentru incluziune strict˘a, fie f : IR → IR, f (x) = x2 ¸si A = [0, 1]. Atunci f (A) = [0, 1] ¸si f −1 (f (A)) = f −1 ([0, 1]) = [−1, 1]. Se vede c˘a A este strict inclus˘a ˆın f −1 (f (A)). v) Fie y ∈ f (f −1 (B)). Atunci exist˘a x ∈ f −1 (B) astfel ˆıncˆ at y = f (x). Din x ∈ f −1 (B), avem f (x) ∈ B deci y ∈ B ¸si astfel, f (f −1 (B)) ⊂ B. Pentru incluziune strict˘a, consider˘am f : IR → IR, f (x) = x2 ¸si B = [−1, 0]. Avem f −1 (B) = {0} ¸si f (f −1 (B)) = {0}. Deci mult¸imea f (f −1 (B)) este strict inclus˘a ˆın B. vi) Fie y ∈ f (X)\f (A). Rezult˘a y ∈ f (X) ¸si y ∈ / f (A). Din y ∈ f (X) rezult˘a c˘a exist˘a x ∈ X astfel ˆıncˆ at y = f (x). Dar y ∈ / f (A) ⇒ x ∈ / A ⇒ x ∈ cA. ˆIn consecint¸˘a, y ∈ f (cA). Pentru incluziune strict˘a, alegem aceea¸si funct¸ie ca mai sus, f : IR → IR, f (x) = x2 ¸si A = [0, 1]. Avem

42 f (IR)\f (A) = [0, ∞)\[0, 1] = (1, ∞) care este strict inclus˘a ˆın f (cA) = f ((−∞, 0) ∪ (1, ∞)) = IR∗ . 1.28. a) Fie x1 , x2 ∈ X astfel ˆıncˆ at f (x1 ) = f (x2 ). Rezult˘ a g(f (x1 )) = g(f (x2 )) care este echivalent cu (g ◦ f ) (x1 ) = (g ◦ f ) (x2 ) . Dar g ◦ f este injectiv˘a deci x1 = x2 . Prin urmare f este injectiv˘a. b) Fie z ∈ T. Cum g ◦ f este surjectiv˘a, exist˘a x ∈ X astfel ˆıncˆ at (g ◦ f ) (x) = z echivalent cu g(f (x)) = z. Notˆ and f (x) = y ∈ Y, avem g(y) = z. Astfel, g este surjectiv˘a. 1.29.

i)⇒ii): fie A1 , A2 ∈ P(X). Cum incluziunea: f (A1 ∩ A2 ) ⊂ f (A1 ) ∩ f (A2 )

are loc ˆıntotdeauna, s˘a ar˘at˘ am incluziunea invers˘ a. Fie y ∈ f (A1 ) ∩ f (A2 ). Rezult˘a c˘a exist˘a x1 ∈ A1 , x2 ∈ A2 astfel ˆıncˆ at y = f (x1 ) = f (x2 ). Din injectivitatea lui f rezult˘a x1 = x2 ∈ A1 ∩ A2 . Deci y ∈ f (A1 ∩ A2 ). ii) ⇒i): fie x1 , x2 ∈ X cu f (x1 ) = f (x2 ). Presupunˆand x1 6= x2 , se consider˘a A1 = {x1 }, A2 = {x2 }. Atunci f (A1 ∩ A2 ) = ∅, f (A1 ) ∩ f (A2 ) = {f (x1 )} ¸si din ii) rezult˘a ∅ = {f (x1 )} care este fals. i)⇒ iii) rezult˘a din ii). iii)⇒i): fie x1 , x2 ∈ X cu f (x1 ) = f (x2 ) ¸si s˘a presupunem c˘a x1 6= x2 . Atunci, pentru A1 = {x1 } ¸si A2 = {x2 } avem A1 ∩ A2 = ∅ ¸si din iii) rezult˘a f (A1 ) ∩ f (A2 ) = ∅ care este fals ˆıntrucˆ at f (x1 ) ∈ f (A1 ) ∩ f (A2 ). i)⇒iv): fie A ∈ P(X). Conform problemei 1.27-vi), este suficient s˘a ar˘at˘am doar incluziunea f (cA) ⊂ f (X)\f (A). Fie y ∈ f (cA). Atunci exist˘a x ∈ cA astfel ˆıncˆat y = f (x). Deci y ∈ f (X). Dac˘ a am avea y ∈ f (A), atunci ar exista x0 ∈ A astfel ˆıncˆ at y = f (x0 ). Deci f (x) = f (x0 ) ¸si din injectivitatea lui f rezult˘a x = x0 , ceea ce contrazice faptul c˘a x ∈ cA ¸si x0 ∈ A. iv)⇒i): fie x1 , x2 ∈ X cu f (x1 ) = f (x2 ) = y ∈ f (X). Presupunem x1 6= x2 ¸si consider˘am A = {x1 }. Atunci x2 ∈ cA de unde y ∈ f (cA). Dar iv) implic˘a (7)

y ∈ f (X)\f (A).

ˆIn acela¸si timp, y = f (x1 ) ∈ f (A), ceea ce contrazice (7). i)⇒v): fie A1 , A2 ∈ P(X) astfel ˆıncˆ at f (A1 ) ⊂ f (A2 ). Avem: x ∈ A1 ⇒ y = f (x) ∈ f (A1 ) ⇒ y ∈ f (A2 ) ⇒ ∃ x0 ∈ A2 a.ˆı. y = f (x0 ). Atunci f (x) = f (x0 ) ¸si pentru c˘a f este injectiv˘a, se obt¸ine x = x0 ∈ A2 . ˆIn concluzie, A1 ⊂ A2 .

43 v)⇒i): fie x1 , x2 ∈ X cu f (x1 ) = f (x2 ). Presupunem x1 6= x2 ¸si consider˘am A1 = {x1 },A2 = {x2 }. Atunci f (A1 ) = f (A2 ) ¸si folosind v) obt¸inem A1 ⊂ A2 care este fals. i)⇒vi): fie A ∈ P(X). Din problema 1.27-iv) avem A ⊂ f −1 (f (A)). S˘ a −1 demonstr˘am incluziunea invers˘a: x ∈ f (f (A)) ⇒ y = f (x) ∈ f (A) ⇒ ∃ x0 ∈ A a.ˆı. y = f (x0 ). Astfel f (x) = f (x0 ) ¸si cum f este injectiv˘a, rezult˘a x = x0 ∈ A. vi)⇒i): fie x1 , x2 ∈ X cu f (x1 ) = f (x2 ). Dac˘a admitem c˘a x1 6= x2 , atunci considerˆand A = {x1 }, rezult˘ a f (x2 ) = f (x1 ) ∈ f (A). De aici obt¸inem −1 c˘a x2 ∈ f (f (A)) = A = {x1 }, ceea ce contrazice presupunerea f˘acut˘ a. i)⇒vii): fie funct ¸ ia g : Y → X, definit˘ a prin: ½ x, f (x) = y (unde x ∈ X este unic determinat din f injectiv˘ a) g(y) = x0 , y ∈ / f (X) (unde x0 ∈ Xeste fixat). Se constat˘a c˘a g este bine definit˘a, surjectiv˘a ¸si (g ◦ f )(x) = g(f (x)) = g(y) = x, ∀ x ∈ X. vii)⇒i) rezult˘a din problema 1.28 -a). i)⇒viii): fie y ∈ Y ¸si presupunem card f −1 ({y}) ≥ 2. Atunci exist˘a x1 , x2 ∈ f −1 ({y}), x1 6= x2 . Rezult˘a f (x1 ) = f (x2 ) = y ¸si pentru c˘a f este injectiv˘a, are loc x1 = x2 deci contradict¸ie. viii)⇒i): fie x1 , x2 ∈ X astfel ˆıncˆ at f (x1 ) = f (x2 ) = y. Atunci x1 , x2 ∈ f −1 ({y}) ¸si din viii) rezult˘a x1 = x2 deci f este injectiv˘a. i)⇒ix): folosind problema 1.26-(ii), va fi suficient s˘a demonstr˘am doar incluziunea: (8)

f −1 ◦ f ⊂ ∆(X).

Fie x1 , x2 ∈ X astfel ˆıncˆat (x1 , x2 ) ∈ f −1 ◦ f. Rezult˘ a c˘a exist˘a y ∈ Y astfel ˆıncˆat (x1 , y) ∈ f ¸si (y, x2 ) ∈ f −1 . Rezult˘a (x1 , y) ∈ f ¸si (x2 , y) ∈ f, adic˘ a f (x1 ) = y = f (x2 ). Cum f este injectiv˘a, rezult˘a x1 = x2 , de unde se obt¸ine (x1 , x2 ) ∈ ∆(X). ix)⇒i): fie x1 , x2 ∈ X astfel ˆıncˆ at f (x1 ) = f (x2 ) = y ⇒ (x1 , y) ∈ f ¸si (x2 , y) ∈ f ⇒ (x1 , y) ∈ f ¸si (y, x2 ) ∈ f −1 ⇒ (x1 , x2 ) ∈ f −1 ◦ f. Din ix) rezult˘a (x1 , x2 ) ∈ ∆(X). Astfel, x1 = x2 deci f este injectiv˘a. i)⇒x): fie T 6= ∅ ¸si h, k : T → X astfel ˆıncˆ at f ◦ h = f ◦ k. Rezult˘a f (h(z)) = f (k(z)), ∀ z ∈ T. Din faptul c˘a f este injectiv˘a se obt¸ine h(z) = k(z), ∀ z ∈ T adic˘a h = k. x)⇒i): fie x1 , x2 ∈ X astfel ˆıncˆ at f (x1 ) = f (x2 ). Considerˆ and T = X ¸si funct¸iile h, k : X → X, h(x) = x1 , k(x) = x2 , avem pentru orice x ∈ X : (f ◦ h)(x) = f (h(x)) = f (x1 ), (f ◦ k)(x) = f (k(x)) = f (x2 ) deci f ◦ h = f ◦ k. Din x) rezult˘a h = k. A¸sadar, x1 = x2 ¸si f este injectiv˘a.

44 1.30. i)⇔ii) reiese imediat din definit¸ia 1.9-b) ¸si observat¸ia 1.8-a). i)⇒iii): fie B ∈ P(Y ), B 6= ∅ ⇒ ∃ y ∈ B. Avem: f surjectiv˘a ⇒ ∃ x ∈ X a.ˆı. f (x) = y ∈ B ⇒ x ∈ f −1 (B) deci f −1 (B) 6= ∅. iii)⇒i): fie y ∈ Y. Dac˘a not˘am B = {y} = 6 ∅, atunci din iii) avem −1 −1 f ({y}) 6= ∅ ⇒ ∃x ∈ f ({y}), adic˘a f (x) = y. Prin urmare, f este surjectiv˘a. i)⇒iv): presupunem c˘a f este surjectiv˘a, deci f (X) = Y. Fie A ∈ P(X). Dac˘a A = ∅, atunci Y \f (∅) = Y = f (X) = f (c∅). Fie atunci A 6= ∅. Dac˘a Y \f (A) = ∅, atunci evident Y \f (A) ⊂ f (cA). Dac˘ a Y \f (A) 6= ∅, fie y ∈ Y \f (A). Rezult˘a y ∈ Y ¸si y ∈ / f (A). Cum f este surjectiv˘a, exist˘a x ∈ X astfel ˆıncˆat f (x) = y. Din y ∈ / f (A), rezult˘a x ∈ cA, deci y = f (x) ∈ f (cA). iv)⇒i): luˆand A = ∅, din iv) rezult˘a Y \f (∅) ⊂ f (c∅) = f (X) ⇒ Y ⊂ f (X) adic˘a f este surjectiv˘a. i)⇒v): fie B1 , B2 ∈ P(Y ) astfel ˆıncˆ at f −1 (B1 ) ⊂ f −1 (B2 ). Dac˘ a B1 = ∅, atunci evident B1 ⊂ B2 . Dac˘ a B1 6= ∅, fie y ∈ B1 . Dar f surjectiv˘ a ⇒ ∃x ∈ X astfel ˆıncˆat f (x) = y ∈ B1 ⇒ x ∈ f −1 (B1 ) ⇒ x ∈ f −1 (B2 ) ⇒ f (x) = y ∈ B2 ¸si astfel, B1 ⊂ B2 . v)⇒i): presupunem c˘a f nu este surjectiv˘a. Atunci exist˘a y ∈ Y astfel ˆıncˆat y ∈ / f (X) ⇒ f −1 (Y ) = f −1 (Y \{y}) = X. Din iv) rezult˘a Y ⊂ Y \{y}, ceea ce este fals. i)⇒vi): utilizˆand problema 1.27-v), vom demonstra doar incluziunea: (9)

B ⊂ f (f −1 (B)).

Pentru aceasta, fie y ∈ B. Cum f este surjectiv˘a, exist˘a x ∈ X astfel ˆıncˆ at −1 −1 f (x) = y ∈ B ⇒ x ∈ f (B) ⇒ f (x) = y ∈ f (f (B)) ¸si are loc (9). vi)⇒i): fie y ∈ Y. Conform v), Y = f (f −1 (Y )) ⇒ y ∈ f (f −1 (Y )) ⇒ ∃ x ∈ f −1 (Y ) ⊂ X astfel ˆıncˆ at f (x) = y ⇒ f este surjectiv˘a. i)⇒vii): deoarece f este surjectiv˘a, pentru orice y ∈ Y, exist˘ a xy ∈ X astfel ˆıncˆat f (xy ) = y. Definim funct¸ia g : Y → X prin g(y) = xy . Atunci g este bine definit˘a, injectiv˘a ¸si (f ◦ g)(y) = f (g(y)) = f (xy ) = y pentru orice y ∈ Y. vii)⇒i) rezult˘a din problema 1.28-b). i)⇒viii): fie y ∈ Y. f fiind surjectiv˘a, exist˘a x ∈ X astfel ˆıncˆ at f (x) = y. Rezult˘a x ∈ f −1 ({y}) deci card(f −1 (y)) ≥ 1. viii)⇒i): fie y ∈ Y. Coform viii), exist˘a x ∈ f −1 ({y}) ⇒ ∃ x ∈ X a.ˆı. f (x) = y ⇒ f este surjectiv˘a. i)⇒ix): folosind problema 1.26-(i), trebuie s˘a ar˘at˘ am doar incluziunea invers˘a: (10)

∆(Y ) ⊂ f ◦ f −1 .

45 Fie (y, y) ∈ ∆(Y ), unde y ∈ Y . Cum f este surjectiv˘a, exist˘a x ∈ X astfel ˆıncˆat f (x) = y. Rezult˘a (x, y) ∈ f ¸si (y, x) ∈ f −1 . Atunci (y, y) ∈ f ◦ f −1 ¸si are loc (10). ix)⇒i): fie y ∈ Y ⇒ (y, y) ∈ ∆(Y ) = f ◦ f −1 ⇒ ∃ x ∈ X astfel ˆıncˆ at −1 (y, x) ∈ f ¸si (x, y) ∈ f ⇒ ∃ x ∈ X astfel ˆıncˆ at f (x) = y ⇒ f este surjectiv˘a. i)⇒x): fie T 6= ∅ ¸si funct¸iile h, k : Y → T astfel ˆıncˆ at h ◦ f = k ◦ f. Fie y ∈ Y. ˆIntrucˆat f este surjectiv˘a, exist˘a x ∈ X astfel ˆıncˆ at f (x) = y. Atunci avem: h(y) = h(f (x)) =(h ◦ f )(x) = (k ◦ f )(x) = k(f (x)) = k(y) ⇒ h = k. x)⇒i): dac˘a avem card Y = 1, atunci evident f este surjectiv˘a. Fie acum card Y ≥ 2 ¸si presupunem, prin reducere la absurd, c˘a exist˘a y0 ∈ Y astfel ˆıncˆat f (x) 6= y0 pentru orice x ∈½X. Fie Z = X ¸si funct¸iile h, k : Y → y, y 6= y0 Y definite astfel: h(y) = y, k(y) = y1 , y = y0 (unde y1 ∈ Y, y1 6= y0 ). Atunci (h◦f )(x) = h(f (x)) = f (x), iar (k ◦f )(x) = k(f (x)) = f (x) deoarece f (x) 6= y0 . Deci h ◦ f = k ◦ f ¸si din x) rezult˘a h = k ⇒ h(y0 ) = k(y0 ) ⇒ y0 = y1 , contradict¸ie! 1.31. Presupunem ˆıntˆai c˘a f este bijectiv˘a, adic˘a injectiv˘a ¸si surjectiv˘a. Conform problemelor 1.29-vii) ¸si 1.30-vii), exist˘a g1 , g2 : Y → X, g1 surjectiv˘a ¸si g2 injectiv˘a astfel ˆıncˆat g1 ◦ f = 1X ¸si f ◦ g2 = 1Y . Atunci avem: g1 = g1 ◦ 1Y = g1 ◦ (f ◦ g2 ) = (g1 ◦ f ) ◦ g2 = 1X ◦ g2 = g2 . Deci exist˘a g1 = g2 = g : Y → X bijectiv˘a astfel ˆıncˆ at g ◦ f = 1X ¸si f ◦ g = 1Y , adic˘ af este inversabil˘a. Implicat¸ia invers˘a rezult˘a imediat din problema 1.28. 1.32. a1 ) ⇒ a2 ): presupunem f injectiv˘ a ¸si fie A1 , A2 ∈ P(X) cu f∗ (A1 ) = f∗ (A2 ) ⇒ f (A1 ) = f (A2 ). Conform problemei 1.29-v), A1 = A2 ¸si f∗ este injectiv˘a. a2 ⇒ a3 ): presupunem f∗ injectiv˘a ¸si fie A ∈ P(X). Avem: (f ∗ ◦f∗ )(A) = ∗ f (f∗ (A)) = f ∗ (f (A)) = f −1 (f (A)). S˘ a ar˘at˘ am c˘a f −1 (f (A)) ⊂ A. Fie −1 0 x ∈ f (f (A)) ⇒ f (x) ∈ f (A) ⇒ ∃ x ∈ A a.ˆı. f (x) = f (x0 ) ⇒ f ({x}) = f ({x0 }). Dar f∗ este injectiv˘a ⇒ {x} = {x0 } ¸si deci x ∈ A. A¸sadar, f −1 (f (A)) ⊂ A. Cum incluziunea invers˘ a are loc ˆıntotdeauna (vezi ∗ problema 1.27-iv), rezult˘a (f ◦ f∗ )(A) = A, ∀ A ∈ P(X). a3 ) ⇒a4 ) rezult˘a din problema 1.28-b). a4 ) ⇒a1 ): presupunem f ∗ surjectiv˘ a. Fie A1 , A2 ∈ P(X). Vom ar˘ata (11)

f (A1 ) ∩ f (A2 ) ⊂ f (A1 ∩ A2 ).

Cum f ∗ este surjectiv˘a, pentru A1 ,A2 , exist˘a B1 , B2 ∈ P(Y ) astfel ˆıncˆ at f ∗ (B1 ) = A1 , f ∗ (B2 ) = A2 , adic˘ a f −1 (B1 ) = A1 , f −1 (B2 ) = A2 . Fie y ∈

46 f (A1 ) ∩ f (A2 ) ⇒ ∃ x1 ∈ A1 , x2 ∈ A2 a.ˆı. y = f (x1 ) = f (x2 ). Atunci avem: y ∈ f (A1 ) = f (f −1 (B1 )) ⊂ B1 , y ∈ f (A2 ) = f (f −1 (B2 )) ⊂ B2 ⇒ y ∈ B1 ∩ B2 ⇒ x1 ∈ f −1 (B1 ∩ B2 ) = f −1 (B1 ) ∩ f −1 (B2 ) = A1 ∩ A2 ⇒ y = f (x1 ) ∈ f (A1 ∩ A2 ). Deci are loc (11). Cum incluziunea invers˘ a are loc (vezi problema 1.14-l)), rezult˘a f (A1 ∩ A2 ) = f (A1 ) ∩ f (A2 ). Din problema 1.29-ii), se obt¸ine faptul c˘a f este injectiv˘a. ˆIn acela¸si mod se arat˘a ¸si echivalent¸a afirmat¸iilor b1 , b2 , b3 , b4 . ½ x, x ∈ Q 1.33. Fie f (x) = . Atunci f nu este injectiv˘a pentru c˘a x2 , x ∈ IR\Q √ √ √ √ √ √ exist˘a 3, − 3 ∈ IR, 3 6= − 3 astfel ˆıncˆ at f ( 3) = f (− 3) = 3. Dar restrict¸ia f |Q (x) = x, ∀ x ∈ Q, este evident injectiv˘a. 1.34. Fie f : IR → IR. Cu ajutorul funct¸iei u : (0, +∞) → IR, u(x) = ln x, se definesc aplicat¸iile g, h : IR → IR prin:   ln x, x > 0   f (x) − ln x, x > 0 f (0), x = 0 , h(x) = 0, x = 0 . g(x) =   f (x) − ln(−x), x < 0 ln(−x), x < 0 Evident g + h = f. ˆIn plus, avem IR = u((0, ∞)) = g((0, ∞)) ⊂ g(IR) ⊂ IR ¸si IR = u((0, ∞)) = h((−∞, 0)) ⊂ h(IR) ⊂ IR. Rezult˘ a g(IR) = IR ¸si h(IR) = IR deci g ¸si h sunt surjective. 1.35. a) Presupunem f injectiv˘ a. Atunci funct¸ia f : X → f (X) este bijectiv˘a, de unde (12)

card X = card(f (X)).

Dar f (X) ⊂ Y. Conform propozit¸iei 1.46, (13)

card(f (X)) ≤ card Y.

Din (12) ¸si (13) rezult˘a card X ≤ card Y. b) Fie f surjectiv˘a. Conform problemei 1.30-vii), exist˘a g : Y → X injectiv˘a astfel ˆıncˆat f ◦ g = 1Y . Aplicˆ and a), rezult˘a card Y ≤ card X. c) Dac˘a f este surjectiv˘a, atunci f (X) = Y deci card(f (X)) = card Y. Din b) avem card Y ≤ card X ¸si de aici rezult˘a card(f (X)) ≤ card X. 1.36 Fie X = {x1 , x2 , . . . , xn } ¸si f (X) = {y1 , y2 , . . . , ym }. a)⇒b) Presupunem c˘a f este injectiv˘a. Atunci funct¸ia f : X → f (X) este bijectiv˘a. Rezult˘a card X = card(f (X)), adic˘a n = m. Cum f (X) ⊂ X, rezult˘a f (X) = X deci f este surjectiv˘a.

47 b)⇒c). Fie f surjectiv˘a. Conform problemei 1.30-vii), exist˘a o funct¸ie injectiv˘a g : X → X astfel ˆıncˆ at f ◦ g = 1X . Cum X este finit˘a ¸si g injectiv˘a, rezult˘a de mai sus c˘a g este surjectiv˘a deci ¸si bijectiv˘a. Atunci g◦f = g◦f ◦1X = g◦f ◦(g◦g −1 ) = g◦(f ◦g)◦g −1 = g◦1X ◦g −1 = g◦g −1 = 1X . Din problema 1.31 rezult˘a c˘a f este bijectiv˘a. Implicat¸ia “c)⇒a)” este evident˘ a. 1.37 Fie A = {a1 , a2 , . . . , an }, n ∈ IN∗ , astfel ˆıncˆ at a1 < a2 < . . . < an . Presupunem, prin reducere la absurd c˘a exist˘a i ∈ {1, 2, . . . , n} astfel ˆıncˆ at f (ai ) 6= ai . I) Dac˘a f (ai ) > ai , atunci i < n. ˆIn caz contrar, am avea f (an ) > an ceea ce este absurd, ˆıntrucˆat f (an ) ∈ A. Din inegalit˘a¸tile ai < ai+1 < . . . < an rezult˘a c˘a f (ai ) < f (ai+1 ) < . . . < f (an ). Dar f (ai ) > ai ⇒ f (ai ) ≥ ai+1 ⇒ f (ai+1 ) > ai+1 . Dup˘a n − i pa¸si va rezulta c˘a f (an ) > an , ceea ce este absurd c˘aci f (an ) ∈ A. II) Dac˘a f (ai ) < ai , atunci i > 1. ˆIn caz contrar, am avea f (a1 ) < a1 ceea ce contrazice faptul c˘a f (a1 ) ∈ A. Din inegalit˘a¸tile a1 < a2 < . . . < ai rezult˘a c˘a f (a1 ) < f (a2 ) < . . . < f (ai ). Avem f (ai ) < ai ⇒ f (ai ) ≤ ai−1 ⇒ f (ai−1 ) < ai−1 . Dup˘a i pa¸si va rezulta f (a1 ) < a1 , ceea ce este absurd deoarece f (a1 ) ∈ A. f (x) f (y) = , rezult˘a x y f (x) > f (y). Rezult˘a c˘a f este strict cresc˘atoare ¸si din problema 1.37 obt¸inem f = 1A . Solut¸ia a 2-a. f (y) Din ipotez˘a rezult˘a c˘a f (x) = k > 0 (constant). Deci f (x) = x = y kx, ∀x ∈ A. Fie a ∈ A fixat, α = min A ¸si β = max A. Dac˘a k < 1, atunci f (α) = kα < α, absurd. Dac˘a k > 1, atunci f (β) = kβ > β, absurd. Rezult˘a k = 1 deci f = 1A . 1.38

Solut¸ia 1. Fie x, y ∈ A cu x > y. Cum

1.39 Solut¸ia 1. Implicat¸ia direct˘a “⇒” este evident˘ a. Pentru implicat¸ia invers˘a “⇐”, fie x, a1 , a2 ∈ A, x 6= a1 6= a2 6= x. Atunci f (x)f (a1 ) = g(x)g(a1 ), f (a1 )f (a2 ) = g(a1 )g(a2 ), f (a2 )f (x) = g(a2 )g(x). ˆInmult¸ind cele trei relat¸ii se obt¸ine: f 2 (x)f 2 (a1 )f 2 (a2 ) = g 2 (x)g 2 (a1 )g 2 (a2 ) ⇔ f 2 (x) = g 2 (x) ⇔ f (x) = g(x).

48 Cum x este arbitrar ˆın A, rezult˘a c˘a f = g. Solut¸ia a 2-a. Fie h = fg . Atunci h(x1 )h(x2 ) = 1, ∀x1 , x2 ∈ A cu x1 6= x2 . Fie a ∈ A 1 fixat. Atunci h(x) = h(a) pentru orice x ∈ A\{a}. 1 Fie x, y ∈ A\{a} cu x 6= y. Atunci h(x) = h(y) = h(a) . Obt¸inem 1 2 ( h(a) ) = 1 ¸si cum h : A → (0, +∞), rezult˘a h(a) = 1, ∀x ∈ A adic˘a f = g. 1.40 Pentru x1 = x2 = 0, din c) obt¸inem f (0) = 0. Dac˘a x1 = x ∈ IR ¸si x2 = −x, tot din c) rezult˘a: 0 = f (0) = f (x) + f (−x) ⇒ f (−x) = −f (x) pentru orice x ∈ IR deci f este impar˘a. Luˆand ˆın d) x1 = x2 = 1 se obt¸ine f (1) = f 2 (1), de unde rezult˘a c˘a f (1) = 0 sau f (1) = 1. Cum f (0) = 0 ¸si f este injectiv˘a, rezult˘a c˘a f (1) = 1. Fie x arbitrar, x > 0. Conform d), √ √ √ f (x) = f ( x · x) = f 2 ( x) ≥ 0 ¸si din injectivitate rezult˘a c˘a f (x) > 0. Deci f (x) > 0 pentru orice x > 0. S˘a ar˘at˘ am c˘a f este strict cresc˘atoare. Fie x1 , x2 ∈ IR cu x1 < x2 . Atunci exist˘a a > 0 astfel ˆıncˆ at x2 = x1 + a ¸si din c) rezult˘a: f (x2 ) = f (x1 + a) = f (x1 ) + f (a) > f (x1 ). Astfel f este strict cresc˘atoare. Ar˘at˘ am c˘a f = 1IR . Presupunem, prin reducere la absurd, c˘a exist˘a x0 ∈ IR astfel ˆıncˆ at f (x0 ) 6= x0 . Avem dou˘a situat¸ii: I) Dac˘a f (x0 ) < x0 , atunci exist˘a r ∈ Q astfel ˆıncˆ at f (x0 ) < r < x0 . Dar f este strict cresc˘atoare deci r = f (r) < f (x0 ), contradict¸ie! II) Dac˘a f (x0 ) > x0 , se rat¸ioneaz˘ a analog. ˆIn concluzie, f = 1IR . 1.41 f fiind periodic˘a, exist˘a T > 0 astfel ˆıncˆ at f (x + T ) = f (x), ∀x ∈ IR. Va fi suficient de ar˘atat c˘a f este constant˘ a pe [0, T ]. S˘a presupunem c˘a f este cresc˘atoare. Atunci pentru orice x ∈ [0, T ], f (0) ≤ f (x) ≤ f (T ). Dar f (T ) = f (0) deci f (x) = f (0) pentru orice x ∈ [0, T ]. Astfel f este constant˘ a pe [0, T ]. 1.42 i) Se observ˘a c˘a pentru orice T ∈ IR∗ , avem f (x+T ) = f (x), ∀x ∈ IR. Rezult˘a c˘a f este periodic˘a ¸si orice T ∈ IR∗ este perioad˘a pentru f dar f nu admite perioad˘a principal˘a. ii) Ar˘at˘am c˘a f nu este periodic˘a. Solut¸ia 1. Presupunem, prin reducere la absurd, c˘a exist˘a T ∈ IR∗ astfel ˆıncˆat f (x + T ) = f (x), ∀x ∈ IR. Rezult˘a a(x + T ) + b = ax + b, ∀x ∈ IR. Deci aT = 0, absurd c˘aci a 6= 0 ¸si T 6= 0.

49 Solut¸ia a 2-a. Rezult˘a direct, folosind problema 1.41. iii) Fie T ∈ Z∗ . Dac˘a x ∈ Z, atunci x + T ∈ Z ¸si avem f (x + T ) = f (x) = 1. Dac˘a x ∈ IR\Z, atunci x + T ∈ IR\Z ¸si f (x + T ) = f (x) = 0. Fie acum T ∈ IR\Z. Pentru x ∈ Z avem f (x) = 1 6= 0 = f (x + T ). Deci mult¸imea perioadelor pentru f este Z∗ . Perioada principal˘a este 1. iv) f este periodic˘a avˆand ca perioad˘a orice T ∈ Q∗ dar f nu are perioad˘a principal˘a. v) Ar˘at˘am c˘a f nu este periodic˘a. Solut¸ia 1. Presupunem, prin reducere la absurd, c˘a exist˘a T ∈ IR∗ astfel ˆıncˆat f (x + T ) = f (x) pentru orice x ∈ IR. Deci are loc (14)

[x + T ] = [x], ∀x ∈ IR.

Pentru x = 0, din (14) rezult˘a [T ] = 0 de unde T ∈ (0, 1). Considerˆand ˆın (14) x = 1 − T , se obt¸ine (15)

1 = [1 − T ].

Deoarece T ∈ (0, 1) rezult˘a c˘a 1 − T ∈ (0, 1) ¸si deci [1 − T ] = 0 ceea ce contrazice (15). Solut¸ia 2. Rezult˘a direct, conform problemei 1.41. vi) Fie T ∈ IR∗ astfel ˆıncˆat f (x + T ) = f (x), ∀x ∈ IR ⇔ ({x + T } = {x}, ∀x ∈ IR) ⇔ (x + T − [x + T ] = x − [x], ∀x ∈ IR) ⇔ (16)

[x + T ] = [x] + T, ∀x ∈ IR.

Luˆand ˆın (16) x = 0, se obt¸ine [T ] = T deci T ∈ Z∗ . Fie acum T ∈ Z∗ . Rezult˘a {x + T } = {x}, ∀x ∈ IR ⇔ f (x + T ) = f (x), ∀x ∈ IR. ˆIn consecint¸˘a, f este periodic˘a de perioad˘a T ∈ Z∗ , iar perioada principal˘a este 1. 1.43

Pentru x arbitrar din IR au loc relat¸iile: f (x + (T1 + T2 )) = f ((x + T1 ) + T2 ) = f (x + T1 ) = f (x) ¸si f (x − T1 ) = f ((x − T1 ) + T1 ) = f (x)

ceea ce arat˘a c˘a (17)

T1 + T2 ¸si − T1 sunt perioade pentru f.

Pentru nT1 (n ∈ IN∗ ) se arat˘a prin induct¸ie. Pentru k = 2, 2T1 = T1 + T1 care este perioad˘a pentru f , conform (17). Presupunem ca nT1 este perioad˘a

50 pentru f ¸si s˘a demonstr˘am c˘a (n + 1)T este perioad˘a pentru f . Pentru orice x ∈ IR avem: f (x + (n + 1)T1 ) = f (x + nT1 + T1 ) = f (x + nT1 ) = f (x) deci nT1 este perioad˘a, ∀n ∈ IN∗ . Din (17) obt¸inem apoi c˘a −nT1 este de asemenea perioad˘a ¸si ˆın concluzie, kT1 este perioad˘a a funct¸iei f pentru orice k ∈ Z∗ . 1.44 Conform problemei 1.43, kT este perioad˘a pentru f, ∀k ∈ Z∗ . Presupunem, prin reducere la absurd, c˘a f admite o perioad˘a T 0 > 0 care nu este de forma kT cu k ∈ Z∗ . Atunci exist˘a p ∈ IN∗ ¸si r ∈ (0, T ) astfel ˆıncˆ at T 0 = pT + r. Atunci avem: f (x + T 0 ) = f (x), ∀x ∈ IR ⇒ f (x) = f (x + T 0 ) = = f (x + pT + r) = f (x + r), ∀x ∈ IR. Ar rezulta c˘a r ∈ (0, T ) este perioad˘a pentru f ceea ce contrazice faptul c˘a T este perioada principal˘a a lui f . 1.45 Fie b = f (a) unde a este fixat ˆın IR\Q. Fie x arbitrar ˆın IR. Dac˘a x ∈ IR\Q, avem f (x) = f (x + a) = f (a) = b deoarece x ¸si a sunt perioade pentru f . Dac˘a x ∈ Q, atunci f (x) = f (x + a) = b pentru c˘a x + a ∈ IR\Q. Deci f este constant˘a, f = b. 1.46 (αf1 + βf2 )(x + T ) = αf1 (x + qT1 ) + βf2 (x + pT2 ) = αf1 (x) + βf2 (x), ∀x ∈ IR deci T este perioad˘a pentru αf1 + βf2 . Analog se arat˘a T1 f1 . Dac˘a ∈ / Q, atunci concluzia nu este ¸si pentru funct¸iile f1 f2 ¸si f2 T2 ˆıntotdeauna √adev˘arat˘ a. De exemplu, √ f (x) = cos x are perioada T1 = 2π, g(x) = cos x 2 are perioada T2 = π 2 dar f + g nu este periodic˘a. 1.47

Mult¸imea A are cel mult 2004 elemente. Dou˘a elemente din A sunt 2n2 + 2 2m2 + 2 egale dac˘a = , n, m ∈ {1, 2, . . . , 2004} ¸si pre3n2 + 2n + 1 3m2 + 2m + 1 supunem n < m. Rezult˘a (n − m)(nm − m − n − 1) = 0. Dar n 6= m ⇒ nm − m+1 2 m − n − 1 = 0 ⇒ n(m − 1) = m + 1. Cum m 6= 1 ⇒ n = = 1+ . m−1 m−1 Se observ˘a c˘a n ∈ IN ⇔ (m − 1) este divizor al lui 2. Se obt¸ine m = 3 ¸si n = 2. ˆIn concluzie, card A = 2003. 1.48

Se demonstreaz˘a prin induct¸ie.

51 Pentru n = 2 : B1 ∩ (B2 \B1 ) = ∅ ¸si B1 ∪ (B2 \B1 ) = B1 ∪ B2 . Rezult˘a (18)

card(B1 ∪ B2 ) = card B1 + card(B2 \B1 ).

Analog, (B1 ∩ B2 ) ∩ (B2 \B1 ) = ∅ ¸si (B1 ∩ B2 ) ∪ (B2 \B1 ) = B2 , de unde (19)

card(B1 ∩ B2 ) + card(B2 \B1 ) = card B2 .

Adunˆand (18) ¸si (19) obt¸inem card(B1 ∪ B2 ) = card B1 + card B2 − card(B1 ∩ B2 ). Presupunem relat¸ia adev˘arat˘a pentru n ∈ IN, n ≥ 2 ¸si fie B1 , B2 , . . . , Bn , Bn+1 mult¸imi finite. Atunci card

³ n+1 [

n n ´ ´ ³[ ´ ³³ [ ´ Bi + Bi = card Bi ∪ Bn+1 = card

+ card Bn+1 − card X



i=1

i=1

i=1

n ³³ [

´

Bi ∩ Bn+1 =

i=1

card(Bi ∩ Bj ) +

1≤i 1, ∀n ∈ IN∗ , deducem c˘a xn < yn , ∀n ∈ IN∗ . Dar (xn ) este m˘arginit inferior (de termenul x1 ) pentru c˘a este un ¸sir cresc˘ator, iar (yn ) este m˘arginit superior (de y1 ) pentru c˘a este un ¸sir descresc˘ator ¸si, ˆın concluzie, ambele ¸siruri sunt m˘arginite. Conform teoremei lui Weierstrass 2.21, ambele ¸siruri sunt convergente. Dac˘a not˘am cu x limita lui (xn ), cu y limita lui (yn ) ¸si trecem la limit˘a ˆın relat¸ia de mai sus obt¸inem y = x. S˘a not˘am aceast˘a limit˘a comun˘a cu e. Din monotonia ¸sirurilor rezult˘a c˘a xn < e < yn , ∀n ∈ IN∗ , adic˘a inegalitatea de demonstrat. c) Demonstr˘am mai intˆai ˆıncadrarea 2 < e < 3. Prima inegalitate este clar˘a: 2 = x1 < e, conform punctului precedent. Pentru a doua vom calcula

82 termeni ai lui (yn ) pˆan˘ a obt¸inem unul mai mic decˆat 3. Astfel g˘asim e < 46656 y5 = 15625 < 3. Pentru a studia convergent¸a ¸sirului (zn ), studiem leg˘atura sa cu ¸sirul (xn ). Folosind binomul lui Newton, avem: 1 1 1 n 1 1 ) = 1 + Cn1 + Cn2 2 + ... + Cnk k + ... + Cnn n = n n n n n n n(n − 1) n(n − 1)...(n − k + 1) n(n − 1)...1 =1+ + + ... + + ... + = n 2!n2 n!nn k!nk 1 1 1 1 k−1 = 1 + 1 + (1 − ) + ... + (1 − )...(1 − ) + ...+ 2! n k! n n 1 1 n−1 + (1 − )...(1 − ). n! n n

xn = (1 +

Pe de o parte, avˆand ˆın vedere c˘a toate parantezele sunt subunitare putem 1 1 scrie xn < 1 + 1 + 2!1 + ... + k! + ... + n! , adic˘a xn < zn , ∀n ∈ IN∗ . Pe de alt˘a 1 parte, fixˆand k ∈ IN∗ , avem xn ≥ 1 + 1 + 2!1 (1 − n1 ) + ... + k! (1 − n1 )...(1 − k−1 n ) 1 1 ¸si trecˆand la limit˘a dup˘a n → ∞, avem e ≥ 1 + 1 + 2! + ... + k! . Cum k a fost ales arbitrar, deducem e ≥ zn , ∀n ∈ IN∗ , deci xn < zn ≤ e, ∀n ∈ IN∗ . Din teorema cle¸stelui 2.26-vi) rezult˘a c˘a zn → e. Presupunem acum, prin reducere la absurd c˘a e ∈ Q. Atunci e poate fi ∗ a e ∈ / IN) ¸si deci scris ˆın forma e = m p , unde m, p ∈ IN , p 6= 1 (pentru c˘ ∗ pe ∈ IN unde p este un numar natural fixat. Evident ¸sirul (zn ) este strict cresc˘ator (deci, ˆın particular, zn < e, ∀n ∈ IN) ¸si pentru orice k ∈ IN∗ putem scrie 1 1 1 0 < zk+p − zp = + + ... + = (p + 1)! (p + 2)! (p + k)! · ¸ 1 1 1 = 1+ + ... + ≤ (p + 1)! p+2 (p + 2)(p + 3)...(p + k) · ¸ 1 1 1 ≤ 1+ + ... + = (p + 1)! p+2 (p + 2)k−1 h i 1 k−1 1 k−1 1 − ( ) (p + 2) 1 − ( p+2 ) p+2 1 = < = 1 (p + 1)! 1 − p+2 (p + 1)!(p + 1)
1 aceast˘a funct¸ie este monoton˘a) (1 + n1 )n < e, demonstrat˘ a la punctul a). Avem 1 n ln n+1 < 1, deci ln(n + 1) − ln n < . Aplic˘ a m funct ¸ia logaritm natural n n inegalit˘a¸tii (cum e > 1 aceast˘a funct¸ie este monoton˘a) e < (1 + n1 )n+1 ¸si 1 obt¸inem 1 < (n + 1)[ln(n + 1) − ln n], deci n+1 < ln(n + 1) − ln n. Scriem succesiv aceast˘a dubl˘a inegalitate pentru k = 1, 2, ..., n.

1 < ln 2 − ln 1 < 2 1 < ln 3 − ln 2 < 3 ...

1 , 1 1 , 2

1 1 < ln(n + 1) − ln n < . n+1 n 1 Sumˆand relat¸iile, avem 12 + 13 + ... + n+1 < ln(n + 1) < 1 + 12 + ... + n1 , ∗ ∀n ∈ IN . Cum lim ln(n + 1) = ∞, ultima inegalitate implic˘a faptul c˘a lim un = ∞. (S-a aplicat propozit¸ia 2.26 -iii)). Consider˘am ¸sirul (vn ) ¸si 1 − ln(n + 1) + ln n < 0, deci studiem monotonia sa. Avem vn+1 − vn = n+1 (vn ) este descresc˘ator. ˆIn particular, (vn ) este m˘arginit superior. Pe de alt˘a parte, vn = 1 + 12 + ... + n1 − ln(n + 1) + ln(n + 1) − ln n > 0, ∀n ∈ IN∗ , deci (vn ) este m˘arginit ¸si inferior. Din teorema lui Weierstrass 2.21 rezult˘a c˘a (vn ) este convergent.

2.6 Aceast˘a problem˘a este o generalizare a punctului c) din problema precedent˘a. Consider˘am ¸sirul (zn ) din problema 2.5. Cum (an ) este un ¸sir strict cresc˘ator de numere naturale, an ≥ n, ∀n ∈ IN. Prin urmare avem relat¸iile 0 < xn ≤ zn < e, deci (xn ) este m˘arginit. Evident ¸sirul este strict cresc˘ator, prin urmare este convergent. Not˘am limita sa cu l. Mai ˆıntˆ ai s˘a observ˘am c˘a acest num˘ar nu poate fi natural. ˆIntr-adev˘ ar, dac˘a 0 ¸si 1 sunt primii doi termeni ai lui (an ), atunci 2 < xn < e, ¸si cum (xn ) este cresc˘ator, l ∈ (2, 3). Dac˘a doar unul dintre numerele 0 ¸si 1 este termen al lui (an ), atunci 1 < xn < zn+1 − 1 < e − 1, ¸si deci l ∈ (1, 2). ˆIn sfˆar¸sit, dac˘a nici 0 nici 1 nu sunt termeni ai lui (an ), avem 0 < xn < zn+2 − 2 < e − 2, ¸si deci l ∈ (0, 1). Deci l ∈ / IN. Presupunem prin reducere la absurd c˘a l ∈ Q. Atunci

84 ∗ e poate fi scris ˆın forma e = m al∈ / IN) p , unde m, p ∈ IN , p 6= 1 (pentru c˘ ∗ ¸si deci pe ∈ IN unde p este un num˘ ar natural fixat. Pentru orice k ∈ IN∗ putem scrie

1 1 1 0 < xk+p − xp = + + ... + = ap+1 ! ap+2 ! ap+k ! · ¸ 1 1 1 + ... + = 1+ ≤ ap+1 ! (ap+1 + 1)...ap+2 (ap+1 + 1)...ap+k · ¸ 1 1 1 1 ≤ 1+ + +...+ ≤ ap+1 ! ap+1 +1 (ap+1 +1)(ap+1 +2) (ap+1 + 1)...ap+k · ¸ 1 1 1 1 ≤ 1+ + ... + = ap+1 ! ap+1 + 1 (ap+1 + 1)2 (ap+1 + 1)ap+k −ap+1 =

1

1 − ( ap+11 +1 )ap+k −ap+1

ap+1 !

1

1 − ( ap+1 +1 )



ap+1 + 1 1 < . ap+1 ! ap+1 ap !ap 1

Prima inegalitate a rezultat din ad˘augarea unor termeni pozitivi suplimentari, a doua din majorarea fract¸iilor (prin minorarea numitorilor), iar ultima inegalitate se poate verifica direct, fiind echivalent˘ a cu ap (ap+1 +1) < ap+1 ap (ap + 1)...ap+1 , evident adev˘arat˘ a. A¸sadar, 0 < xk+p − xp < ap1!ap , ∀k ∈ IN∗ . Facem k → ∞ ¸si obt¸inem 0 < l − xp ≤ ap1!ap . Deducem 0 < ap !e − ap !xp ≤ a1p < 1. Dar ap !l ∈ IN ¸si de asemenea, ap !xp ∈ IN. Avem a¸sadar la fel ca la problema precedent˘ a o diferent¸˘ a dintre dou˘a numere naturale cuprins˘a ˆın intervalul (0, 1), ceea ce este o contradict¸ie. Presupunerea f˘acut˘a este fals˘a, deci l ∈ / Q. 2.7 a) Se disting dou˘a sub¸siruri cu comport˘ari diferite (sub¸sirul format din termenii de rang par ¸si sub¸sirul format din termenii de rang impar). Astfel, (2n)3 + 3−2n 1 x2n = =1+ →1 (2n)3 (2n)3 32n ¸si este evident descresc˘ator ¸si x2n+1 =

−(2n + 1)3 + 3−(2n+1) 1 = −1 + → −1 3 (2n + 1) (2n + 1)3 32n+1

¸si este tot descresc˘ator. Cum cele dou˘a sub¸siruri nu au aceea¸si limit˘a, ¸sirul nu este convergent. ˆIn plus, ...x2n+1 < x2n−1 < ... < x1 < 0 < ... < x2n < x2n−2 < ...x2

85 de unde se observ˘a c˘a ¸sirul nu este monoton. b) S¸irul nu este monoton: x1 = sin1 1 > 0, pentru c˘a 1 ∈ (0, π) x4 = sin 4 a 4 ∈ (π, 2π), iar x7 = sin7 7 > 0, pentru c˘a 7 ∈ (2π, 3π). 4 < 0, pentru c˘ Deci x1 > x4 < x7 , ceea ce ˆınseamn˘ a c˘a ¸sirul nu este nici cresc˘ator, nici descresc˘ator. Observ˘am totu¸si c˘a ¸sirul este convergent ¸si are limita 0, pentru c˘a funct¸ia sin este marginit˘a, iar n1 → 0 (vezi propozit¸ia 2.26-v)). c) Pentru a studia monotonia ¸sirului consider˘am funct¸ia f : [0, ∞) → IR, f (x) = ex (sin x + 2). Atunci, f 0 (x) = ex (sin x + cos x + 2) ≥ 0, ∀x ∈ [0, ∞). Prin urmare funct¸ia este cresc˘atoare, deci ¸sirul este cresc˘ator. Ar˘atat¸i c˘a xn → ∞. 2.8

Calculˆand raportul

xn+1 xn ,

avem

xn+1 y1 y2 ...yn yn+1 = = yn+1 → l. xn y1 y2 ...yn Ambele concluzii rezult˘a din propozit¸ia 2.28. 2.9

a) Folosind exercit¸iul 2.4-f), lim n3 ( 47 )n = 0. Pe de alt˘a parte,

lim n2 sin 2π 3n = lim de funct¸ii

2π 3n 2π 3n

sin

lim sin x x→0 x

2π 2 3n n

= 0; am folosit pe rˆand limita fundamental˘ a

= 1 ¸si exercit¸iul 2.4-d). A¸sadar limita c˘autat˘ a este 0. q n n b) Fie an ¸sirul din enunt¸. Putem scrie an = √ = n nn! . Suntem ˆın n n! nn si este suficient n! ¸ 1 n n ( n+1 n ) = (1+ n ) → e.

situat¸ia ˆın care putem aplica propozit¸ia 2.29: not˘am xn = n+1

n! . Avem xxn+1 = (n+1) s˘a determin˘am lim xxn+1 (n+1)! nn = n n Limita c˘autat˘a este e. c) Este u¸sor de v˘azut c˘a putem aplica propozit¸ia 2.28 (sau exercit¸iul 2.8) (3n−1) 3 pentru xn = 25 59 ...... (4n+1) . Astfel, xxn+1 = 3n+2 si deci lim xn = 0. 4n+5 → 4 < 1 ¸ n d) Vom aplica propozit¸ia 2.30 (criteriul Stolz-Ces`aro). Not˘am an = √ 1 ln 2+ 13 ln 3+... n1 ln n ¸si bn = n n. Evident ¸sirul (bn ) este strict cresc˘ator ¸si 2

cu limita +∞. Atunci

√ √ ln(n+1) (n+1) n+1+n n n+1 3n2 +3n+1

an+1 −an bn+1 −bn

=

1 ln(n+1) n+1 √

√ (n+1) n+1−n n

=

√ √ ln(n+1) (n+1) n+1+n n n+1 (n+1)3 −n3

=

= 0 (conform exercit¸iului 2.4-c) ¸si limitei (L1) din

observat¸ia 2.34). e) Fie xn = n21+1 + n22+2 + ... + n2n+n . Vom majora ¸si minora fract¸iile folosind acela¸si numitor (cel mai mare ˆın cazul minor˘arii ¸si cel mai mic ˆın cazul major˘arii). Astfel, xn > n21+n + n22+n + ... + n2n+n = 1+2+...+n = n2 +n n(n+1) 2(n2 +n)

=

1 2

¸si xn
0 astfel ˆınˆ at |f 0 (x)| ≤ M,

90 ∀x ∈ [0, 1]. Atunci, ¯Z 1 ¯ Z 1 Z 1 ¯ ¯ n+1 0 ¯ xn+1 f 0 (x)dx¯ ≤ x |f (x)|dx ≤ M xn+1 dx = ¯ ¯ 0 0 0 ¯1 n+2 ¯ x ¯ = M → 0. =M n + 2 ¯0 n + 2 R ˆIn consecint¸˘a, 1 xn+1 f 0 (x)dx → 0, ¸si, revenind la xn , avem lim xn = f (1). 0 2.11 S¸irul este format din termeni pozitivi. Avˆand ˆın vedere c˘a sinn+1 x ≤ sinn x, ∀n ∈ IN, ∀x ∈ [0, π2 ], ¸sirul este descresc˘ator. ˆIn consecint¸˘ a ¸sirul este convergent. Se observ˘a c˘a forma ¸sirului este asem˘an˘ atoare celei de la exercit¸iul 2.10-c). Totu¸si, metoda folosit˘a acolo nu mai funct¸ioneaz˘ a pen( π2 )n tru determinarea limitei pentru c˘a lim n+1 = ∞. Vom face un calcul direct al valorii integralei. Avem: Z xn =

0

π/2

Z sinn x d x = xn =

π/2

(− cos x)0 sinn−1 x d x=

0

Z π/2 ¯ n−1 ¯π/2 = − cos x sin (n − 1) cos2 x sinn−2 xdx = ¯0 + 0 Z π/2 = (n − 1) (1 − sin2 x) sinn−2 xdx = (n − 1)xn−2 − (n − 1)xn . 0

Deci, nxn = (n − 1)xn−2 , adic˘a xn = n−1 am aceast˘a n xn−2 , ∀n ≥ 2. Aplic˘ relat¸ie pentru valori pare ¸si respectiv impare ale rangului. Astfel, 2n − 1 2n − 3 2n − 1 x2n−2 = x2n−4 = ... = 2n 2n 2n − 2 2n − 1 2n − 3 1 1 · 3 · ... · (2n − 1) π = ... x0 = 2n 2n − 2 2 2 · 4 · ... · (2n) 2

x2n =

¸si 2n 2n 2n − 2 x2n−1 = x2n−3 = ... = 2n + 1 2n + 1 2n − 1 2n 2n − 2 2 2 · 4 · ... · (2n) = ... x1 = , 2n + 1 2n − 1 3 3 · 5 · ... · (2n + 1)

x2n+1 =

valorile pentru x0 ¸si x1 calculˆ andu-se direct. Conform exercit¸iului 2.10-i), lim x2n = 0. Folosind aceea¸si metod˘a ca ˆın exercit¸iul ment¸ionat (ˆınlocuind

91 de aceast˘a dat˘a pe k cu volori impare), obt¸inem x2n+1 → 0. ˆIn consecint¸˘ a, lim xn = 0. 2.12 Fie (xn ) ¸sirul din problema precedent˘ a ¸si (yn ) ¸sirul din problema de fat¸˘a. Evident ¸sirul (yn ) este format din termeni pozitivi ¸si √ √ yn+1 2n + 1 2n + 3 8n3 + 20n2 + 14n + 3 √ = =√ ≤ 1, ∀n ∈ IN∗ , yn 2n + 2 2n + 1 8n3 + 20n2 + 16n + 1 deci ¸sirul este descresc˘ator ¸si ˆın consecint¸˘ a este convergent. Conform calculelor f˘acute ˆın problema precedent˘ a, x2n 1 · 3 · ... · (2n − 1) π 3 · 5 · ... · (2n + 1) π = = yn2 . x2n+1 2 · 4 · ... · (2n) 2 2 · 4 · ... · (2n) 2 n = n−1 a lim xxn−1 = 1. Cum Dar din relat¸ia de recurent¸˘a, xxn−2 n , deducem c˘ n (xn ) este descresc˘ator, xn−2 > xn−1 > xn , de unde, folosind criteriul cle¸sq n telui, lim xxn−1 = 1. Obt¸inem deci c˘a lim yn2 π2 = 1, adic˘ a lim yn =

2 π.

2.13 Fie (xn )n∈IN un ¸sir nem˘arginit de numere reale. Atunci (xn )n∈IN este nem˘arginit inferior sau superior. Presupunem, f˘ar˘ a a restrˆange generalitatea c˘a este nem˘arginit superior. Aceasta ˆınseamn˘ a, negˆand definit¸ia, c˘a pentru orice M > 0, exist˘a nM ∈ IN astfel ˆınˆ at xnM > M. Pentru M = 1, exist˘a n1 ∈ IN astfel ˆıncˆat xn1 > 1. Consider˘ am acum ¸sirul (xn )n>n1 . Acest ¸sir, obt¸inut prin ˆındep˘artarea unui num˘ ar finit de termeni din ¸sirul init¸ial este ˆın continuare nem˘arginit superior (contrar (xn )n∈IN este m˘arginit superior). Pentru M = 2, exist˘a n2 > n1 astfel ˆıncˆ at xn1 > 2. Consider˘ am acum ¸sirul nem˘arginit (xn )n>n2 ¸si lu˘am M = 3. Procedˆand similar obt¸inem un ¸sir strict cresc˘ator de numere naturale (nk )k≥1 astfel ˆıncˆ at xnk > k, ∀k ≥ 1. S˘ a ar˘at˘am c˘a (xnk )k≥1 are limita +∞. Fie ε > 0; exist˘a kε ∈ IN astfel ˆıncˆ at kε > ε. Atunci ∀k > kε , xk > k > kε > ε, adic˘ a exact caracterizarea cu ε a faptului c˘a xnk → ∞. Similar, dac˘a ¸sirul init¸ial este nem˘arginit inferior, atunci va avea un sub¸sir cu limita −∞. 2.14 Fie (xn )n≥1 un ¸sir de numere reale convergent la x ∈ IR, iar an = x1 +x2 +...+xn , ¸sirul mediilor aritmetice asociat. Pentru determinarea limitei n ¸sirului (an ) putem utiliza criteriul Stolz-Ces`aro: obt¸inem lim an = lim xn+1 = x. Reciproca este fals˘a: consider˘am urm˘atorul contraexemplu: xn = (−1)n , 0 −1 ∀n ∈ IN∗ . Atunci a2n = 2n = 0, iar a2n+1 = 2n+1 , ∀n ∈ IN∗ , deci an → 0, dar (xn ) este divergent.

92 Fie acum (xn )n≥1 un ¸sir de numere reale pozitive convergent la x ∈ IR √ ¸si gn = n x1 x2 ...xn , n ≥ 2 ¸sirul mediilor geometrice asociat. Folosind propozit¸ia 2.29, avem lim gn = lim xn+1 = x. Reciproca este fals˘a. Consider˘am ¸sirul xn = n1 , dac˘a n este num˘ ar par nenul ¸si xn = 1, dac˘a n este q q q impar. Atunci g2n = 2n n1n = n1 , ∀n ≥ 1 ¸si g2n+1 = 2n+1 n1n ,∀n ≥ 2. ˆIn consecint¸˘a gn → 0, dar (xn ) este divergent. Fie acum (xn )n≥1 un ¸sir de numere reale pozitive convergent la x ∈ IR, iar hn = 1 + 1 n+...+ 1 ¸sirul mediilor armonice. Folosind criteriul Stolz-Ces`aro x1

x2

xn

pentru ¸sirul ( h1n ) (sunt ˆındeplinite condit¸iile aplic˘arii acestui criteriu) este 1 u¸sor de v˘azut c˘a lim h1n = lim xn+1 = x1 , deci lim hn = x. Reciproca este din nou fals˘a. Este de ajuns s˘a consider˘am urm˘atorul contraexemplu: xn = 21 , dac˘a n este num˘ar par nenul ¸si xn = 13 , dac˘a n este impar. Z 2.15

Fie xn = f (1) + f (2) + . . . + f (n) −

n

f (x) dx, ∀n ≥ 1. Studiem, 1

pe rˆand monotonia ¸si m˘arginirea ¸sirului. Monotonia se studiaz˘a f˘acˆ and diferent¸a dintre doi termeni consecutivi ai ¸sirului ¸si comparˆand-o cu 0. Avem Z

Z

n+1

n

xn+1 − xn = f (n + 1) −

f (x) dx + f (x) dx = 1 1 Z n Z n+1 Z n = f (n + 1) − f (x) dx − f (x) dx + f (x) dx = 1 n 1 Z n+1 = f (n + 1) − f (x) dx. n

S¸tim c˘a funct¸ia f este descresc˘atoare; atunci pentru orice x ∈ [n, n + 1], ≥ f (n + 1),R de unde, prin integrare obt¸inem ¯ Rf (x) n+1 n+1 f (x) dx ≥ n f (n + 1) dx = f (n+1)·x ¯n+1 = f (n+1). Prin urmare, n n xn+1 − xn ≤ 0, ∀n ≥ 1, deci (xn ) este descresc˘ator. Este suficient s˘a mai demonstr˘am c˘a ¸sirul este m˘arginit inferior. Pentru aceasta scriem Z 2 xn = f (1) + f (2) + . . . + f (n) − f (x) dx− 1 Z 3 Z n − f (x) dx − ... − f (x) dx = 2

Z

= f (1) + (f (2) −

n−1 2

f (x) dx) + (f (3)− 1

93 Z

Z

3



n

f (x) dx) + ... + (f (n) − 2

f (x) dx). n−1

Prin ipotez˘a, f (1) ≥ 0, iar fiecare parantez˘ a cont¸ine o expresie de tip f (k) − Rk a ca mai sus c˘a este nenegativ˘a (f (k) ≤ k−1 f (x) dx despre care se arat˘ f (x), ∀x ∈ [k − 1, k]). Prin urmare xn ≥ 0, ∀n ≥ 1. Folosind teorema lui Weierstrass 2.21 concluzion˘am c˘a ¸sirul (xn ) este convergent. Pentru a obt¸ine convergent¸a ¸sirului 1+ 12 + 31 +..... n1 −ln n particulariz˘ am rezultatul de mai sus pentru f : [1, ∞) → IR, f (x) = x1 , funct¸ie care verific˘ a toate ipotezele cerute. √ Pentru ¸sirul 1 + √12 + ... + √1n − 2 n lu˘ am funct¸ia f : [1, ∞) → IR, 1 1 1 f (x) = √x . ˆIn sfˆar¸sit, pentru 1 + 2 ln 2 + ... + n ln lu˘am f : n − ln (ln n) Z n 1 sirul xn = f (2)+. . .+f (n)− [2, ∞) → IR, f (x) = x ln f (x) dx x . Evident ¸ 2

este de asemenea convergent (ca mai sus) ¸si difer˘a de ¸sirul dat doar printr-o constant˘a. A¸sadar ¸sirul considerat este convergent. 2.16

Discut˘am limita ³

´ p lim n an + n2 + bn + c = lim n2

n→∞

n→∞

Ã

r a+

c b 1+ + 2 n n

! .

Evident, pentru a < −1, limita este −∞, iar pentru a > −1 limita este +∞. Consider˘am a = −1. Pentru a < 0 avem ˆın parantez˘ a o nedeterminare de tipul (∞ − ∞), motiv pentru care amplific˘am paranteza cu expresia conjugat˘a. Astfel, ³ ´ p n2 − n2 − bn − c lim n −n + n2 + bn + c = lim n √ = n2 + bn + c + n −bn2 − cn = lim √ . n2 + bn + c + n Dac˘a b 6= 0 limita este −∞ · sgn(b), iar dac˘a b = 0 limita este − 2c . √ Discut˘am cea de-a doua limit˘a. Scoatem factor comun n. Avem à ! r r √ √ √ √ 1 2 lim(a n + b n + 1 + c n + 2) = lim n a + b 1 + + c 1 + n n Se observ˘a c˘a dac˘a a + b + c 6= 0 limita este +∞ · sgn(a + b + c). Dac˘ a a + b + c = 0, atunci c = −a − b ¸si avem

94

√ √ √ lim(a n + b n + 1 + c n + 2) = √ √ √ √ = lim(a n + b n + 1 − a n + 2 − b n + 2) = √ √ √ £ √ ¤ = lim a( n − n + 2) + b( n + 1 − n + 2) = ¸ · −1 −2 √ √ + b√ = 0. = lim a √ n+ n+2 n+1+ n+2 ¡ ¢¡ ¢ ¡ ¢ 2.17 Fie xn = 1 + n12 1 + n22 ... 1 + nn2 , n ≥ 1. Dup˘a o metod˘a deja folosit˘a, logaritm˘am ¸sirul ¸si utiliz˘am inegalitatea (I2) din observat¸ia 2.35. µ ¶ µ ¶ ³ 1 2 n´ ln xn = ln 1 + 2 + ln 1 + 2 + ... + 1 + 2 . n n n Atunci 1 12 2 22 n n2 1 2 n − + − + ... + − ≤ ln xn ≤ 2 + 2 + ... + 2 . 2 4 2 4 2 4 n 2n n 2n n 2n n n n Efectu˘am calculele ¸si obt¸inem n(n + 1) n(n + 1)(2n + 1) n(n + 1) − ≤ ln xn ≤ . 2 4 2n 2n 2n2 Cum ambii membri care ˆıncadreaz˘ a ln xn au limita 12 , lim ln xn = 21 , adic˘a 1 lim xn = e 2 . 2.18 Fie ¸sirurile xn = cos nα ¸si yn = sin nα. Vom trata simultan problema convergent¸ei celor dou˘a ¸siruri. Presupunem c˘a xn → x. Din formula trigonometric˘a cos(n + 1)α = cos nα cos α − sin nα sin α, avem xn+1 = xn cos α − yn sin α. Dac˘ a α 6= kπ, k ∈ Z (adic˘a sin α 6= 0) avem yn =

xn cos α − xn+1 , sin α

ceea ce ˆınseamn˘a c˘a (yn ) este convergent (operat¸ii cu ¸siruri convergente). Notˆand limita sa cu y avem x = x cos α − y sin α. Pe de alt˘a parte, sin 2nα = 2 sin nα cos nα ¸si trecˆand la limit˘a obt¸inem y = 2xy, deci y = 0 sau x = 12 .

95 Cazul y = 0. Din formula sin2 nα + cos2 nα = 1, avem x2 + y 2 = 1, deci x = 1 sau x = −1. Din relat¸ia de mai sus obt¸inem cos α = 1, adic˘ a α = ± π2 +2kπ. Dar, ˆın oricare dintre aceste cazuri, xn = cos nπ , ¸ s ir divergent 2 (pentru c˘a sub¸sirurile de rang 4p√¸si 4p + 1 au limite√diferite). Cazul x = 12 . Obt¸inem y = ± 23 ¸si 12 = 12 cos α∓ 23 sin α. De exemplu, ˆın √

situat¸ia ˆın care lu˘am semnul +, avem 12 = 12 cos α + 23 sin α, adic˘a cos π3 = cos( π3 + α). Ajungem la contradict¸ia α = 2kπ, k ∈ Z (am luat α 6= kπ) sau α = − 2π ın care (xn ) nu este convergent. Cel˘alalt caz se 3 + 2kπ, k ∈ Z, caz ˆ rezolv˘a la fel. S¸irul nu este convergent. ˆIn final, trat˘am cazul α = kπ, k ∈ Z. Dac˘a α = 2kπ, k ∈ Z, atunci x = 1 ¸si y = 0. Dac˘a α = (2k + 1)π, k ∈ Z, xn = cos n(2k + 1)π = cos nπ = (−1)n , ¸sir divergent. Prin urmare ¸sirul (xn ) este convergent doar dac˘a α = 2kπ, k ∈ Z. S ¸ irul (yn ) este convergent doar dac˘a α = kπ, k ∈ Z. Reciproca este imediat˘a. 2.19 a) xn+1 = (1− 4n1 2 )xn , ∀n ≥ 1, x1 = 1. Cum x1 > 0 ¸si 0 < (1− 4n1 2 ) < 1 pentru orice n ≥ 1, rezult˘a prin induct¸ie c˘a xn > 0, ∀n ≥ 1 ¸si (xn ) este descresc˘ator. Obt¸inem c˘a ¸sirul este convergent. Se poate ar˘ata c˘a limita este 0. b) xn+1 = x2n − 2xn + 2, n ∈ IN∗ , x1 = a > 0. Avem xn+1 − 1 = (xn − 1)2 . Not˘am xn − 1 = yn . Deci yn+1 = yn2 , ∀n ∈ IN∗ , y1 = a − 1 > −1. Studiem monotonia ¸sirului (yn ). Avem yn+1 − yn = yn (yn − 1). Distingem mai multe situat¸ii. Dac˘a y1 > 1, atunci prin induct¸ie se obt¸ine pe baza relat¸iei de recurent¸˘ a c˘a tot¸i termenii ¸sirului sunt supraunitari, deci yn+1 − yn ≥ 0, adic˘ a ¸sirul este cresc˘ator. Prin urmare ¸sirul are limit˘a ˆın IR. Presupunˆand c˘a ¸sirul are limita real˘a y ¸si trecˆand la limit˘a ˆın relat¸ia de recurent¸˘ a, avem y 2 = y, adic˘ a y = 0 sau y = 1. Un ¸sir de numere supraunitare ¸si cresc˘ator nu poate avea nici una dintre aceste limite, deci ¸sirul are limita +∞, adic˘ a este divergent. Revenind la (xn ), concluzion˘am c˘a acest ¸sir este divergent dac˘a a > 2. Dac˘a y1 = 1, atunci yn = 1, ∀n ∈ IN, deci lim yn = 1. Deci, pentru a = 2, lim xn = 2. Dac˘a y1 ∈ (0, 1), atunci prin induct¸ie se obt¸ine c˘a tot¸i termenii ¸sirului sunt ˆın acest interval ¸si deci diferent¸a yn+1 − yn este negativ˘a; ¸sirul este descresc˘ator. Prin urmare ¸sirul este convergent. Ca mai sus obt¸inem c˘a singura limit˘a posibil˘a este 0. Deci pentru a ∈ (1, 2), (xn ) este convergent la 1. Dac˘a y1 = 0, yn = 0, ∀n ∈ IN, deci lim yn = 0. Deci, pentru a = 1, lim xn = 1.

96 Dac˘a y1 ∈ (−1, 0), atunci y2 = y12 ∈ (0, 1) ¸si practic revenim la unul din cazurile anterioare: tot¸i termenii ¸sirului, ˆıncepˆ and cu ragul 2, sunt ˆın (0, 1). S¸irul (yn ) este convergent la 0. Deci pentru a ∈ (0, 1), (xn ) este convergent la 1. c) xn+1 = e−1+xn , n ≥ 1, x1 ∈ (0, ∞) . Evident, tot¸i termenii ¸sirului sunt pozitivi. Din inegalitatea ex−1 ≥ x, ∀x ∈ IR (egalitate pentru x = 1), rezult˘ a c˘a ¸sirul este cresc˘ator S¸irul are limita ˆın IR. Distingem urm˘atoarele cazuri. Dac˘a x1 > 1, atunci xn > 1, ∀ n ≥ 1. Presupunˆand c˘a limita ¸sirului este num˘arul real x, avem ex−1 = x, adic˘a x = 1, contradict¸ie. Prin urmare ¸sirul este divergent (are limita ∞). Dac˘a x1 = 1, atunci ¸sirul este constant 1, deci este convergent la 1. Dac˘a x1 < 1, ¸sirul este m˘arginit superior de 1 pentru c˘a tot¸i termenii sunt subunitari (induct¸ie). Limita ¸sirului este 1. 2.20 Dac˘a x0 < 0, atunci x1 > 1, deci x2 ∈ (0, 1), adic˘a x3 < 0. Prin induct¸ie, rezult˘a c˘a sub¸sirul (x3k )k este format din numere negative, iar (x3k+1 )k din numere supraunitare. S¸irul nu poate fi convergent. Celelalte situat¸ii se trateaz˘a asem˘an˘ ator. 2.21 Fie xn = 1 + 12 + ... + n1 , n ∈ IN∗ . Am demonstrat c˘a limita acestui ¸sir este +∞. Fie p ∈ IN∗ fixat. Avem 0 ≤ xn+p − xn =

1 1 p 1 + + ... + ≤ → 0, n+1 n+2 n+p n+p

deci lim (xn+p − xn ) = 0. n→∞

2.22 Studiul convergent¸ei ¸sirului nu se poate face prea u¸sor prin metodele utilizate pˆan˘a acum. Vom apela la funct¸ia f : [1, ∞) → IR, f (x) = πx − 2x arctg x. Studiem monotonia funct¸iei, calculˆand derivata de ordinul ˆıntˆ ai. Astfel, 2x f 0 (x) = π − 2 arctg x − . 1 + x2 Pentru a determina semnul derivatei ˆıntˆ ai vom calcula derivata a doua. f 00 (x) = −2

1 2 − 2x2 −4 − = < 0, ∀x ≥ 1. 2 2 2 1+x (1 + x ) (1 + x2 )2

Deci derivata ˆıntˆai este o funct¸ie strict descresc˘atoare. Cum lim f 0 (x) = 0, x→∞

deducem c˘a f 0 (x) > 0, ∀x ≥ 1, adic˘a funct¸ia f este cresc˘atoare. Prin urmare

97 ¸sirul (xn ) este cresc˘ator, deci are limit˘a ˆın IR. Pentru a determina limita ¸sirului este suficient s˘a calcul˘am lim f (x) = lim

x→∞

π − 2 arctg x

x→∞

1 x

=

−2 1+x2 lim x→∞ − 12 x

= 2.

S¸irul este convergent ¸si are limita 2. 2.23 Mai ˆıntˆai determin˘am constantele x, y, z astfel ˆıncˆ at limita ¸sirului (an ) s˘a fie 1. Evident, dac˘a z ≥ 0, an → ∞. Deci z < 0. Atunci avem de rezolvat (ˆın parantez˘a) o nedeterminare de tip ∞ − ∞. p n2 + nx + y − n2 z 2 an = n( n2 + nx + y + nz) = n p = n2 + nx + y − nz n3 (1 − z 2 ) + n2 x + ny . = p n2 + nx + y − nz Pentru ca lim an s˘a fie 1 trebuie s˘a avem 1√− z 2 = 0, x = 0, y = 1 − z, adic˘ a 2 z = −1, x = 0, y = 2. Prin urmare an = n( n + 2 − n). Acum α

lim nα ln an = lim ln(an )n . Dac˘a α ≤ 0, lim nα ln an = 0, prin urmare este necesar ca α > 0. ˆIn acest caz sub logaritm avem o nedeterminare de tip 1∞ . Deci p α lim nα ln an = lim ln(1 + n n2 + 2 − n2 − 1)n = ¸nα (n√n2 +2−n2 −1) · 1 p √ 2 2 2 = lim ln (1 + n n2 + 2 − n − 1) n n +2−n −1 = = ln elim n



α (n

n2 +2−n2 −1)

= p n4 + 2n2 − n4 − 2n2 − 1 √ = lim nα (n n2 + 2 − n2 − 1) = lim nα = n n2 + 2 + n2 + 1 −1 = lim nα √ . n n2 + 2 + n2 + 1

Pentru ca ultima limit˘a s˘a fie finit˘a ¸si nenul˘ a trebuie ca α = 2, caz ˆın care 1 limita este − 2 . 2.24 Din relat¸ia de recurent¸˘a avem xn+1 − xn = −x2n ≤ 0, ∀n ∈ IN, deci ¸sirul este descresc˘ator. Pe de alta parte, xn+1 = xn (1 − xn ). Cum x0 ∈ [0, 1], se obt¸ine u¸sor prin induct¸ie c˘a xn ∈ [0, 1], ∀n ∈ IN, deci ¸sirul este m˘arginit.

98 ˆIn concluzie (xn ) este convergent. Trecˆ and la limit˘a ˆın relat¸ia de recurent¸˘ a, 2 avem lim xn = 0. Cum xk = xk − xk+1 , avem yn = x0 − xn+1 , ¸sir convergent cu limita x0 . 2.25 a) Evident, ¸sirul (xn ) este cresc˘ator. Pe de alt˘a parte, xn+1 = xn (1 + xn ), ∀n ≥ 1. ˆIn funct¸ie de pozit¸ia primului termen fat¸˘ a de −1 ¸si 0 distingem urm˘atoarele situat¸ii. Dac˘a x1 > 0, atunci xn > 0,∀n ≥ 1. Presupunˆ and c˘a ¸sirul are limita un num˘ar real x, obt¸inem din relat¸ia de recurent¸˘ a x = 0, ceea ce este imposibil. Deci limita este +∞. Dac˘a x1 = 0, atunci ¸sirul este constant 0, deci convergent la 0. Dac˘a x1 ∈ (−1, 0), atunci prin induct¸ie se arat˘a imediat c˘a tot¸i termenii ¸sirului sunt ˆın acest interval, deci ¸sirul este m˘arginit ¸si ˆın consecint¸˘ a convergent. Limita sa este 0. Dac˘a x1 = −1, atunci x2 = 0 ¸si xn = 0, ∀n ≥ 2. S¸irul are limita 0. Dac˘a x1 < −1, atunci x2 > 0 ¸si revenim la primul caz. b) Se observ˘a c˘a yn+1 − a = (yn − a) + (yn − a)2 . Problema se reduce la cazul precedent cu ¸sirul (yn − a) ˆın loc de (xn ). 2.26 Se observ˘a c˘a tot¸i termenii ¸sirului sunt pozitivi. Aplicˆand inega√ litatea mediilor ˆın relat¸ia de recurent¸˘ a obt¸inem c˘a xn ≥ a pentru orice n ≥ 2. Studiem monotonia ¸sirului: 1 a 1 xn+1 − xn = ( − xn ) = (a − x2n ) ≤ 0. 2 xn 2xn Prin urmare, ¸sirul este monoton descresc˘ator. Cum (xn ) este ¸si m˘arginit inferior, rezult˘a c˘a este convergent. Trecˆ and la limit˘a ˆın relat¸ia de recurent¸˘ a √ √ √ obt¸inem limita a (valoarea − a nu poate fi limita ¸sirului pentru c˘a − a < 0 ¸si termenii ¸sirului sunt pozitivi). 2.27 Se arat˘a f˘ar˘a dificultate c˘a tot¸i termenii ¸sirului sunt ˆın intervalul [0, 6] (induct¸ie), deci ¸sirul este m˘arginit. Studiem monotonia ¸sirului xn+1 − xn =

√ 6 − xn − x2n (2 − xn )(3 + xn ) 6 − xn − xn = √ = √ . 6 − xn + xn 6 − xn + xn

Trebuie s˘a studiem a¸sadar pozit¸ia ¸sirului fat¸˘ a de 2. ˆIncepem cu x0 . √ √ Dac˘a x0 < 2, atunci x1 = 6 − x0 > 2, x2 = 6 − x1 < 2. Inductiv, obt¸inem c˘a x2k < 2, ∀k ∈ IN ¸si x2k+1 > 2, ∀k ∈ IN. Studiem monotonia

99 sub¸sirului (x2k ). √ √ 6 − 6 − x2k − x22k = 6 − 6 − x2k − x2k = p = √ 6 − 6 − x2k + x2k 36 − 12x22k + x42k − 6 + x2k = p . √ √ ( 6 − 6 − x2k + x2k )(6 − x22k + 6 − x2k ) q

x2k+2 − x2k

Cum numitorul este pozitiv, trebuie s˘a decidem semnul expresiei de la num˘ar˘ator. Avem x42k − 12x22k + x2k + 30 = (x2k − 2)(x32k + 2x22k − 8x2k − 15) = £ ¤ = (x2k − 2) x2k (x22k + 2x2k − 8) − 15 > 0, deci acest sub¸sir este cresc˘ator. Similar se arat˘a c˘a sub¸sirul (x2k+1 ) este descresc˘ator. Demonstrat¸ia monotoniei celor dou˘a sub¸siruri se poate face ¸si prin induct¸ie! Deci cele dou˘a sub¸siruri considerate sunt convergente. Not˘am a obt¸inem √cu l1 ¸si respectiv l2 . Din relat¸ia de recurent¸˘ √ cele dou˘a limite l1 = 6 − l2 ¸si l2 = 6 − l1 , de unde rezult˘a l1 = l2 = 2. S¸irul (xn ) este convergent la 2. Cazul x0 > 2 se rezolv˘a similar. Dac˘a x0 = 2, atunci ¸sirul este constant. A se vedea exercit¸iul 2.38 pentru alt˘a solut¸ie. 2.28

Studiem monotonia: xn+1 − xn =

√ xn + 2 − x2n (2 − xn )(1 + xn ) xn + 2 − xn = √ = √ . xn + 2 + xn xn + 2 + xn

Trebuie s˘a compar˘am termenii ¸sirului cu 2. Dac˘a x0 > 2, atunci (induct¸ie) xn > 2, ∀n ∈ IN. S¸irul este descresc˘ator ˆın acest caz, deci convergent; limita sa este 2. Dac˘a x0 = 2 ¸sirul este constant, deci convergent la 2. Dac˘a x0 < 2, termenii ¸sirului sunt mai mici decˆat 2 ¸si ¸sirul este cresc˘ator. Din nou limita ¸sirului este 2. 2.29 Dac˘a x < y, exist˘a ε > 0 astfel ˆıncˆ at x + ε < y − ε (de exemplu ε = y−x ). Cum x → x, exist˘ a n ∈ IN, astfel ˆıncˆ at ∀n ≥ n0 , |xn − x| < ε, n 0 4 adic˘a x − ε < xn < x + ε. Asem˘an˘ ator exist˘a n1 ∈ IN, astfel ˆıncˆ at ∀n ≥ n1 , |yn − y| < ε, adic˘a y−ε < yn < y+ε. A¸sadar, pentru orice m ≥ max(n0 , n1 ), xm < x + ε < y − ε < ym , adic˘ a ceea ce trebuia s˘a ar˘at˘ am. Dac˘a x = y concluzia nu mai este adev˘arat˘a: este suficient s˘a lu˘am ¸sirurile xn = (−1)n n1 ¸si yn = (−1)n+1 n1 , care au ambele limita 0.

100 2.30

Putem scrie

µ xn − yn = yn

¶ xn − 1 → 0, yn

pentru c˘a (yn ) este m˘arginit. Dac˘a (yn ) este nem˘arginit, rezultatul nu se mai p˘astreaz˘a. D˘am un contraexemplu: xn = n+1, yn = n. Evident, lim xynn = 1, dar xn − yn = 1 → 1. 2

2.31 Avem urm˘atoarea relat¸ie: xn+1 − xn = − xnn ≤ 0, ∀n ≥ 1, deci (xn ) este descresc˘ator. a) Dac˘a x1 < 0, atunci tot¸i termenii ¸sirului sunt negativi. Prin urmare ¸sirul are limita ˆın IR. Deci x2n ≥ x21 , ∀n ≥ 1. Avem xn − xn+1 =

x2n x2 ≥ 1 , ∀n ≥ 1. n n

Deci x21 1 x21 x2 − x3 ≥ 2 ... x1 − x2 ≥

xn − xn+1 ≥

x21 . n

Prin adunarea acestor relat¸ii obt¸inem x1 − xn+1 ≥ x21 (1 +

1 1 + ... + ), 2 n

deci

1 1 + ... + ) → −∞, 2 n de unde deducem c˘a limita lui (xn ) este −∞. b) Dac˘a x1 ∈ [0, 1], atunci x2 = x1 (1 − x1 ) ∈ [0, 1]. Inductiv, obt¸inem c˘a ¸sirul este cuprins ˆıntre 0 ¸si 1, deci este m˘arginit. Prin urmare ¸sirul este convergent ¸si not˘am l = lim xn . Dac˘a x1 = 0, atunci ¸sirul este constant 0, deci cu limita 0. Dac˘ a x1 = 1, atunci x2 = 0 ¸si ¸sirul este constant 0 ˆıncepˆand cu acest rang. Limita este din nou 0. Dac˘ a x1 ∈ (0, 1), atunci, folosind criteriul Stolz-Ces`aro avem: xn+1 ≤ x1 − x21 (1 +

(n + 1)xn+1 − x2n+1 − nxn + x2n nxn − x2n = lim = n 1 = lim [n(xn+1 − xn ) + xn+1 ] = lim(−x2n + xn+1 ) = −l2 + l.

l = lim xn+1 = lim

101 Prin urmare limita este 0. c) Dac˘a x1 > 1, atunci x2 < 0 ¸si revenim la primul caz. Limita ¸sirului este −∞. 2.32

Avem xn+2 ≤

2 max{xn, xn+1 } xn+1 + xn ≤ = yn , ∀n ≥ 0. 2 2

Cum (xn ) este m˘arginit, (yn ) este minorat. Fie n fixat. Dac˘a xn+1 ≥ xn+2 , atunci yn+1 = xn+1 ≤ max{xn, xn+1 } = yn ; dac˘a xn+1 < xn+2 , atunci yn+1 = xn+2 ≤ yn , din relat¸ia de mai sus. Prin urmare ¸sirul (yn ) este descresc˘ator. Concluzion˘am c˘a ¸sirul (yn ) este convergent. S˘a studiem convergent¸a ¸sirului (xn ). Adunˆ and xn+1 la relat¸ia 2 xn xn+2 ≤ xn+12+xn , obt¸inem xn+2 + xn+1 ≤ x + , ∀n ≥ 0. Prin urmare ¸sirul n+1 2 2 xn zn = xn+1 + 2 este monoton descresc˘ator ¸si m˘arginit ((xn ) este m˘arginit), deci este convergent. Not˘am limita sa cu l. Pentru orice ε > 0, exist˘a nε ∈ IN astfel ˆıncˆat pentru orice n ≥ nε , l − ε < zn < l + ε, adic˘a, l − ε < xn+1 +

xn < l + ε. 2

Putem presupune, f˘ar˘a a restrˆange generalitatea c˘a relat¸iile de mai sus au loc pentru orice n ∈ IN. Pa¸sii de urmat ˆın continuare sunt urm˘atorii: se scrie relat¸ia de mai sus pentru k = 0, n ¸si se ˆınmult¸e¸ste fiecare convenabil pentru a se obt¸ine prin adunare o sum˘a telescopic˘a: prima cu (− 12 )n−1 , a doua cu (− 12 )n−2 ¸s.a.m.d. Prin sumare obt¸inem o ˆıncadrare care ne permite s˘a concluzion˘am c˘a lim xn = 2l3 . 2.33 Se obt¸ine u¸sor prin induct¸ie c˘a xn > 0, ∀n ∈ IN. Scriem relat¸ia de recurent¸˘a pentru (xn ) pentru k ∈ 1, n ¸si prin adunare obt¸inem xn+1 = x1 + (

1 1 1 + + ... + ). x1 x2 xn

Ridicˆand la p˘atrat relat¸ia de recurent¸˘ a, avem x2n+1 = x2n + x12 + 2. Scriem ¸si n aceast˘a relat¸ie pentru k ∈ 1, n ¸si prin adunare obt¸inem 1 1 1 + 2 + ... + 2 ) + 2n = 2 xn x1 x2 1 1 1 = (x21 + 2 ) + ( 2 + ... + 2 ) + 2n ≥ 2n + 2. x x1 x2 n

x2n+1 = x21 + (

102 Prin urmare x2n > 2n, ∀n ≥ 2, adic˘ a 1 ≤ dedus˘a prin adunare, avem

√xn . 2n

Revenind la prima relat¸ie

n

xn+1 = 2 +

X 1 1 1 √ = + ... + ≤2+ x2 xn 2k k=2

n n X X 1 1 1 2 √ ≤2− √ + √ =2− √ + = √ 2 k=1 2k 2 k=1 2k + 2k − 2 n √ X √ √ 1 1 =2− √ + ( 2k − 2k − 2) = 2 − √ + 2n. 2 k=1 2

√ √ Prin urmare xn+1 ≤ 2− √12 + 2n, ∀n ≥ 1, adic˘a xn ≤ 2− √12 + 2n − 2, ∀n ≥ 2, deci √ 2 − √12 + 2n − 2 xn √ ≤ √ → 1. 2n 2n n Din teorema cle¸stelui obt¸inem c˘a √x2n → 1. 2.34 Dac˘a a = 1, atunci ¸sirul (xn ) este o progresie aritmetic˘a de rat¸ie b ¸si prin urmare, xn = x1 + (n − 1)b. Acest ¸sir are limita x1 , dac˘a b = 0 ¸si limita (sgn b)(+∞) ˆın caz contrar. Dac˘a a 6= 1, atunci xn−1 = axn−2 + b, de unde deducem xn − xn−1 = a(xn−1 − xn−2 ) = a2 (xn−2 − xn−3 ) = ... = = an−2 (x2 − x1 ), ∀n ≥ 2. Deci x2 − x1 = x2 − x1 x3 − x2 = a(x2 − x1 ) x4 − x3 = a2 (x2 − x1 ) ... xn − xn−1 = an−2 (x2 − x1 ). Adunˆand aceste egalit˘a¸ti obt¸inem ˆın final xn = an−1 x1 +

an−1 − 1 b. a−1

Convergent¸a se studiaz˘a acum ˆın funct¸ie de a, b, x1 folosind limita (L1) din observat¸ia 2.34.

103 2.35 Vom tranforma relat¸ia de recurent¸˘ a pentru a reduce problema la problema precedent˘a. Mai ˆıntˆai observ˘am c˘a dac˘a b = 0 suntem ˆıntr-un caz particular al problemei precedente. Presupunem b 6= 0. Scriem relat¸ia de recurent¸˘a sub forma xn − axn−1 − bxn−2 = 0 ˆın care ˆınlocuim a = α + β, b = −αβ (i.e. α, β sunt solut¸iile, posibil complexe, ale ecuat¸iei x2 − ax − b = 0). Obt¸inem xn − αxn−1 − βxn−1 + αβxn−2 = 0, adic˘a xn − αxn−1 = β(xn−1 + αxn−2 ), ∀n ≥ 2. Luˆand yn = xn −αxn−1 avem yn = βyn−1 , ∀n ≥ 2. Obt¸inem c˘a yn = β n−1 y1 , adic˘a xn − αxn−1 = β n−1 y1 , ∀n ≥ 1. Fie ¸sirul zn =

xn βn

(remarc˘am c˘a β 6= 0 pentru c˘a b 6= 0). Atunci zn =

α y1 zn−1 + , ∀n ≥ 1. β β

Dac˘a α 6= β, din problema precedent˘ a avem µ ¶n−1 ( αβ )n−1 − 1 y1 α zn = z1 + α β β β −1 de unde deducem xn =

αn − β n αn−1 − β n−1 x1 − αβ x0 , ∀n ≥ 1. α−β α−β

Dac˘a α = β ˆın relat¸ia de mai sus scriem ˆın loc de

αn −β n α−β

αn−1 + αn−2 β + ... + αβ n−1 + β n−1 n−1

n−1

−β ¸si analog pentru α α−β . Convergent¸a se studiaz˘a ˆın funct¸ie de a, b, x1 , x0 folosind limita (L1) din observat¸ia 2.34. Concluzionˆand, termenul general al ¸sirului are forma de mai sus, unde α si β sunt solut¸iile ecuat¸iei x2 − ax − b = 0, numit˘ a ecuat¸ia caracteristic˘a a ¸sirului.

2.36

Folosim execit¸iul precedent.

104 a) Ecuat¸ia caracteristic˘ a a ¸sirului lui Fibonacci este x2 − x − 1 = 0, de √ √ unde obt¸inem α = 1+2 5 , β = 1−2 5 . Din exercit¸iul precedent, obt¸inem "à √ !n à √ !n # 1 1+ 5 1− 5 xn = √ − , ∀n ∈ IN. 2 2 5 b) Ecuat¸ia caracteristic˘a asociat˘a ¸sirului (xn ) este x2 − 2x + 1 = 0, deci α = β = 1. Obt¸inem xn = (αn−1 + αn−2 β + ... + αβ n−1 + β n−1 )x1 − − (αn−2 + αn−3 β + ... + αβ n−3 + β n−2 )x0 = = n + 1, ∀n ∈ IN. c) Ecuat¸ia caracteristic˘a asociat˘a ¸sirului (xn ) este x2 − x + 1 = 0, deci 2π 2π α = cos 2π si β = cos 2π ın forma 3 + i sin 3 ¸ 3 − i sin 3 (am scris numerele ˆ trigonometric˘a pentru a putea efectua mai u¸sor calculele). Obt¸inem ¸ · 2(n − 1)π 2nπ 2 √ − i sin , ∀n ∈ IN. sin xn = 3 3 3 d) x0 = 3, x1 = 4, (n + 1)(n + 2)xn = 4(n + 1)(n + 3)xn−1 − 4(n + 2)(n + 3)xn−2 , ∀n ≥ 2. ˆImpart¸ind relat¸ia de recurent¸˘ a prin (n + 1)(n + 2)(n + 3), obt¸inem xn−1 xn−2 xn =4 −4 n+3 n+2 n+1 xn ¸si notˆand yn = n+3 , avem yn = 4yn−1 − 4yn−2 , ∀n ≥ 2. Ecuat¸ia caracteristic˘a asociat˘a ¸sirului (yn ) este x2 − 4x + 4 = 0, deci α = β = 2. Efectuˆand calculele obt¸inem yn = 2n−1 (−n + 2), ∀n ∈ IN, deci yn = (n + 3)(2 − n)2n−1 , ∀n ∈ IN 2.37 Este clar c˘a termenii ambelor ¸siruri sunt pozitivi. Dac˘a x0 = y0 , atunci se constat˘a f˘ar˘ a dificultate c˘a ¸sirurile sunt constante ¸si problema este rezolvat˘a. Presupunem acum c˘a y0 < x0 . Folosim urm˘atoarele√inegalit˘a¸ti cunoscute (¸si u¸sor de probat): ∀a > b > 0, b < a+b ab < b ¸si 2 < a, b
x0 se trateaz˘a analog. Pentru am˘anunte privind limita comun˘ a precum ¸si pentru generaliz˘ari ale acestei probleme a se vedea cartea “Priveli¸sti matematice”, a autorului Isaac J. Schoenberg [16]. 2.38 (i) Pentru ˆınceput s˘a observ˘am c˘a nu putem preciza pozit¸ia lui x1 = f (x0 ) fat¸˘a de x0 , a¸sa c˘a vom considera dou˘a cazuri. Dac˘a x1 ≥ x0 , atunci folosind monotonia funct¸iei putem scrie x2 = f (x1 ) ≥ f (x0 ) = x1 . Inductiv se arat˘a c˘a (xn ) este cresc˘ator. Dac˘a x1 ≤ x0 , atunci se arat˘a c˘a ¸sirul este descresc˘ator. Evident, din definit¸ia funct¸iei f tot¸i termenii ¸sirului (xn ) sunt ˆın intervalul [a, b], deci ¸sirul este m˘arginit ¸si prin urmare convergent. Notˆand limita sa cu x, trecˆand la limit˘a ˆın relat¸ia de recurent¸˘ a ¸si folosind continuitatea funct¸iei f, obt¸inem c˘a x = f (x), adic˘a limita lui (xn ) este un punct fix al lui f . (ii) Cum f este descresc˘atoare, f ◦ f este cresc˘atoare. Evident, x2n = (f ◦ f )(x2n−2 ), ∀n ≥ 1 ¸si x2n+1 = (f ◦ f )(x2n−1 ), ∀n ≥ 1. ˆIn plus, dac˘a x2 ≥ x0 , atunci x3 ≤ x1 , iar dac˘a x2 ≤ x0 , atunci x3 ≥ x1 . Aplicˆand punctul precedent sub¸sirurilor (x2n ) si (x2n+1 ) obt¸inem c˘a ele sunt monotone de monotonii diferite ¸si convergente. Evident, pentru ca ¸sirul (xn ) s˘a fie convergent trebuie ca cele dou˘a limite s˘a fie egale. Studiem cazurile particulare. (a) x0 = 1, xn+1 = sin xn , ∀n ≥ 0. Consider˘am funct¸ia f : [0, 1] → [0, 1], f (x) = sin x. Acest˘a funct¸ie este continu˘ a ¸si cresc˘atoare pe intervalul de definit¸ie. Aplicˆand punctul (i), obt¸inem c˘a ¸sirul este monoton descresc˘ator (x1 = sin 1 < 1 = x0 ) ¸si convergent la solut¸ia ecuat¸iei sin x = x care este 0. (b) x0 = 12 , xn+1 = (xn − 1)2 , ∀n ≥ 0. Consider˘ am funct¸ia f : [0, 1] → 2 [0, 1], f (x) = (x − 1) . Acest˘a funct¸ie este continu˘ a ¸si descresc˘atoare pe intervalul de definit¸ie. Aplicˆand punctul (ii), obt¸inem c˘a sub¸sirul (x2n ) 9 este monoton cresc˘ator (x2 = 16 > x0 ), iar sub¸sirul (x2n+1 ) este monoton 1 49 descresc˘ator (x1 = 4 > x3 = 256 ). Ambele sub¸siruri converg la limitele notate a ¸si respectiv b. Constat˘am c˘a a 6= b pentru c˘a a > 21 ¸si b < 41 . S¸irul (xn ) este divergent. Cum x2n+1 = (x2n −1)2 ¸si x2n+2 = (x2n+1 −1)2 obt¸inem b = (a − 1)2 ¸si a = (b − 1)2 . F˘acˆ and calculele ¸si ¸tinˆ and cont de inegalit˘a¸tile de mai sus se obt¸ine a = 1, b = 0.

106 (c) Se procedeaz˘a ca la punctul (a). S˘a mai remarc˘am c˘a ¸sirurile de la exercit¸iile 2.27 ¸si 2.28 puteau fi tratate ¸si prin metoda expus˘a la acest exercit¸iu. 2.39 Studiem convergent¸a ¸sirurilor de la acest exercit¸iu folosind teorema lui Cauchy: demonstr˘am c˘a ¸sirurile sunt fundamentale. a) Fie ε > 0; trebuie s˘a determin˘am un num˘ ar natural nε astfel ˆıncˆ at pentru orice n ≥ nε ¸si orice p ∈ IN, s˘a aib˘a loc relat¸ia |xn+p − xn | < ε. Evalu˘am diferent¸a |xn+p − xn | : ¯ ¯ 1 1 1 + ...+ |xn+p − xn | = ¯¯1 + 2 + ... + 2 + 2 n (n + 1)2 ¯ 1 1 1 ¯¯ + − 1 − 2 − ... − 2 ¯ = (n + p)2 2 n 1 1 1 = + + ... + . 2 2 (n + 1) (n + 1) (n + p)2 Pentru a putea continua calculele facem majorarea 1 k , ∀k ≥ 2. Astfel

1 k2


0 astfel ˆıncˆat pentru orice n ∈ IN, |an | ≤ M. Fie ε > 0 ¸si n, m ∈ IN. Avem ¯ ¯ a1 a2 an an+1 |xn+m − xn | = ¯¯ p + p + ... + p + + ...+ 1 2 n (n + 1)p ¯ a1 a2 an ¯¯ an+m − − p − ... − p ¯ = + (n + m)p 1p 2 n ¯ ¯ ¯ ¯ an+1 a n+m ¯ + ... + = ¯¯ ≤ (n + 1)p (n + m)p ¯ ¯ ¯ ¯ ¯ ¯ an+1 ¯ ¯ an+m ¯ ¯ ¯= ¯ ¯ ≤¯ + ... + ¯ (n + 1)p ¯ (n + m)p ¯ |an+m | |an+1 | + ... + ≤ = p (n + 1) (n + m)p M M M ≤ + + ... + ≤ (n + 1)p (n + 2)p (n + m)p M M M ≤ + + ... + = 2 2 (n + 1) (n + 2) (n + m)2 1 1 1 = M( + + ... + )< 2 2 (n + 1) (n + 2) (n + m)2 1 M 1 )< , ∀m ∈ IN < M( − n n+m n ¸si concluzia rezult˘a. e) Proced˘am ca la punctele precedente.

108 f) Fie ε > 0 ¸si n, p ∈ IN. Atunci ¯ ¯ ¯ ¯ 1 1 1 n+2 n+3 n+p+1 ¯= |xn+p − xn | = ¯¯(−1) + (−1) + ... + (−1) n+1 n+2 n + p¯ ¯ ¯ ¯ 1 ¯ 1 p−1 1 ¯ ¯ − + ... + (−1) =¯ . n+1 n+2 n + p¯ Dac˘a p este par, atunci 1 1 1 1 1 −( − ) − ... − ( − )− n+1 n+2 n+3 n+p−2 n+p−1 1 1 1 − < < , n+p n+1 n

|xn+p − xn | =

iar dac˘a p este impar, |xn+p −xn | =

1 1 1 1 1 1 −( − ) − ... − ( − )< . n+1 n+2 n+3 n+p−1 n+p n

Din nou ¸sirul este fundamental, deci convergent. g) Folosind faptul c˘a arctg n ∈ [0, π2 ), ∀n ∈ IN problema se reduce la punctul b). 2.40 Este suficient s˘a demonstr˘am c˘a ¸sirurile nu sunt fundamentale, adic˘a ∃ε > 0 a.ˆı. ∀n ∈ IN∗ , ∃p ∈ IN a.ˆı. |xn+p − xn | ≥ ε. a) Pentru ¸sirul xn = 1 + 12 + ... + n1 avem |xn+p − xn | =

1 1 p 1 + + ... + > . n+1 n+2 n+p n+p

Luˆand ε = 12 , n ∈ IN∗ arbitrar ¸si p = n, obt¸inem |xn+p − xn | ≥ ε. S¸irul nu este fundamental, deci este divergent. ˆIn plus, pentru c˘a ¸sirul este strict cresc˘ator, putem trage concluzia c˘a are limita +∞. b) ˆIn cazul ¸sirului xn = 1 + √12 + ... + √1n putem s˘a proced˘am ca la punctul precedent sau s˘a constat˘am c˘a termenii acestui ¸sir sunt mai mari decˆat termenii corespunzatori ai ¸sirului precedent, deci ¸sirul are limita +∞. 2.41 S¸irul (cn ) este convergent, deci majorat: ∃M > 0 a.ˆı. cn < M, ∀n ∈ IN. ˆIn plus, an cn → ac, iar (an ) este ¸sir fundamental. Prin urmare, ∀ε > ε 0, ∃n0 ∈ IN a.ˆı. ∀p ∈ IN, |an cn − ac| < 2ε ¸si |an+p − an | < 2M . Putem scrie

109 (pentru n ≥ n0 ) ε = 2 ε = |an+1 bn+1 + ... + a2n b2n − an bn+2 − ... − an b2n | + ≤ 2 ε ≤ bn+1 |an+1 − an | + ... + b2n |a2n − an | + < 2 ε ε ε ε < (bn+1 + ... + b2n ) + ≤ M+ =ε 2M 2 2M 2 ¸si demonstrat¸ia este ˆıncheiat˘a. |xn − ac| ≤ |xn − an cn | + |an cn − ac| < |xn − an cn | +

2.42 a) Se arat˘a f˘ar˘a dificultate c˘a ¸sirul este strict descresc˘ator. Deci inf xn = lim xn = 1, iar sup xn = x0 = 2.

n∈IN

n∈IN

b) Pentru a studia ¸sirul ˆıl vom partit¸iona ˆın mai multe sub¸siruri convergente. T ¸ inˆand cont c˘a funct¸ia sin are perioada 2π vom studia separat sub¸sirurile (x4k ) , (x4k+1 ) , (x4k+2 ) , (x4k+3 ) . Este clar c˘a orice termen al ¸sirului este termen al unuia dintre aceste sub¸siruri. Astfel x4k = 1 + 0 = 1. Evident, inf x4k = sup x4k = 1. 1 (4k+1)2

k∈IN sin(2kπ + π2 )

k∈IN

1 x4k+1 = + sir este strict des= (4k+1) 2 + 1. Acest ¸ cresc˘ator ¸si are limita 1. Deci inf x4k+1 = 1 ¸si sup x4k+1 = x1 = 2. k∈IN

k∈IN

x4k+2 = 1 + sin(2kπ + π) = 1; inf x4k+2 = sup x4k+2 = 1. (Puteam s˘a k∈IN

k∈IN

trat˘am ˆıntr-un singur sub¸sir (x2k ) sub¸sirurile (x4k ) ¸si (x4k+2 )). 1 1 3π x4k+3 = (4k+3) sir este 2 + sin(2kπ + 2 ) = (4k+3)2 − 1; evident, acest ¸ strict descresc˘ator ¸si are limita −1. Deci inf x4k+3 = −1 ¸si sup x4k+3 = k∈IN

− 98 .

k∈IN

Din problema 1.59 putem trage concluzia c˘a inf xn este minimul x3 = infimumurilor sub¸sirurilor considerate, adic˘a −1, iar sup xn este maximul supremumurilor considerate, adic˘a 2. c) Proced˘am la fel: ˆımp˘art¸im ¸sirul ˆın dou˘a sub¸siruri (x2n ) ¸si (x2n+1 ). Se obt¸ine inf x2n = 1 ¸si sup x2n = 2, iar inf x2n+1 = 12 ¸si sup x2n+1 = 1. Deci inf xn = 21 ¸si sup xn = 2. 2.43 Metoda de determinare a celor dou˘a limite extreme pe care o vom folosi este urm˘atoarea: partit¸ion˘ am ¸sirul ˆın sub¸siruri convergente ¸si apoi alegem dintre aceste limite pe cea mai mare care va fi limita superioar˘a ¸si pe cea mai mic˘a care va fi limita inferioar˘a. a) T ¸ inˆand cont de periodicitatea funct¸iei cos ¸si de numitorul 3 vom ˆımp˘art¸i ¸sirul ˆın cele 6 sub¸siruri corespunz˘atoare claselor de resturi modulo 6. Astfel

110 x6k = cos(2kπ) = 1 → 1; x6k+1 = cos(2kπ + π3 ) = 12 → 12 ; x6k+2 = 1 1 cos(2kπ + 2π 3 ) = − 2 → − 2 ; x6k+3 = cos(2kπ + π) = −1 → −1; x6k+4 = 4π 1 1 1 1 cos(2kπ + 3 ) = − 2 → − 2 ¸si x6k+5 = cos(2kπ + 5π 3 ) = 2 → 2 . Prin urmare lim sup xn = 1 ¸si lim inf xn = −1. b) Consider˘am sub¸sirurile (x2n ) ¸si (x2n+1 ) care au limitele 1 ¸si respectiv −1. Deci lim sup xn = 1 ¸si lim inf xn = −1. c) Consider˘am sub¸sirurile (x4n ), (x4n+1 ), (x4n+2 ) ¸si (x4n+3 ). Obt¸inem lim sup xn = 21 ¸si lim inf xn = − 12 . d) Consider˘am sub¸sirurile (x4n ), (x4n+1 ), (x4n+2 ) ¸si (x4n+3 ). De exemplu sub¸sirul (x4k ) are limita 1 + e. n e) xn = (−1)[ 3 ] + sin nπ ¸iile posibile trebuie 4 . Pentru a epuiza toate situat s˘a consider˘am sub¸sirurile (x12k ), ..., (x12k+11 ). 2.44 Fie x = 0; luˆand de exemplu sub¸sirul (x5n )n , acest sub¸sir este format doar din termeni nuli, deci limita sa este 0; prin urmare, 0 ∈ L((xn )). Fie x ∈ (0, ∞). Dac˘a x ∈ Q, atunci exist˘a m, k ∈ IN∗ astfel ˆıncˆ at x = m k. Sub¸sirul (x2kp 3mp )p este constant x, deci are limita x. Prin urmare, (0, ∞) ∩ Q ⊂ L((xn )). Fie acum x ∈ (0, ∞)\Q. Conform teoremei 2.35, exist˘a un l si (kl ) sunt ¸siruri ¸sir de numere rat¸ionale (rl )l = ( m kl )l → x, unde (ml ) ¸ de numere naturale nenule. Sub¸sirul (x2kl 3ml )l coincide cu ¸sirul (rl ), deci are limita x. Obt¸inem c˘a [0, ∞) ⊂ L((xn )). Cum ¸sirul (xn ) este format din numere nenegative, nu poate cont¸ine un sub¸sir cu limita negativ˘a ¸si deci L((xn )) ⊂ [0, ∞), de unde rezult˘a egalitatea. Rat¸ionˆand ca mai sus se poate ar˘ata c˘a ¸sirul (m , dac˘a exist˘a m, k ∈ IN∗ , 4k ≤ m ≤ 5k astfel ˆıncˆ at n = 2k 3m yn = k 1, ˆın rest satisface condit¸ia L((yn )) = {1} ∪ [4, 5]. 2.45 Fie x ∈ L((xn )); conform definit¸iei, exist˘a un sub¸sir (xnk )k convergent la x. Atunci (xnk − ynk )k este sub¸sir al lui (xn − yn )n , deci are limita 0. Prin urmare lim ynk = x, deci x ∈ L((yn )). Am demonstrat c˘a L((xn )) ⊂ k→∞

L((yn )). Schimbˆand rolurile celor dou˘a ¸siruri, obt¸inem incluziunea invers˘ a, deci egalitatea. Cum lim inf xn = min L((xn )) ¸si lim sup xn = max L((xn )) (vezi propozit¸ia 2.31) rezult˘a imediat ¸si egalitea limitelor extreme. 2.46

Se rat¸ioneaz˘a ca la exercit¸iul precedent.

2.47

a) Pornim de la definit¸ie: L = lim sup xn = inf sup xk . Fie c > L, n∈IN k≥n

adic˘a c > inf sup xk . Din definit¸ia marginii inferioare, rezult˘a c˘a exist˘a nc ∈ n∈IN k≥n

111 IN astfel ˆıncˆat c > sup x, deci c > xn , ∀n ≥ nc . Fie c < L; rezult˘a c˘a pentru k≥nc

orice n ∈ IN, sup xk > c, deci exist˘a k > n astfel ˆıncˆ at xk > c. Am demonstrat k≥n

ambele propriet˘a¸ti. Invers, presupunem c˘a L satisface propriet˘a¸tile. Prima dintre ele ˆınseamn˘a c˘a pentru orice c > L, c ≥ sup x ≥ inf sup xk , deci k≥nc

n∈IN k≥n

L ≥ lim sup xn , iar a doua ˆınseamn˘ a c˘a pentru orice c < L, c ≤ sup xk k≥n

pentru orice n, deci c ≤ inf sup xk ¸si prin urmare L ≤ lim sup xn . Obt¸inem n∈IN k≥n

egalitatea. b) Se procedez˘a la fel. +1 2.48 Not˘am yn = n( xn+1 − 1) ¸si presupunem prin reducere la absurd c˘a xn lim sup yn < 1. Din caracterizarea limitei superioare (exercit¸iul precedent) rezult˘a c˘a exist˘a n0 ∈ IN∗ astfel ˆıncˆ at pentru orice n ≥ n0 , yn < 1. F˘ acˆ and calculele obt¸inem c˘a pentru n ≥ n0 , n < (n + 1)xn − nxn+1 . Prin ˆınmult¸ire 1 cu n(n+1) obt¸inem

1 xn xn+1 < − , ∀n ≥ n0 . n+1 n n+1 Scriind succesiv aceast˘a relat¸ie pentru k ∈ n0 , n ¸si sumˆand obt¸inem 1 1 xn xn+1 xn 1 + + ... + < 0 − < 0. n0 + 1 n0 + 2 n n0 n+1 n0 Cum lim ( n01+1 + n01+2 + ... + n1 ) = ∞, relat¸ia de mai sus este absurd˘a, deci n→∞ presupunerea facut˘a este fals˘a. 2.49 a) Fie (xnk + ynk )k un sub¸sir al ¸sirului (xn + yn )n convergent la lim inf(xn + yn ) (conform propozit¸iei 2.31 un asemenea sub¸sir exist˘a). Sub¸sirul (xnk ) este m˘arginit ¸si prin urmare admite un sub¸sir convergent, (xnkp )p . Evident lim xnkp ≥ lim inf xn . Sub¸sirul (ynkp )p este de asemenea convergent pentru c˘a (xnkp +ynkp )p este sub¸sir al ¸sirului (xnk +ynk )k , deci xnkp +ynkp → lim inf(xn + yn ). Din nou lim ynkp ≥ lim inf yn . Prin urmare lim inf(xn + yn ) = lim(xnkp + ynkp ) = lim xnkp + lim ynkp ≥ ≥ lim inf xn + lim inf yn . Prima inegalitate este dovedit˘a. Inegalitatea a doua este evident˘ a. Cea de-a treia inegalitate se arat˘a la fel ca prima. Demonstrat¸i ¸si folosind definit¸iile limitelor extreme.

112 b) Proced˘am ˆıntocmai ca la punctul precedent. c) Pentru punctul a) consider˘am xn = 2 · (−1)n ¸si yn = (−1)n+1 . Cum xn + yn = (−1)n , ∀n ∈ IN, avem lim inf xn + lim inf yn = −3 < lim inf(xn + yn ) = −1 < lim sup(xn + yn ) = 1 < lim sup xn + lim supn = 3. Pentru punctul b) lu˘am xn = 2 + (−1)n ¸si yn = 1 + (−1)n+1 . Atunci xn yn = 3 + (−1)n+1 . Se constat˘a c˘a inegalit˘a¸tile sunt stricte. 2.50 Presupunem c˘a xn → x. Atunci lim inf xn = x. Din exercit¸iul precedent ¸stim c˘a x + lim inf yn ≤ lim inf(xn + yn ). Demonstr˘ am inegalitatea invers˘a. Fie (ynk )k → lim inf yn . Atunci (xnk + ynk ) → x + lim inf yn ≥ lim inf(xn + yn ) pentru c˘a limita inferioar˘a a unui ¸sir este minimul mult¸imii punctelor limit˘a ale acelui ¸sir. A¸sadar x + lim inf yn = lim inf(xn + yn ). Invers, ¸stim c˘a pentru orice ¸sir (yn ) are loc lim inf xn +lim inf yn = lim inf(xn + yn ), deci, ˆın particular, are loc egalitatea ¸si pentru ¸sirul (−xn ). Obt¸inem lim inf xn + lim inf(−xn )=0, adic˘ a lim inf xn − lim sup xn =0, deci lim inf xn = lim sup xn , deci ¸sirul este convergent. Pentru lim sup se procedeaz˘a analog. O proprietate asem˘an˘ atoare are loc ¸si pentru ˆınmult¸ire, corespunz˘ator punctului b) din exercit¸iul precedent. Scrierea ¸si demonstrarea acestei propriet˘a¸ti este l˘asat˘a pe seama cititorului. 2.51 Toate inegalit˘a¸tile rezult˘a direct din definit¸iile limitelor extreme. Astfel, de exemplu, lim sup xn = inf sup xk ≤ sup xk , ∀n ∈ IN. Pentru n = 0 n∈IN k≥n

k≥n

inegalit˘a¸tile devin inf xn ≤ lim inf xn ≤ lim sup xn ≤ sup xn . Pentru a dovedi c˘a aceste inegalit˘a¸ti pot fi stricte consider˘am urm˘atorul en−2 xemplu: xn = (−1)n 2n−3 . Se arat˘a ca la exercit¸iile precedente c˘a lim inf xn = 1 1 − 2 , lim sup xn = 2 , sup xn = x0 = 23 , iar inf xn = x1 = −1. 2.52 a) Pentru ˆınceput presupunem c˘a lim sup xn = L < +∞. Din caracterizarea limitei superioare (exercit¸iul 2.45), avem ∀c > L, ∃nc ∈ IN, a.ˆı. ∀n ≥ nc , xn < c. Fie c > L fixat. Lu˘am M = max{c, x0 , x1 , ..., xnc −1 }. Atunci xn ≤ M, ∀n ∈ IN, adic˘a ¸sirul este majorat. Invers, presupunem c˘a ¸sirul este majorat, adic˘a sup xn < ∞. Din problema precedent˘a, lim sup xn ≤ sup xn < ∞ ¸si demonstrat¸ia este ˆıncheiat˘ a. b) Proced˘am analog.

113 2.53 Fie y = inf yn ∈ IR ∪ {−∞}. Dac˘ a y > −∞, conform caracteriz˘arii marginii inferioare, ∀ε > 0, ∃m ∈ IN∗ a.ˆı.

xm < y + ε. m

Pentru ε fixat, fie n ≥ m. Din teorema ˆımp˘ art¸irii cu rest exist˘a q, r ∈ IN astfel ˆıncˆat n = mq + r, r ∈ {0, 1, 2, ..., m − 1}. Avem xn = xmq+r ≤ xmq + xr ≤ qxm + xr . Obt¸inem xn qxm + xr xm mq xr xm mq xr xr < = + = + 0 rezult˘a c˘a exist˘a lim yn = y. Dac˘a y = −∞, atunci ∀ε > 0, ∃m ∈ IN∗ a.ˆı. xmm < −ε. Ca mai sus se arat˘a c˘a xr xn < −ε + , ∀n ≥ m, n n deci lim sup yn ≤ −ε, ∀ε > 0. Prin urmare, lim sup yn = −∞, deci exist˘a lim yn = −∞. 2.54 Presupunem c˘a a) este adev˘arat˘ a, adic˘a sup A = inf B. Din caracterizarea marginii superioare ¸si marginii inferioare deducem c˘a pentru orice num˘ar natural nenul exist˘a xn ∈ A astfel ˆıncˆ at sup A −

1 < xn . n

¸si yn ∈ B astfel ˆıncˆat yn < inf B + Astfel sup A −

1 . n

1 1 < xn ≤ yn < inf B + , ∀n ∈ IN∗ , n n

deci 0 ≤ yn − xn < ceea ce implic˘a yn − xn → 0.

2 n

114 Presupunem c˘a este adev˘arat˘ a condit¸ia b). Din ipoteza asupra mult¸imilor A ¸si B avem sup A ≤ inf B. Dac˘a sup A < inf B, atunci pentru orice (xn ) ⊂ A ¸si pentru orice (yn ) ⊂ B avem yn − xn ≥ inf B − sup A > 0, ∀n ∈ IN. Cum inf B−sup A este o constant˘ a, nu putem avea yn −xn → 0, contradict¸ie.

Capitolul 3 SERII NUMERICE

Definit¸ia 3.1. Fie (an )n≥n0 (n0 ∈ IN) un ¸sir de numere reale ¸si (sn )n≥n0 ¸sirul definit prin: sn0 = an0 , sn0 +1 = an0 + an0 +1 , sn0 +2 = an0 + an0 +1 + an0 +2 , ....................................... sn = an0 + an0 +1 + ... + an , n ∈ IN, n ≥ n0 . Se nume¸ste serie de numere reale (sau serie numeric˘ a) perechea {(an )n , (sn )n } format˘ a din ¸sirurile (an )n ¸si (sn )n . an se nume¸ste termenul general al seriei, iar (sn )n se nume¸ste ¸sirul sumelor part¸iale asociat ¸sirului (an )n . Vom nota seria prin: ∞ P P an , an sau an0 + an0 +1 + an0 +2 + ... + an + .... n=n0

n≥n0

Definit¸ia 3.2. Fie sumelor part¸iale. a) Seria

∞ P n=n0

∞ P n=n0

an (n0 ∈ IN) o serie numeric˘ a ¸si (sn ) ¸sirul

an se nume¸ste convergent˘ a (sau seria

∞ P n=n0

an converge) dac˘a

(sn ) este convergent. ˆIn acest caz, limita ¸sirului (sn ) se nume¸ste suma ∞ P seriei ¸si se noteaz˘a prin an . n=n0

115

116 b) Seria

∞ P n=n0

an se nume¸ste divergent˘ a (sau seria

∞ P n=n0

an diverge) dac˘a

nu este convergent˘ a deci dac˘a (sn ) este divergent. Dac˘a limita ¸sirului (sn ) este +∞ sau −∞, atunci se spune c˘a suma seriei este +∞ sau ∞ ∞ P P −∞ ¸si se noteaz˘a an = +∞ sau an = −∞. n=n0

n=n0

Observat¸ia 3.3. (Serii remarcabile) a) Seria geometric˘a de rat¸ie q:

∞ P

q n , q ∈ IR (q 0 = 1).

n=0

Dac˘a q ∈ (−∞, −1], atunci seria este divergent˘ a. Dac˘a q ∈ (−1, 1), atunci seria este convergent˘ a ¸si are suma

∞ P

qn =

n=0

1 1−q .

Dac˘a q ≥ 1, atunci seria este divergent˘ a ¸si are suma b) Seria armonic˘a generalizat˘a:

∞ P n=1

∞ P

q n = +∞.

n=0 1 nα , α

∈ IR.

Dac˘a α > 1, atunci seria este convergent˘ a. Dac˘a α ≤ 1, atunci seria este divergent˘ a. Pentru α = 1, seria

n=1

nume¸ste seria armonic˘a. Observat¸ia 3.4. Fie seria

∞ P

∞ P n=0

1 n

se

α cu termen general an = α ∈ IR, ∀n ∈ IN

(¸sirul (an ) este constant). Dac˘a α = 0, atunci seria este convergent˘ a ¸si are suma 0. Dac˘a α 6= 0, atunci seria este divergent˘ a. Definit¸ia 3.5. Fie (bn )n≥n0 (n0 ∈ IN) un ¸sir de numere reale. O serie ∞ P de forma (bn − bn−1 ) se nume¸ste serie telescopic˘a. ˆIn acest caz, ¸sirul n=n0 +1

sumelor part¸iale este sn = bn − bn0 , ∀n ∈ IN∗ , n ≥ n0 + 1. Definit¸ia 3.6. Dac˘a dou˘a serii

∞ P n=n0

an ¸si

∞ P n=n0

bn (n0 ∈ IN) au aceea¸si

natur˘a (adic˘a sunt ˆın acela¸si timp convergente sau divergente), atunci vom ∞ ∞ P P nota an ∼ bn . n=n0

n=n0

117 Teorema 3.7. (Condit¸ia necesar˘ a de convergent¸˘ a) ∞ P Dac˘a seria an (n0 ∈ IN) este convergent˘ a, atunci lim an = 0. n→∞

n=0

ˆIn consecint¸˘a, rezult˘a: Corolar 3.8. Dac˘a (an )n este convergent ¸si lim an 6= 0 sau dac˘a (an ) n→∞ ∞ P an este este divergent (ceea ce vom nota prin an 9 0), atunci seria n=n0

divergent˘a.

Propozit¸ia 3.9. (Propriet˘ a¸ti generale ale seriilor convergente) i) Fie seria

∞ P n=n0

an (n0 ∈ IN). Dac˘a din ¸sirul (an ) se elimin˘a sau se

adaug˘a un num˘ar finit de termeni, atunci natura seriei nu se schimb˘ a (dar ˆın caz de convergent¸˘a, sumaPseriei se modific˘a). Astfel, vom face convent¸ia de a nota o serie prin an atunci cˆand ne va interesa doar natura seriei (nu ¸si suma seriei). a se asociaz˘a termenii seriei ˆın grupe finite, ii) Dac˘a ˆıntr-o serie convergent˘ cu p˘astrarea ordinii termenilor, atunci se obt¸ine tot o serie convergent˘ a ¸si cu aceea¸si sum˘a. Dac˘a seria este divergent˘ a, atunci rezultatul nu se ∞ P mai p˘astreaz˘a. De exemplu, fie seria divergent˘ a (−1)n−1 ¸si seria: n=1

(1)

[1 + (−1)] + [1 + (−1)] + ... + [1 + (−1)] + ....

obt¸inut˘a prin asocierea termenilor ˆın grupe de cˆate doi termeni. Se observ˘a c˘a seria (1) este convergent˘ a ¸si are suma 0. ∞ ∞ ∞ P P P iii) Fie seria an (n0 ∈ IN) ¸si k ∈ IN∗ . Atunci an ∼ an+k . ˆIn n=n0

caz de convergent¸˘a, dac˘a

∞ P n=n0

an = s, atunci

an0 +1 + ... + an0 +k−1 ). Invers, dac˘a t + (an0 + an0 +1 + ... + an0 +k−1 ). iv) Fie

∞ P n=n0 ∞ P

∞ P n=n0

n=n0 ∞ P

n=n0

n=n0

an+k = s − (an0 +

an+k = t, atunci

∞ P n=n0

an =

an (n0 ∈ IN) o serie numeric˘ a ¸si pentru orice p ∈ IN, fie seria

n=n0 +p+1

an . Atunci

∞ P n=n0

an ∼

∞ P n=n0 +p+1

an . ˆIn caz de convergent¸˘ a, se

118 ∞ P

noteaz˘a

n=p+1

an =rp (numit restul de ordin p al seriei

∞ P n=n0

an ) ¸si avem

lim rp = 0.

p→∞

Teorema 3.10. (Teorema lui Cauchy de caracterizare) ∞ P an (n0 ∈ IN) este convergent˘ a dac˘a ¸si numai dac˘a ∀ε > 0, O serie n=n0

∃N (ε) = N ∈ IN, astfel ˆıncˆ at ∀n ≥ N, n ≥ n0 ¸si ∀p ∈ IN∗ , |an+1 + an+2 + ... + an+p | < ε. Observat¸ia 3.11. Dac˘a seria

∞ P n=n0

an (n0 ∈ IN) are proprietatea c˘a

an ≥ 0, ∀n ≥ n0 , atunci ¸sirul sumelor part¸iale este cresc˘ator. ˆIn acest caz, se spune c˘a seria este cu termeni nenegativi. P Teorema 3.12. Fie an P o serie cu termeni nenegativi ¸si (sn ) ¸sirul sumelor part¸iale. Atunci seria an este convergent˘ a dac˘a ¸si numai dac˘a (sn ) este majorat. Observat¸ia 3.13. a) O serie cu termeni nenegativi

∞ P n=n0

an (n0 ∈ IN) este divergent˘ a dac˘a

¸si numai dac˘a (sn ) este nemajorat ceea ce este echivalent cu faptul c˘a lim sn = +∞. n→∞

b) O serie cu termeni nenegativi are ˆıntotdeauna sum˘a ˆın [0, +∞]. Teorema 3.14. (Criteriul de comparat¸ie de specia I) P P Fie an ¸si bn (n0 ∈ IN) serii cu termeni nenegativi astfel ˆıncˆ at an ≤ n≥n0

bn , ∀n ≥ n0 .

n≥n0

P bn converge, atunci seria an converge. P P ii) Dac˘a seria an diverge, atunci seria bn diverge. i) Dac˘a seria

P

Teorema de comparat¸ie de specia a II-a) P 3.15. (Criteriul P Fie seriile an ¸si bn (n0 ∈ IN) astfel ˆıncˆ at an > 0, bn > 0 ¸si an+1 an ≤ bn+1 bn

n≥n0

n≥n0

pentru orice n ≥ n0 . P P i) Dac˘a seria bn este convergent˘ a, atunci seria an este convergent˘ a.

119 ii) Dac˘a seria

P

an este divergent˘ a, atunci seria

P

bn este divergent˘ a.

Teorema 3.16. a) P P (Criteriul de comparat¸ie cu limit˘ Fie an ¸si bn serii cu termeni pozitivi astfel ˆıncˆ at exist˘a limita an = λ ∈ [0, +∞]. lim n→∞ bn P P i) Dac˘a λ ∈ (0, +∞), atunci an ∼ bn . ii) Pentru λ = 0, P P • dac˘a seria bn converge, atunci seria an converge; P P • dac˘a seria an diverge, atunci seria bn diverge. iii) Pentru λ = +∞, P P • dac˘a seria an converge, atunci seria bn converge; P P • dac˘a seria bn diverge, atunci seria an diverge. Corolar 3.17. Fie seria

∞ P

an bn (n0 ∈ IN) unde an > 0, bn > 0, ∀n ≥ P P n0 . Dac˘a exist˘a limita lim bn = λ ¸si λ ∈ (0, +∞), atunci an bn ∼ an . n=n0

n→∞

Teorema 3.18. (Criteriul lui Cauchy de condensare) P Fie an o serie nenegativi astfel ˆıncˆ at ¸sirul (an ) este descresc˘aP cu termeni P tor. Atunci an ∼ 2n a2n . Teorema ad˘ acinii cu m˘ arginire) P 3.19. (Criteriul r˘ Fie seria an (n0 ∈ IN) cu termeni nenegativi. n≥n0

P √ i) Dac˘a exist˘a M < 1 astfel ˆıncˆ at n an ≤ M , ∀n ≥ n0 , atunci seria an converge. P √ ii) Dac˘a exist˘a M ≥ 1 astfel ˆıncˆ at n an ≥ M , ∀n ≥ n0 , atunci seria an diverge. Teorema 3.20. (Criteriul r˘ ad˘ acinii cu limit˘ a superioar˘ a) P an o serie cu termeni nenegativi. P √ i) Dac˘a lim sup n an < 1, atunci seria an este convergent˘ a. n→∞ √ P ii) Dac˘a lim sup n an > 1, atunci seria an este divergent˘ a.

Fie

n→∞

Teorema ad˘ acinii cu limit˘ a) P 3.21. (Criteriul r˘ √ Fie seria an cu termeni nenegativi astfel ˆıncˆ at exist˘a limita lim n an = α. n→∞

120 P i) Dac˘a α < 1, atunci seria P an converge. ii) Dac˘a α > 1, atunci seria an diverge.

Fie

Teorema 3.22. (Criteriul raportului cu m˘ arginire) P an (n0 ∈ IN) o serie cu an > 0 pentru orice n ≥ n0 . n≥n0

at i) Dac˘a exist˘a M < 1 astfel ˆıncˆ este convergent˘a. ii) Dac˘a exist˘a M ≥ 1 astfel ˆıncˆ at este divergent˘a.

Fie

an+1 an

≤ M, ∀n ≥ n0 , atunci seria

an+1 an

≥ M, ∀n ≥ n0 , atunci seria

P P

an an

Teorema 3.23. (Criteriul raportului cu limite extreme) P an (n0 ∈ IN) o serie cu an > 0 pentru orice n ≥ n0 . n≥n0

P an+1 i) Dac˘a lim sup < 1, atunci seria an converge. an n→∞ P an+1 > 1, atunci seria an diverge. ii) Dac˘a lim inf n→∞ an

Fie

Teorema 3.24. (Criteriul raportului cu limit˘ a) P an (n0 ∈ IN) o serie cu an > 0 pentru orice n ≥ n0 astfel ˆıncˆ at exist˘a n≥n0

limita lim

n→∞

an+1 = α. an

P i) Dac˘a α < 1, atunci seria P an este convergent˘ a. ii) Dac˘a α > 1, atunci seria an este divergent˘ a. Teorema 3.25. (Criteriul lui Raabe - Duhamel) P Fie an (n0 ∈ IN) o serie cu an > 0 pentru orice n ≥ n0 astfel ˆıncˆ at exist˘a n≥n0 µ ¶ an limita lim n − 1 = β. n→∞ an+1 P i) Dac˘a β > 1, atunci seria P an converge. ii) Dac˘a β < 1, atunci seria an diverge. Teorema X 3.26. (Criteriul lui Gauss) Fie seria an (n0 ∈ IN) cu an > 0 pentru orice n ≥ n0 astfel ˆıncˆ at n≥n0 an poate scrie ˆın forma: an+1 = 1+ ¸si (xn ) ⊂ IR este un ¸sir m˘arginit.

β n

+

xn , n1+α

an an+1

se

∀n ≥ n0 , unde α, β ∈ IR, α > 0

121 P i) Dac˘a β > 1, atunci seria P an converge. ii) Dac˘a β ≤ 1, atunci seria an diverge. X Corolar 3.27. (Gauss) Fie seria an (n0 ∈ IN) cu an > 0 pentru n≥n0

orice n ≥ n0 avˆand proprietatea:

an P1 (n) P2 (n) Pk (n) xn =1+ + + ··· + + 1+α , an+1 Q1 (n) Q2 (n) Qk (n) n ∀n ≥ n0 , unde Pi , Qi sunt polinoame cu coeficient¸i reali astfel ˆıncˆ at: grad Qi − grad Pi = 1, ∀i = 1, k, α ∈ (0, ∞) ¸si (xn ) ⊂ IR este un ¸sir m˘arginit. Notˆand cu bi respectiv ci , coeficientul dominant al polinomului Pi respectiv k b P i al polinomului Qi , ∀i = 1, k ¸si cu β = , avem: c i=1 i P • pentru β > 1, seria P an converge, • pentru β ≤ 1, seria an diverge. Teorema P 3.28. (Criteriul lui Dirichlet) Fie seria an bn (n0 ∈ IN) care verific˘ a condit¸iile: n≥n0

i) seria

P

n≥n0

an are ¸sirul sumelor part¸iale (sn ) m˘arginit (adic˘a exist˘a α ≥

0 astfel ˆıncˆat |sn | ≤ α, ∀n ≥ n0 ), ii) (bn ) este un ¸sir descresc˘ator cu lim bn = 0. n→∞

X Atunci seria an bn este convergent˘ a.

Teorema 3.29. (Criteriul lui Abel) P Fie an bn o serie pentru care au loc afirmat¸iile: P i) seria an este convergent˘a, ii) (bn ) este un ¸sir monoton ¸si m˘arginit. P Atunci seria an bn este convergent˘ a. P Definit¸ia 3.30. O serie numeric˘ a an (n0 ∈ IN) se nume¸ste serie alternat˘a dac˘a

n≥n0

an · an+1 < 0, ∀n ≥ n0 . ˆIn acest caz, an se mai scrie ˆın forma an = (−1)n bn pentru orice n ≥ n0 sau an = (−1)n+1 bn pentru orice n ≥ n0 , unde bn > 0 pentru orice n ≥ n0 (se observ˘a c˘a bn = |an | pentru orice n ≥ n0 ).

122 Teorema 3.31. (Criteriul lui Leibniz) P n Fie (−1) bn (n0 ∈ IN) o serie alternat˘a (bn > 0, ∀n ≥ n0 ) astfel ˆıncˆ at n≥n0 P (−1)n bn este ¸sirul (bn ) este descresc˘ator ¸si lim bn = 0. Atunci seria n→∞

n≥n0

convergent˘a.

Definit¸ia 3.32. P P a) O serie an se nume¸ste absolut convergent˘ a dac˘a seria |an | este convergent˘a. P b) Seria anP se nume¸ste semiconvergent˘ a (sauPcondit¸ionat convergent˘ a) dac˘a seria an este convergent˘ a, iar seria |an | este divergent˘ a. P a o serie an este absolut convergent˘ a, atunci seria P Teorema 3.33. Dac˘ an este convergent˘ a. Observat¸ia 3.34. a) Reciproca teoremei 3.33 nu este adev˘arat˘ a (vezi problema 3.1-10). b) Dac˘ d˘ acinii pentru seria P a se aplic˘a criteriul raportului sau cel al r˘aP |an | ¸si aceasta este divergent˘ a, atunci ¸si seria an este divergent˘ a (vezi problema 3.5). Teorema P 3.35. (Produs cu un scalar) P P Fie seria an (n0 ∈ IN) ¸si λ ∈ IR. Dac˘a λ 6= 0, atunci (λan ) ∼ an n≥n0 P P ¸si ˆın caz de convergent¸˘ a avem (λan ) = λ an . n≥n0

n≥n0

Teorema a dou˘ a serii) P 3.36. (Suma P Fie seriile an ¸si bn (n0 ∈ IN). n≥n0

n≥n0

P i) Dac˘a ambele serii sunt convergente, atunci ¸ s i seria (an + bn ) este P P P convergent˘a ¸si (an + bn ) = an + bn . n≥n0P n≥n0 P n≥n0 ii) Dac˘a P seria an converge ¸si seria bn diverge (sau invers), atunci seria (an + bn ) diverge. Definit¸ia 3.37. (Produsul Cauchy al dou˘ a serii) ∞ ∞ P P Fie seriile an , bn ¸si fie (cn ) ¸sirul definit prin: n=0

n=0

c0 = a0 b0 , c1 = a0 b1 + a1 b0 ,

123 c2 = a0 b2 + a1 b1 + a2 b0 , ........................................., cn = a0 bn + a1 bn−1 + ... + an b0 = Atunci seria

∞ P n=0

n P

ak bn−k , ∀n ∈ IN.

k=0

cn se nume¸ste produs Cauchy al seriilor

∞ P n=0

an ¸si

∞ P n=0

bn .

Teorema 3.38. (Mertens) ∞ ∞ P P Fie an ¸si bn serii convergente. Dac˘a cel put¸in una dintre serii este n=0

n=0

∞ P absolut convergent˘a, atunci seria produs Cauchy, cn , este convergent˘ a ¸si n=0 ³P ´³ P ´ ∞ ∞ ∞ P cn = an bn . n=0 n=0 n=0 ´³ P ´ ³P ´2 ³P ∞ ∞ ∞ Dac˘a an = bn , ∀n ∈ IN, atunci vom nota an bn prin an .

A ridica seria ∞ P n=0

∞ P n=0

n=0

n=0

n=0

an la p˘atrat ˆınseamn˘ a a efectua produsul Cauchy al seriei

an cu ea ˆıns˘a¸si.

Corolar 3.39. (Teorema lui Cauchy) ∞ ∞ P P Dac˘a dou˘a serii an ¸si bn sunt absolut convergente, atunci seria produs n=0 n=0 ³P ´³ P ´ ∞ ∞ ∞ ∞ P P Cauchy, cn , este absolut convergent˘ a ¸si cn = an bn . n=0

n=0

n=0

n=0

Observat¸ia 3.40. (Calculul aproximativ al sumelor de serii) ∞ P Fie an o serie convergent˘a cu suma s. ˆIn acest caz, ¸sirul sumelor part¸iale n=0

sn = a0 + a1 + · · · + an converge la s iar restul de ordin n al seriei, ∞ P rn = ak = s − sn , converge la 0. Pentru determinarea cu aproximat¸ie k=n+1

a sumei s, se poate folosi formula de aproximare: s∼ = sn , fiind necesar˘a o evaluare a erorii absolute |rn | = |s − sn |. De exemplu:

¯ ¯ ¯ an+1 ¯ ¯≤λ a) Presupunem c˘a exist˘a n0 ∈ IN ¸si λ ∈ (0, 1) astfel ˆıncˆ at ¯¯ an ¯ pentru orice n ≥ n0 . Atunci |an+1 | ≤ λ|an |, |an+2 | ≤ λ|an+1 | ≤ λ2 |an |,

124 |an+3 | ≤ λ3 |an |, ·¯· · , |an+p¯| ≤ λp |an |, ∀p ∈ IN∗ . Conform problemei 3.19, ¯ P ¯ ∞ ∞ P P λ ¯ ∞ ¯ |s − sn | = |rn | = ¯ . Deci ak ¯ ≤ |ak | ≤ |an | · λk = |an | · ¯k=n+1 ¯ k=n+1 1−λ k=1 λ |s − sn | ≤ |an | · , ∀n ≥ n0 . 1−λ S˘a presupunem c˘a vrem s˘a calcul˘am s cu o aproximat¸ie dat˘a ε > 0. Pentru aceasta, vom determina N ∈ IN, N minim ¸si N ≥ n0 astfel ˆıncˆ at λ |an | · ≤ ε pentru n ≥ N . Atunci s ∼ = sN . 1−λ b) Fie (an ) un ¸sir descresc˘ator de numere pozitive, cu an → 0 astfel ˆıncˆ at ∞ P seria (−1)n an este convergent˘ a ¸si are suma s. S˘a ar˘at˘ am c˘a |rn | ≤ an+1 , n=0

∀n ∈ IN. Fixˆand n ∈ IN, pentru orice k ∈ IN∗ avem: an+1 − an+2 ≤an+1 − an+2 + · · · + (−1)k+1 an+k + (−1)k+2 an+k+1 ≤an+1 . Trecˆand la limit˘a pentru k → ∞ se obt¸ine an+1 −an+2 ≤ (−1)n+1 rn ≤ an+1 . Rezult˘a |s − sn | = |rn | ≤ an+1 , ∀n ∈ IN. Dac˘a vrem s˘a calcul˘am s cu o aproximat¸ie dat˘a ε > 0, atunci vom determina N ∈ IN, N minim astfel ˆıncˆat an+1 ≤ ε pentru orice n ≥ N . Atunci s ∼ = sN . Scrierea p−adic˘ a a numerelor reale Urm˘atoarele enunt¸uri au fost extrase din cartea d-nei prof. dr. Anca Maria Precupanu, Bazele Analizei Matematice [14]. Teorema 3.41. Fie p ∈ IN, p ≥ 2 ¸si (an )n∈IN∗ ⊂ Z cu proprietatea ∞ a P n 0 ≤ an ≤ p − 1, ∀n ∈ IN∗ . Atunci seria este convergent˘ a iar suma sa n n=1 p este un num˘ar a ∈ [0, 1]. Teorema 3.42. Fie p ∈ IN, p ≥ 2 ¸si a ∈ (0, 1]. Atunci exist˘a un ¸sir (an )n∈IN∗ ⊂ Z, cu an 6= 0 pentru o infinitate de valori ale lui n, astfel ˆıncˆ at ∞ a P n ∗ 0 ≤ an ≤ p − 1, ∀n ∈ IN ¸si = a. n n=1 p ∞ a P n din teorema precedent˘ a se nume¸ste dezn p n=1 voltare p−adic˘a a num˘ arului a. Se mai scrie a = 0,p a1 a2 a3 .... Vom spune dezvoltare diadic˘a ˆın cazul p = 2. Dac˘a p = 10, atunci se obt¸ine dezvoltarea zecimal˘a obi¸snuit˘ a a num˘ arului a.

Definit¸ia 3.43. Seria

125 Numerele din (0, 1] pot avea mai multe dezvolt˘ ari p−adice. Teorema 3.44. Fie p ∈ IN, p ≥ 2, a ∈ (0, 1] ¸si a = p−adic˘a a lui a.

∞ a P n dezvoltarea n n=1 p

Dac˘a (bn )n∈IN∗ ⊂ Z, 0 ≤ bn ≤ p − 1 pentru orice n ∈ IN∗ ¸si a = atunci exist˘a n0 ∈ IN∗ astfel ˆıncˆat:

∞ b P n , n p n=1

i) bn = an , 1 ≤ n < n0 : ii) bn0 = an0 + 1; iii) an = p − 1, bn = 0, n > n0 , adic˘a a are o singur˘a dezvoltare p−adic˘a cu elemente nenule. Mai mult, a are dou˘a dezvolt˘ari p−adice dac˘a ¸si numai dac˘a exist˘a m, n0 ∈ IN∗ astfel m ˆıncˆat a = n0 unde a < pn0 . p

126

Probleme 3.1 Stabilit¸i natura urm˘atoarelor serii folosind definit¸ia 3.2 ¸si ˆın caz de convergent¸˘a determinat¸i sumele lor: 1. 2. 3. 4. 5. 6. 7. 8. 9. 10. 11.

∞ X n=1 ∞ X

1 ; −1

4n2

1 , x > 0; (n + x)(n + x + 1) n=1 ¶ ∞ µ X 1 n+1 ; + ln n n n=1 ∞ X 1 √ ; √ n+ n+1 n=1 ∞ X 1 arctg 2 ; n +n+1 n=0 ∞ 2 X n +n−1 ; (n + 1)! n=1 ¶ ∞ µ X n 1 + ln ; n n+1 n=1 ∞ X (n − 1)! , p fixat din IN∗ ; (n + p)! n=1 ∞ X 2n + 1 p , unde (2n)!! = 2 · 4 · 6 · · · (2n); √ (2n)!!(2n + 2 + 2n + 2) n=1 ∞ (−1)n−1 P ; n n=1 ∞ X (n + 1)xn , x ∈ IR; n=0 ∞ X

xn , x ∈ IR; 3n+2 n=0 ¶ ½ ¾ ∞ µ X 2a + 3 n 1 , a ∈ IR \ . 13. 3a − 1 3 12.

n=0

3.2 Ar˘atat¸i c˘a seria aproximat¸ie de 10−2 .

∞ P n=1

n+1 n2 ·n!

este convergent˘ a ¸si calculat¸i suma seriei cu

127 3.3 Demonstrat¸i c˘a seria seriei cu aproximat¸ie de

∞ P

n=0 10−3 .

(−1)n (2n+1)3

este convergent˘ a ¸si calculat¸i suma

3.4 S˘a se studieze convergent¸a urm˘atoarelor serii folosind teoremele precizate. Corolarul 3.8: ¶n ∞ µ 2 X n −n+1 1. ; n2 + n + 1 n=1 ∞ X 5n 2. ; n3 + 5n n=1

Teorema lui Cauchy de caracterizare (3.10): 3. 4.

∞ X cos n n=0 ∞ X n=1

; 7n √ n n ; n2 + 1

Teorema 3.12: 5. 6.

∞ X n=0 ∞ X n=1

1 ; (n + 1) · 3n 1 + sin n ; + 3n + 4

n2

Criteriul de comparat¸ie de specia I (3.14): 7. 8.

∞ X

arctg

n=1 ∞ X n=1

1 ; n3

n ; n2 + cos2 n

Criteriul de comparat¸ie de specia a II-a (3.15): 9.

10.

∞ X

(2n + 1)!! √ , unde (2n + 1)!! = 1 · 3 . . . (2n + 1); (2n)!! · n + 1 n=1 ∞ X

1

1

1

(2 − e 2 )(2 − e 3 )...(2 − e n );

n=2

128 Criteriul de comparat¸ie cu limit˘a (3.16, 3.17): 11.

∞ P

n+3 ; 2n4 +1 n=1 √ ∞ 2 X n + 2n n3

√ 4 + n7 √ ; 12. √ 3 4 + n3 n + 3 n n=1 √ ∞ √ 3 X n+3− 3n+1 √ 13. ; 3 n2 + 4 n=1 Criteriul lui Cauchy de condensare (3.18): 14. 15.

∞ X n=2 ∞ X n=2

1 ; n ln n 1 ; (ln n)ln n

Criteriul r˘ad˘acinii cu m˘arginire (3.19): ¶ ∞ µ X 8n + 1 n 16. ; n3 + 2 n=1

Criteriul r˘ad˘acinii cu limit˘a superioar˘a (3.20): 17.

∞ X

an , unde an =

n=0

1 1 pentru n par ¸si an = n pentru n impar; n 3 7

Criteriul r˘ad˘acinii cu limit˘a (3.21): 18. 19.

∞ X ¡√ ¢n √ n+2− n+1 ; n=1 ∞ µ 2 X 1 n=1

+ 22 + ... + n2 n − n2 3

¶n ;

Criteriul raportului cu m˘arginire (3.22): 20.

∞ X n=0

32n ; (n + 1)!

Criteriul raportului cu limit˘a (3.24): 21.

∞ X 1 · 3 · 5 . . . (2n − 1) n=1

2 · 5 · 8 . . . (3n − 1)

;

129 22.

∞ X n! n=1

nn

;

Criteriul lui Raabe - Duhamel (3.25): 23. 24.

∞ X 2 · 7 · 12 . . . (5n − 3) n=1 ∞ X n=1

3 · 8 · 13 . . . (5n − 2)

;

(2n)! ; (n!)2 · 22n

Criteriul lui Gauss (3.26, 3.27): ∞ X 12 · 52 · 92 . . . (4n − 3)2 . 25. 32 · 72 · 112 . . . (4n − 1)2 n=1

Observat¸ie a) Dac˘a M 6= 1 ˆın criteriul r˘ad˘ acinii cu m˘arginire, atunci se poate aplica criteriul r˘ad˘acinii cu limit˘a superioar˘a sau criteriul r˘ad˘ acinii cu limit˘a. Ast√ √ fel, dac˘a M < 1 ¸si n an ≤ M pentru orice n ≥ n0 , atunci lim sup n an ≤ n→∞ √ M < 1, iar dac˘a M > 1 ¸si n an ≥ M pentru orice n ≥ n0 , atunci √ lim sup n an ≥ M > 1 ¸si se obt¸in concluziile din criteriul r˘ad˘ acinii cu limit˘a n→∞ √ superioar˘a sau din criteriul r˘ad˘acinii cu limit˘a dac˘a exist˘a lim n an . n→∞

La fel stau lucrurile ¸si ˆın cazul criteriului raportului. Pentru M = 1, exist˘a serii pentru care se poate aplica criteriul r˘ad˘ acinii cu m˘arginire dar criteriul r˘ad˘acinii cu limit˘a superioar˘a (sau criteriul r˘ad˘ acinii cu limit˘a) nu poate preciza natura seriei. De exemplu, fie seria ∞ X en · n2n . (n + 1)2n

n=1

√ n a n =

e·nn (n+1)n

e 1 n , ∀n (1+ n )

∈ IN∗ ¸si conform criteriului r˘ad˘ acinii √ n cu m˘arginire, seria dat˘a este divergent˘ a. Dar lim an = 1 ¸si nu se poate n→∞ folosi criteriul r˘ad˘acinii cu limit˘a. Aceea¸si situat¸ie are loc pentru M = 1 ˆın criteriul raportului. De exem∞ P 2·5·8·...·(3n+2) an+1 3n+5 plu, fie seria si conform 1·4·7·...·(3n+1) . Avem an = 3n+4 > 1, ∀n ∈ IN ¸ Atunci

n=0

=

criteriului raportului cu m˘arginire, seria este divergent˘ a. Dar lim an+1 =1 n→∞ an ¸si nu putem aplica criteriul raportului cu limit˘a.

130 b) Este evident c˘a criteriul r˘ad˘ acinii cu limit˘a (respectiv criteriul raportului cu limit˘a) implic˘a criteriul r˘ad˘ acinii cu limit˘a superioar˘a (respectiv cri√ teriul raportului cu limite extreme) deoarece dac˘a exist˘a lim n an (respectiv an+1 ), n→∞ an

lim

n→∞

atunci lim sup n→∞

(respectiv lim sup an+1 an = lim inf n→∞

n→∞

√ √ n an = lim n an n→∞

an+1 an

an+1 ). n→∞ an

= lim

Exist˘a ˆıns˘a situat¸ii ˆın care se poate folosi criteriul r˘ad˘ acinii cu limit˘a superioar˘a (respectiv criteriul raportului cu limite extreme) ¸si nu se poate aplica criteriul r˘ad˘acinii cu limit˘a (respectiv criteriul raportului cu limit˘a). ∞ P De exemplu, pentru seria din exercit¸iul 3.4-17, an cu an = 31n pentru n=0 √ n par ¸si an = 71n pentru n impar, se vede c˘a nu exist˘a lim n an (deci nu n→∞ √ putem utiliza criteriul r˘ad˘ acinii cu limit˘a), dar lim sup n an = 13 < 1 ¸si n→∞

conform criteriului r˘ad˘ acinii cu limit˘a superioar˘a, seria este convergent˘ a. c) Criteriul raportului cu limit˘a (sau cu limite extreme) implic˘a criteriul r˘ad˘acinii cu limit˘a (sau cu limit˘a superioar˘a) ¸si acest lucru rezult˘a din propozit¸ia 2.29. Astfel dac˘a an > 0, ∀n ∈ IN, atunci (2)

lim inf n→∞

√ √ an+1 an+1 ≤ lim inf n an ≤ lim sup n an ≤ lim sup . n→∞ an an n→∞ n→∞

Dar exist˘a situat¸ii ˆın care natura unei serii se poate preciza cu criteriul r˘ad˘acinii ¸si nu cu criteriul raportului. Astfel, seria de la b) este convergent˘ a an+1 conform criteriului r˘ad˘ acinii cu limita superioar˘a. Dar lim inf an = 0 < 1 n→∞

¸si lim sup an+1 si nu se poate aplica criteriul raportului cu limit˘a an = ∞ > 1 ¸ n→∞

sau cu limite extreme. √ d) Dac˘a lim an+1 = 1, atunci din (2) rezult˘a c˘a ¸si lim n an = 1 ¸si a n n→∞ n→∞ nu se poate utiliza nici criteriul raportului cu limit˘a, nici criteriul r˘ad˘ acinii ∞ P a 1 n+1 cu limit˘a. Spre exemplu, fie seria = 1 deci n ln n . Avem lim an n=2

n→∞

nu merge aplicat nici criteriul raportului, nici criteriul r˘ad˘ acinii. Trebuie g˘asit un alt criteriu, de exemplu criteriul lui Cauchy de condensare. Astfel ∞ P 1 1 1 2n · a2n = 2n · 2n ·ln si seria a. Rezult˘a c˘a ¸si 2n = n ln 2 ¸ n ln 2 este divergent˘ seria

∞ P n=2

n=2

1 n ln n

este divergent˘ a.

e) Criteriul raportului cu limit˘a implic˘a criteriul lui Raabe-Duhamel.

131 an+1 n→∞ an

ˆIntr-adev˘ar, dac˘a lim

µ lim n

n→∞

= α 6= 1, atunci

¶ ( −∞, dac˘a α > 1 an −1 = an+1 +∞, dac˘a α < 1

¸si putem aplica criteriul lui Raabe-Duhamel. Exist˘a serii pentru care lim an+1 = 1 deci nu putem folosi criteriul ran→∞ an portului cu limit˘a dar se poate utiliza criteriul lui Raabe-Duhamel. ³ De exem´ an+1 an −1 = plu, pentru seria din exercit¸iul 3.4-23, lim an = 1 dar lim n an+1 1 5

n→∞

n→∞

< 1 ¸si conform criteriului lui Raabe-Duhamel, seria diverge. 3.5 S˘a se arate c˘a dac˘a seria

n≥n0

urm˘atoarele condit¸ii: (i) ∃M ≥ 1 a.ˆı.

P

an (n0 ∈ IN) ˆındepline¸ste una din

|an+1 | ≥ M, ∀n ≥ n0 , |an |

|an+1 | > 1, |an | |an+1 | ∃ lim = α ¸si α > 1, n→∞ |an | p ∃M ≥ 1 a.ˆı. n |an | ≥ M, ∀n ≥ n0 , p lim sup n |an | > 1, p ∃ lim n |an | = α ¸si α > 1,

(ii) lim inf (iii) (iv) (v) (vi)

n→∞

P

atunci seria

n≥n0

an este divergent˘ a.

3.6 Determinat¸i natura urm˘atoarelor serii: 1. 2. 3. 4.

∞ X

µ e

−n 3

n=1 ∞ ³ X n=1 ∞ X n=2 ∞ X n=1

·

n+1 n

¶n2 ;

´n √ √ 3 n3 + n2 + 2 − 3 n3 − n2 + 2 ;

1 ; ln(n!) sin3

1 ; n

132 ∞ X

1 √ √ √ ; 3 1 + 2 + 3 + ... + n n n=1 ∞ X 1 6. arctg ; n

5.

7. 8. 9. 10. 11. 12. 13. 14.

n=1 ∞ X n=1 ∞ X n=2 ∞ X n=1 ∞ X n=1 ∞ X n=2 ∞ X n=0 ∞ X n=1 ∞ X n=1 ∞ X

(2n)n

p

; (4n2 + 3n + 1)n+1 ¶ µ ¶ µ 1 1 · ln 1 + 2 ; ln n · ln 1 + n n

1 n+1 ln ; n n 1+

1 2

+ ... + n2

1 n

;

1 ; [n + (−1)n ]2 5 + sin 4n ; 2n (2n − 1)!! 1 · ; (2n)!! 2n + 1 1 ; [3 + (−1)n ]n

nn ; en · n! n=1 ¶n ∞ µ X 1 2 n 16. + + ... + 2 ; n2 + 2 n2 + 4 n + 2n

15.

17.

n=1 ∞ X n=2

ln(n + 1) . n2

3.7 Precizat¸i natura seriilor urm˘atoare ˆın funct¸ie de parametrii respectivi: 1. 2.

∞ X n , x > 0; xn

n=1 ∞ X n=1

1 , x > 0; n(1 + x + x2 + . . . + xn )

133 3. 4. 5. 6. 7.

∞ X

a

n=1 ∞ X

√ n

, a > 0;

aln n , a > 0;

n=1 ∞ X

nα ln n, α ∈ IR;

n=2 ∞ µ X n=1 ∞ X n=1 ∞ X

an2 + n + 1 n2

¶n , a > 0;

an √ , a > 0; n n!

(an)n , a > 0; n! n=1 · ¸ ∞ X α(α − 1) . . . (α − n + 1) 2 (2n + 1) 9. , α ∈ IR \ {−1, −2, −3, . . .}; (α + 1)(α + 2) . . . (α + n)

8.

10. 11. 12. 13. 14.

n=1 ∞ X n=2 ∞ X

ln(1 + xn ) , x > 0, α ∈ IR; nα

√ √ n+1 ( n + 1 − n)α · ln , α ∈ IR; n−1

n=2 ∞ X

1

1

a1+ 2 +...+ n , a > 0;

n=1 ∞ X

(−1)n+1 1

n=1 ∞ X n=2

nα+ n

, α ∈ IR;

1 , α ∈ IR. n(ln n)α

3.8 Studiat¸i convergent¸a urm˘atoarelor serii folosind teoremele precizate: ∞ X cos nx √ , x ∈ IR; a) Criteriul lui Dirichlet: n n=1 ∞ X cos n · cos n1 √ ; 7 3 n n=1 ∞ X (−1)n ; c) Criteriul lui Leibniz: ln n

b) Criteriul lui Abel:

n=2

134 ∞ X

d) Teorema 3.33:

(−1)

n2 +n 2

n=1

·

n . 5n

3.9 Dat¸i exemple de serii divergente a c˘aror sum˘a s˘a fie: a) convergent˘a; b) divergent˘a. 3.10 Dat¸i exemple de: a) serie alternat˘a divergent˘ a,

P n≥n0

(−1)n an (an > 0, ∀n ≥ n0 ), cu propri-

etatea an → 0; b) serie alternat˘a convergent˘ a care nu verific˘ a condit¸iile din criteriul lui Leibniz. ∞ P

3.11 Fie seria

an ¸si seria

n=1

(∗)

(a1 + a2 + ... + an1 ) +(an1 +1 + an1 +2 + ... + an2 ) + . . . ... + (ank +1 + ank +2 + ... + ank+1 ) + ...

obt¸inut˘a prin gruparea termenilor ˆın grupe finite, cu p˘astrarea ordinii termenilor, astfel ˆıncˆat termenii din fiecare grup˘aP au acela¸si semn. Ar˘atat¸i c˘a dac˘a seria (∗) este convergent˘ a, atunci ¸si seria an este convergent˘ a ¸si are aceea¸si sum˘a. √ ∞ X (−1)[ n] Aplicat¸ie: stabilit¸i natura seriei . n n=1

3.12 Studiat¸i natura seriilor: ∞ X sin n cos n2 √ ; 1. n+ n n=1 ∞ X (−1)n ; 2. 3 + sin n

3. 4.

n=0 ∞ X

(−1)n−1

n=1 ∞ X n=1

(1 +

1 2

ln(5 + e3n ) ; ln(3 + e5n )

+ ... + n1 ) sin nx , x ∈ IR; n

135 5. 6.

∞ X (n!)2 · xn n=1 ∞ X

(2n)!

, x ∈ IR;

√ √ ( n2 + 1 − a n2 − 1) · xn , a, x ∈ IR

n=1

(Prof. dr. A.M. Precupanu, examen Analiz˘a matematic˘a I); ¶n ∞ µ X n+a 7. b· ; a, b ∈ IR; n+a−1 n=1 ¶n µ ∞ X n(n+1)(n+2) n 3 ; 8. (−1) · 4n + 1 9. 10. 11. 12. 13. 14. 15. 16. 17.

n=1 ∞ X n=1 ∞ X n=1 ∞ X n=2 ∞ X n=1 ∞ X n=1 ∞ X n=1 ∞ X n=1 ∞ X n=1 ∞ X n=1 ∞ X

nα · an ; a, α ∈ IR; xn , x ∈ IR \ {1, −1}; 1 − xn xn

xn √ , x ∈ IR; + n

xn , x ∈ IR \ {−1}; (1 + x)(1 + x2 ) . . . (1 + xn ) an · bn ; a, b ∈ IR cu a + b 6= 0; an + bn 1 + (−1)n n · x , x ∈ IR; n xn − 1 n · y ; x, y ∈ IR; n nx + 3 n · a ; x, y, a ∈ IR; ny + 7 n! · an ; a ∈ IR, α > 0; α(α + 1) . . . (α + n)

an ; a ∈ IR, b > 0 n(1 + bn ) n=1 (Prof. dr. A.M. Precupanu, examen Analiz˘a matematic˘a I); ∞ p X ( n2 + 1 − an) · xn ; a, x ∈ IR 19.

18.

n=1

(Prof. dr. A.M. Precupanu, examen Analiz˘a matematic˘a I);

136 20.

21. 22. 23. 24. 25.

∞ µp X

¶ β + 1 − αn − · xn ; α, β, x ∈ IR n n=1 (Prof. dr. A.M. Precupanu, examen Analiz˘a matematic˘a I); ¸ ∞ · X (2n − 1)!! α n · x ; α, x ∈ IR; (2n)!! n=1 ∞ X x(x + 1) . . . (x + n) n · a ; a ∈ IR, 0 < x < y; y(y + 1) . . . (y + n) n=1 √ ∞ X (α − 1)(α − 2) . . . (α − n + 1) n , α ∈ IR \ Z− ; (α + 1)(α + 2) . . . (α + n) n=1 ∞ X n! ; α ∈ IR, b > 0; b(b + 1) . . . (b + n) · nα n=1 √ √ √ ∞ X n(n+1) e · e · 3 e... n e (−1) 2 , α ∈ IR; nα n2

n=1

26.

∞ P

¢n2 ¡ , a ∈ IR, a > 2 (a − 2)n · 1 + n1

n=1

(Examen de Licent¸˘ a-iunie 2004).

3.13 Criteriul P logaritmic Fie seria an (n0 ∈ IN) cu an > 0, ∀n ≥ n0 . n≥n0

a) Dac˘a lim inf

ln a1n

b) Dac˘a lim sup

ln n ln a1n ln n

> 1, atunci seria < 1, atunci seria

P

an converge.

P

an diverge.

ln a1n c) Presupunˆand c˘a exist˘a lim = α, avem: n→∞ ln n P • dac˘a α > 1, atunci seria an este convergent˘ a; P • dac˘a α < 1, atunci seria an este divergent˘ a. 3.14 Folosind criteriul logaritmic (problema 3.13), studiat¸i convergent¸a seriilor: ¶ln(n+1) ∞ µ X 1 a) ; n3 + n + 3 n=1 ∞ X 1 , α ∈ IR. b) (ln n)α n=2

137 3.15 Precizat¸i natura seriei

∞ √ √ P ( n + a n + 1 + b) · cn , unde a, b, c ∈ IR n=1

¸si determinat¸i suma seriei ˆın cazul a= − c, b=1, |c| 0. a) Determinat¸i a ¸si α astfel ˆıncˆ at (bn ) s˘a fie convergent ¸si calculat¸i lim bn . n→∞

b) Pentru a = 1, studiat¸i natura seriei

∞ P

bn · xn , x ∈ IR.

n=1

3.17 S˘a se studieze natura seriei ∞ X (2n + 1)(a − 1)(a − 2) . . . (a − n + 1) · xn

(a + 1)(a + 2) . . . (a + n + 1)

n=1

unde a, x ∈ IR ¸si pentru a = 0, x = 1 s˘a se calculeze suma seriei. (Prof. dr. A.M. Precupanu, examen Analiz˘a matematic˘a I). 3.18 Fie seria convergent˘a

∞ P

an cu an > 0, ∀n ∈ IN∗ . Ar˘atat¸i c˘a seriile

n=1

∞ ∞ √ P P −1 −1 an an+1 ¸si (a−1 sunt convergente. n + an+1 ) n=1

n=1

∞ P

3.19 Dac˘a seria

n=n0

an (n0 ∈ IN) este convergent˘ a, atunci ¯ ∞ ¯ ∞ ¯X ¯ X ¯ ¯ an ¯ ≤ |an |. ¯ ¯ ¯ n=n0

3.20 Fie seria

∞ P

n=n0

an cu an > 0, ∀n ∈ IN∗ . Ar˘atat¸i c˘a

n=1

3.21 Fie

∞ P

∞ P

n=1

an o serie cu an > 0, ∀n ∈ IN∗ ¸si sn =

n=1

gente; b) dac˘a seria

∞ P

an converge, atunci seriile

n=1 ∞ P

an diverge, atunci seria

n=1

n P

∞ P n=1

an 1+an .

ak pentru orice

k=1

n ∈ IN∗ . Ar˘atat¸i c˘a: a) dac˘a seria

an ∼

∞ P n=1

∞ P n=1 an sn

an sn

¸si

∞ P n=1

diverge ¸si

an s2n ∞ P

n=1

sunt converan s2n

converge.

138 3.22 Dac˘a (an )n∈IN∗ ⊂ (0, ∞), precizat¸i natura seriilor: a) b)

∞ X an ; 1 + a3n n=1 ∞ X an n=1

1 + nα an

, α ∈ IR.

3.23 S˘a se arate c˘a dac˘a seria

∞ P

nan este convergent˘ a, atunci ¸si seria

n=1

∞ P

an converge.

n=1

3.24 Dac˘a seria

∞ P

a2n este convergent˘ a, care este natura seriei

n=1

3.25 Fie

∞ P n=1

∞ P n=1

an n

?

an o serie cu termeni nenegativi.

a seria a) Teorema lui Olivier: dac˘ descresc˘ator, atunci lim nan = 0.

∞ P

an este convergent˘ a ¸si (an ) este

n=1

n→∞

b) Ar˘atat¸i c˘a ipoteza de monotonie este esent¸ial˘ a. c) Reciproca teoremei lui Olivier este adev˘arat˘ a? ∞ P

3.26 Fie seria n P

an cu termeni pozitivi astfel ˆıncˆ at an → 0, fie sn =

n=1

ak termenul general al ¸sirului sumelor part¸iale ¸si tn = sn − [sn ] (partea

k=1

fract¸ionar˘a a lui sn ), n ∈ IN∗ . S˘a se arate c˘a seria

∞ P

an este convergent˘ a

n=1

dac˘a ¸si numai dac˘a ¸sirul (tn )n este convergent. (Concursul student¸esc “Traian Lalescu”, faza final˘a Constant¸a - 2003) 3.27 Ar˘atat¸i c˘a

∞ P n=1

1 n2

=

π2 6

¸si

∞ P n=1

1 n4

=

π4 90 .

3.28 Dat¸i exemple pentru urm˘atoarele situat¸ii: a) dou˘a serii convergente care au produsul Cauchy divergent; b) dou˘a serii divergente pentru care produsul Cauchy este absolut convergent.

139 3.29 S˘a se efectueze produsul Cauchy al seriilor ∞ P

deduc˘a de aici suma seriei

n=0

n=0

1 (−1) · √ ( 2)n n=0

n!

1 n!

¸si

∞ P n=0

(−1)n n!

¸si s˘a se

(−1)n n! .

3.30 Demonstrat¸i c˘a à ∞ !à ∞ X X1 n=0

∞ P

n

! = e(2 −

√ 2).

3.31 S˘a se ridice la p˘atrat urm˘atoarele serii, s˘a se arate c˘a seriile obt¸inute sunt convergente ¸si s˘a se determine sumele lor: a) b) c)

∞ X

(−1)n ·

n=0 ∞ X

1 ; 5n

nan cu a ∈ (−1, 1);

n=1 ∞ X n=1

n−1 . 2n+1

3.32 Dac˘a |x| < 1 (x0 = 1), ar˘atat¸i c˘a ∞ ³X

xn

n=0

3.33 Ar˘atat¸i c˘a seria

∞ P n=0

xn n!

´2

=

∞ X

(n + 1) · xn .

n=0

este absolut convergent˘ a pentru orice x ∈ IR

(x0 = 1) ¸si apoi, notˆand cu E(x) suma acestei serii, demonstrat¸i relat¸ia: E(x + y) = E(x) · E(y), ∀x, y ∈ IR. 3.34 S˘a se demonstreze c˘a seriile

∞ P

(−1)n ·

n=0

x2n (2n)!

¸si

∞ P

(−1)n ·

n=0

x2n+1 (2n+1)!

sunt absolut convergente pentru orice x ∈ IR ¸si apoi, notˆand sumele lor cu C(x) ¸si respectiv S(x), s˘a se arate c˘a: S(x + y) = S(x)C(y) + C(x)S(y) ¸si C(x + y) = C(x)C(y) − S(x)S(y), ∀x, y ∈ IR. a ¸si descresc˘atoare. Fie 3.35 Fie f : [0, +∞) → [0, +∞) o funct¸ie continu˘ ∞ ∞ P P seria an cu termen general an = f (n), n ∈ IN. S˘a se arate c˘a seria an n=0 n=0 Rn a ¸si este este convergent˘a dac˘a ¸si numai dac˘a limita lim 0 f (x)dx exist˘ n→∞ finit˘a.

140

Solut¸ii 1 2

3.1

³

1. Seria este telescopic˘a avˆ and an = ³ ´ n n P P 1 1 1 1 Atunci sn = = − 2 2k−1 2k+1 = 4k2 −1 c˘a lim sn = n→∞

k=1 1 2 deci

k=1

1 2n−1

1 2

³



1−

1 2n+1

1 2n+1

´

, ∀n ∈ IN∗ .

´ . Se observ˘a

seria este convergent˘ a ¸si are suma 12 .

1 1 2. Seria este telescopic˘a: an = n+x − n+x+1 , ∀n ∈ IN∗ . Rezult˘ a sn = ∗ 1 1 1 a cu suma 1+x . 1+x − n+x+1 , ∀n ∈ IN deci seria este convergent˘ 1 1 3. an = n + ln(n + 1) − ln n, sn = 1 + 2 + . . . + n1 + ln(n + 1) → +∞ deci seria diverge. √ √ 4. an = √n+1√n+1 = n + 1 − n, ∀n ∈ IN∗ (serie telescopic˘a). Rezult˘a √ sn = n + 1 − 1 → +∞ deci seria este divergent˘ a. 5. an = arctg(n + 1) − arctgn (serie telescopic˘a). Rezult˘a ∞ P 1 = π2 . arctg n2 +n+1

n=0

6. Este o serie telescopic˘a: an = ∞ P n=0

n2 +n−1 (n+1)!

1 (n−1)!



1 (n+1)! , ∀n

∈ IN∗ . Atunci

= 1.

7. an = n1 + ln n − ln(n + 1), ∀n ∈ IN∗ ⇒ sn = 1 + 21 + . . . + n1 − ln(n + 1) = n vn + ln n+1 → c, unde vn = 1 + 12 + . . . + n1 − ln n → c (constanta lui Euler), conform problemei 2.5-d). h i 8. an =

1 p

1 n(n+1)...(n+p−1)

1 (n+1)(n+2)...(n+p) 1 este p·p! . ∗



, ∀n ∈ IN∗ (avem o serie

telescopic˘a). Astfel suma seriei 9. an = √ 1 − √ 1 , ∀n ∈ IN (serie telescopic˘a). Suma seriei este (2n)!!

(2n+2)!!

√1 . 2 n−1

n−1

10. an = (−1)n ¸si sn ¡= 1 − 12 + 31 − . . . + ¢(−1)n ¡ , ∀n ∈ IN∗ . Avem ¢ 1 1 1 s2n = 1 − 21 + 13 − . . . − 2n = 1 + 12 + 13 + . . . + 2n − 2 12 + 14 + . . . + 2n = v2n + ln(2n) − vn − ln n = v2n − vn + ln 2 → ln 2 ¸si s2n+1 = 1 − 12 + 13 − 14 + 1 1 1 . . . − 2n + 2n+1 = s2n + 2n+1 → ln 2. Deci seria converge ¸si are suma ln 2. ∞ ∞ ∞ P P P (−1)n−1 1 Se observ˘a c˘a seria |an | = este divergent˘ a . Astfel seria n n n=1

n=1

n=1

este convergent˘a dar nu este absolut convergent˘ a. ∞ P 11. Pentru x = 1, seria devine (n+1) care diverge pentru c˘a termenul n=0

general nu tinde la 0 (conform corolarului 3.8). Pentru x 6= 1, ´10 + ³ sn = 1−xn 2 n 2 3 n+1 0 = 2x + 3x + . . . + (n + 1)x = (x + x + x + . . . + x ) = x 1−x

141 1−(n+1)xn +n·xn+1 (1−x)2

1 care converge la (1−x) a |x| < 1 ¸si este divergent dac˘a 2 dac˘ x ∈ (−∞, −1] ∪ (1, ∞). ∞ ∞ ¡ ¢ ¡ ¢n P P x n 12. an = 312 · x3 ¸si conform teoremei 3.35, an ∼ care 3 n=0

n=0

este seria geometric˘a cu rat¸ia q = x3 (vezi observat¸ia 3.3-a)). ˆIn consecint¸˘ a, ∞ P 1 1 1 seria an este convergent˘a cu suma 32 · 1− x = 3(3−x) dac˘ a ¸si numai dac˘a 3 ¯ x ¯ n=0 ¯ ¯ < 1 adic˘a pentru x ∈ (−3, 3) ¸si divergent˘ a pentru x3 ∈ (−∞, −1] ∪ (1, ∞) 3 adic˘a pentru x ∈ (−∞, −3] ∪ [3, ∞). 13. Este o serie geometric˘a cu rat¸ia q = 2a+3 3a−1 . 3.2

Fie an =

n+1 . n2 ·n!

Atunci

an+1 an

=

n2 (n+2) (n+1)4

→ 0 < 1 ¸si din criteriul

1 raportului cu limit˘a rezult˘a c˘a seria converge. Se observ˘a c˘a an+1 an ≤ 2 , ∀n ≥ λ 1. Aplicˆand observat¸ia 3.40-a) cu λ = 12 vom avea |an | · 1−λ = nn+1 2 ·n! ≤ −2 2 10 ⇔ 100(n + 1) ≤ n · n!, ∀n ≥ 5. Putem atunci considera N = 5 ¸si astfel s∼ = 2, 46. = s5 = a1 + a2 + . . . + a5 = 21 + 223·2! + 324·3! + 425·4! + 526·5! ∼

3.3

Fie an =

(−1)n (2n+1)3

¸si bn =

1 (2n+1)3

> 0, ∀n ∈ IN. Se observ˘a c˘a ¸sirul (bn ) ∞ P este descresc˘ator ¸si cu limita 0 deci seria an este convergent˘ a datorit˘a n=0

1 −3 criteriului lui Leibniz. S˘a determin˘am n minim astfel ˆıncˆ at (2n+3) 3 ≤ 10 (vezi observat¸ia 3.40-b)). Ultima inegalitate este echivalent˘ a cu (2n + 3)3 ≥ 3 10 care este adev˘arat˘a pentru n ≥ 4. Atunci |s − sn | = |rn | ≤ an+1 < 10−3 pentru n ≥ 4. Prin urmare, s ∼ = s4 = a0 + a1 + a2 + a3 + a4 = 1 − 313 + 513 − 1 ∼ 1 + 93 = 0, 969. 73 2 ³ 2 ´n ³ ´n lim −2n −2n n→∞ n2 +n+1 = e−2 6= 0 3.4 1. lim nn2 −n+1 = lim 1 + = e +n+1 n2 +n+1 n→∞ n→∞ deci seria diverge. n 2. lim n35+5n = lim n31 = 1 6= 0 ¸si seria este divergent˘ a.

n→∞

n→∞ 5n +1 ∗ cos n 7n , ∀n ∈ IN ¯, atunci |an+1 + an+2 + . . . + an+p | = ¯ ¯ cos(n+1) cos(n+2) cos(n+p) ¯ 1 1 1 + 7n+2 + . . . + 7n+p = ¯ 7n+1 + 7n+2 + . . . + 7n+p ¯ ≤ 7n+1 1 ¡ ¢ ¡ ¢ 1− 1 1 1 1 + 17 + . . . + 7p−1 = 7n+1 · 1−71p = 6·71 n 1 − 71p < 6·71 n , ∀p ∈ IN∗ . 7n+1 7 at Dar lim 6·71 n = 0 ¸si atunci pentru orice ε > 0, exist˘a n0 ∈ IN astfel ˆıncˆ n→∞ 1 a |an+1 +an+2 +. . .+an+p | < ε, ∀n ≥ n0 6·7n < ε pentru orice n ≥ n0 . Rezult˘ ∗

3. Dac˘a an =

¸si ∀p ∈ IN . Astfel seria este convergent˘ a.

142 4. Fie an =

√ n n , ∀n n2 +1

∈ IN∗ . Se observ˘a c˘a an ≥

|an+1 + an+2 + . . . + an+p | =

1 ∗ n+1 , ∀n ∈ IN . √Atunci √ n+2 n+p + (n+2) + . . . + (n+p) ≥ (n+2)2 +1 (n+p)2 +1 p n 1 p = n, n+p+1 = 2n+1 ≥ 3 , ∀n ∈ IN∗ . ∗

√ (n+1) n+1 (n+1)2 +1

p 1 1 1 n+2 + n+3 +. . .+ n+p+1 ≥ n+p+1 . Pentru Deci exist˘a ε = 31 > 0 astfel ˆıncˆ at pentru orice n ∈ IN , exist˘a p = n cu proprietatea |an+1 + an+2 + . . . an+p | ≥ 13 . Astfel seria este divergent˘ a. 1 5. Fie an = (n+1)·3n , ∀n ∈ IN. Atunci sn = a0 + a1 + a2 + . . . + an = 1 ¡ 1− n+1 1 1 1 1 1 3 3 1 + 2·3 + 3·31 2 + . . . (n+1)·3 = n ≤ 1 + 3 + 32 + . . . + 3n = 1 2 1− 1− 3 ∞ P ∀n ∈ IN ¸si conform teoremei 3.12, seria an converge.

¢

1

3n+1

< 23 ,

n=0

≥ 0, ∀n ∈ IN∗ ¸si avem 6. Observ˘am c˘a an = ´ ³ ´ n n n ³ P P P 1+sin k 2 1 1 1 1 sn = ≤ ≤ 2 − =2 − 2 2 k+1 k+2 2 n+2 0, ∀n ∈ IN∗ . Avem an = arctg n13 ≤ n13 , ∀n ∈ ∞ P 1 este seria armonic˘a generalizat˘a cu α = 3 > 1 care este IN∗ , iar n3 n=1

convergent˘a (vezi observat¸ia 3.3-b)). Criteriul de comparat¸ie de specia I ∞ P stabile¸ste atunci c˘a ¸si seria an este convergent˘ a. n=1

8. Avem an =

n

n2 +cos2

n



1 n+1

= bn > 0, ∀n ∈ IN∗ ¸si cum seria

∞ P n=1

1 n+1

este divergent˘a, acela¸si criteriu de comparat¸ie de specia I ne arat˘a c˘a seria ∞ P an diverge. n=1



(2n+3) n+1 n √ > 0, ∀n ∈ IN∗ . Avem an+1 an = (2n+2) n+2 ≥ n+1 = 1 ∞ P ∗ 1 n+1 a. Din criteriul de comparat¸ie 1 , ∀n ∈ IN , iar seria n este divergent˘

9. Fie an =

n

(2n+1)!! √ (2n)!! n+1

n=1

de specia a II-a se obt¸ine c˘a ¸si seria

∞ P n=1

1

an este divergent˘ a.

1

1

10. Fie an = (2 − e 2 )(2 − e 3 ) . . . (2 − e n ) > 0, ∀n ≥ 2. Avem an+1 an = ¢ ¡ 1 1 n+1 , ∀n ∈ IN∗ . Rezult˘a c˘a e n+1 < 1+ n1 , ∀n ∈ 2−e n+1 . Se ¸stie c˘a e < 1 + n1 ∞ 1 P 1 1 n−1 n IN∗ . Atunci an+1 > 2 − 1 − = = , ∀n ≥ 2, iar seria 1 an n n n−1 este n−1

n=2

divergent˘a. Conform criteriului de comparat¸ie de specia a II-a, seria diverge.

∞ P n=2

an

143 11. an =

n+3 2n4 +1

3 n(1+ n )´ ³ 4 n 2+ 14

=

=

1 n3

3 1+ n 2+ 14

· bn , unde bn =

. Avem lim bn = n→∞ P P 1 1 ¸si conform corolarului 3.17 obt¸inem c˘a an ∼ . Cum 2 ∈ (0, n3 P ∞) 1 seria este convergent˘ a (este seria armonic˘ a generalizat˘ a cu α = 3 > 1) n3 P rezult˘a c˘a ¸si seria an este convergent˘ a. n

n

12. an =

√ √ 4 n2√+2n n3 + n7 √ 3 4 3 n +n n+3

−1

=

5

7

n2 +2n 2 +n 4 4

7

n 3 +n 2 +3

5

=

1

3

n 2 (n− 2 +2+n− 4 ) 7

n 2 (n−

=

13 7 6 +1+3n− 2 )

1 n ·bn ,

unde

−3

bn = n− 132 +2+n −47 . Avem lim bn = 2 ∈ (0, ∞) ¸si conform corolarului 3.17, n→∞ +1+3n 2 P n 6P P1 P 1 an ∼ . Dar seria armonic˘ a a deci ¸si seria an n n este divergent˘ este divergent˘a. 13. √ √ 3 n+3− 3n+1 √ = an = 3 n2 + 4 2 p p p = √ = 3 3 3 2 2 n + 4[ (n + 3) + (n + 3)(n + 1) + 3 (n + 2)2 ] 2 1 ·q ¸ = 4 · bn , = q¡ q¡ q ¡ ¢ ¢ ¡ ¢ ¢ 2 2 2 2 n3 n 3 3 1 + n42 · n 3 3 1+ n3 + 3 1+ n3 1+ n1 + 3 1+ n1 unde bn =

q 3

1+ 42 n

·q 3

q

3 2 1+ n +3

(

)

2

(

3 1+ n

)(

1 1+ n

q 3

1 2 1+ n

)+ (

)

(0, +∞) ¸si la fel ca la seria precedent˘ a, obt¸inem c˘a

¸.

P

Avem lim bn = n→∞

an ∼

P

1

4

n3

2 3



care este

convergent˘ a (ca serie armonic˘a generalizat˘a cu α = 43 > 1). Astfel seria P an converge. 1 14. Fie an = n ln a c˘a (an ) este descresc˘ator. n > 0, ∀n ≥ 2. Se observ˘ ∞ ∞ P P Aplicˆand criteriul lui Cauchy de condensare, an ∼ 2n · a2n . Fie n=2 P n=2P 1 1 bn = 2n · a2n = n ln . Conform teoremei 3.35, b ∼ n 2 n care este P divergent˘a. ˆIn consecint¸˘a, seria an diverge. 1 15. Fie an = (ln n)ln n > 0, ∀n ≥ 2. S¸irul (an ) fiind descresc˘ator, putem ∞ ∞ P P aplica criteriul lui Cauchy de condensare, deci an ∼ 2n · a2n . Fie n

n=2

n=2

1 2 1 bn = 2n · a2n = 2n · (n ln 2) n ln 2 = (ln 2)ln 2 · nn ln 2 . Conform teoremei 3.35, P P 2n P 2n bn ∼ . Pentru seria se aplic˘a criteriul r˘ad˘ acinii cu limit˘a. nn ln 2 nn ln 2 √ 2n 2 n Astfel, dac˘a not˘am cn = nn ln 2 , avem lim cn = lim nln 2 = 0 < 1. Rezult˘a n→∞ n→∞ P P c˘a seria cn este convergent˘a ¸si, prin urmare, seria an este de asemenea convergent˘a.

144 ³ ´n √ 16. Fie an = 8n+1 > 0, n ∈ IN∗ . Avem n an = 8n+1 ≤ 12 < 1 pentru 3 n +2 n3 +2 n ≥ 4. Rezult˘a c˘a seria este convergent˘ a. √ √ 17. Pentru n par, n an = 31 ¸si pentru n impar, n an = 17 . Rezult˘a c˘a √ lim sup n an = 13 < 1. Astfel seria este convergent˘ a. n→∞ √ √ √ 18. Notˆand an = ( n + 2− n + 1)n > 0 pentru n≥1, avem lim n an = n→∞ √ √ lim ( n + 2 − n + 1) = lim √n+2+1 √n+1 = 0 < 1. Rezult˘a c˘a seria conn→∞ n→∞ verge. ´n ³ 2 2 2 n − . Se ¸stie c˘a 12 + 22 + . . . + n2 = 19. Fie an = 1 +2 n+...+n 2 3 µ n(n+1)(2n+1) ¶ √ n(n+1)(2n+1) n ∗ 6 , ∀n ∈ IN . Atunci lim n an = lim −3 = 6 n2 n→∞

3n+1 n→∞ 6n

lim

=

1 2

n→∞

< 1 ¸si seria este convergent˘ a. 2n

3 9 1 20. Fie an = (n+1)! > 0, ∀n ∈ IN. Avem an+1 an = n+2 ≤ 2 < 1 pentru n ≥ 16. Conform criteriului raportului cu m˘arginire, seria converge. an+1 ∗ 2n+1 2 21. Fie an = 1·3·5...(2n−1) si 2·5·8...(3n−1) , ∀n ∈ IN . Atunci an = 3n+2 → 3 < 1 ¸ seria este convergent˘a. nn 1 1 22. Fie an = nn!n , ∀n ∈ IN∗ . Atunci an+1 an = (n+1)n = (1+ 1 )n → e < 1. n

Astfel seria converge. 5n+2 23. Dac˘a an = 2·7·12...(5n−3) , se observ˘a c˘a an+1 an = 5n+3 → 1. Atunci ³ ´ 3·8·13...(5n−2) ³ ´ an 1 lim n an+1 − 1 = lim n 5n+3 si conform criteriului lui 5n+2 − 1 = 5 < 1 ¸ n→∞ n→∞ Raabe-Duhamel, seria diverge. (2n)! an+1 2n+1 24. Fie an = (n!) 2 ·22n . Avem an = 2(n+1) → 1. Atunci µ ¶ an 1 lim n −1 = 0 ¸si xn = 4+ 21+ 1 care converge la 14 deci este n

n2

m˘arginit. Deoarece β = 1 rezult˘a c˘a seria diverge. | 3.5 (i) Din |a|an+1 ≥ M ≥ 1, ∀n ≥ n0 se obt¸ine |an+1 | ≥ |an |, ∀n ≥ n0 n| adic˘a ¸sirul |an | este cresc˘ator.PAtunci |an | 9 0 ceea ce implic˘a an 9 0 ¸si conform corolarului 3.8, seria an este divergent˘ a.

145 (ii) dac˘a lim inf |ak+1 k≥n0 |ak |

ˆıncˆat inf

|an+1 | |an |

|ak+1 | |ak | k≥n n∈IN

= sup inf

> 1, atunci exist˘a n0 ∈ IN astfel

> 1. Rezult˘a c˘a exist˘a n0 ∈ IN astfel ˆıncˆ at

|ak+1 | |ak |

> 1 pentru

orice k ≥ n0 . Astfel exist˘a n0 ∈ IN astfel ˆıncˆ aP t |ak+1 | > |ak |, ∀k ≥ n0 . De aici se obt¸ine c˘a |an | 9 0 deci an 9 0 ¸si seria an este divergent˘ a. |an+1 | |an+1 | (iii) Rezult˘a din (ii) deoarece lim inf |an | = lim |an | > 1. n→∞ p (iv) Din n |an | ≥ M ≥ 1, ∀n ≥ n0 , rezult˘a |an | ≥ 1, ∀n ≥ n si deci 0 ¸ P |an | 9 0. Aceasta implic˘a an 9 0 ¸si conform corolarului 3.8, seria an este divergent˘a. p p p (v) lim sup n |an | = inf sup k |ak | > 1 ⇒ ∀n ∈ IN, sup k |ak | > 1 ⇒ n∈IN k≥n p k≥n kn ∀n ∈ IN, ∃kn ≥ n a.ˆı. |akn | >P1 ⇒ ∀n ∈ IN, ∃kn ≥ n a.ˆı. |akn | > 1 ⇒ |an | 9 0 ⇒ an 9 0. Astfel seria an diverge. p p (vi) rezult˘a din (v) ˆıntrucˆat lim sup n |an | = lim n |an | > 1. n→∞

¢n2 ¡ n , ∀n ∈ N ∗ . Aplic˘am criteriul r˘ad˘ acinii cu 3.6 1. Fie an = e− 3 · n+1 n ¡ ¢n 2 1 √ 1 − limit˘a: lim n an = lim e 3 1 + n = e 3 > 1 ¸si seria este divergent˘ a. n→∞ n→∞ 2. Se aplic˘a criteriul r˘ad˘acinii cu limit˘a: √ lim n an = n→∞ p p 3 3 = lim ( n3 + n2 + 2 − n3 − n2 + 2) = n→∞

(n3 + n2 + 2) − (n3 − n2 + 2) p p = (n3 +n2 +2)2 + 3 (n3 +n2 +2)(n3 −n2 +2)+ 3 (n3 −n2 +2)2

= lim p 3 n→∞

2 < 1. 3 Rezult˘a c˘a seria converge. 3. Avem n! ≤ nn , ∀n ≥ 2 ⇒ ln(n!) ≤ n ln n, ∀n ≥ 2 ⇒ =

1 1 ≥ , ∀n ≥ 2. ln(n!) n ln n

(1)

Am ar˘atat (ˆın problema 3.4-14) c˘a seria

∞ P n=2

1 n ln n

este divergent˘ a. Atunci

din (1) ¸si criteriul de comparat¸ie de specia I, rezult˘a c˘a seria

∞ P n=2

1 ln(n!)

este

de asemenea divergent˘a. 4. Fie an = sin3 n1 > 0 ¸si bn = n13 , ∀n ∈ IN∗ . Avem lim abnn = n→∞ ³ 1 ´3 P sin n = 1. Conform criteriului de comparat¸ie cu limit˘a, an ∼ lim 1

n→∞

n

146 P

1 n3

care este a. P seria armonic˘a generalizat˘a cu α = 3 > 1 deci convergent˘ Astfel seria an converge. 1 √ 5. Fie an = √ √ > 0, ∀n ∈ IN∗ . 3 n 1+ 2+

Solut¸ia 1. Fie

3+...+ n bn = n1 , n ∈ IN∗ .

Avem

criteriul lui Stolz-Ces`aro (propozit¸ia 2.30):

√ √ √ 1+ 2+ 3 3+...+ n n . Folosim n √ n+1 bn+1 −bn n+1 lim an+1 −an = lim = 1 n→∞ n→∞ P

bn an

=

1 ⇒ lim abnn = 1. Conform criteriului de comparat¸ie cu limit˘a, an ∼ P n→∞ P P bn . Cum seria bn este divergent˘ a, rezult˘a c˘a seria an diverge. Solut¸ia a 2-a. √ √ √ √ S¸tim c˘a n n < 2, ∀n ∈ IN∗ . Rezult˘a c˘a 1 + 2 + 3 3 + . . . + n n < 1 2(n + 1), ∀n ∈ IN∗ . De aici obt¸inem an > 2(n+1) , ∀n ∈ IN∗ ¸si cum seria ∞ P 1 a, conform criteriului de comparat¸ie de specia I, 2(n+1) este divergent˘

n=1

seria

∞ P n=1

an este divergent˘ a.

∈ IN∗ . Atunci lim abnn = 1 ¸si din n→∞ P criteriul de comparat¸ie cu limit˘a, seria an diverge. 7. Termenul general se scrie 6. Fie an = arctg n1 ¸si bn =

an = =

unde bn =



q¡ nn+1 · 4+

1 · q¡ n 1+

3 1+ 4n +

3 4n

1 n , ∀n

nn · 2n ¢ q 3 1 n + · 4+ n n2

1 ¢n q + 4n1 2 · 4 +

1 ´n q 3 + · 4+ n

1 4n2

1 n2

3 n

+

3 n

+

1 n2

1 n2

=

=

1 · bn , n

. Avem lim bn = √1 3 · n→∞

e4

1 2

¸si conform

P P1 P1 criteriului de comparat a, n . Seria n fiind divergent˘ P ¸ie cu limit˘a, an ∼ rezult˘a c˘a ¸si seria an este divergent˘ a . ¡ ¢ ¡ ¢ 8. Fie an = ln n · ln 1 + n1 ln 1 + n12 . Utilizˆand inegalit˘a¸tile ln x ≤ x, > 0 ¸si ln(1 + x) ≤ x, ∀x ≥ 0, rezult˘a: an ≤ n12 , ∀n ≥ 2. Cum seria P∀x 1 este convergent˘a, din criteriul de comparat¸ie de specia I se obt¸ine c˘a n2 P seria an converge. 9. Se procedeaz˘a ca la 8. Va rezulta c˘a seria este convergent˘ a. 10. Se scrie termenul general ˆın forma: an = nvn2 + lnn2n , unde vn = 1 + 12 + . . .+ n1 −ln n. Not˘am xn = nvn2 ¸si yn = lnn2n . S¸irul (vn ) fiind convergent rezult˘a c˘a este m˘arginit deci exist˘a P α > 0 astfel ˆıncˆ at |vn | ≤ α, ∀n ∈ IN∗ . Atunci 1 |xn | ≤ nα2 , ∀n ∈ IN∗ . Seria fiind convergent˘ a, conform criteriului de n2 P comparat¸ie de specia I, seria |xn | este convergent˘ a. Folosind teorema

147 P 3.33, rezult˘a c˘a seria xn converge. P S¸irulP(ynn ) este descresc˘ P n·ln 2ator. Din n criteriul lui Cauchy de condensare, yn ∼ 2 · y2 = care este q 2n n n ln 2 1 convergent˘a datorit˘a criteriului r˘ad˘ acinii cu limit˘a: lim 2n = 2 < 1. n→∞ P P Astfel seria yn converge. ˆIn sfˆar¸sit, conform teoremei 3.36, seria an este convergent˘a. 1 1 11. Fie an = [n+(−1) n ]2 > 0, ∀n ≥ 2. Avem an ≤ (n−1)2 pentru orice P 1 P 1 ∼ care este n ≥ 2, iar din criteriul de comparat¸ie cu limit˘a, (n−1)2 n2 convergent˘ P a. Atunci criteriul de comparat¸ie de specia I asigur˘a faptul c˘a seria an este convergent˘a. P 6 P 1 4n 12. Avem 0 ≤ 5+sin ≤ 26n , ∀n ∈ IN, iar 2n 2n ∼ 2n care converge 1 (ca serie geometric˘ a cu rat ¸ ia q = ). Conform criteriului de comparat¸ie de 2 P 5+sin 4n specia I, seria converge. 2n 13. Not˘am cu an termenul³ general al ´ seriei. Se aplic˘a criteriul lui Raabean Duhamel ¸si se obt¸ine lim n an+1 − 1 = 32 > 1 deci seria converge. n→∞ ∞ P 1 14. Analog cu seria din 11 se demonstreaz˘a c˘a seria [3+(−1)n ]n este n=1

convergent˘a. 15. Fie an =

nn ∗ a criteriul de comparat¸ie de en ·n! , n ∈ nIN . Se utilizeaz˘ 1 1 1 n 1+ 1+ ( n) ( n) an+1 n 1 n+1 = = > = = 1 1 , iar seria n+1 an e n+1 1+ n (1+ n1 ) n

specia a II-a: P1 P a c˘a ¸si seria an diverge. n diverge. Rezult˘ 16. Fie an > 0 termenul general al seriei. Se aplic˘a criteriul r˘ad˘ acinii n P √ ∗ k k . Pentru orice 1 ≤ k ≤ n ¸si n ∈ IN , n2 +2n ≤ cu limit˘a: n an = n2 +2k k=1

k n2 +2k



Rezult˘a

n P k=1

k n2 +2n



n P

k n2 +2k

k=1 n(n+1) = 21 2



n P k=1

k , ∀n n2 +2

∈ IN∗ . Avem:

n P k ¸si analog, lim = 12 . 2 n→∞ k=1 n→∞ k=1 n +2 √ Conform teoremei cle¸stelui (propozit¸ia 2.26-vi)), lim n an = 12 < 1 ¸si seria n→∞ P an converge. ln n 17. Not˘am an = ln(n+1) , n ≥ 2. Se scrie an = ln(n+1) si se observ˘a ln n · n2 ¸ n2 ∞ ∞ P P ln n c˘a lim ln(n+1) = 1. Conform corolarului 3.17, an ∼ . Am ar˘atat ln n n2

lim

n P

k . n2 +2 k n2 +2n

=

lim 2 1 n→∞ n +2n

·

n→∞

la seria de la punctul 10 c˘a seria P seria an este convergent˘a. 3.7

∞ P n=2

n=2

ln n n2

n=2

este convergent˘ a. ˆIn consecint¸˘ a,

Convenim s˘a not˘am termenul general al seriilor cu an .

148 1. Se aplic˘a criteriul r˘ad˘ acinii cu limit˘a: lim

n→∞

√ n a n = lim

√ n

n n→∞ x

=

1 x.

Pentru x1 < 1 ⇔ x > 1, seria converge iar pentru x1 > 1 ⇔ x < 1, seria diverge. Pentru x1 = 1 ⇔ x = 1, ˆınlocuind ˆın an obt¸inem an = n 9 0 deci seria diverge. ˆIn concluzie, seria converge pentru x > 1 ¸si diverge pentru 0 < x ≤ 1. 1 2. Pentru x = 1, an = n(n+1) . Aplicˆand criteriul de comparat¸ie cu limit˘a P P 1 P se obt¸ine: an ∼ care este convergent˘ a. Deci seria an converge. n2 Pentru x > 1, an = n(xx−1 > 0. Conform criteriului r˘ a d˘ a cinii cu limit˘a, n+1 −1) P √ 1 1−x an converge. Pentru x < 1, an = n(1−x lim n an = x < 1 deci seria n+1 ) . n→∞ ˆIntrucˆat lim 1−x ¸ie n+1 = 1 − x ∈ (0, ∞), folosind criteriul de comparat n→∞ 1−x P P1 P cu limit˘a, rezult˘a c˘a an ∼ a deci ¸si seria an n care este divergent˘ diverge. Astfel: seria converge pentru x ≥ 1 ¸si diverge pentru x < 1. 3. Dac˘a a ≥ P 1, atunci an ≥ 1, pentru orice n ≥ 1 deci an 9 0 de unde rezult˘a c˘a seria an diverge. Dac˘a a < 1 se aplic˘a criteriul lui RaabeDuhamel: µ ¶ ³ √ √ ´ ³ √ −1 ´ an √ lim n − 1 = lim n a n− n+1 − 1 = lim n a n+ n+1 − 1 = n→∞ n→∞ n→∞ an+1 a −n √ · = lim √ n→∞ n+ n+1

√ √ −1 n+ n+1

−1

√ √ −1 n+ n+1

= −∞ · ln a = +∞ > 1

P ¸si rezult˘a c˘a seria an converge. Deci seria diverge pentru a ≥ 1 ¸si converge pentru a ∈ (0, 1). P 4. Dac˘a a ≥ 1, atunci an ≥ 1, ∀n ≥ 1 deci an 9 0 ¸si seria an diverge. Dac˘a a < 1, se aplic˘a criteriul lui Raabe-Duhamel: ³ ´ £ ¤ an lim n an+1 − 1 = lim n aln n−ln(n+1) − 1 = n→∞

n→∞

n

n aln n+1 − 1 = lim n ln · = n n→∞ n+1 ln n+1 µ ¶n ln n n a n+1 − 1 1 = lim ln · = (−1) · ln a = ln a−1 = ln . n n→∞ n+1 ln n+1 a P Dac˘a ln a1 < 1 ⇔ a > 1e , seria an diverge, iar dac˘a ln a1 > 1 ⇔ a < 1e , seria ¡ ¢ln n P = n1 ¸si seria an converge. Pentru ln a1 = 1 ⇔ a = 1e se obt¸ine an = 1e ∞ ¡ ¢ P 1 a. ˆIn concluzie, dac˘a a ∈ 0, 1e seria este convergent˘ a n este divergent˘

n=1

iar dac˘a a ∈ [ 1e , ∞) seria este divergent˘ a.

149 P 5. Dac˘a α ≥ 0, atunci lim nα ln n = +∞ ¸si astfel seria an diverge. n→∞ Pentru α < 0, exist˘a n0 ∈ IN astfel ˆıncˆ at an ≥ an+1 pentru n ≥ n0 . Folosind ∞ ∞ P P criteriul lui Cauchy de condensare avem an ∼ 2n · a2n . Fie bn = n=2 n=2 P 2n · a2n = n · √ 2n(1+α) · ln 2. Pentru seria bn se aplic˘a criteriul acinii cu P r˘ad˘ n 1+α 1+α . Dac˘a 2 < 1 ⇔ α < −1, seria bn converge limit˘a: lim bn = 2 n→∞ P P 1+α deci ¸si seria an converge. Dac˘a 2 > 1 ⇔ α ∈ (−1, 0), seria bn P 1+α = 1 ⇔ α = −1, b = n ln 2 9 diverge. Rezult˘ a c˘ a a diverge. Dac˘ a 2 n n P P 0 deci seria bn este divergent˘a ¸si seria an este la fel: divergent˘ a. Rezult˘a c˘a seria converge pentru α < −1 ¸si diverge pentru α ∈ [−1, ∞). √ 6. Se aplic˘a criteriul r˘ad˘acinii cu limit˘a: lim n an = a. Dac˘ a a < 1, seria n→∞ P P an converge, iar dac˘a a > 1, seria an diverge. Pentru a = 1, lim an = n→∞ ¡ ¢ P n lim 1 + n+1 = e = 6 0 ¸ s i seria a este divergent˘ a . Deci pentru a ∈ (0, 1) n 2 n n→∞

seria este convergent˘a ¸si pentru a ∈ [1, ∞) seria este divergent˘ a. an 1 1 √ , ∀n ≥ 1. Cum 7. Presupunem ˆıntˆai a ≥ 1. Atunci an = √ ≥ ≥ n n n n! n! P1 seria P n este divergent˘a, din criteriul de comparat¸ie de specia I rezult˘a c˘a an ≤ an , iar seria geometric˘a seria an diverge. Dac˘a a ∈ (0, 1), atunci √ n n! P n criteriului de comparat¸ie de specia I, seria P a este convergent˘a. Conform an este convergent˘a. ˆIn concluzie, seria converge pentru a ∈ (0, 1) ¸si diverge pentru a ≥ 1. 8. Se aplic˘a criteriul raportului cu limit˘a: an+1 n→∞ an

lim

an+1 ·(n+1)n+1 (n+1)! n→∞ a(n + 1)n

= lim

= lim

n→∞

n! an ·nn

= ¶ µ 1 n = lim a 1 + = ae. n→∞ n

nn

·

P Dac˘a ae < 1 ⇔ a < 1e , seria an converge iar dac˘a ae > 1 ⇔ a > 1e , seria n P (1+ n1 ) = . Conform problemei 2.5 -b), an diverge. Pentru a = 1e , an+1 an e 1 1 n ¡ ¢ 1+ n+1 ( n) 1 n n+1 e < 1 + n1 . Rezult˘a an+1 an > (1+ 1 )n+1 = 1+ 1 = n+1 = 1 , ∀n ≥ 1, n n n P1 este divergent˘ a . Datorit˘ a criteriului de comparat ¸ ie de specia iar seria nP a II-a, seria an este de asemenea divergent˘ a. Astfel, dac˘a a ∈ (1, 1e ) seria 1 este convergent˘a, iar dac˘a a ≥ e seria este divergent˘ a. 9. Se aplic˘a criteriul lui Raabe-Duhamel: µ lim n

n→∞

an −1 an+1



·

¸ (2n + 1)(α + n + 1)2 = lim n − 1 = 4α + 1. n→∞ (2n + 3)(α − n)2

150 P Dac˘a P 4α + 1 > 1 ⇔ α > 0, seria an converge iar dac˘a 4α + 1 < 1 ⇔ αP < 0, seria an diverge. Pentru α = 0, an = 0 pentru orice n ≥ 1 ¸si seria an este convergent˘a. A¸sadar, seria converge pentru α ≥ 0 ¸si diverge pentru α < 0. 10. I) Dac˘a x ³ ∈ (0, ´ 1), se aplic˘a criteriul raportului cu limit˘a: α n+1 ) P an+1 n lim an = lim n+1 · ln(1+x si seria an converge. n) = x < 1 ¸ ln(1+x n→∞ n→∞ P 2 II) Dac˘a x = 1, an = ln an converge pentru α > 1 ¸si nα . Astfel seria diverge pentru α ≤ 1. n ln(1+( x1 ) ) ln xn (1+ x1n ) ln x = = bn + cn , III) Dac˘a x > 1, se scrie an = + nα nα nα−1 1 n ln(1+( x ) ) x unde bn = nlnα−1 ¸si cn = . Deoarece x1 < 1, folosind I) rezult˘a c˘a nα ∞ ∞ P P P ln x 1 ∼ seria cn este convergent˘ a pentru orice α ∈ IR. Seria nα−1 nα−1 n=2

n=2

care converge pentru α−1 > 1 ⇔ α > 2 ¸si diverge pentru α−1 ≤ 1 ⇔ α ≤ 2. Conform teoremei 3.36, avem: P • α > 2 ⇒ an converge ¸si P • α ≤ 2 ⇒ an diverge. ˆIn concluzie, dac˘a • x ∈ (0, 1) ¸si α ∈ IR, seria converge; • x = 1 ¸si α > 1, seria converge; • x = 1 ¸si α ≤ 1, seria diverge; • x > 1 ¸si α > 2, seria converge; • x > 1 ¸si α ≤ 2, seria diverge. 11.

³ · ln 1 +

an =

1 √ √ ( n+1+ n)α

=

√ n−1√ ( n+1+ n)α

=

√1 √ (n−1)( n+1+ n)α

1

·

2 n−1

2 ln(1+ n−1 ) 1 n−1

·

´ =

=

2 ln(1+ n−1 ) 1 n−1

, n ≥ 2. Atunci lim abnn = 12 ¸si din criteriul de n→∞ P comparat¸ie cu limit˘a, an ∼ bn care P este seria armonic˘a generalizat˘ P a. bn converge deci ¸si seria an Astfel, pentru 1 + α2 > 1 ⇔ α > 0, seria

Fie bn =

√1 n( n)α

=

1 α n1+ P2

151 P α converge ¸ s i pentru 1 + ≤ 1 ⇔ α ≤ 0, seria bn diverge ¸si deci seria 2 P P an diverge. Prin urmare, seria an este convergent˘ a pentru α > 0 ¸si divergent˘a pentru α ≤ 0. 12. Dac˘ P a a ≥ 1, atunci an ≥ 1 pentru orice n ≥ 1 deci an 9 0 ¸si rezult˘a c˘a seria an diverge. Dac˘a a ∈ (0, 1), se aplic˘a criteriul lui Raabe-Duhamel: ³ ´ ³ −1 ´ −1 n+1 an −n lim n an+1 − 1 = lim n a n+1 − 1 = lim n+1 · a −1 −1 = (−1) ln a. n→∞ n→∞ n→∞ n+1 P 1 Pentru − ln a > 1 ⇔ a < e , seria an converge. P Pentru − ln a < 1 ⇔ a > 1e , seria an diverge. 1 1 1 1 Pentru − ln a = 1 ⇔ a = e , an = 1+ 1 +...+ 1 = 1+ 1 +...+ 1 −ln n+ln n = evn ·n , e

2

n

e

n

2

vn c c. Atunci unde vn = 1 + 21 + . . . + n1 − ln n → P Pe1 → e ∈ (0, ∞) ¸si conform criteriului , an ∼ a. Deci n care este divergent˘ P de comparat¸ie cu limit˘aP 1 ¸si seria an diverge. Astfel, seria an converge pentru a ∈ (0, e ) ¸si diverge pentru a ≥ 1e . ( +∞, α < 0, 13. Fie α ≤ 0. Atunci lim |an | = . Rezult˘a c˘a an 9 0 n→∞ 1 α=0 P 1 si exist˘a n0 ∈ IN astfel deci seria an diverge. Pentru α > 0, lim α+ 1 = 0 ¸ n→∞ n n ˆıncˆat P an ≥ an+1 , pentru orice n ≥ n0 . Din criteriul lui Leibniz obt¸inem c˘a seria an este convergent˘a. Deci seria diverge pentru α ≤ 0 ¸si converge pentru a > 0. P 14. Dac˘a α = 0, an = n1 ¸si seria an este divergent˘ a. Dac˘a α > 0, an este ir descresc˘ator ¸si se aplic˘a criteriul lui Cauchy condensare. P un ¸sP P de P 1 Astfel 1 n·a n ∼ an ∼ 2n · a2n . Avem 2n · a2n = nα ·(ln , iar 2 α 2 nα . Deci 2) P P dac˘a α > 1, seria an converge iar dac˘a α ≤ 1, seria an diverge. Dac˘a α < 0, lim a1n = +∞ ¸si conform criteriului de comparat¸ie cu limit˘a, seria n→∞ n P an este divergent˘a. A¸sadar, seria converge pentru α > 1 ¸si diverge pentru α ≤ 1.

3.8

a) Dac˘a x = 2kπ(k ∈ Z), atunci seria devine

Pentru x 6= 2kπ(k ∈ Z), fie an = cos nx ¸si bn = descresc˘ator ¸si cu limita 0, iar

√1 , n n

∞ P n=1

√1 n

care diverge.

≥ 1. S¸irul (bn ) este

| cos x + cos 2x + . . . + cos nx| = =

|2 sin x2 cos x + 2 sin x2 cos 2x + . . . + 2 sin x2 cos nx| = 2| sin x2 |

152

=

(2n+1)x x 5x 3x | sin 3x − sin (2n−1)x | 2 − sin 2 + sin 2 − sin 2 + . . . + sin 2 2 = x 2| sin 2 |

=

| sin (2n+1)x − sin x2 | 1 2 ≤ , ∀n ∈ IN∗ . x 2| sin 2 | | sin x2 | ∞ P

Astfel, sunt ˆındeplinite condit¸iile din criteriul lui Dirichlet deci seria

n=1

cos √ nx n

este convergent˘a. n √ b) Fie an = cos si bn = cos n1 , n ∈ IN∗ . Folosind criteriul lui Dirichlet 7 3 ¸ n P cos n √ se arat˘a (ca la punctul a)) c˘a seria a. Din faptul c˘a 7 3 este convergent˘ n

−1 ≤ cos n1 ≤ 1 pentru orice n ≥ 1, rezult˘a c˘a (bn ) este m˘arginit. Funct¸ia cosinus fiind descresc˘atoare pe [0, π2 ] iar ¸sirul n1 este descresc˘ator, conform propozit¸iei 1.37, ¸sirul (bn ) este cresc˘ator. Deoarece sunt ˆındeplinite condit¸iile P cos n·cos n1 √ din criteriul lui Abel, seria converge. 7 3 c) Fie an =

n

1 ln n , n

≥ 2. Avem an > 0 pentru orice n ≥ 2, ¸sirul (an ) este ∞ P (−1)n descresc˘ator ¸si lim an = 0. Din criteriul lui Leibniz, seria ln n este n→∞

convergent˘a. d) Fie an = (−1)

n=2

n2 +n 2

·

n 5n , n ≥ ∞ X

1. Se consider˘a seria modulelor

|an | =

n=1

∞ X n 5n

n=1

p ¸si pentru aceast˘a serie se aplic˘a criteriul r˘ad˘ acinii cu limit˘a: lim n |an | = n→∞ √ P nn 1 lim = 5 < 1 deci seria |an | este convergent˘ a. Aplicˆand teorema 3.33, n→∞ 5 P rezult˘a c˘a ¸si seria an converge. 3.9

a) Seriile

∞ P n=0

an =

∞ P

(−3)n ¸si

n=0 (an ) ¸si (bn )

∞ P n=0

bn =

∞ P

(−1)n+1 ·3n sunt divergente

n=0

nu converg la zero. Atunci an + bn = (−3)n + ∞ P (−1)n+1 ·3n = (−3)n [1+(−1)] = 0 pentru orice n ∈ IN∗ deci seria (an +bn ) pentru c˘a ¸sirurile

n=1

este convergent˘a. ∞ ∞ P P b) Seriile n2 ¸si n sunt divergente pentru c˘a lim n2 = lim n = ∞, iar seria

∞ P

n=0

n=0

n→∞

n→∞

(n2 + n) este divergent˘ a din acela¸si motiv.

n=0

3.10 a) Fie seria alternat˘a 1 − 17 + 12 − 712 + 31 − 713 + . . . + n1 − 71n + . . . ¸si not˘am ¸sirul sumelor part¸iale cu (sn ). Se observ˘a c˘a an → 0 dar seria

153 este divergent˘a pentru c˘a lim s2n = lim (1 − 17 + 12 − 712 + . . . + n1 − 71n ) = n→∞ ¢¤ £¡ ¢ n→∞ ¡ lim 1 + 21 + . . . + n1 − 17 + 712 + . . . + 71n = +∞. n→∞ ∞ P b) Fie seria alternat˘a (−1)n−1 an = 1 − 212 + 313 − 412 + 513 − 612 + . . . + 1 (2n−1)3

n=1

1 − (2n) 2 + . . ., unde an =

1 n3

1 n2

pentru n par. P 1 Avem pentru orice n ∈ IN ¸si cum seria P n2 este conn| ≤ vergent˘a, conform criteriului de comparat¸ie de specia I, seria |(−1)n−1 an | P converge. Atunci teorema 3.33 asigur˘a faptul c˘a ¸si seria (−1)n−1 an converge. Se observ˘a c˘a lim an = 0 dar (an ) nu este descresc˘ator. 1 n2

|(−1)n−1 a

pentru n impar ¸si an = ∗

n→∞

3.11

S˘a not˘am prin (sn ) ¸sirul sumelor part¸iale pentru seria

∞ P n=1

an , prin

(tk ) ¸sirul sumelor part¸iale pentru seria (∗) ¸si prin t suma seriei (∗). Deci t = lim tk ⇔ (∀ε > 0, ∃k0 (ε) = k0 ∈ IN∗ a.ˆı. t − ε < tk < t + ε, ∀k ≥ k0 ). k→∞

Pentru c˘a ˆın seria (∗), termenii din fiecare grup˘a au acela¸si semn, avem tk−1 < sn < tk sau tk < sn < tk−1 , n ∈ {nk−1 +1, . . . , nk }, unde tk−1 = snk−1 ¸si tk = snk . Pentru k ≥ k0 + 1, avem t − ε < tk−1 < tk < t + ε sau t − ε < tk < tk−1 < t + ε. Rezult˘ a c˘a t − ε < sn < t + ε, ∀n ≥ nk0 + 1 adic˘a ∞ P lim sn = t sau altfel spus, seria an converge ¸si are suma t. n→∞

Pentru aplicat¸ie:

∞ P n=1

n=1



(−1)[ n

n]

= −1 − 12 − 13 + 41 + 15 + 16 + 17 + 18 − 19 − . . ..

√ [ n] √ √ Not˘am an = (−1)n , n ≥ 1. Fie [ n] = k ∈ IN∗ ⇔ k ≤ n < k + 1 ⇔ k 2 ≤ n < (k + 1)2 ⇔ k 2 ≤ n ≤ (k + 1)2 − 1 = k 2 + 2k. Atunci pentru n ∈ {k 2 , k 2 +1, . . . , k 2 +2k}, termenii an au acela¸si semn. Grupˆand termenii cu acela¸si semn (ca ˆın problema precedent˘ a), obt¸inem seria alternat˘a: ¶ µ ¶ µ 1 1 1 1 1 1 1 + + + + + − − 1+ + 2 3 4 5 6 7 8 µ ¶ · ¸ (2) ∞ X 1 1 1 1 1 k − + + ... + + ... = (−1) + ... + 2 9 10 15 k2 k + 2k

k=1

S˘a not˘am bk = k12 + . . . +

1

³

k2 +2k

xk + yk , unde xk =

1 k2

=

1 k2

+ ... +

+ ... +

1 k2 +k−1

1

´

k2 +k−1

¸si yk =

³ +

1 k2 +k

1

k2 +k

+ ... +

+ ... +

1 . k2 +2k

1 1 1 1 ≤ xk ≤ 2 + . . . + 2 + ... + 2 {z k } |k | + k {z k + k} de k ori de k ori k2

1

´

k2 +2k

Avem

=

154 ¸si

1 1 1 1 + ... + ≤ yk ≤ 2 ⇒ + ... + 2 2 2 (k + 1) (k + 1) |k + k {z k + k} | {z } de k+1 ori de k+1 ori k k2 +k

≤ xk ≤

k k2

k+1 (k+1)2 2 obt¸ine k+1

¸si

≤ yk ≤

k+1 k2 +k

1 k+1 ≤ xk 2 2 k ⇔ k+1



1 k

1 1 k+1 ≤ yk ≤ k . bk ≤ k2 , ∀k ∈ IN∗ .



¸si

≤ x k + yk ≤ ≤ Prin adunare se Rezult˘a lim bk = 0 din teorema cle¸stelui (propozit¸ia 2.6-vi)). ˆIn plus, din k→∞

2 2 inegalit˘a¸tile de mai sus avem: k+2 < bk+1 < k+1 < bk < k2 , ∀k ∈ IN∗ de unde rezult˘a c˘a ¸sirul (bn ) este descresc˘ator. Din criteriul lui Leibniz, P seria (2) este convergent˘a. Folosind acum problema 3.11 rezult˘a c˘a ¸si seria an converge.

3.12 1. S˘a not˘am an = sin n cos n2 ¸si (bn ) = este descresc˘ator cu lim bn = 0. Apoi avem

1√ ,n n+ n

≥ 1. Atunci (bn )

n→∞

|a1 + a2 + . . . + an | = ¯ n ¯ ¯ ¯ 1 ¯P = 2 ¯ [sin k(k + 1) − sin k(k − 1)]¯¯ = 21 | sin n(n + 1)| ≤ 21 , ∀n ≥ 1 k=1

P sin n cos n2 √ ¸si se aplic˘a criteriul lui Dirichlet. Rezult˘a c˘a seria este convern+ n gent˘a. (−1)n 2. Fie an = 3+sin a −1 ≤ sin n ≤ 1 pentru orice n , n ∈ IN. Din faptul c˘ 1 1 n ∈ IN se obt¸ine 4 ≤ |an | ≤ 2 pentru orice n ∈ IN de unde rezult˘a c˘a an 9 0 ¸si conform corolarului 3.8, seria diverge. 3n ) 3n ) 3. Fie an = (−1)n−1 · ln(5+e , n ≥ 1. Avem lim |an | = lim ln(5+e 5n ) = ln(3+e5n ) ln(3+e n→∞ n→∞ P 3 si seria an diverge. 5 . Deci an 9 0 ¸ 1+ 1 +...+ 1

4. Se aplic˘a criteriul lui Dirichlet. Fie an = sin nx ¸si bn = 2 n n , n ≥ ∞ P 1. Se arat˘a la fel ca ˆın seria din problema 3.8-a), c˘a seria sin nx are ¸sirul n=1

sumelor part¸iale m˘arginit. Pentru a calcula limita ¸sirului (bn ) se aplic˘a 1 (1+ 12 +...+ n+1 )−(1+ 21 +...+ n1 ) criteriul Stolz-Ces`aro (propozit¸ia 2.30): lim = (n+1)−n 1 n+1

lim n→∞ 1 −n < (n+1)2

n→∞

= 0. Rezult˘a c˘a lim bn = 0. Pentru monotonie: bn+1 − bn = n→∞

0 pentru orice n ≥ 1 deci ¸sirul (bn ) este descresc˘ator. Deoarece P sunt ˆındeplinite condit¸iile din criteriul lui Dirichlet, seria an bn converge pentru orice x ∈ IR.

155 5. Pentru x = 0, seria evident converge. Presupunem x 6= 0. Fie an = ≥ 1. Se aplic˘a criteriul raportului cu limit˘a pentru seria modulelor. P | Avem lim |a|an+1 = |x| a |x| < 4, seria an converge iar dac˘a 4 . Astfel, dac˘ n| n→∞ P |an+1 | |x| > 4, seria an diverge. Pentru |x| = 4, avem |an | = 2n+2 2n+1 > 1, ∀n ≥ 1. P Conform problemei 3.5-(i), seria an diverge. Prin urmare, seria converge pentru x ∈ (−4, 4) pentru x ∈ (−∞, −4] ∪ [4, ∞) √ ¸si diverge √ 6. Fie an = ( n2 + 1 − a n2 − 1) · xn , n ≥ 1. Se aplic˘a criteriul rapor| tului cu limit˘a pentru seria modulelor. Avem lim |a|an+1 = |x|. n| n→∞ P Pentru |an | este convergent˘ a ¸si conform teoremei 3.33, P |x| < 1, seria seria an este convergent˘a. P Pentru |x| > 1 se utilizeaz˘a problema 3.5-(ii) P ¸si seria an diverge. Dac˘a |x| = 1 ¸si a 6= 1, atunci an 9 0 ¸si seria an diverge. Pentru x = 1 ¸si a = 1, an = √n2 +1+2 √n2 −1 . ˆIn acest caz, aplicˆand criteriul ∞ ∞ P P 1 de comparat¸ie cu limit˘a, seria an are aceea¸si natur˘a cu seria n deci n=1 n=1 P seria an diverge. P (−1)n √ Dac˘a x = −1 ¸si a = 1, atunci an = √n2 +1+ ¸si seria an este n2 −1 convergent˘a datorit˘a criteriului lui Leibniz. ˆIn concluzie: (n!)2 ·xn (2n)! , n

• • • • •

dac˘a dac˘a dac˘a dac˘a dac˘a

|x| < 1 ¸si a ∈ IR, atunci seria converge; |x| > 1 ¸si a ∈ IR, atunci seria diverge; |x| = 1 ¸si a 6= 1, atunci seria diverge; x = 1 ¸si a = 1, atunci seria diverge; x = −1 ¸si a = 1, atunci seria converge.

7. S˘a not˘am cu an termenul general al seriei.pSe aplic˘a criteriul r˘ad˘ acinii n cu limit˘a pentru seria modulelor ¸si avem lim |an | = |b|. n→∞ P P Pentru |b| < 1, seria |an | converge deci ¸si seria P an converge. Pentru |b| > 1 se folose¸ste problema 3.5-(ii)P ¸si seria an diverge. Pentru |b| = 1, lim |an | = e 6= 0 deci seria an este divergent˘ a. n→∞ ´n ³ n(n+1)(n+2) n 3 8. Fie an = (−1) , n ≥ 1. Conform criteriului r˘ad˘ aci· 4n+1 p P 1 nii cu limit˘a pentru seria modulelor, lim n |an | = 4 < 1 deci seria |an | n→∞ P converge ¸si conform teoremei 3.33, seria an este convergent˘ a.p 9. Fie an = nα ·an , n ≥ 1. La fel ca la seria precedent˘ a, lim n |an | = |a|. n→∞ P Astfel, seria an converge pentru |a| < 1 ¸si diverge pentru |a| > 1. Pentru ∞ P 1 a = 1 se obt¸ine seria armonic˘a generalizat˘a care converge pentru n−α n=1

156 −α > 1 ⇔ α < −1 ¸si diverge P pentru −α ≤ 1 ⇔ α ≥ −1. Pentru a = −1 P ¸si α ≥ 0, an 9 0 deci seria an diverge. Pentru a = −1 ¸si α < 0, seria an converge datorit˘a criteriului lui Leibniz. Deci pentru: • • • • • • P

|a| < 1 ¸si α ∈ IR, seria converge; |a| > 1 ¸si α ∈ IR, seria diverge; a = 1 ¸si α < −1, seria converge; a = 1 ¸si α ≥ −1, seria diverge; a = −1 ¸si α ≥ 0, seria diverge; a = −1 ¸si α < 0, seria converge.

10. S˘a not˘am an =

xn 1−xn , n

≥ 1. Dac˘a |x| > 1, atunci an 9 0 ¸si seria

| an diverge. Dac˘a |x| < 1, lim |a|an+1 = |x| < 1 ¸si criteriul raportului cu P n→∞ n | P limit˘a stabile¸ste c˘a seria |an | converge deci ¸si seria an converge. A¸sadar, seria converge pentru x ∈ (−1, 1) ¸si diverge pentru x ∈ (−∞, −1) ∪ (1, ∞). n 11. S˘a not˘am an = xnx+√n ¸si ¸sirul sumelor part¸iale cu (sn )n≥2 . P Pentru |x| > 1, an 9 0 deci seria an diverge. P |an+1 | Pentru |x| < 1, lim |an | = |x| < 1 ¸si seria an converge. n→∞ ∞ P √1 Pentru x = 1, seria devine care diverge pentru c˘a are aceea¸si n+1

natur˘a cu seria

∞ P n=2

n=2

1 n1/2

(conform criteriului de comparat¸ie cu limit˘a).

Pentru x = −1, an =

(−1)n √ (−1)n + n

¸si

1 −1 1 −1 1 s2n = √ +√ +√ +√ +√ + ...+ 2+1 3−1 4+1 5−1 6+1 −1 1 1 −1 1 +√ +√ 1, atunci lim

n=1

cu rat¸ia 12 ). 13. Fie an =

an ·bn an +bn , n

|an+1 | |an | = |ab| |an+1 | lim = |a|. n→∞ |an |

≥ 1. Avem

¯ n n ¯ ¯ a +b ¯ · ¯ an+1 , n ≥ 1. +bn+1 ¯

I) Cazul |a| < |b|. Atunci P Dac˘a |a| < 1, seria a converge. P n Dac˘a |a| > 1, seria an diverge. P Dac˘a |a| = 1 < |b|, atunci an 9 0 deci seria an diverge. II) Cazul |a| > |b| se trateaz˘a analog. P n III) Cazul |a| = |b| ⇔ a = b. ˆIn acest caz, an = a2 ¸si seria an este convergent˘a pentru |a| < 1 ¸si divergent˘ a pentru |a| ≥ 1. n 14. Fie an = 1+(−1) · xn , ∀n ∈ IN∗ . Pentru x = 0, seria este evident n q p n convergent˘a. S˘a presupunem c˘a x 6= 0. Pentru n par, |an | = n n2 · |x| > 0 p p ¸si pentru n impar, n |an | = 0. Rezult˘a c˘a lim sup n |an | = |x|. Dac˘a |x| < 1,Patunci se aplic˘a criteriul r˘aP d˘ acinii cu limit˘a superioar˘a. Rezult˘a c˘a seria |an | converge deci ¸si seria an converge. P Dac˘a |x| > 1, atunci se aplic˘a problema 3.5-(v) ¸si se obt¸ine c˘a seria an diverge. n P1 Dac˘a |x| = 1, atunci an = n1 + (−1) a n . Cum seria n este divergent˘ P (−1)n ¸si seria este convergent˘ a (din criteriul lui Leibniz), rezult˘a conform n P teoremei 3.36 c˘a seria an diverge. 15. Seria converge pentru x = 1 sau y = 0. Presupunem x 6= 1 ¸si y 6= 0. n n xn+1 −1 Fie an = x n−1 · y n , n ≥ 1. Rezult˘a an+1 an = y · n+1 · xn −1 . |an+1 | = n→∞ |an | P

I. |x| < 1 ⇒ lim

|y|.

(i) Dac˘a |y| < 1, seria a este convergent˘ a. P n (ii) Dac˘a |y| > 1, seria an este divergent˘ a. n n (iii) Dac˘a y =P1, an = xn − n1 = bn + cn , unde bn = xP si cn = n ¸ −1 , n ≥ 1. Seria c este evident divergent˘ a . Pentru seria b n n avem n p P n lim |bn | = |x| < 1 ¸si conform criteriului r˘ad˘ acinii cu limit˘a, seria bn n→∞ P converge. Aplicˆand teorema 3.36, se obt¸ine c˘a seria an diverge. n n (−1)n+1 (−1)n ·xn + n = xn + yn , unde xn = (−1)n ·x (iv) Dac˘a y = −1, an = n n+1 P ¸P si yn = (−1)n , n ≥ 1. Ca mai sus se arat˘a c˘a seria xn converge, iar seria yn este convergent˘ P a utilizˆand criteriul lui Leibniz. Aplicˆand teorema 3.36, obt¸inem c˘a seria an converge. | II. |x| > 1 ⇒ lim |a|an+1 = |xy|. n| n→∞

158 P (i) Dac˘a |xy| < 1, seria a converge. P n (ii) Dac˘a |xy| > 1, seria an diverge. (iii) Dac˘a y = x1 , atunci an = reduce la cazul I-(iii).

xn −1 n

1 xn

·

n

= −

( x1 )

−1

n

¯ ¯ cu ¯ x1 ¯ < 1 ¸si se n

(−1)n +(−1)n+1 ·( x1 ) (−1)n −1 xn −1 cu n ¯ 1 ¯ (iv) Dac˘a y = x , atunci an = n · xn = ¯ ¯ < 1 ¸si se reduce la cazul I-(iv). x P III. Dac˘a x = 1, atunci an = 0 pentru orice n ≥ 1 ¸si seria an converge. ∞ ∞ n P P 1−(−1)n IV. Dac˘a x = −1, atunci an = (−1)n −1 · y n ¸si an ∼ · yn . n n=1 n=1 p n n Fie bn = 1−(−1) · y n , n ≥ 1. Atunci lim n P sup |bn | = |y|. Conform criteriului r˘ad˘acinii cu limit˘a superioar˘a, seria bn converge pentru |y| < 1 ¸si diverge ∞ n P (−1)n 1 pentru |y| > 1. Dac˘a |y| = 1, atunci an = (−1) − . Cum seria n n n

este convergent˘a (din criteriul lui Leibniz) iar seria

n=1

∞ P

rezult˘a conform teoremei 3.36 c˘a seria

n=1

x

∞ P

n=1

1 n

este divergent˘ a,

an este divergent˘ a.

+3 16. Fie an = nny +7 · an , n ≥ 1. Dac˘ a a = 0, atunci an = 0, ∀n ≥ 1 ¸si P (n+1)x +3 ny +7 seria an converge. Pentru a = 6 0 avem an+1 si an = nx +3 (n+1)y +7 · a ¸ P |an+1 | an converge pentru |a| < 1 ¸si diverge lim |an | = |a|. Rezult˘a c˘a seria n→∞

pentru |a| > 1. x +3 Dac˘a a = 1, an = nny +7 , n ≥ 1. Avem urm˘atoarele situat¸ii: P (i) x ≥ y. Atunci an 9 0 deci seria an diverge. ∞ ∞ P P 1 (ii) 0 < x < y. Atunci an ∼ (conform criteriului de ny−x n=1

n=1

comparat¸ie cu limit˘a) deci converge pentru y − x > 1 ¸si diverge pentru y − x ≤ 1. ∞ ∞ P P 1 (iii) x < 0 ≤ y. Atunci an ∼ ny (conform criteriului de comparan=1

n=1

¸tie cu limit˘a) deci converge pentru y > 1 ¸si diverge pentru y ≤ 1. P (iv) x < y < 0. Atunci an 9 0 ¸si seria an este divergent˘ a. x +3 Dac˘a a = −1, an = nny +7 · (−1)n , n ≥ 1. P (i) Dac˘a x ≥ y, atunci an 9 0 ¸si seria an diverge. x +3 x +3 (ii) Dac˘a 0 < x < y sau x < 0 < y, atunci lim nny +7 = 0 iar ¸sirul nny +7 n→∞ P este descresc˘ator. Utilizˆand criteriul lui Leibniz, seria an converge. P (iii) Dac˘a x < y ≤ 0, atunci an 9 0 ¸si seria an este divergent˘ a. 17. S˘a not˘am cu an termenul general al seriei. Pentru a = 0, seria a(n+1) | converge. Presupunem a 6= 0. Avem an+1 si lim |a|an+1 = |a|. an = α+n+1 ¸ n| n→∞

159 P I. Pentru |a| < 1, seria an converge conform criteriului raportului cu P limit˘a pentru seria |an |. P II. Pentru |a| > 1, seria an diverge conform problemei 3.5-(iii). III.³Pentru a ´= 1 se aplic˘a criteriul lui Raabe-Duhamel: an lim n an+1 − 1 = α. n→∞ P (i) Dac˘a α > 1, atunci seria an este convergent˘ a. P (ii) Dac˘a 0 < α < 1, atunci seria an este divergent˘ a. P 1 (iii) Dac˘a α = 1, atunci an = n+1 ¸si seria an diverge. n! n IV. Pentru a = −1, an = (−1) ·bn , unde bn = α(α+1)·...·(α+n) > 0, ∀n ≥ 1. n+1 Avem bn+1 ator. Aplicˆand de dou˘a bn = α+n+1 < 1, ∀n ≥ 1 deci bn este descresc˘ ori teorema lui Stolz-Ces`aro (propozit¸ia 2.30) ¸sirului bn , obt¸inem lim bn = n→∞ P 0. Prin urmare, seria an este convergent˘ a cu criteriul lui Leibniz. an+1 an n 1+bn 18. Fie an = n(1+b si avem n ) , n ≥ 1. Atunci an = a · n+1 · 1+bn+1 ¸ cazurile: | I. 0 < b < 1 ⇒ lim |a|an+1 = |a| (am aplicat criteriul raportului cu limit˘a P n→∞ n | pentru seria |an |). P (i) Pentru |a| < 1, seria a este convergent˘ a. P n (ii) Pentru |a| > 1, seria an este divergent˘ a. ∞ ∞ P P 1 1 (iii) Pentru a = 1, an = n(1+b si an ∼ n) ¸ n (conform criteriului de n=1 P n=1 comparat¸ie cu limit˘a) deci seria an diverge. (−1)n 1 1 (iv) Pentru a = −1, an = n(1+b sirul n(1+b n ) . Avem lim n(1+bn ) = 0, ¸ n n→∞ P ) este descresc˘ator ¸si aplicˆand criteriul lui Leibniz se obt¸ine c˘a seria an converge. P | II. b = 1 ⇒ lim |a|an+1 = |a| deci seria an converge pentru |a| < 1 ¸si n| n→∞

diverge pentru |a| > 1. Dac˘a a = 1, atunci an =

P 1 si seria an diverge. 2n ¸ P (−1)n an converge an = 2n ¸si seria

Dac˘a a = −1, atunci Leibniz). | III. b > 1 ⇒ lim |a|an+1 = n| n→∞

diverge pentru |a| > b. Dac˘a a = b, atunci an =

|a| b

deci seria

bn n(1+bn )

la cazul I -(iii). Dac˘a a = −b, atunci an =

=

P

an converge pentru |a| < b ¸si

1 n , n[1+( 1b ) ]

(−1)n ·bn n(1+bn )

=

(din criteriul lui

unde

(−1)n n , n[1+( 1b ) ]

1 b

∈ (0, 1) ¸si se reduce

unde

reduce la cazul I-(iv). √ 19. Fie an = ( n2 + 1 − an) · xn , n ≥ 1. Avem lim

1 b

|an+1 | n→∞ |an |

∈ (0, 1) ¸si se = |x|.

160 P I. Pentru |x| < 1, seria an converge conform criteriului raportului cu P limit˘a pentru seria |an |. P II. Pentru |x| > 1, seria an diverge conform problemei 3.5-(iii). III. x = 1. (i) Dac˘a a 6= 1, atunci an 9 0 ¸si seria

P

an diverge.

1 (ii) Dac˘a a = 1, atunci an = √n2 +1+n ¸si conform criteriului de comparat¸ie ∞ ∞ P P P1 1 cu limit˘a, an ∼ a c˘a ¸si seria n . Cum seria n diverge, rezult˘ n=1 n=1 P an diverge.

IV. x = −1. (i) Dac˘a a 6= 1, atunci an 9 0 ¸si seria n

P

an diverge. P ¸si seria an este convergent˘ a din

(ii) Dac˘a a = 1, atunci an = √n(−1) 2 +1+n criteriul lui Leibniz. p √ 20. Fie an = ( n2 + 1 − αn − βn ) · xn , ∀n ∈ IN∗ . Avem lim n |an | = |x|. n→∞ P I. Dac˘a |x| < 1, atunci conform criteriului r˘ a d˘ a cinii cu limit˘ a , seria |an | P converge deci ¸si seria an converge (din teorema 3.33). P II. Dac˘a |x| > 1, atunci seria an diverge conform problemei 3.5-(vi). P III. Dac˘a |x| = 1 ¸si α 6= 1, atunci an 9 √ 0 ¸si seria an diverge. 2 + 1 − n − β . Se observ˘ IV. Dac˘ a x = 1 ¸ s i α = 1, atunci a = n a c˘a n n P P P1 1 (conform criteriului de comparat ¸ ie cu limit˘ a ). Cum seria an ∼ n n P diverge, rezult˘a c˘a ¸si seria an diverge. √ V. Dac˘a x = −1 ¸si α = 1, atunci an = ( n2 + 1 − n − βn ) · (−1)n = P (−1)n P (−1)n (−1)n (−1)n √ √ − β · . Cum seriile ¸ s i sunt convergente 2 2 n n n+ n +1 n+ n +1 P (din criteriul lui Leibniz), rezult˘a c˘a seria an este convergent˘ a conform teoremelor 3.35 ¸si h3.36. i α | 21. Fie an = (2n−1)!! = |x| deci seria · xn , n ≥ 1. Avem lim |a|an+1 (2n)!! n| n→∞ P an converge pentru |x| < 1 ¸si diverge ³ ´αpentru |x| > 1. an+1 2n+1 ≥ 1, ∀n ≥ 1 ⇒ an este cresc˘ator I. x = 1 ¸si α ≤ 0 ⇒ an = 2n+2 P deci an 9 0 ¸si seria an diverge. II. x = 1 ¸si α > 0. Se pot demonstra prin induct¸ie urm˘atoarele inegalit˘a¸ti: (3)

1 1 1 · 3 · . . . · (2n − 1) √ ≤ ≤√ , ∀n ∈ IN∗ . 2 · 4 · . . . · (2n) 2 n 2n + 1

161 Rezult˘a c˘a

1 2α ·nα/2

≤ an ≤

1 , ∀n (2n+1)α/2 ∞ P

comparat¸ie de specia I se obt¸ine c˘a

n=1

∈ IN∗ ¸si aplicˆand criteriul de ∞ P 1 an ∼ care converge pennα/2 n=1

tru α > 2 ¸si diverge pentru α ≤ 2. | III. x = −1 ¸si α ≤ 0 ⇒ |a|an+1 ≥ 1, ∀n ≥ 1. Rezult˘a c˘a ¸sirul |an | este Pn | cresc˘ator deci an 9 0 ¸si seria an diverge. h iα IV. x = −1 ¸si α > 0. Atunci an = (−1)n · bn , unde bn = 1·3·...·(2n−1) > 2·4·...·(2n) ´α ³ 2n+1 0, ∀n ≥ 1. S¸irul (bn ) este descresc˘ator ˆıntrucˆ at bn+1 < 1, ∀n ≥ bn = 2n+2 1. Din (3) rezult˘a c˘a 0 < bn
1, atunci seria an diverge conform problemei 3.5-(iii). III. Dac˘a a = 1, atunci lim an+1 = lim x+n+1 = 1 ¸si aplic˘am criteriul n→∞ an n→∞ y+n+1 lui Raabe-Duhamel. ³ ´ an Avem lim n an+1 − 1 = y − x deci seria converge pentru y − x > 1 ¸si n→∞ x diverge pentru y − x < 1. Dac˘a y − x = 1 ⇔ y = x + 1, atunci an = x+n+1 P P1 ¸si ˆın acest caz, ¸ie cu limit˘a). n (conform criteriului dePcomparat P 1 an ∼ este divergent˘ a , rezult˘ a c˘ a seria a este de asemenea Cum seria n n divergent˘a. IV. Dac˘a a = −1, atunci an = (−1)n · bn , unde bn = x(x+1)·...·(x+n) y(y+1)·...·(y+n) > ∗ 0, ∀n ∈ IN . x+n+1 ∗ Avem bn+1 sirul (bn ) este descresc˘ator. bn = y+n+1 < 1, ∀n ∈ IN deci ¸ Aplicˆand ¸sirului (bn ) de dou˘a ori criteriul lui Stolz-Ces`aro (propozit ¸ia 2.30), P obt¸inem c˘a lim bn = 0. Conform criteriului lui Leibniz, seria an este n→∞ convergent˘a. √ n+1 √ 23. Not˘am cu an termenul general al seriei. Avem an+1 = · an n α−n α+n+1 , n ≥ 1. P I. Pentru α >³ 14 se aplic˘a´criteriul lui Raabe-Duhamel pentru seria |an | P n| ¸si avem: lim n |a|an+1 − 1 = 4α+1 > 1. Rezult˘a c˘a seria |an | este con2 | n→∞ P vergent˘a ¸si conform teoremei 3.33, seria an converge de asemenea. −1 |an+1 | II. Pentru α ≤ 4 , |an | ≥ 1, ∀n ≥ 1 deci ¸sirul |an | este cresc˘ator. Atunci P an 9 0 ¸si seria an diverge.

162 III. Pentru α = 14 , an = (−1)n−1 bn , unde bn = 4 · 0, n ≥ 2. S¸irul (bn ) este descresc˘ator deoarece

3·7·11··...·(4n−5) 5·9·13·...·(4n−3)

bn+1 bn

·

√ n 4n+1

>

≥ 1, ∀n ≥ 2. Pen-

3·7·11·...·(4n−5) 5·9·13·...·(4n−3) ≤ √ 8 √n , ∀n ≥ 2. Din teo(4n+1) 4n−1 P

tru a determina limita ¸sirului (bn ) se folose¸ste inegalitatea √ 2 , ∀n 4n−1

≥ 2, de unde rezult˘a c˘a 0 < bn ≤ rema cle¸stelui (propozit¸ia 2.26-vi)) obt¸inem lim bn = 0. Astfel seria n→∞ converge cu criteriul lui Leibniz.

an

IV. Pentru α ∈ (− 41 , 14 ), an = (−1)n · bn , unde bn =

Avem

√ (1 − α)(2 − α) . . . (n − 1 − α) n > 0, ∀n ≥ 2. (1 + α)(2 + α) . . . (n + α)

bn+1 bn

≤ 1, ∀n ≥ 2 deci (bn ) este descresc˘ator ¸si lim bn = 0. Conform n→∞ P criteriului lui Leibniz, seria an este convergent˘ a.

24. Notˆand cu an termenul general ´ al seriei, utiliz˘am criteriul lui Gauss ¡ ¢ ³ an 1 α b ¸si scriem: an+1 = 1 + n 1 + n+1 , n ≥ 1. Se aplic˘a formula lui Taylor ˆın punctul 0 cu restul Lagrange de ordinul 2 (teorema 6.26) ¡ funct ¢ ¸iei f (x) = ∗ 1 α (1 + x) ¸si obt¸inem: pentru orice n ∈ IN , exist˘a tn ∈ 0, n astfel ˆıncˆ at ¡ ¢ α(α−1) an 1 α α α−2 1+ n = 1 + n + 2n2 (1 + tn ) . Atunci vom putea scrie an+1 = α(α−1) xn α−2 este un ¸ sir m˘arginit. 1 + α+b n + n2 , n ≥ 1, unde xn = 2P (1 + tn ) Astfel, din criteriul lui Gauss, seria an converge pentru α + b > 1 ¸si diverge pentru α + b ≤ 1.

25. Fie an = (−1)n(n+1)/2 · bn , unde bn =

√ √ √ e· e· 3 e·...· n e nα

=

1

1

e1+ 2 +...+ n nα

. P

Dac˘a α ≤ 0, atunci an 9 0 deoarece lim bn = +∞. Astfel seria an n→∞ este divergent˘a. ∞ P Dac˘a α > 0, atunci se aplic˘a criteriul lui Dirichlet: seria (−1)n(n+1)/2 n=1

are ¸sirul sumelor part P¸iale m˘arginit iar ¸sirul (bn ) este descresc˘ator cu limita 0. Rezult˘a c˘a seria an converge. ¡ ¢n2 > 0, ∀n ∈ IN∗ ¸si aplic˘am criteriul 26. Not˘am an = (a − 2)n · 1 + n1 √ r˘ad˘acinii cu limit˘a: lim n an = (a − 2)e. n→∞ P an converge. I. Dac˘a (a − 2)e < 1 ⇔ a < 2 + 1e , atunci seria P 1 II. Dac˘a (a − 2)e > 1 ⇔ a > 2 + e , atunci seria an diverge. ¡ ¢n2 III. Dac˘a (a − 2)e = 1 ⇔ a = 2 + 1e , atunci an = e1n · 1 + n1 , n ∈ IN∗ .

163 ¢n+1 ¡ , ∀n ∈ IN∗ . Atunci avem: Conform problemei 2.5-b), e < 1 + n1 µ ¶ 1 (n+1)n 1 1 e < 1+ ⇒ n >¡ ¢(n+1)n ⇒ 1 n e 1+ n ¡ ¢n2 1 + n1 1 ¢ , ∀n ∈ IN∗ . ⇒ an > ¡ ¢ 2 +n = ¡ 1 n n 1 1 + 1+ n n

n

1 1 n n→∞ (1+ n )

Dar lim

=

1 e

de unde rezult˘a c˘a an 9 0 deci seria

P

an este diver-

gent˘a. 3.13

ln a1 n ln n . Din ln a1 ˆıncˆ at ln nn >

a) Alegem α astfel ˆıncˆat 1 < α < lim inf

definit¸ia limitei

inferioare rezult˘a c˘a exist˘a n0 ∈ IN∗ astfel α pentru orice 1 n ≥ n0 . Rezult˘a P c˘a an < nα , ∀n ≥ n0 ¸si conform criteriului de comparat¸ie de specia I, seria an converge. b) Se procedeaz˘a analog. c) Rezult˘a din a) ¸si b) folosind teorema 2.25. ³ ´ln(n+1) ln 1 1 a) Dac˘a not˘am an = n3 +n+3 , n ≥ 1, atunci lim lnann = n→∞ P ∞ > 1 deci seria an este convergent˘ a. P ln 1 b) Fie an = (ln1n)α , n ≥ 2. Atunci lim lnann = 0 < 1 deci seria an n→∞ diverge. ¯ ¯ √ √ ¯ ¯ 3.15 Fie an = ( n + a n + 1 + b) · cn , n ≥ 1. Avem lim ¯ an+1 ¯ = |c|. a n n→∞ P I. Dac˘a |c| < 1, atunci seriaP |an | converge conform criteriului raportului cu limit˘a. Atunci ¸si seria Pan converge (datorit˘a teoremei 3.33). II. Dac˘a |c| > 1, atunci seria an diverge conformPproblemei 3.5-(iii). III. Dac˘a |c| = 1 ¸si a 6= −1, atunci an 9 0 ¸si seria √an diverge. √ n IV. Dac˘ P a |c| = 1, a = −1 ¸si b 6= 0, atunci an = ( n − n + 1 + b) · c 9 0 ¸si seria an diverge. √ P √ V. Dac˘a c = 1, a = −1 ¸si b = 0, atunci an = n − n√ + 1 ¸si seria an ˆ este telescopic˘ a. In acest caz, sn = a1 + . . . + an = 1 − n + 1 → −∞ ¸si P seria an diverge. √ √ VI. Dac˘a c = −1, a = −1 ¸si b = 0, atunci an = ( n − n + 1) · (−1)n = P (−1)n+1 √ √ ¸si seria an converge conform criteriului lui Leibniz. n+ n+1 √ ˆIn cazul a = −c, b = 1 ¸si |c| < 1, sn = a1 +a2 +. . .+an = c−cn+1 n + 1+ 3.14



1−cn 1−c , n

≥ 1. Suma seriei este lim sn = n→∞

2c−c2 1−c .

164 1

q a+



n n , n ≥ 1. Dac˘a α + 12 < 0 ⇔ α < − 12 , atunci lim bn = 0. n→∞ √ Dac˘a α + 12 = 0 ⇔ α = − 12 , atunci lim bn = a. n→∞ Dac˘a α + 12 > 0 ⇔ α > − 12 , atunci lim bn = ∞. n→∞ p √ | α n b) an = n ( n + n) · x , n ≥ 1. Avem lim |a|an+1 n| n→∞ P

3.16

a) bn = nα+ 2

= |x|.

Dac˘a |x| < 1, atunci seria |an | converge conform Pcriteriului raportului cu limit˘a. Folosind teorema 3.33 obt¸inem c˘a ¸si seria an converge. P Dac˘a |x| > 1, atunci seria an diverge conform problemei 3.5-(iii). q q √ √ n 1 Pentru x = 1, an = −α− · 1 + , n ≥ 1 ¸ s i lim 1 + nn = 1. Din cri1 n n→∞ 2 n ∞ ∞ P P 1 teriul de comparat¸ie cu limit˘a se obt¸ine c˘a an ∼ care converge −α− 1 n=1

n=1 n

2

pentru −α − 12 > 1 ¸si diverge pentru −α − 21 ≤ 1. Pentru x = −1, an = (−1)n · bn , n ≥ 1. P Dac˘a α ≥ − 12 , atunci bn 9 0 (vezi punctul a)), deci an 9 0 ¸si seria an diverge. Dac˘a α < − 12 , atunci bn → 0 (conform a)). Pentru a demonstra c˘a ¸sirul (bn ) este descresc˘ ¸sirului (bn ) i se asociaz˘a funct¸ia f : [1, ∞) → IR, f (x) = p ator, √ bx = xα x + x (vezi observat¸ia 2.32) ¸si se studiaz˘a monotonia funct¸iei f .

1 √ xα− 2 [(4α+2) x+(4α+1)] √ √ . Cum α < − 12 , avem 4α + 2 < 0 deci 4 x+ x ´2 ³ 4α+1 0 f (x) < 0 pentru x > −4α−2 . Atunci f este descresc˘atoare pe intervalul ·³ ¶ ´2 4α+1 , ∞ . Rezult˘a c˘a exist˘a n0 ∈ IN∗ astfel ˆıncˆ at bn ≥ bn+1 , ∀n ≥ n0 −4α−2

Avem f 0 (x) =

∗ (adic˘a ¸sirul (bn ) este descresc˘ator ˆıncepˆ and cu un P anumit rang n0 ∈ IN ). Acum putem aplica criteriul lui Leibniz ¸si seria an converge.

3.17

Not˘am cu an termenul general al seriei.

Avem

an+1 an

=

2n+3 2n+1

·

n−a n+a+2

P

|an+1 | n→∞ |an |

· x, n ≥ 1 ¸si lim

= |x|.

I. Pentru |x| < 1, seria |anP | converge (din criteriul raportului cu limit˘a) ¸si conform teoremei 3.33, seria an converge. P II) Pentru |x| > 1, seria an diverge ³ conform´problemei 3.5-(iii). n| III) Pentru |x| = 1 ¸si a > 0, lim n |a|an+1 si din cri| − 1 = 2a + 1 > 1 ¸ n→∞ P P teriul lui Raabe-Duhamel, seria |an | converge deci ¸si seria an converge. 1 IV) Pentru |x| = P1 ¸si a ≤ − 2 , |an+1 | ≥ |an | pentru orice n ∈ IN. Rezult˘a c˘a an 9 0 ¸si seria an este divergent˘ a.

165 V) Pentru x = 1 ¸si a = 0 seria devine cu seria

∞ P n=1

∞ P n=1

1 n

2n+1 n(n+1)

care are aceea¸si natur˘a

(din criteriul de comparat¸ie cu limit˘a) deci seria diverge.

VI) Pentru x = 1 ¸si a ∈ (− 12 , 0), an = (−1)n · (2n+1)(1−a)(2−a)...(n−1−a) ¸si (1+a)(2+a)...(n+1+a) P folosind criteriul lui Leibniz seria an converge. ∞ P (2n+1)(−1)n VII) Pentru x = −1 ¸si a = 0 seria devine care este conn(n+1) n=1

vergent˘a cu criteriul lui Leibniz. 2(2a+2) an −2 VIII) Pentru x = −1 ¸si a ∈ (− 12 , 0), an+1 = 1 + 2n+3 + 2a+2 n−a − (2n+3)(n−a) . Se aplic˘a criteriul lui Gauss (corolarul P 3.27). Avem β = −1 + 2a + 2 = 2a + 1 < 1 de unde rezult˘a c˘a seria an diverge. 3.18

Rezult˘a din inegalit˘a¸tile mediilor: 1 an

2 an + an+1 √ ,n ≥ 1 ≤ an an+1 ≤ 1 2 + an+1

¸si din criteriul de comparat¸ie de specia I. 3.19

Not˘am cu (sn )n≥n0 ¸sirul sumelor part¸iale pentru seria P (tn )n≥n0 ¸sirul sumelor part¸iale pentru seria |an |. Avem

P n≥n0

an ¸si cu

n≥n0

(4) |sn | = |an0 +an0 +1 +. . .+an | ≤ |an0 |+|an0 +1|+. . .+|an | = tn , ∀n ≥ n0 . Conform observat¸iei 3.13-b), exist˘a lim tn ∈ [0, +∞]. Trecˆ and ˆın (4) la n→∞ P P limit˘a se obt¸ine | lim sn | ≤ lim tn , adic˘a | an | ≤ |an |. n→∞

n→∞

n≥n0

n≥n0

P 3.20 PEste suficient de ar˘atat c˘a seria an converge dac˘a ¸si numai dac˘a an seria converge. 1+an P an i) Presupunem c˘a seria an converge. Din inegalitatea 1+a < an , ∀n ∈ n P an ∗ IN ¸si din criteriul de comparat¸ie de specia I rezult˘a c˘a seria 1+an converge. P an an ii) Dac˘a seria a, atunci lim 1+a = 0. Rezult˘a 1+an este convergent˘ n n→∞

c˘a lim an = 0. De aici se obt¸ine lim aann = 1 ¸si conform criteriului de n→∞ n→∞ 1+an P comparat¸ie cu limit˘a, seria an este convergent˘ a. 3.21

a) Presupunem c˘a seria

∞ P n=1

este cresc˘ator ¸si majorat. Fie tn =

an este convergent˘ a. Rezult˘a c˘a ¸sirul (sn ) n P k=1

ak sk

¸sirul sumelor part¸iale pentru seria

166 ∞ P n=1

an sn .

Pentru c˘a s1 ≤ sk , ∀k ∈ IN∗ , avem tn ≤

n P

k=1 ∞ P

(tn ) este majorat. Conform teoremei 3.12, seria Pentru seria

∞ P n=1

b) Dac˘a seria

an s2n

ak s1

n=1

= an sn

sn s1 , ∀n

∈ IN∗ deci

este convergent˘ a.

se procedeaz˘a la fel.

∞ P n=1

an diverge, atunci ¸sirul (sn ) este nemajorat (din obser-

vat¸ia 3.13) ¸si sn → +∞. Rezult˘a c˘a pentru orice n ∈ IN∗ , exist˘a p ∈ IN∗ astfel ˆıncˆat sn+p ≥ 2sn . Utiliz˘am teorema lui Cauchy de caracterizare: P an an+p sn+p −sn an+1 an+2 ≥ 12 . Astfel seria Analog sn+1 + sn+2 + . . . + sn+p ≥ sn+p sn diverge. ¯ ¯ ∞ ¯ ¯ P a an se arat˘a pentru seria . Astfel ¯¯ as2n+1 + as2n+2 + . . . + s2n+p ¯¯ ≤ sna·sn+1 + s2 n+1 n+p n+1 n+2 n=1 n an+p an+2 1 1 1 1 1 + . . . + = − + − + . . . + sn+p−1 − s1n = sn+1 ·sn+2 sn+p−1 ·sn+p sn sn+1 sn+1 sn+2 1 1 ∗ 1 1 si conform teoremei lui Cauchy de sn − sn+p < sn , ∀p ∈ IN . Dar sn → 0 ¸ ∞ P an caracterizare, seria converge. s2 n=1 n

an a) S˘a not˘am bn = 1+a 3 , n ≥ 1. n ∞ P P I. Dac˘a seria an converge, atunci lim an = 0. Rezult˘a c˘a bn ∼ n→∞ n=1 P P an din criteriul de comparat¸ie cu limit˘a deci seria bn converge. ∞ P II. Dac˘a seria an diverge, atunci nu se poate spune nimic despre n=1 P n seria bn . Astfel, dac˘a an = n pentru orice n ≥ 1, atunci bn = 1+n si 3 ¸ ∞ ∞ P P P 1 bn ∼ (conform criteriului de comparat¸ie cu limit˘a) deci seria an n2 n=1 n=1 P converge. Iar dac˘a an = 1 pentru orice n ≥ 1, atunci bn = 12 ¸si seria bn este divergent˘a. b) Fie bn = 1+nanα an , n ≥ 1 ¸si (sn ) ¸sirul sumelor part¸iale pentru seria ∞ P bn . n=1 P I. Dac˘a seria an converge, atunci din faptul c˘a P bn < an , ∀n ≥ 1 ¸si din criteriul de comparat¸ie de specia I se obt¸ine c˘a seria bn converge. P II. Presupunem c˘a seria an diverge ¸si α > 1. Avem bn < n1α , ∀n ≥ 1. ∞ P 1 P Cum seria a, rezult˘a c˘a seria bn este convergent˘ a nα este convergent˘

3.22

n=1

(conform criteriului de comparat¸ie de specia I). P III. Presupunem c˘a seria an diverge ¸si α = 1.Avem mai multe situat¸ii:

167 1) S¸irul (nan ) este majorat. Atunci exist˘a M > 0 astfel ˆıncˆ at na n ≤ P an ∗ ∗ 1 M, ∀n ∈ IN . Se obt¸ine bn = 1+nan ≥ 1+M an , ∀n ∈ IN . Cum seria an P 1 P a diverge. Atunci seria diverge, rezult˘a c˘a ¸si seria b diverge, n 1+M n conform criteriului de comparat¸ie de specia I. an nan 2) nan → +∞. ˆIn acest caz, bn = 1+na = 1+na · n1 pentru orice n n nan n ∈ IN∗ , iar lim 1+na = 1. Conform criteriului de comparat¸ie cu limit˘a, n P P 1 n→∞ P1 P bn ∼ a, rezult˘a c˘a seria bn este n . Cum seria n este divergent˘ divergent˘a. 3) (nan ) nu este majorat ¸si nan 9 +∞. Atunci exist˘a (nk ank )k∈IN∗ un sub¸sir al ¸sirului (nan )n astfel ˆıncˆ at lim nk ank = +∞. Conform cazului 2), k→∞ ∞ P ank seria a. Notˆand cu (tk )k ¸sirul sumelor part¸iale 1+nk an este divergent˘ k

k=1

pentru seria

∞ P k=1

ank 1+nk ank ,

avem lim tk = +∞. Dar snk ≥ tk , ∀k ∈ IN∗ , de k→∞

unde rezult˘a c˘a lim snk = +∞. Deci (sn ) este divergent ¸si ˆın consecint¸˘ a, k→∞ P seria bn este divergent˘a. P IV. Presupunem c˘a seria an diverge ¸si α < 1. Atunci nα < n ¸si de an aici rezult˘a c˘a bn = 1+nanα an ≥ 1+na , ∀n ∈ IN∗ . Conform cazului III, seria n P an 1+na Pn diverge. Utilizˆand criteriul de comparat¸ie de specia I, obt¸inem c˘a seria bn diverge. 3.23

Se scrie an = nan · n1 , n ∈ IN∗ ¸si se utilizeaz˘a criteriul lui Abel.

¯ ¯ Conform ingalit˘a¸tii ¯ ann ¯ ≤ 12 a2n + 2n1 2 , n ≥ 1 ¸si criteriului de compara∞ P an ¸tie de specia I, seria n converge. 3.24

n=1

3.25

a) Seria

∞ P n=1

an fiind convergent˘ a, din criteriul lui Cauchy de ca-

racterizare rezult˘a: ∀ε > 0, ∃n0 (ε) = n0 ∈ IN∗ a.ˆı. ∀n ≥ n0 ¸si ∀m > n, an+1 + an+2 + . . . + am < 2ε . ˆIn particular, pentru m > 2n0 ¸si deoarece (an ) este descresc˘ator, avem: m 2 am

< (m − n0 )am < an0 +1 + an0 +2 + . . . + am
2n0 ⇒ nan → 0. ( b) ˆIntr-adev˘ar, pentru an =

1 k n, n = 3 , 1 , n 6= 3k n2

ε 2



168 seria

∞ P n=1

an este convergent˘ a, ¸sirul (an ) nu este descresc˘ator iar nan 9 0.

1 c) Reciproca este fals˘a. De exemplu, dac˘a an = n ln n pentru n ≥ 2, ∞ P an este divergent˘ a. atunci (an ) este descresc˘ator, lim nan = 0 dar seria n→∞

3.26

Presupunem c˘a seria

n=2

∞ P

an converge deci (sn ) este n=1 [sn ] este partea ˆıntreag˘ a a lui

convergent, strict

cresc˘ator ¸si majorat. Dac˘a sn , atunci [sn ] ≤ [sn+1 ], ∀n ∈ IN∗ deci ¸sirul ([sn ]) este de asemenea cresc˘ator. Din inegalitatea [sn ] ≤ sn , ∀n ∈ IN∗ , rezult˘a c˘a ¸sirul ([sn ]) este ¸si majorat ¸si prin urmare, este convergent. Atunci ¸sirul (tn ) este convergent. Presupunem c˘a ¸sirul (tn ) este convergent. Avem an+1 = sn+1 − sn = [sn+1 ] − [sn ] + tn+1 − tn , de unde obt¸inem lim ([sn+1 ] − [sn ]) = 0. Cum n→∞

[sn+1 ] − [sn ] ∈ IN pentru orice n ∈ IN∗ , rezult˘a c˘a ¸sirul ([sn ]) este constant de la un rang ˆıncolo deci este convergent. Dar sn = [sn ] + tn , ∀n ∈ IN∗ . ˆIn ∞ P consecint¸˘a, ¸sirul (sn ) este convergent. Astfel seria an converge. n=1

3.27 Aceast˘a solut¸ie se g˘ase¸ste ˆın Iaglom A.M., Iaglom I.M. [10]. ˆIn primul rˆand, din inegalitatea: sin x < x < tgx, ∀x ∈ (0, π ) rezult˘a 2 (5)

(6)

ctg2 x
0 pentru fiecare termen din 2 2 2 ∗ = 2(n+1) |cn | se obt¸ine: |cn | ≥ + + ... + n+2 ≥ 1, ∀n ∈ IN , n + 2 n + 2 n + 2 {z } | de unde rezult˘a c˘a seria



∞ P n=0

de n+1 ori cn este divergent˘ a.

b) Fie a1 = 2, a2 = 2, a3 = 22 , . . . , an = 2n−1 , . . . ¸si b1 = −1, bn = 1 ∞ ∞ P P pentru n ≥ 2. Evident seriile an ¸si bn sunt divergente. Avem: n=1

n=1

170

c1 = a1 b1 = −1 c2 = a1 b2 + a2 b1 = 0 ............................................ cn = a1 bn + a2 bn−1 + . . . + an−1 b2 + an b1 = 0 ............................................ ∞ P

deci seria

n=1

3.29

cn este absolut convergent˘ a. ∞ P

Seriile

n=0

1 n!

¸si

∞ P n=0

(−1)n n

sunt absolut convergente ¸si conform teore∞ P

mei lui Cauchy (3.39), seria produs Cauchy

n=0

¸si ∞ X

(11)

n=0

à cn =

∞ X 1 n!

n=0

Conform problemei 2.5-c), are loc n ≥ 1 deci

∞ P n=0



∞ P n=0

∞ X (−1)n

!

n!

n=0 1 n!

cn este absolut convergent˘ a

.

= e. Avem c0 = 1, cn = 0 pentru

cn = 1 ¸si din (11) rezult˘a c˘a

∞ P n=0

(−1)n n!

= 1e .

3.30 Se procedeaz˘a ca ˆın problema precedent˘ a, observˆand c˘a seria ∞ P −1 (−1)n · (√12)n este seria geometric˘a cu rat¸ia √ . 2

n=0

a ¸si are suma 3.31 a) Seria geometric˘a cu rat¸ia − 15 este absolut convergent˘ ∞ P (−1)n 51n = 1+1 1 = 56 . Se aplic˘a teorema lui Mertens (3.38) ¸si se obt¸ine: n=0 µ ¶ 5µ ∞ ¶ ∞ ∞ ¡ 5 ¢2 25 P n P (−1)n P (−1) · = (−1)n · n+1 = 36 . n n 5 5 5n = 6 n=0

n=0

n=0

Pentru b) ¸si c) se rat¸ioneaz˘ a analog. 3.32

S˘a not˘am an = xn , n ∈ IN. Seria

∞ P

xn este absolut convergent˘ a ¸si

n=0

se aplic˘a teorema lui Mertens 3.38. Avem cn =

n P k=0

ak · an−k = (n + 1)xn

pentru orice n ∈ IN, de unde rezult˘a concluzia problemei.

171 n

Pentru x ∈ IR not˘am an = xn! , n ∈ IN. Se aplic˘a criteriul raportu∞ ∞ P P lui cu limit˘a pentru seria |an | ¸si se obt¸ine c˘a seria an este absolut 3.33

n=0

n=0

yn n! , n

convergent˘a. Pentru y ∈ IR, fie bn =

∈ IN. Conform teoremei lui ∞ ∞ ∞ P P P bn , notat˘a cn , Mertens 3.38, seria produs Cauchy al seriilor an ¸si n=0 n=0 n=0 µ ¶ µ ¶ ∞ ∞ ∞ P P P este convergent˘a ¸si are loc: cn = an bn = E(x)E(y). Dar

cn =

n P k=0

ak · bn−k =

Rezult˘a c˘a enunt¸.

∞ P n=0

cn =

n=0

n P k=0 ∞ P n=0

xk k!

y n−k (n−k)!

·

(x+y)n n!

=

n=0 n P

1 n!

k=0

n=0

Cnk

· xk y n−k =

(x+y)n n! , ∀n

∈ IN.

= E(x + y) ¸si de aici se obt¸ine relat¸ia din

3.34 Folosind criteriul raportului cu limit˘a pentru seriile modulelor se obt¸ine c˘a ambele serii sunt absolut convergente. Pentru x, y ∈ IR, not˘am ∞ P y 2n x2n+1 an = (−1)n · (2n+1)! , bn = (−1)n · (2n)! , n ∈ IN. Dac˘a not˘am prin cn ∞ P

seria produs Cauchy al seriilor

bn , conform teoremei lui Cauchy µ∞ ¶µ ∞ ¶ ∞ ∞ P P P P 3.39, seria cn este absolut convergent˘ a ¸si cn = an bn = n=0

n=0

S(x)C(y). Avem cn =

n P k=0

an ¸si

n=0

∞ P

n=0

n=0

ak bn−k =

(−1)n (2n+1)!

n P

k=0

∞ ˆIn mod analog rezult˘a C(x)S(y) = P dn , unde dn = n=0

y 2n−2k+1 , n ∈ IN. Utilizˆand teorema 3.36 obt¸inem: S(x)C(y) + C(x)S(y) =

∞ X

n=0

n=0

2k+1 C2n+1 · x2k+1 · y 2n−2k , n (−1)n (2n+1)!

n P k=0

∈ IN.

2k C2n+1 · x2k ·

∞ X (−1)n (cn + dn ) = (x + y)2n+1 = S(x + y). (2n + 1)!

n=0

n=0

A doua relat¸ie se arat˘a analog. 3.35

Fie sn =

n P k=0

ak ¸si yn =

Rn 0

f (x)dx, n ∈ IN. Deoarece f este des-

cresc˘atoare, avem f (k + 1) ≤ f (x) ≤ f (k), ∀x ∈ [k, k + 1], ∀k ∈ IN. Cum f este continu˘a, rezult˘a Z (12)

ak+1 = f (k + 1) ≤

k

k+1

f (x)dx ≤ f (k) = ak , ∀k = 0, n, n ∈ IN.

172 Adunˆand inegalit˘a¸tile (12), rezult˘a c˘a

n P k=0

ak+1 ≤

R n+1 0

f (x)dx ≤

n P k=0

adic˘a sn+1 − a0 ≤ yn+1 ≤ sn , ∀n ∈ IN. De aici se obt¸ine u¸sor concluzia.

ak ,

Capitolul 4 NOT ¸ IUNI DE TOPOLOGIE ˆIN IR

Definit¸ia 4.1. Dac˘a x ∈ IR ¸si A ⊂ IR este o mult¸ime nevid˘a, se define¸ste distant¸a de la punctul x la mult¸imea A prin d(x, A) = inf{|x − a| | a ∈ A}. Diametrul mult¸imii A se define¸ste prin diam A = sup |x − y|.S x,y∈A

Distant¸a dintre dou˘a submult¸imi nevide A, B ⊂ IR se define¸ste prin d(A, B) = inf{|a − b| | a ∈ A, b ∈ B}. Se observ˘a c˘a ˆın definit¸ia 4.3 bila deschis˘ a (ˆınchis˘ a) depinde de metrica folosit˘a. Definit¸ia 4.2. a) Fie x ∈ IR. O submult¸ime V a lui IR se nume¸ste vecin˘ atate a lui x dac˘a exist˘a ε > 0 astfel ˆıncˆ at (x − ε, x + ε) ⊂ V ). b) O submult¸ime V ⊂ IR se nume¸ste vecin˘ atate a lui x = +∞ (respectiv x = −∞) dac˘a exist˘a a ∈ IR astfel ˆıncˆ at (a, +∞] ⊂ V (respectiv [−∞, a) ⊂ V ). Not˘am cu V(x) mult¸imea tuturor vecin˘ at˘ a¸tilor lui x ∈ IR. Definit¸ia 4.3. Fie x ∈ IR. O familie U de submult¸imi ale lui IR se nume¸ste sistem fundamental de vecin˘ at˘ a¸ti ale lui x dac˘ a: (a) U ⊂ V(x); (b) ∀V ∈ V(x), ∃U ∈ U a.ˆı. U ⊂ V. Definit¸ia 4.4. O submult¸ime D ⊂ IR se nume¸ste deschis˘ a dac˘a D = ∅ sau D este vecin˘atate pentru orice punct al s˘au. Not˘am cu τ0 familia mult¸imilor deschise ale lui IR. τ0 se nume¸ste topologia uzual˘a a lui IR. Dac˘a ∅ 6= X ⊂ IRp , atunci o mult¸ime A ⊂ X se nume¸ste deschis˘ a ˆın X dac˘a A = X ∩ D cu D ∈ τ0 . ˆIn acest caz, familia τ0 (X) = {X ∩ D | D ∈ τ0 } se nume¸ste topologia uzual˘a indus˘a pe X. 173

174 Definit¸ia 4.5. O submult¸ime F a lui IR se nume¸ste ˆınchis˘ a dac˘a IR\F este deschis˘a. Dac˘a ∅ 6= X ⊂ IRp , atunci o mult¸ime F ⊂ X se nume¸ste ˆınchis˘ a ˆın X dac˘a X\F ∈ τ0 (X). Teorema 4.6. (i) Orice reuniune de mult¸imi deschise este o mult¸ime deschis˘ a. (ii) Intersect¸ia unui num˘ ar finit de mult¸imi deschise este o mult¸ime deschis˘a. (iii) Orice intersect¸ie de mult¸imi ˆınchise este mult¸ime ˆınchis˘ a. (iv) Reuniunea unui num˘ ar finit de mult¸imi ˆınchise este mult¸ime ˆınchis˘ a. Definit¸ia 4.7. O mult¸ime A ⊂ IR se nume¸ste de tip Gδ dac˘ a se poate reprezenta ca o intersect¸ie num˘ arabil˘ a de mult¸imi deschise. O mult¸ime A ⊂ IR se nume¸ste de tip Fσ dac˘a se poate reprezenta ca o reuniune num˘arabil˘a de mult¸imi ˆınchise. Definit¸ia 4.8. O submult¸ime A a lui IR se nume¸ste m˘arginit˘ a dac˘a exist˘a ε > 0 astfel ˆıncˆ at A ⊂ [−ε, ε]. Definit¸ia 4.9. Fie A ⊂ IR. Un punct x ∈ IR se nume¸ste punct interior mult¸imii A dac˘a A ∈ V(x). ◦

Mult¸imea tuturor punctelor interioare ale lui A se noteaz˘a cu A. Uneori, din motive de scriere, vom mai nota interiorul unei mult¸imi A cu int A. ◦



Propozit¸ia 4.10. Fie A ⊂ IR. Atunci A este o mult¸ime deschis˘ a, A ⊂ A, ◦

iar A este deschis˘a dac˘a ¸si numai dac˘a A = A. Definit¸ia 4.11. Fie A ⊂ IR. Un punct x ∈ IR se nume¸ste punct aderent mult¸imii A dac˘a ∀V ∈ V(x), V ∩ A 6= ∅. Pentru ca x s˘a fie aderent mult¸imii A este necesar ¸si suficient ca pentru orice ε > 0, (x − ε, x + ε) ∩ A 6= ∅. Mult¸imea tuturor punctelor aderente ale lui A se noteaz˘a cu A ¸si se nume¸ste aderent¸a (sau ˆınchiderea) lui A. Propozit¸ia 4.12. Fie A ⊂ IR. Atunci A este o mult¸ime ˆınchis˘ a , A ⊂ A, iar A este ˆınchis˘a dac˘a ¸si numai dac˘a A = A. Propozit¸ia 4.13. Fie A ⊂ IR. Atunci au loc relat¸iile int(IR\A) = IR\A ¸si IR\A = IR\ int A.

175 Definit¸ia 4.14. Fie A ⊂ IR. Se nume¸ste frontiera mult¸imii A mult¸imea ◦

Fr A = A\A = A ∩ IR\A. Evident, Fr(IR\A) = Fr A. Definit¸ia 4.15. Fie A ⊂ IR. Un punct x ∈ IR se nume¸ste punct de acumulare pentru mult¸imea A dac˘ a ∀V ∈ V(x), (V \{x}) ∩ A 6= ∅. Dac˘a A ⊂ IR ¸si x = +∞ sau x = −∞ definit¸ia se p˘astreaz˘ a neschimbat˘ a. Pentru ca x s˘a fie punct de acumulare pentru mult¸imea A este necesar ¸si suficient ca pentru orice ε > 0, ((x − ε, x + ε)\{x}) ∩ A 6= ∅. Mult¸imea punctelor de acumulare ale mult¸imii A se noteaz˘a cu A0 ¸si se nume¸ste mult¸imea derivat˘a a lui A. Un punct din A care nu este de acumulare pentru mult¸imea A se nume¸ste punct izolat al mult¸imii A. Propozit¸ia 4.16. Fie A ⊂ IR. Atunci A = A ∪ A0 . Mult¸imea A este ˆınchis˘a dac˘a ¸si numai dac˘a A0 ⊂ A. Urm˘atoarele caracteriz˘ari ale punctelor aderente ¸si de acumulare cu ajutorul ¸sirurilor sunt foarte importante. Propozit¸ia 4.17. Fie A ⊂ IR ¸si x ∈ IR. (i) x este aderent mult¸imii A dac˘a ¸si numai dac˘a exist˘a un ¸sir de puncte din A convergent la x; (ii) x este punct de acumulare pentru mult¸imea A dac˘ a ¸si numai dac˘a exist˘a un ¸sir de puncte din A\{x} convergent la x. Definit¸ia 4.18. O submult¸ime A ⊂ IR se nume¸ste a) compact˘a dac˘a este m˘arginit˘ a ¸si ˆınchis˘ a; b) secvent¸ial compact˘a dac˘a orice ¸sir de puncte din A admite un sub¸sir convergent la un punct din A. Teorema 4.19. Fie A ⊂ IR. Urm˘atoarele afirmat¸ii sunt ehivalente: (i) A este compact˘a; (ii) A este secvent¸ial compact˘a; (iii) din orice acoperire cu mult¸imi deschise a lui AS se poate extrage o subacoperire finit˘a (adic˘a ∀(Di )i∈I ⊂ τ0 cu A ⊂ Di , exist˘ a J ⊂ I, J i∈I S finit˘a astfel ˆıncˆat A ⊂ Dj ). j∈J

176

Probleme 4.1 S˘a se determine aderent¸a, interiorul, mult¸imea derivat˘ a, frontiera, aderent¸a interiorului, interiorul aderent¸ei pentru mult¸imile de mai jos. S˘a se precizeze de asemenea ˆın fiecare caz dac˘a mult¸imea este m˘arginit˘ a sau compact˘a. a) A ⊂ IR, A = [2, 3) ∪ {4} ∪ (5, 7); b) A ⊂ IR, A = { n1 | n ∈ IN∗ }; nπ ∗ c) A ⊂ IR, A = { n+1 n sin 2 | n ∈ IN }; d) A ⊂ IR, A = (IR\Q)∩[−1, 5]; 4.2 Fie α, β : P(IR) → P(IR), α(A) = int A, β(A) = int A. Ar˘ atat¸i c˘a: a) dac˘a A este deschis˘ a, atunci A ⊂ α(A); b) dac˘a A este ˆınchis˘ a, atunci β(A) ⊂ A; c) ∀A ⊂ IR, α(α(A)) = α(A) ¸si β(β(A)) = β(A). Dat¸i exemplu de mult¸imi A pentru care mult¸imile A, int A, A, α(A), β(A), α(int A), β(A) sunt toate diferite ˆıntre ele. 4.3 Aratat¸i c˘a A ⊂ IR este deschis˘ a dac˘a ¸si numai dac˘a A ∩ Fr A = ∅ iar A este ˆınchis˘a dac˘a ¸si numai dac˘a Fr A ⊂ A. 4.4 Fie (xn ) un ¸sir de puncte din IR. Ar˘atat¸i c˘a mult¸imea L((xn )) = {x ∈ IR | x = lim xnk , (xnk ) sub¸sir al lui (xn )} este ˆınchis˘a. 4.5 Fie ¸sirul (xn ) ⊂ IR un ¸sir ¸si An = {xm | m ≥ n}. Ar˘ atat¸i c˘a (xn ) este convergent dac˘a ¸si numai dac˘a diam An → 0. 4.6 Fie a ∈ IR. Ar˘atat¸i c˘a mult¸imile de forma (a − 2ε, a + ε), ε > 0 formeaz˘a un sistem fundamental de vecin˘ at˘ a¸ti pentru punctul a. Aceea¸si 3 1 problem˘a pentru mult¸imile (a − n , a + n+1 ), n ∈ IN∗ . a L ∈ (0, 1) astfel 4.7 Fie f : IR→IR cu proprietatea c˘a exist˘a o constant˘ ˆıncˆat ∀x, y ∈ IR, |f (x) − f (y)| ≤ L|x − y|. Fie ¸sirul (xn ) definit prin x0 ∈ IRp , xn+1 = f (xn ), ∀n ≥ 0. Ar˘ atat¸i c˘a (xn ) este convergent, iar limita sa este un punct fix al lui f. Deducet¸i c˘a o funct¸ie cu proprietatea de mai sus admite un punct fix unic. R˘amˆ an adev˘arate rezultatele dac˘a L = 1?

177 4.8 S˘a se arate c˘a dac˘a D ⊂ IR este o mult¸ime deschis˘ a, iar A ⊂ IR este o mult¸ime arbitrar˘a, atunci mult¸imea A + D = {a + d | a ∈ A, d ∈ D} este deschis˘a. 4.9 Fie A, D dou˘a submult¸imi ale lui IR. S˘ a se arate c˘a dac˘a D este deschis˘a, atunci D ∩ A ⊂ D ∩ A. S˘ a se arate c˘a ipoteza asupra submult¸imii D este esent¸ial˘a. 4.10 Fie A, D dou˘a submult¸imi ale lui IR. S˘a se arate c˘a dac˘a D este deschis˘a, atunci D ∩ A 6= ∅ dac˘ a ¸si numai dac˘a D ∩ A 6= ∅. S˘ a se arate c˘a ipoteza asupra submult¸imii D este esent¸ial˘ a. 4.11 Fie A, D dou˘a submult¸imi ale lui IR. S˘a se arate c˘a dac˘a D este deschis˘a, atunci D + A = D + A. S˘a se arate c˘a ipoteza asupra submult¸imii D este esent¸ial˘a. a se arate c˘a A + B 4.12 Fie A, B dou˘a submult¸imi compacte ale lui IR. S˘ este compact˘a. Demonstrat¸i ˆın trei moduri: cu acoperiri deschise, cu ¸siruri, cu definit¸ia mult¸imilor compacte ˆın IR. 4.13 Ar˘atat¸i c˘a orice reuniune finit˘a de mult¸imi compacte este compact˘a ¸si orice intersect¸ie de mult¸imi compacte este mult¸ime compact˘a. Este orice reuniune de mult¸imi compacte o mult¸ime compact˘a? a 4.14 Fie (xn ) ⊂ IR un ¸sir convergent ¸si A = {xn | n ∈ IN} ∪ {lim xn }. S˘ se arate ca A este compact˘a. a. Atunci A ¸si A0 sunt compacte. 4.15 Fie A ⊂ IR o mult¸ime m˘arginit˘ 4.16 Fie A, B ⊂ IR mult¸imi nevide, disjuncte. S˘a se arate c˘a dac˘a A este ˆınchis˘a ¸si B compact˘a, atunci d(A, B) > 0. R˘amˆ ane adev˘arat˘ a proprietatea dac˘a B este doar ˆınchis˘a? 4.17 Ar˘atat¸i c˘a pentru orice submult¸ime A a lui IR exist˘a o submult¸ime B a lui IR cel mult num˘arabil˘a astfel ˆıncˆ at B ⊂ A ⊂ B. a A ⊂ IR exist˘a un ¸sir de 4.18 Ar˘atat¸i c˘a pentru orice mult¸ime ˆınchis˘ puncte (xn ) astfel ˆıncˆat L((xn )) = A (reciproca exercit¸iului 4.14). 4.19 S˘a se arate c˘a pentru orice submult¸ime a lui IR mult¸imea punctelor izolate este cel mult num˘arabil˘a.

178 4.20 Fie A ⊂ IR nevid˘a, cel mult num˘ arabil˘ a. S˘a se arate c˘a (IR\A)0 = IR. 4.21 Fie A ⊂ IR nenum˘ arabil˘ a. S˘a se arate c˘a A0 6= ∅. 4.22 Fie A ⊂ IR nevid˘a, cel mult num˘ arabil˘ a. S˘a se arate c˘a int A = ∅.

179

Solut¸ii 4.1 Pentru rezolvarea acestui exercit¸iu vom face uz de definit¸iile punctelor interioare, aderente, de acumulare f˘ar˘ a a le mai preciza de fiecare dat˘a. ◦

a) A = (2, 3) ∪ (5, 7), A = [2, 3] ∪ {4} ∪ [5, 7], A0 = [2, 3] ∪ [5, 7], Fr A = {2, 3, 4, 5, 7}. Evident, A ⊂ D(0, 7) = [−7, 7], deci A este m˘arginit˘ a. ¯ Mult¸imea A nu este compact˘a pentru c˘a nu este ˆınchis˘ a (A 6= A). b) Mult¸imea este format˘a din termenii unui ¸sir de numere reale cu limita 0. Pentru a deduce A, A0 putem utiliza ¸si caracterizarea cu ¸siruri a punctelor ◦

aderente ¸si de acumulare. Astfel A = ∅ deoarece nu exist˘a nici un interval deschis) inclus ˆın A; A = A ∪ {0}; A0 = {0}; Fr A = A. Mult¸imea A este m˘arginit˘a: A ⊂ D(0, 1) = [−1, 1] ¸si nu este compact˘a, nefiind ˆınchis˘ a. Problema se poate generaliza astfel: dac˘a (xn ) ⊂ IR, (xn ) este monoton, ◦

convergent la x ∈ IR ¸si A este mult¸imea termenilor s˘ai, atunci A = ∅, A = A ∪ {x}, A0 = {x}, Fr A = A. c) Mult¸imea este format˘a din termenii unui ¸sir divergent de numere reale. Pentru a studia punctele aderente este nevoie s˘a determin˘am punctele limit˘a nπ 4k+2 si ale ¸sirului. Fie xn = n+1 n sin 2 . Atunci x2k = 0, x4k+1 = 4k+1 → 1 ¸ ◦

x4k+3 = − 4k+4 4k+3 → 1. Prin urmare, A = ∅ (ca mai sus), A = A ∪ {−1, 1}, 0 A = {−1, 1}, Fr A = A. A este m˘arginit˘ a dar nu este compact˘a. d) T ¸ inˆand cont de densitatea mult¸imilor Q ¸si IR\Q ˆın IR se obt¸ine ◦

A = ∅ (ˆın orice interval deschis exist˘a atˆat numere rat¸ionale cˆat ¸si numere irat¸ionale), A = [−1, 5], A0 = [−1, 5], Fr A = [−1, 5]. Mult¸imea este m˘arginit˘a dar nu este ˆınchis˘a, deci nu este compact˘a. 4.2 a) Dac˘a A este deschis˘a, atunci int A = A ⊂ A, deci int A ⊂ int A = α(A). b) Dac˘a A este ˆınchis˘a, atunci A = A, deci β(A) = int A ⊂ int A ⊂ A. c) Deorece α(A) este deschis˘ a, α(A) ⊂ α(α(A)). Dar A este ˆınchis˘ a, deci β(A) ⊂ A, de unde rezult˘a int(β(A)) ⊂ α(A), adic˘a α(α(A)) ⊂ α(A). Are loc egalitatea. Pentru cealalt˘a egalitate se procedeaz˘a analog. Fie A = [0, 1) ∪ (1, 2] ∪ {3} ∪ [Q ∩ (4, 5)]. Aceast˘a mult¸ime verific˘ a propriet˘a¸tile cerute. 4.3

Avem echivalent¸ele: ◦



A ∩ Fr A = ∅ ⇔ A ∩ (A\A) = ∅ ⇔ A ∩ (A ∩ (IR\A)) = ∅ ⇔ (A ∩ A) ∩ ◦





(IR\A) = ∅ ⇔ A ∩ (IR\A) = ∅ ⇔ A ⊂ A ⇔ A deschis˘ a

180 ¸si A este ˆınchis˘a ⇔ IR\A este deschis˘ a ⇔ IR\A∩Fr(IR\A) = ∅ ⇔ Fr(IR\A) ⊂ A ⇔ Fr A ⊂ A. 4.4 Fie x ∈ IR\L((xn )). Exist˘a ε > 0 astfel ˆıncˆ at (x − ε, x + ε) nu cont¸ine nici un element al ¸sirului (xn ). Ar˘at˘ am c˘a (x − 2ε , x + 2ε ) ⊂ IR\L((xn )); din aceast˘a incluziune rezult˘a c˘a IR\L((xn )) este deschis˘ a, deci L((xn )) este ˆınchis˘a. ˆIntr-adev˘ar, dac˘a exist˘a y ∈ (x − 2ε , x + 2ε ) ∩ L((xn )), atunci (y − ε ε ¸ine o infinitate de termeni ai ¸sirului; cum (y − 2ε , y + 2ε ) ⊂ 2 , y + 2 ) ar cont (x − ε, x + ε), aceasta reprezint˘ a o contradict¸ie. 4.5 Remarc˘am c˘a dac˘a n ≤ p, atunci Ap ⊂ An , deci diam Ap ≤ diam An . Presupunem c˘a (xn ) este convergent; atunci (xn ) este ¸sir fundamental: pentru orice ε > 0, exist˘ a nε ∈ IN, ˆıncˆ at pentru orice n, m ≥ nε , |xn − xm | < ε. Aceasta ˆınseamn˘a c˘a supm,n≥nε |xn − xm | ≤ ε, adic˘a diam Anε ≤ ε. Cum pentru orice n ≥ nε , diam An ≤ diam Anε ≤ ε, rezult˘a c˘a diam An → 0. Invers, presupunem c˘a diam An → 0. Pentru orice ε > 0, exist˘a nε ∈ IN, ˆıncˆat pentru orice n ≥ nε , diam An < ε, adic˘a pentru orice m, p ≥ nε , |xm − xp | < ε. Prin urmare ¸sirul (xn ) este fundamental, deci convergent. 4.6 Vom verifica cele dou˘a propriet˘a¸ti ale sistemelor fundamentale de vecin˘at˘a¸ti. Astfel pentru orice ε > 0, (a − ε, a + ε) ⊂ (a − 2ε, a + ε) ¸si prin urmare (a − 2ε, a + ε) ∈ V(a). Fie acum V ∈ V(a); conform definit¸iei, exist˘a ε > 0 astfel ˆıncˆat (a − ε, a + ε) ⊂ V. Atunci intervalul (a − ε, a + 2ε ) are forma c˘autat˘a ¸si este inclus ˆın V. Prin urmare, familia (a − 2ε, a + ε), ε > 0 formeaz˘a un sistem fundamental de vecin˘ at˘ a¸ti pentru punctul a. 1 1 1 Evident, (a − n+1 , a + n+1 ) ⊂ (a − n3 , a + n+1 ), ∀n ∈ IN∗ , deci (a − n3 , a + 1 a ε > 0 astfel ˆıncˆ at (a − ε, a + ε) ⊂ V. n+1 ) ∈ V(a). Fie V ∈ V(a); exist˘ 3 1 C˘aut˘am un num˘ar natural n astfel ˆıncˆ at (a − n , a + n+1 ) ⊂ (a − ε, a + ε). 3 Este de ajuns s˘a g˘asim n cu proprietatea n ≤ ε. Un astfel de num˘ ar este n = [ 3ε ] + 1. Obt¸inem concluzia. 4.7 Fie x0 ∈ IR, xn+1 = f (xn ), ∀n ≥ 0. Ar˘at˘ am c˘a ¸sirul (xn ) este ¸sir Cauchy. Avem |x2 − x1 | = |f (x1 ) − f (x0 )| ≤ L|x1 − x0 |, |x3 − x2 | = |f (x2 ) − f (x1 )| ≤ L|x2 − x1 | ≤ L2 |x1 − x0 |. Prin induct¸ie rezult˘a c˘a |xn+1 − xn | ≤ Ln |x1 − x0 |, ∀n ∈ IN∗ .

181 Fie acum n, p ∈ IN∗ . Avem inegalit˘a¸tile |xn − xn+p | ≤ |xn − xn+1 | + |xn+1 − xn+2 | + ... + |xn+p−1 − xn+p | ≤ ≤ Ln |x1 − x0 | + Ln+1 |x1 − x0 | + ... + Ln+p−1 |x1 − x0 | = = |x1 − x0 |(Ln + Ln+1 + ... + Ln+p−1 ) = |x1 − x0 |Ln ≤ |x1 − x0 |

1 − Lp ≤ 1−L

Ln . 1−L

Dac˘a x1 = x0 , din relat¸ia de mai sus deducem c˘a ¸sirul este constant ¸si deci limita sa este x0 ; ˆın plus x0 = f (x0 ), adic˘ a x0 este punct fix al funct¸iei f. n Dac˘a x1 6= x0 , atunci deoarece lim L = 0 obt¸inem c˘a pentru orice ε > 0, exist˘a nε ∈ IN∗ astfel ˆıncˆat pentru orice n ≥ nε ¸si pentru orice p ∈ IN∗ are loc Ln |xn − xn+p | ≤ |x1 − x0 | < ε. 1−L Prin urmare ¸sirul (xn ) este fundamental, deci convergent. S˘a not˘am limita sa cu x. Avem: |x − f (x)| ≤ |x − xn+1 | + |xn+1 − f (x)| = |x − xn+1 |+ + |f (xn ) − f (x)| ≤ |x − xn+1 | + L|xn − x| → 0. Deducem c˘a |x − f (x)| = 0, adic˘a x este punct fix al funct¸iei f. Prespunˆ and ca ar exista dou˘a puncte fixe x, y, x 6= y ale funct¸iei f se obt¸ine contradict¸ia |x − y| = |f (x) − f (y)| ≤ L|x − y| < |x − y|. Punctul fix este unic. Dac˘a L = 1 rezultatele nu sunt adev˘arate. De exemplu funct¸ia f : IR → IR, f (x) = x+1 (L = 1) nu are puncte fixe ¸si orice ¸sir definit ca mai sus este divergent la +∞. 4.8

Putem scrie A+D =

S

(a+D) unde a+D = {a+d | d ∈ D}. Avˆand

a∈A

ˆın vedere c˘a orice reuniune de mult¸imi deschise este mult¸ime deschis˘ a, este suficient s˘a ar˘at˘am c˘a a+D este deschis˘ a pentru orice a ∈ A. Fie a+d ∈ a+D (i.e. d ∈ D). Cum D este deschis˘ a, exist˘a ε > 0 astfel ˆıncˆ at (d−ε, d+ε) ⊂ D. Dar a + (d − ε, d + ε) = B(a + d, ε); ˆıntr-adev˘ ar dac˘a y ∈ (a + d − ε, a − d + ε), atunci y−a ∈ (d−ε, d+ε), deci ky − a − dk < ε, adic˘ a y ∈ B(a+d, ε). Invers, dac˘a y ∈ (a + d − ε, a + d + ε), atunci |y − a − d| < ε, deci y − a ∈ (d − ε, d + ε), adic˘a y ∈ a + (d − ε, d + ε). Cum a + (d − ε, d + ε) ⊂ a + D, obt¸inem c˘a acest˘a mult¸ime este deschis˘a, adic˘a ceea ce doream s˘a ar˘at˘ am.

182 4.9 Fie x ∈ D ∩ A, adic˘ a x ∈ D ¸si x ∈ A. Fie V ∈ V(x). Atunci, cum D este deschis˘a, D ∈ V(x), deci D ∩ V ∈ V(x). Prin urmare A ∩ (D ∩ V ) 6= ∅, deci (A ∩ D) ∩ V 6= ∅. Cum vecin˘ atatea V a fost aleas˘a arbitrar, obt¸inem x ∈ D ∩ A. Dac˘a D nu este deschis˘ a incluziunea nu se p˘astreaz˘ a. Fie de exemplu D = (0, 1] ¸si A = (1, 2). D ∩ A = {1} ¸si D ∩ A = ∅. 4.10 Presupunem c˘a D ∩ A 6= ∅, adic˘a exist˘a x ∈ D ∩ A. Cum D este deschis˘a, D ∈ V(x) iar din x ∈ A obt¸inem D ∩ A 6= ∅. Presupunem acum c˘a D ∩ A 6= ∅. Cum D ∩ A ⊂ D ∩ A, rezult˘ a c˘a D ∩ A 6= ∅. Dac˘ a D nu este deschis˘a proprietatea nu mai este adev˘arat˘ a. Putem considera acela¸si exemplu ca la exercit¸iul precedent. 4.11 Este clar c˘a D + A ⊂ D + A. R˘ amˆ ane s˘a ar˘at˘ am incluziunea invers˘ a. Fie x ∈ D + A; exist˘a d ∈ D ¸si a ∈ A astfel ˆıncˆ at x = a + d. Cum a ∈ A, exist˘a un ¸sir (an ) ⊂ A, an → a. Dar a − an + d → d ∈ D ¸si cum D este deschis˘a, exist˘a n0 ∈ IN ˆıncˆ at pentru orice n ≥ n0 , a − an + d ∈ D. Atunci a + d = an + (a − an + d) ∈ A + D. Rezult˘a egalitatea dorit˘a. Dac˘a D nu este deschis˘a proprietatea nu este adev˘arat˘ a. Fie D = [0, 1] ¸si A = (0, 1]. D + A = (0, 2] ¸si D + A = [0, 2]. 4.12

Demonstrat¸ia este imediat˘a.

4.13

Fie A1 , A2 , ..., An mult¸imi compacte ¸si (xn ) ⊂

n S

Ak . Avˆ and ˆın

k=1

vedere c˘a mult¸imea termenilor ¸sirului este num˘ arabil˘ a ¸si c˘a avem un num˘ ar finit de mult¸imi, exist˘a o mult¸ime Ai care s˘a cont¸in˘ a un num˘ ar infinit de termeni ai ¸sirului. Atunci (xn ) are un sub¸sir inclus ˆın Ai . Deoarece Ai este compact˘a, exist˘a un sub¸sir al acestui sub¸sir (deci un sub¸sir al ¸sirului (xn )) n n S S convergent la un punct x ∈ Ai ⊂ Ak . A¸sadar orice ¸sir din Ak are un k=1 n S

sub¸sir convergent la un punct din

Ak , adic˘a

k=1

n S

k=1

Ak este compact˘a. O

k=1 n S

solut¸ie chiar mai simpl˘a poate fi dat˘a ar˘atˆ and c˘a

Ak este m˘arginit˘ a ¸si

k=1

ˆınchis˘a. O reuniune arbitrar˘a de mult¸imi compacte nu este compact˘a: de e∞ S xemplu [−n, n] = IR. Fie acum (Ai )i∈I o familie arbitrar˘a de mult¸imi n=1 T compacte. Evident Ai este ˆınchis˘ a (intersect¸ie de mult¸imi ˆınchise) ¸si i∈I T m˘arginit˘a (este inclus˘a ˆın Ai , ∀i ∈ I). Rezult˘a c˘a Ai este compact˘a. i∈I

183 4.14 Orice ¸sir de elemente din A este convergent la lim xn ∈ A, deci A este compact˘a. Remarc˘am c˘a pentru un ¸sir (xn ) convergent mult¸imea termenilor s˘ai nu este neap˘arat compact˘a, nefiind ˆınchis˘ a. De exemplu xn = n1 , n ∈ IN∗ . 4.15 Dac˘a A este m˘arginit˘a, exist˘a M > 0 astfel ˆıncˆ at A ⊂ D(0, M ). a), deci A0 ¸si A Atunci A0 ⊂ A ⊂ [−M, M ] ([−M, M ] este mult¸ime ˆınchis˘ sunt m˘arginite. Evident A este ˆınchis˘ a, deci este compact˘a. Fie x ∈ / A0 . Dac˘ a x ∈ A, atunci x este punct izolat al mult¸imii A, deci exist˘a ε > 0 astfel ˆıncˆ at (x − ε, x + ε) ∩ A = {x}. Fie y ∈ (x − 2ε , x + 2ε ). Cum (y − 2ε , y + 2ε ) ⊂ (x, ε), rezult˘a c˘a ( ³ ∅, dac˘ a y 6= x ε ε´ y − ,y + ∩A= 2 2 {x}, dac˘a y = x adic˘a y ∈ / A0 . Dac˘a x ∈ / A, atunci exist˘a ε > 0 astfel ˆıncˆ at (x−ε, x+ε)∩A = ∅. Fie y ∈ (x − 2ε , x + 2ε ). Cum (y − 2ε , y + 2ε ) ⊂ (x − ε, x + ε), rezult˘a c˘a (y − 2ε , y + 2ε ) ∩ A = ∅. Prin urmare, complementara lui A0 este deschis˘ a, deci A0 este ˆınchis˘a. Obt¸inem c˘a A0 este compact˘a. 4.16 Fie A, B mult¸imi compacte nevide. Presupunem prin reducere la absurd c˘a d(A, B) = 0, adic˘a inf |a − b| = 0. Din caracterizarea marginii a∈A,b∈B

inferioare, pentru orice n ∈ IN∗ , exist˘ a an ∈ A, bn ∈ B astfel ˆıncˆ at |an −bn | < 1 a, ¸sirul (bn ) admite un sub¸sir n , deci an − bn → 0. Deoarece B este compact˘ (bnk )k convergent la un punct x ∈ B. Atunci ank → x ∈ A. Cum A este ˆınchis˘a, rezult˘a c˘a x ∈ A, adic˘a x ∈ A ∩ B, contradict¸ie. Dac˘ nu se p˘astreaz˘ a. De exemplu √a mult¸imile sunt ˆınchise, proprietatea √ A = { n + 1 | n ∈ IN} ¸si √ B = { n | n ∈ IN}. Mult¸imile sunt ˆınchise ¸si √ nem˘arginite iar d(A, B) ≤ ( n + 1 − n) → 0, deci d(A, B) = 0. 4.17 Dac˘a A = ∅ lu˘am B = ∅ ¸si proprietatea este verificat˘ a. Presupunem c˘a A 6= ∅. Mult¸imea numerelor rat¸ionale este numarabil˘ a ¸si atunci o putem pune ˆın forma Q = {r1 , r2 , ..., rn ...}. Evident S Q = IR. Fie mult¸imile Bn,q = (rn − q, rn + q), q ∈ Q∗ . Atunci IR = Bn,q . Evident, n∈IN∗ ,q∈Q

A = A ∩ IR = A ∩

[

Bn,q =

n∈IN∗ ,q∈Q

[

(A ∩ Bn,q ).

n∈IN∗ ,q∈Q

Fie I = {(n, q) ∈ IN∗ × Q | A ∩ Bn,q 6= ∅}. Pentru fiecare i ∈ I, alegem xi ∈ A ∩ Bn,q ¸si lu˘am B = {xi | i ∈ I}. Evident, B este cel mult num˘ arabil˘ a ¸si B ⊂ A. Fie a ∈ A; cum Q = IR, pentru orice ε > 0, exist˘ a rn ∈ Qp astfel ˆıncˆat |a − rn | < 2ε . Fie q ∈ Q astfel ˆıncˆ at |a − rn | < q < 2ε . Rezult˘a c˘a

184 a ∈ Bn,q . Fie y elementul din A∩Bn,q . Atunci |a−y| ≤ |a−rn |+|rn −y| < ε. Rezult˘a c˘a putem construi un ¸sir de puncte din B cu limita a, adic˘a A ⊂ B. 4.18 Folosind exercit¸iul precedent, exist˘a o mult¸ime cel mult num˘ arabil˘ a B astfel ˆıncˆat A = B. Putem exprima mult¸imea B ˆın forma B = {bn | n = 1, 2, · · · }. Fie ¸sirul (xn ) = (b1 , b1 , b2 , b1 , b2 , b3 , b1 , b2 , b3 , b4 , b1 , b2 , b3 , b4 , b5 , · · · ) . Pentru acest ¸sir mult¸imea punctelor limit˘a coincide cu A. 4.19 Fie A ⊂ IR. Conform exercit¸iului 4.17, exist˘a B astfel ˆıncˆ at B ⊂ A ⊂ B. Fie a un punct izolat al mult¸imii A; atunci exist˘a V ∈ V(x) astfel ˆıncˆ at V ∩ A = {a}. Dar x ∈ B, deci B ∩ V 6= ∅. Dar B ∩ V ⊂ V ∩ A = {a}, ceea ce ˆınseamn˘a c˘a a ∈ B. A¸sadar mult¸imea punctelor izolate ale lui A este inclus˘a ˆın B, deci este cel mult num˘ arabil˘ a. 4.20 Presupunem prin reducere la absurd c˘a exist˘a x ∈ IR\(IR\A)0 . Atunci exist˘a ε > 0 astfel ˆıncˆ at (x − ε, x + ε)\{x} ∩ (IR\A) = ∅, ceea ce ˆınseamn˘ a c˘a (x − ε, x + ε)\{x} ⊂ A. Cum (x − ε, x + ε)\{x} este nenum˘ arabil˘ a am ajuns la o contradict¸ie. 4.21 Presupunem prin reducere la absurd c˘a A0 = ∅. Pentru orice n ∈ IN∗ definim 1 An = {x ∈ A | d(x, A\{x}) ≥ }. n ∞ ∞ S S Ar˘at˘am c˘a An = A : ˆın caz contrar, exist˘a x ∈ A\ An , adic˘a pentru n=1

n=1

orice n ∈ IN∗ , d(A\{x}, x) < n1 , deci exist˘a xn 6= x, xn ∈ A, cu|xn − x| < n1 . Aceasta implic˘a faptul c˘a xn → x, deci x ∈ A0 , contradict¸ie. Fie acum 1 1 1 x ∈ An . Atunci (x − 2n , x + 2n ) ∩ A = {x} iar dac˘a y ∈ A, y 6= x, (x − 2n , x+ 1 1 1 1 ) ∩ (y − , y + ) = ∅. Cum ). Dac˘ a Q = IR putem alege q ∈ Q ∩ B(x, x 2n 2n 2n 2n x 6= y, qx 6= qy . Definim funct¸ia injectiv˘a fn : An → Q, f (x) = qx . Cum Q este num˘arabil˘a, rezult˘a c˘a An este cel mult numarabil˘ a. Prin urmare, ∞ S An este cel mult num˘ arabil˘ a, deci A este cel mult num˘ arabil˘ a. Am ajuns n=1

la o contradict¸ie, deci problema este rezolvat˘ a. 4.22 Dac˘a interiorul mult¸imii A ar fi nevid, atunci A ar trebui s˘a cont¸in˘ a un interval deschis; cum orice interval deschis este nenum˘ arabil, rezult˘a c˘a nu poate fi cont¸inut ˆın A.

Capitolul 5 LIMITE DE FUNCT ¸ II. CONTINUITATE

Definit¸ia 5.1. Fie f : A → IR, A ⊂ IR ¸si a punct de acumulare pentru A. Spunem c˘a elementul l ∈ IR este limita funct¸iei f ˆın punctul a, dac˘ a pentru orice V ∈ V(l), exist˘a U ∈ V(a) astfel ˆıncˆ at dac˘a x ∈ U ∩ A, x 6= a, are loc f (x) ∈ V. ˆIn acest caz vom scrie lim f (x) = l. x→a

Teorema 5.2. Fie f : A → IR, A ⊂ IR, l ∈ IR ¸si a punct de acumulare pentru A. Urm˘atoarele afirmat¸ii sunt echivalente: (i) lim f (x) = l; x→a

at dac˘a |x − a| < δ, x ∈ (ii) pentru orice ε > 0, exist˘a δ > 0, astfel ˆıncˆ A, x 6= a are loc |f (x) − l| < ε (caracterizarea ε − δ); (iii) pentru orice (xn ) ⊂ A \ {a}, xn → a implic˘a f (xn ) → l (caracterizarea cu ¸siruri). Observat¸ia 5.3. Dac˘a exist˘a dou˘a ¸siruri (x0n ), (x00n ) ⊂ A\{a}, x0n → → a astfel ˆıncˆat f (x0n ) → l0 , f (x00n ) → l00 ¸si l0 6= l00 , atunci funct¸ia f nu are limit˘a ˆın a ∈ A0 . a, x00n

Definit¸ia 5.4. Fie a ∈ IR, A ⊂ IR ¸si not˘am As = A ∩ (−∞, a], Ad = A ∩ [a, ∞). Punctul a se nume¸ste punct de acumulare la stˆanga (respectiv dreapta) pentru A, dac˘a este punct de acumulare pentru mult¸imea As (respectiv Ad ). Vom nota mult¸imea punctelor de acumulare la stˆanga (respectiv dreapta) cu A0s (respectiv A0d ). Definit¸ia 5.5. Fie f : A → IR, A ⊂ IR ¸si a punct de acumulare la stˆanga (respectiv dreapta) pentru A. Spunem c˘a elementul l ∈ IR este limita la stˆanga (respectiv dreapta) a funct¸iei f ˆın punctul a dac˘ a pentru orice vecin˘atate V ∈ V(l) exist˘a U ∈ V(a), astfel ˆıncˆ at dac˘a x ∈ U ∩ As 185

186 (respectiv x ∈ U ∩ Ad ), x 6= a, are loc f (x) ∈ V. ˆIn acest caz vom scrie lim f (x) = l (respectiv x→a lim f (x) = l). x→a xa

Vom mai nota uneori x→a lim f (x) = ls ¸si x→a lim f (x) = ld . xa

Teorema 5.6. Fie f : I → IR, I ⊂ IR, I interval deschis, a ∈ I ¸si l ∈ IR. Atunci exist˘a lim f (x) = l dac˘a ¸si numai dac˘a exist˘a limitele laterale (la x→a stˆanga ¸si la dreapta) ˆın a ¸si sunt egale. ˆIn acest caz toate cele trei limite sunt egale: x→a lim f (x) = x→a lim f (x) = l. xa

Teorema 5.7. (Criteriul major˘ arii) Fie f : A → IR, A ⊂ IR, g : A → 0 IR ¸si a ∈ A . Dac˘a exist˘a l ∈ IR ¸si U ∈ V(a) astfel ˆıncˆ at |f (x) − l| ≤ g(x) pentru orice x ∈ U ∩ A, x 6= a ¸si lim g(x) = 0, atunci exist˘a lim f (x) = l. x→a

x→a

Teorema 5.8. (Trecerea la limit˘ a ˆın inegalit˘ a¸ti). a) Fie f, g : A → 0 IR, A ⊂ IR, a ∈ A astfel ˆıncˆ at exist˘a lim f (x) = l1 ¸si lim g(x) = l2 , finite. x→a

x→a

Dac˘a exist˘a U ∈ V(a) astfel ˆıncˆ at oricare ar fi x ∈ U ∩ A, x 6= a, f (x) ≤ g(x) (sau f (x) < g(x)) atunci l1 ≤ l2 . b) Fie f : A → IR, A ⊂ IR, a ∈ A0 astfel ˆıncˆ at exist˘a lim f (x) = l, x→a

finit˘a. Dac˘a exist˘a U ∈ V(a) astfel ˆıncˆ at oricare ar fi x ∈ U ∩ A, x 6= a, α ≤ f (x) ≤ β (sau α < f (x) < β) atunci α ≤ l ≤ β. Teorema 5.9. (Operat¸ii cu limite de funct¸ii) Dac˘a funct¸iile f, g : A → IR, A ⊂ IR, au limita finit˘a ˆın punctul a ∈ A0 , iar α ∈ IR, atunci funct¸iile f + g, αf, f g au limita ˆın a ¸si au loc relat¸iile: (i) lim (f (x) + g(x)) = lim f (x) + lim g(x), x→a

x→a

x→a

(ii) lim (αf (x)) = α lim f (x), x→a

x→a

(iii) lim (f (x)g(x)) = lim f (x) lim g(x). x→a

x→a

x→a

(iv) Dac˘a ˆın plus, lim g(x) 6= 0, atunci exist˘a U ∈ V(a) astfel ˆıncˆ at oricare x→a

ar fi x ∈ U ∩ A, x 6= a, s˘a avem g(x) 6= 0, funct¸ia lim f (x) x→a g(x)

=

f g

are limit˘a ˆın a ¸si

lim f (x)

x→a

lim g(x) .

x→a

Teorema 5.10. Fie f, g : A → IR, A ⊂ IR ¸si a ∈ A0 . Dac˘a lim f (x) = x→a

0 ¸si exist˘a U ∈ V(a) astfel ˆıncˆ at g este m˘arginit˘ a pe U , atunci exist˘a lim f (x)g(x) = 0. x→a

187 Teorema 5.11. Fie f : A → IR, A ⊂ IR ¸si a ∈ A0 . Dac˘ a exist˘a lim f (x) = x→a

l, l > 0 (respectiv l < 0), atunci exist˘a U ∈ V(a) astfel ˆıncˆ at pentru orice x ∈ U ∩ A, x 6= a, are loc f (x) > 0 (respectiv f (x) < 0). Teorema 5.12. Fie f : A → IR, A ⊂ IR ¸si a ∈ A0 . Dac˘ a exist˘a lim f (x) = l ∈ IR, atunci exist˘a U ∈ V(a) astfel ˆıncˆ at f este m˘arginit˘ a pe x→a U (adic˘a exist˘a M˙ > 0 astfel ˆıncˆ at pentru orice x ∈ U, are loc |f (x)| ≤ M ). Propozit¸ia 5.13. Fie l ∈ IR, f : A → IRq , A ⊂ IR, B ⊂ A ¸si a ∈ B 0 . Dac˘a exist˘a lim f (x) = l, atunci exist˘a ¸si lim (f |B ) (x) = l. ˆIn acest caz x→a

vom nota lim (f |B ) (x) = x→a lim f (x) . x→a

x→a

x∈B

Definit¸ia 5.14. Fie f : A → IR, A ⊂ IR ¸si a ∈ A0 . Spunem c˘a funct¸ia f are limita ∞ (respectiv −∞) ˆın punctul a, dac˘ a pentru orice V ∈ V(∞) (respectiv V ∈ V(−∞)), exist˘a U ∈ V(a) astfel ˆıncˆ at pentru orice x ∈ U ∩ A, x 6= a, are loc f (x) ∈ V. ˆIn acest caz vom scrie lim f (x) = ∞ (respectiv x→a

lim f (x) = −∞).

x→a

Teorema 5.15. Fie f : A → IR, A ⊂ IR ¸si a ∈ A0 . Atunci exist˘a lim f (x) = ∞ (respectiv lim f (x) = −∞), dac˘a ¸si numai dac˘a oricare ar fi

x→a

x→a

ε > 0, exist˘a δε > 0, astfel ˆıncˆ at dac˘a |x − a| < δε , x ∈ A, x 6= a are loc f (x) > ε (respectiv f (x) < −ε).

Definit¸ia 5.16. Fie f : A → IR, A ⊂ IR, astfel ˆıncˆ at ∞ (respectiv − ∞) este punct de acumulare pentru A. Spunem c˘a elementul l ∈ IR este limita funct¸iei f ˆın punctul ∞ (respectiv −∞), dac˘ a pentru orice V ∈ V(l), exist˘a U ∈ V(∞) (respectiv U ∈ V(−∞)) astfel ˆıncˆ at pentru orice x ∈ U ∩ A, are ˆ loc f (x) ∈ V. In acest caz vom scrie lim f (x) = l (respectiv lim f (x) = l). x→∞

x→−∞

Teorema 5.17. Fie l ∈ IR, f : A → IR, A ⊂ IR, astfel ˆıncˆ at ∞ (respectiv −∞) este punct de acumulare pentru A. Atunci exist˘a lim f (x) = l (respectiv

x→∞

lim f (x) = l) dac˘a ¸si numai dac˘a oricare ar fi ε > 0, exist˘a

x→−∞

δ > 0, astfel ˆıncˆat dac˘a x > δ (respectiv x < −δ), x ∈ A are loc |f (x) − l| < ε. Definit¸ia 5.18. Fie f : A → IR, A ⊂ IR, astfel ˆıncˆ at ∞ (respectiv − ∞) este punct de acumulare pentru A. Spunem c˘a limita funct¸iei f ˆın punctul

188 ∞ (respectiv −∞ ) este ∞, dac˘a pentru orice V ∈ V (∞) , exist˘ a U ∈ V (∞) (respectiv U ∈ V (−∞)) astfel ˆıncˆ at pentru orice x ∈ U ∩A, are loc f (x) ∈ V. ˆIn acest caz vom scrie lim f (x) = ∞ (respectiv lim f (x) = ∞). x→∞

x→−∞

Teorema 5.19. Fie f : A → IR, A ⊂ IR, astfel ˆıncˆ at ∞ (respectiv −∞) este punct de acumulare pentru A. Atunci exist˘a lim f (x) = ∞ (respectiv x→∞

lim f (x) = ∞) dac˘a ¸si numai dac˘a oricare ar fi ε > 0, exist˘a δε > 0, astfel

x→−∞

ˆıncˆat dac˘a x > δε (respectiv x < −δε ), x ∈ A are loc f (x) > ε. Teorema 5.20. Fie f, g : A → IR, A ⊂ IR, a ∈ A0 ¸si exist˘a lim f (x) = ∞ x→a

(respectiv lim g(x) = −∞). Dac˘a exist˘a U ∈ V(a) astfel ˆıncˆ at oricare ar fi x→a

x ∈ U ∩ A, x 6= a, f (x) ≤ g(x), atunci exist˘a lim g(x) = ∞ (respectiv x→a

lim f (x) = −∞).

x→a

Definit¸ia 5.21. Fie f : A → IR, A ⊂ IR astfel ˆıncˆ at ∞ (respectiv −∞) este punct de acumulare pentru A. Spunem c˘a dreapta y = l, l ∈ IR, este asimptot˘a orizontal˘a la +∞ (respectiv −∞) pentru funct¸ia f dac˘ a exist˘a lim f (x) = l (respectiv lim f (x) = l).

x→∞

x→−∞

Definit¸ia 5.22. Fie f : A → IR, A ⊂ IR astfel ˆıncˆ at ∞ (respectiv −∞) este punct de acumulare pentru A. Spunem c˘a dreapta y = mx + n, m, n ∈ IR, m 6= 0, este asimptot˘a oblic˘a la +∞ (respectiv −∞) pentru funct¸ia f , dac˘a exist˘a lim |f (x) − mx − n| = 0 (respectiv lim |f (x) − mx − n| = 0). x→∞

x→−∞

Teorema 5.23. Fie f : A → IR, A ⊂ IR astfel ˆıncˆ at ∞ (respectiv −∞) este punct de acumulare pentru A. Dreapta y = mx + n, m, n ∈ IR, m 6= 0, este asimptot˘a oblic˘a la +∞ (respectiv −∞) pentru funct¸ia f dac˘a ¸si numai si lim (f (x) − mx) = n (respectiv lim f (x) dac˘a exist˘a lim f (x) x =m¸ x =m x→∞

x→∞

x→−∞

¸si lim (f (x) − mx) = n). x→−∞

Definit¸ia 5.24. a) Spunem c˘a dreapta x = a este asimptot˘a vertical˘ a la stˆanga (respectiv dreapta) pentru funct¸ia f : A → IR, A ⊂ IR, a ∈ A0s (a ∈ A0d ) dac˘a exist˘a x→a lim f (x) = +∞ sau −∞ (respectiv x→a lim f (x) = +∞ sau −∞). x>a

x 0; x→0

v) lim sinx x = 1; x→0

vi) lim tgx x = 1; x→0

x vii) lim arcsin = 1; x x→0

viii) lim arctgx = 1; x x→0

ln x = 0; x→∞ x n lim xx = 0, n x→∞ a

ix) lim x)

∈ IN, a > 1.

Observat¸ia 5.26. Eliminarea nedetermin˘arilor se face, de obicei, astfel: (i) Cazurile 00 , ±∞ a fie folosind limitele fundamentale, fie cu ±∞ se elimin˘ 0 regula lui L Hˆospital (vezi capitolul 6). ±∞ (ii) Cazurile 0 · ∞, 0 · (−∞) se reduc la cazurile 00 , ±∞ astfel: fie 0 f, g : A → IR, a ∈ A , astfel ˆıncˆ at exist˘a lim f (x) = 0, lim g(x) = x→a

x→a

∞(−∞) ¸si exist˘a U ∈ V(a) astfel ˆıncˆ at, f (x) 6= 0, g(x) 6= 0, ∀x ∈ f (x) U \ {a}; putem scrie f (x)g(x) = 1 ¸si astfel nedeterminarea dat˘a g(x)

se reduce la o nedeterminare nedeterminare

0 0

sau f (x)g(x) =

∞ ∞.

g(x) 1 f (x)

¸si vom obt¸ine o

(iii) Cazul ∞−∞ se reduce, de obicei, la cazul 0·∞ astfel: fie f, g : A → IR, a ∈ A0 , astfel ˆıncˆat exist˘a lim f (x) = ∞, lim g(x) = ∞ ¸si exist˘a x→a

x→a

U ∈ V(a) astfel ˆıncˆat f (x) 6= 0, ∀x ∈ U \ {a}; putem scrie (f (x) − g(x)) = f (x)(1 − fg(x) a lim fg(x) a (x) ). Dac˘ (x) = 1, atunci nedeterminarea dat˘ x→a

se reduce la o nedeterminare 0 · ∞; dac˘a lim fg(x) (x) > 1(< 1), atunci lim (f (x) − g(x)) = −∞(+∞).

x→a

x→a

190 (iv) Cazurile 00 , ∞0 se reduc la cazul 0 · ∞ astfel: fie f, g : A → IR, a ∈ A0 , astfel ˆıncˆ at exist˘a lim f (x) = 0 (+∞), lim g(x) = 0 ¸si ex→a

x→a

xist˘a U ∈ V(a) astfel ˆıncˆ at f (x) > 0, ∀x ∈ U ∩ A \ {a}. Putem scrie f (x)g(x) = eg(x) ln f (x) ¸si limita de la exponent este o nedeterminare 0 · (−∞) (respectiv 0 · ∞). (v) Cazul 1∞ se reduce tot la cazul 0 · ∞ fie prin metoda de la punctul iv), fie astfel: dac˘a f, g : A → IR, a ∈ A0 , astfel ˆıncˆ at exist˘a lim f (x) = 1, x→a

lim g(x) = ∞ ¸si exist˘a U ∈ V (a) astfel ˆıncˆ at , f (x) 6= 1, ∀x ∈ U \ {a},

x→a

1

atunci vom scrie f (x)g(x) = {[1 + (f (x) − 1)] f (x)−1 }g(x)(f (x)−1) . Avem 1

lim [1+(f (x)−1) f (x)−1 = e, iar lim g(x)(f (x)−1) este o nedeterminare x→a x→a 0 · ∞. Definit¸ia 5.27. Fie f : A → IR, A ⊂ IR ¸si a ∈ A. Spunem c˘a funct¸ia f este continu˘a ˆın punctul a dac˘ a oricare ar fi V ∈ V(f (a)), exist˘a U ∈ V(a) astfel ˆıncˆat pentru orice x ∈ U ∩ A, are loc f (x) ∈ V. Dac˘a f nu este continu˘ a ˆın a, vom spune c˘a f este discontinu˘ a ˆın a sau c˘a a este punct de discontinuitate al funct¸iei f . Teorema 5.28. Fie f : A → IR, A ⊂ IR ¸si a ∈ A0 ∩ A. Atunci f este continu˘a ˆın a dac˘a ¸si numai dac˘a exist˘a lim f (x) = f (a). x→a

Teorema 5.29. Fie f : A → IR, A ⊂ IR ¸si a ∈ A. Urm˘atoarele afirmat¸ii sunt echivalente: (i) f este continu˘a ˆın a; (ii) pentru orice ε > 0, exist˘ a δ > 0, astfel ˆıncˆ at dac˘a |x − a| < δ, x ∈ A, s˘a rezulte |f (x) − f (a)| < ε (caracterizarea ε − δ); (iii) pentru orice (xn ) ⊂ A, xn → a implic˘ a f (xn ) → f (a) (caracterizarea cu ¸siruri). Definit¸ia 5.30. Fie f : A → IR, A ⊂ IR ¸si a ∈ A. Spunem c˘a funct¸ia f este continu˘a la stˆanga (respectiv dreapta) ˆın punctul a, dac˘a oricare ar fi V ∈ V(f (a)), exist˘ a U ∈ V(a) astfel ˆıncˆ at dac˘a x ∈ U ∩ As (respectiv x ∈ U ∩ Ad ) are loc f (x) ∈ V. Teorema 5.31. Funct¸ia f : A → IR, A ⊂ IR este continu˘ a la stˆanga (respectiv la dreapta) ˆın a ∈ A, punct de acumulare la stˆanga (respectiv la

191 dreapta) pentru A, dac˘a ¸si numai dac˘a exist˘a x→a lim f (x) = f (a) (respectiv lim f (x) = f (a)). x→a

xa

Teorema 5.32. Fie f : I → IR, I ⊂ IR, I interval deschis ¸si a ∈ I. Atunci f este continu˘a ˆın a dac˘ a ¸si numai dac˘a f este continu˘ a la stˆanga ¸si la dreapta ˆın a. Definit¸ia 5.33. Fie f : I → IR, I ⊂ IR, I interval, a ∈ I ¸si presupunem c˘a exist˘a V ∈ V(a) astfel ˆıncˆat f este m˘arginit˘ a pe V ∩ I. Numim oscilat¸ia funct¸iei f ˆın punctul a num˘arul ωf (a) = inf {diam (f (V ∩ I)) |V ∈ V (a)} . Pentru ∅ = 6 A ⊂ R, se noteaz˘a diam A = sup{d(x, y) | x, y ∈ A}, numit diametrul mult¸imii A. Teorema 5.34. Fie f : I → IR, I ⊂ IR, I interval, a ∈ I. Funct¸ia f este continu˘a ˆın a dac˘a ¸si numai dac˘a ωf (a) = 0. Definit¸ia 5.35. Spunem c˘a funct¸ia f : A → IR, A ⊂ IR este continu˘ a (pe A), dac˘a f este continu˘a ˆın orice punct a ∈ A. Teorema 5.36. (de caracterizare a continuit˘ a¸tii globale) Dac˘ a A ⊂ IR, f : A → IR, atunci urm˘atoarele afirmat¸ii sunt echivalente: (i) f este continu˘a pe A; (ii) pentru orice mult¸ime deschis˘ a D ⊂ IR, f −1 (D) este mult¸ime deschis˘ a ˆın A; (iii) pentru orice mult¸ime ˆınchis˘ a F ⊂ IR, f −1 (F ) este mult¸ime ˆınchis˘ a ˆın A; A A (iv) pentru orice mult¸ime B ⊂ A, are loc f (B ) ⊂ f (B) (unde notat¸ia B semnific˘a aderent¸a mult¸imii B ˆın topologia indus˘a pe A. Amintim c˘a A B = {x ∈ A|D ∩ B 6= ∅, ∀D ∈ τ0 (A) cu x ∈ D}. ) Teorema 5.37. Dac˘a f, g : A → IR, A ⊂ IR sunt funct¸ii continue pe A, λ ∈ IR atunci: (i) f ± g, λf sunt funct¸ii continue pe A; (ii) f · g este continu˘a pe A;

192 (iii)

f g

este continu˘a pe mult¸imea A\{x ∈ A | g(x) = 0};

(iv) |f |, min(f, g), max(f, g) sunt funct¸ii continue pe A. Teorema 5.38. Fie f : A → B ⊂ IR, A ⊂ IR, g : B → IRs . (i) Dac˘a f este continu˘ a ˆın a ∈ A, iar g este continu˘ a ˆın f (a), atunci g ◦ f este continu˘a ˆın a. a pe A, iar g este continu˘ a pe B atunci g ◦ f este (ii) Dac˘a f este continu˘ continu˘a pe A. Teorema 5.39.

Dac˘a f : A\{a} ( → IR, A ⊂ IR, a ∈ IR ¸si exist˘a f (x), x ∈ A\{a} lim f (x) = l ∈ IR, atunci funct¸ia fe(x) = este continu˘ a x→a l, x=a ˆın a (fe se nume¸ste prelungirea prin continuitate a funct¸iei f ). Definit¸ia 5.40. Funct¸ia f : IR → IR, se nume¸ste homeomorfism dac˘a f este bijectiv˘a ¸si f, f −1 sunt funct¸ii continue. Definit¸ia 5.41. Fie f : I → IR, I ⊂ IR, I interval ¸si a ∈ I. Spunem c˘a a este punct de discontinuitate de specia I dac˘a exist˘a limitele laterale ˆın a ¸si ele sunt finite. ˆIn caz contrar vom spune c˘a a este punct de discontinuitate de specia a II-a. Teorema 5.42. Dac˘a f : I → IR, I ⊂ IR, I interval, este o funct¸ie monoton˘a atunci: (i) punctele sale de discontinuitate sunt de specia I; arabil˘ a. (ii) mult¸imea acestor puncte este cel mult num˘ Teorema 5.43. Dac˘a A ⊂ IR, f : A → IR este continu˘ a ¸si K este o submult¸ime compact˘a ˆın IR, K ⊂ A, atunci f (K) este compact˘a ˆın IR. Teorema 5.44. (Weierstrass) Dac˘a f : K → IR este continu˘ a, K ⊂ IR, K o mult¸ime compact˘a, atunci f este m˘arginit˘ a pe K ¸si ˆı¸si atinge marginile (adic˘a exist˘a a, b ∈ K, astfel ˆıncˆ at sup f (x) = f (a) ¸si inf f (x) = f (b)). x∈K

x∈K

Definit¸ia 5.45. Spunem c˘a funct¸ia f : I → IR, I ⊂ IR, I interval, are proprietatea lui Darboux dac˘a pentru orice a, b ∈ I, a < b ¸si orice λ ∈ (f (a), f (b)) sau λ ∈ (f (b), f (a)) exist˘a cλ ∈ (a, b) astfel ˆıncˆ at f (cλ ) = λ. Teorema 5.46. Fie I ⊂ IR, I interval. Dac˘a funct¸ia f : I → IR, are proprietatea lui Darboux ¸si a, b ∈ I, a < b, astfel ˆıncˆ at f (a)f (b) < 0, atunci ecuat¸ia f (x) = 0 are cel put¸in o solut¸ie ˆın intervalul (a, b).

193 Teorema 5.47. Fie I ⊂ IR, I interval. Funct¸ia f : I → IR are proprietatea lui Darboux dac˘a ¸si numai dac˘a pentru orice J ⊂ I, J interval, f (J) este interval. Teorema 5.48. Fie I ⊂ IR, I interval. Dac˘a f : I → IR este continu˘ a, atunci f are proprietatea lui Darboux. Teorema 5.49. Fie I ⊂ IR, I interval. Dac˘a f : I → IR are proprietatea lui Darboux, atunci f are numai discontinuit˘ a¸ti de specia a II-a. Teorema 5.50. Fie I ⊂ IR, I interval. Dac˘a f : I → IR are proprietatea Darboux ¸si f este monoton˘a, atunci f este continu˘ a. Teorema 5.51. Fie I, J ⊂ IR, I, J intervale. Dac˘a f : I → J, f surjectiv˘a, continu˘a ¸si strict monoton˘a, atunci f este inversabil˘ a ¸si f −1 este continu˘a. Definit¸ia 5.52. Funct¸ia f : A → IR, A ⊂ IR se nume¸ste uniform continu˘a pe mult¸imea B ⊂ A dac˘a pentru orice ε > 0, exist˘ a δ > 0 astfel ˆıncˆ at pentru orice x, y ∈ B, |x − y| < δ are loc |f (x) − f (y)| < ε. Teorema 5.53. Fie f : A → IR, A ⊂ IR o funct¸ie uniform continu˘ a pe A ¸si B ⊂ A. Atunci: a) f este continu˘a pe A; b) f este uniform continu˘a ¸si pe mult¸imea B. Definit¸ia 5.54. Funct¸ia f : A → IR, A ⊂ IR se nume¸ste lipschitzian˘ a, dac˘a exist˘a L > 0 astfel ˆıncˆat: |f (x)−f (y)| ≤ L|x−y|, pentru orice x, y ∈ A (L se nume¸ste constanta Lipschitz a funct¸iei f ). Teorema 5.55. Dac˘a f : A → IR, A ⊂ IR este lipschitzian˘ a, atunci f este uniform continu˘a. Teorema 5.56. (Cantor). Dac˘a f : A → IR, A ⊂ IR este continu˘ a, iar A este compact˘a, atunci f este uniform continu˘ a. Teorema 5.57. Fie f : A → IR, A ⊂ IR, A m˘ arginit˘ a, f continu˘ a. Atunci f este uniform continu˘a dac˘a ¸si numai dac˘a f se poate prelungi prin continuitate la A.

194

Probleme 5.1 Fie f : IR → IR, f (x) = 3x − 2. Demonstrat¸i c˘a: a) exist˘a δ¡> 0 astfel ˆıncˆ at ¢dac˘a x ∈ (2 − δ, 2 + δ), atunci 1 1 f (x) ∈ 4 − 100 , 4 + 100 ; b) exist˘a δ1 > 0 astfel ˆıncˆ at dac˘a |x − 2| < δ1 , atunci |f (x) − 4| < c) demonstrat¸i, folosind caracterizarea ε − δ, c˘a lim f (x) = 4.

1 1000 ;

x→2

at dac˘a |x − 1| < δ atunci 5.2 a) Dovedit¸i c˘a exist˘a δ > 0 astfel ˆıncˆ µ ¶ 1 1 2 x +x∈ 2− ,2 + . 100 100 b) Demonstrat¸i, folosind caracterizarea ε − δ iar apoi caracterizarea cu ¸siruri, c˘a lim (x2 + x) = 2. x→1

5.3 Demonstrat¸i c˘a: a) exist˘a α ∈ IR+ astfel ˆıncˆ at dac˘a x > α, s˘a rezulte µ ¶ 2x + 3 1 1 ∈ 2 − 3,2 + 3 ; x+4 10 10 2x+3 x→∞ x+4 lim x+1 x→∞ 2x+3

b) lim

= 2, folosind caracterizarea ε − δ;

c)

= 12 , folosind definit¸ia.

5.4 Folosind caracterizarea ε−δ, iar apoi caracterizarea cu ¸siruri, dovedit¸i c˘a: 2 +1 = ∞; a) lim xx+1 x→∞

b)

2 lim x +2 x→−∞ 2x+1

= −∞.

5.5 Demonstrat¸i c˘a exist˘a α, m ≥ 0, astfel ˆıncˆ at pentru orice x ≥ α ¯ 3 ¯ ¯ 2x + 5x + 1 ¯ m ¯ ¯ ¯ 5x7 + 2x3 + 1 ¯ ≤ x4 . 5.6 Demonstrat¸i c˘a funct¸iile urm˘atoare nu au limit˘a ˆın punctele indicate: a) f (x) = sin x la ∞; b) f (x) = cos x21−x ˆın 0; £ ¤ c) f (x) = x1 ˆın 0, la dreapta. 5.7 Calculat¸i limitele laterale ˆın punctele precizate:

195 1

a) f (x) = e 4−x2 , x = 2; 1 b) f (x) = , x = 1; 1 x2 −1 1+2 £ ¤ c) f (x) = x x1 , x = 0; ¯ ¯ ¯ ¯ d) f (x) = ln ¯ 12 − sin(x−1) , x = 1. 2 (x−1) ¯ ospital: 5.8 Calculat¸i, f˘ar˘a a folosi regula lui L 0 Hˆ 1−cos3 x a) lim sin2 x ; x→0 ³ ´ 2 2sin x −1 ln(1+x2 )

b) lim

arcsin(x4 +x)

x→0

;

x3 +1 ln x; 2 x→0 x +1 x>0 3 lim x2 +1 1 ; x→∞ ¡x +1 ln x ¢ lim x3 ex − 2x ; x→∞

c) lim d) e)

f) lim (x − ln (x + 1)) ; x→∞ ³ √ ´√x √x g) lim x+ ; x− x x→∞

1

h) lim (2x + sin x) x ; x→0 ³ 1 ´sin(x−1)2 i) lim e x−1 ; x→1 x0

k) lim

x→∞

x

³

xα +x+1 xβ −1

;

´γx−√x2 −x

, α, β, γ ∈ IN∗ (Examen Definitivat Profesori-

august 1999). (

5.9 Fie funct¸ia f : IR → IR, f (x) =

sin x1 , α,

x 6= 0 x=0

a) f este continu˘a ˆın punctele x = p1 , unde p ∈ IN∗ ? b) Determinat¸i α ∈ IR, astfel ˆıncˆ at : i) f s˘a fie continu˘ a; ii) f s˘a aib˘a proprietatea lui Darboux. ( 6 0 sin x sin x1 , x = c) Aceea¸si problem˘a pentru funct¸ia f1 (x) = . Geα, x=0 neralizare.

196 5.10 Precizat¸i dac˘a urm˘atoarele funct¸ii sunt continue ¸si dac˘a au proprietatea lui Darboux: ( 1 e− x2 sin x1 , x 0 mult¸imea Aε = {x ∈ I|ωf (x) ≥ ε} este ˆınchis˘ a. c) mult¸imea Df , a punctelor de discontinuitate ale lui f , este de tip Fσ . 5.14 a) Exist˘a funct¸ii f : IR → IR, continue pe Q ¸si discontinue pe IR− Q? b) Exista funct¸ii f : [0, 1] → IR, continue pe [0, 1] − Q ¸si discontinue pe [0, 1] ∩ Q ? 5.15 Fie f, g : IR → IR funct¸ii continue pe IR, astfel ˆıncˆ at f (x) = g(x) pentru orice x ∈ Q. Ar˘atat¸i c˘a f = g. 5.16 Fie α, β ∈ IR, α < β ¸si f : IR → IR, o funct¸ie continu˘ a. Demonstrat¸i c˘a mult¸imile: A = {x ∈ IR|f (x) > α} , B = {x ∈ IR|f (x) < β} , C = {x ∈ IR|α < f (x) < β} sunt deschise,

197 iar mult¸imile D = {x ∈ IR|f (x) ≥ α} , E = {x ∈ IR|f (x) ≤ β} , F = {x ∈ IR|α ≤ f (x) ≤ β} sunt ˆınchise. 5.17 Demonstrat¸i c˘a dac˘a f : IR → IR atunci urm˘atoarele afirmat¸ii sunt echivalente: a) f este continu˘a; ◦

b) ∀B ⊂ IR, f −1 (B) ⊂ int f −1 ¡ (B); ¢ −1 −1 c) ∀B ⊂ IR, f (B) ⊂ f B . 5.18 a) Dac˘a f¡ :¢IR → IR este continu˘ a, este adev˘arat c˘a pentru orice A ⊂ IR are loc f A ⊃ f (A)? b) Dac˘a f este bijectiv˘a, atunci f este homeomorfism dac˘a ¸si numai dac˘a ∀A ⊂ IR, f (A) = f (A). 5.19 Determinat¸i mult¸imile A ⊂ IR cu proprietatea c˘a funct¸ia caracteristic˘a asociat˘a, ϕA , este continu˘ a pe IR. 5.20 a) Dac˘a f : [a, ∞) → IR este continu˘ a, iar graficul lui f admite asimptot˘a la +∞, atunci f este uniform continu˘ a. b) Dac˘a f : IR → IR este continu˘ a, iar graficul lui f admite asimptot˘a la +∞ ¸si la −∞, atunci f este uniform continu˘ a. c) Dac˘a f : (a, ∞) → IR este continu˘ a, iar graficul lui f admite asimptot˘a vertical˘a la stˆanga ˆın x = a, atunci f nu este uniform continu˘ a. 5.21 Dac˘a f : IR → IR este continu˘ a ¸si periodic˘a atunci f este uniform continu˘a. 5.22 S˘a se studieze continuitatea uniform˘a a funct¸iilor: 1 a) f (x) = x−2 pe intervalele:(3, 4]; (2, 4]; [1, 4]; [4, ∞); 2

b) f (x) = x x−1 pe intervalele: (1, 2); (1, ∞); (0, ∞); √ c) ex , ln x, sin x, cos x, tgx, arctgx, sin x2 , x2 + x + 1 pe domeniul maxim de definit¸ie. 5.23 Studiat (¸i continuitatea uniform˘a a funct¸iilor: ln x ln (1 − x) , x ∈ (0, 1) a) f (x) = ; 1 ctg (x − 1) − x−1 , x ∈ (1, 2) ( x sin x ¡ π ¢ a −a , x ∈ −2,0 3 , a > 0; b) f (x) = ¡ 2 x ¢1 x , x ∈ (0, 1) π arccos x

198 ( arctg(1+x)−arctg(1−x) c) f (x) =

2x −2sin x

α+

¡ tgx ¢ x1 x

,

, x ∈ (0, 1) ¡ ¢; x ∈ − π4 , 0

5.24 S˘a se arate c˘a o funct¸ie f : D → IR, D ⊂ IR, uniform continu˘ a, transform˘a ¸sirurile Cauchy ˆın ¸siruri Cauchy. Dat¸i un exemplu de funct¸ie continu˘a pentru care proprietatea nu este adev˘arat˘ a. 5.25 Demonstrat¸i urm˘atoarea reciproc˘a a teoremei lui Weierstrass: dac˘a K ⊂ IR ¸si orice funct¸ie continu˘ a f : K → IR este m˘arginit˘ a, atunci K este compact˘a. 5.26 Dac˘a f : IR → IR este continu˘ a ¸si f ◦ f admite puncte fixe, atunci ¸si f admite puncte fixe.

199

Solut¸ii 5.1

a) Are loc echivalent¸a µ ¶ ½ 1 1 1 3x − 2 > 4 − 100 f (x) ∈ 4 − ,4 + ⇐⇒ 1 3x − 2 < 4 + 100 100 100 ¡ ¢ 1 1 Mult¸imea solut¸iilor acestui sistem este S = 2 − 300 , 2 + 300 . Vom alege 1 δ¡ > 0, astfel ˆıncˆ ¢ at (2 − δ, 2 + δ) ⊂ S, de exemplu ¡ δ 1= 300 . Dac˘ ¢a x ∈ 1 1 1 , 2 + 300 , atunci x ∈ S ¸si deci f (x) ∈ 4 − 100 , 4 + 100 . Sunt 2 − 300 ¸si alte valori posibile pentru δ? b) Avˆand ˆın vedere c˘a |x − 2| < δ1 ⇐⇒ x ∈ (2 − δ1 , 2 + δ1 ) ¸si

1 ⇐⇒ f (x) ∈ |f (x) − 4| < 1000

µ 4−

1 1 ,4 + 1000 1000



rezolvarea este similar˘a cu cea de la punctul a). c) Fie ¡ε > 0. Relat¸ia |f (x) − 4| < ε se mai scrie |x − 2| < 3ε sau echiva¢ lent, x ∈ 2 − 3ε , 2 + 3ε . Not˘am S = (2 − 3ε , 2 + 3ε ). Orice δ ≤ 3ε (δ > 0), verific˘a condit¸ia din ipotez˘a: dac˘a x ∈ (2 − δ, 2 + δ) rezult˘a c˘a x ∈ S ¸si deci |f (x) − 4| < ε, ceea ce trebuia demonstrat. 5.2

a) Are loc echivalent¸a µ ¶ ½ 2 1 1 x +x>2− 2 x +x∈ 2− ,2 + ⇐⇒ 2+x (1 − δ, 1 + δ) ⊂ S este suficient ca δ < − 12 + 20 ¡ 0!). 1Cu un δ 1astfel ¢ ales, dac˘a |x − 1| < δ va rezulta x ∈ S ¸si deci x2 + x ∈ 2 − 100 , 2 + 100 . b) i) Considerˆand ε > 0, fixat, trebuie demonstrat ¯ 2 ¯c˘a exist˘a δ > 0 astfel ¯ ˆıncˆat pentru orice x cu |x − 1| < δ, are loc x + x − 2¯ < ε. Varianta I. Avem: ¯ 2 ¯ ¯x + x − 2¯ = |x − 1| |x + 2| < δ |(x − 1) + 3| ≤ δ 2 + 3δ ≤ 4δ,

200 ultima inegalitate fiind adev˘arat˘ a¡ˆın ipoteza δ ≤ 1 (care nu restrˆa¯nge genera¯ ¢ ε litatea!). Alegˆand acum δ < min 4 , 1 va rezulta 4δ 5000 ⇐⇒ x > 4996 sau x < −5004.

Deci pentru α = 4996, dac˘ a x ≥ α are loc |x + 4| > 5000 ¸si deci 2x+3 x+4 ∈ V. =2 b) Deoarece x → ∞ putem presupune c˘a x > 0. Avem: lim 2x+3 x→∞ x+4 dac˘a pentru orice ε > 0, exist˘a α > 0, astfel ˆıncˆ at dac˘a x > α are loc 2x+3 ∈ (2 − ε, 2 + ε) . Dar aceast˘ a relat ¸ ie este echivalent˘ a cu x > 5ε − 4. Deci x+4 exist˘a α = 5ε − 4, astfel ˆıncˆ at este verificat˘ a relat¸ia din definit¸ie. 5.4 a) i) Vom demonstra c˘a pentru orice A > 0, exista δA > 0 astfel ˆıncˆat dac˘a x > δA are loc f (x) > A. F˘ar˘ a a restrˆange generalitatea putem presupune c˘a A > 1, x > 0 ¸si avem echivalent¸ele: x2 + 1 > A ⇔ x2 − Ax + 1 − A > 0 (x + 1 > 0!) x+1 Ã ! Ã ! √ √ A − A2 + 4A − 4 A + A2 + 4A − 4 ⇔ x ∈ −∞, ∪ , ∞ = S. 2 2 √

2

Impunem condit¸ia (δA , ∞) ⊂ S ¸si obt¸inem c˘a δA ≥ A+ A 2+4A−4 . De exem√ 2 plu, putem lua δA = A+ A 2+4A−4 . ˆIntr-adev˘ ar dac˘a x > δA , atunci x ∈ S 2 x +1 ¸si, ˆın final, x+1 > A, ceea ce trebuia demonstrat. ¯ 7 5 4¯ 7 +5x5 +x4 ¯ +5x +x ¯ 5.5 Relat¸ia dat˘a se mai scrie: ¯ 2x ≤ m, x ≥ α. Notˆ and 2x 5x7 +2x3 +1 ¯ 5x7 +2x3 +1 = f (x) ¸si observˆand c˘a lim f (x) = 25 , rezult˘ a c˘a exist˘a o vecin˘ atate [α, ∞] x→∞

a punctului ∞, astfel ˆıncˆ at pentru orice x ∈ [α, ∞) are loc |f (x)| ≤ m (teorema 5.12), ceea ce trebuia demonstrat. 5.6 a) Rezolvˆand ecuat¸iile sin x = 0 ¸si sin x = 1 obt¸inem solut¸iile: x = kπ, k ∈ Z ¸si x = π2 + kπ, k ∈ Z. Considerˆand ¸sirurile x0n = nπ → ∞, x00n =

201 π 2

+ 2nπ → ∞, observ˘am c˘a f (x0n ) = 0, ∀n ∈ IN ¸si f (x00n ) = 1, ∀n ∈ IN. Prin urmare f (x0n ) → 0, f (x00n ) → 1 ¸si, conform observat¸iei 5.3, rezult˘a concluzia. b) La fel ca ˆın exercit¸iul anterior g˘asim ! 4 1− 1+ → 0, x0n < 0, 2nπ Ã ! s 1 4 x00n = 1− 1+ → 0, x00n < 0, ∀n ∈ IN∗ . 2 2nπ + π2 x0n

1 = 2

Ã

r

Avem f (x0n ) → 1, f (x00n ) → 0 ¸si deci nu exist˘a lim f (x) . Demonstrat¸i c˘a nu x→0 x0

c) Demonstr˘am c˘a lim f (x) = ∞. Conform definit¸iei funct¸iei parte £1¤

x→0 x>0

ˆıntreag˘a, x = k, dac˘a k ≤ x1 < k + 1, pentru£ orice x ∈ IR∗ . Dac˘ a ¤ 1 1 x → 0, x > 0 din k +1 > x rezult˘a k → ∞, adic˘ a lim x = ∞. Demonstr˘am x→0 x>0

analog c˘a lim f (x) = −∞. ˆIn final, concluzion˘am c˘a nu exist˘a lim f (x) . x→0

x→0 x 0 ¸si avem

5.7

1

ls = e 0+ = e∞ = ∞. Dac˘a x → 2, x > 2 atunci 4 − x2 → 0, 4 − x2 < 0 ¸si 1 avem ld = e 0− = e−∞ = 0. O alt˘a variant˘ a ar fi s˘a substituim 4 − x2 = y, 1 caz ˆın care limita la stˆanga devine: ls = lim e y = e∞ = ∞. y→0 y>0

b) Se procedeaz˘a ca la punctul a) ¸si g˘asim ls = 1 ¸si ld = 0. £ ¤ £ ¤ c) x1 − 1 < x1 ≤ x1 , ˆınmult¸ind cu x > 0 obt¸inem: 1 − x < x x1 ≤ 1, deci lim f (x) = 1. Calculat¸i ¸si limita la stˆanga. x→0 x>0

d)

¯ ¯ ¯ ¯ ¯ 1 sin(x − 1) ¯ ¯ 1 sin y 1 ¯ ¯ ¯ ¯ ¯ = ∞. lim ln ¯ − = lim ln − x→1 2 (x − 1)2 ¯ y→0 ¯ 2 y y¯ x 0 ¸si qε ∈ IN∗ , astfel ˆıncˆ at 1 < ε. Pentru orice q ∈ {1, 2, .., qε } , exist˘ a m ∈ ³ qε ´ m+1 IN, astfel ˆıncˆat x0 ∈ m . Notˆ and acest interval cu Iq , s˘ a remarc˘am q , q c˘a el nu cont¸ine nici un T num˘ ar rat¸ional cu numitorul q, oricare ar fi q ∈ {1, 2, .., qε } . Dac˘a I = Iq , x = pq ∈ I, atunci, conform observat¸iei 1≤q≤qε

precedente, q ≥ qε + 1 ¸si vom avea : ¯ µ ¶¯ ¯ p ¯¯ 1 1 1 |f (x) − f (x0 )| = |f (x)| = ¯¯f = ≤ < < ε. ¯ q q qε + 1 qε ˆIn cazul cˆand x ∈ I ∩ (IR\Q) , f (x) = 0 ¸si inegalitatea anterioar˘ a este evident˘a. Am demonstrat astfel c˘a, oricare ar fi ε > 0, exist˘ a I ∈ V (x0 ) , astfel ˆıncˆat dac˘a x ∈ I, rezult˘a c˘a |f (x) − f (x0 )| < ε, prin urmare f este continu˘a ˆın x0 . ˆIn mod asem˘an˘ ator se demonstreaz˘a c˘a f este continu˘ a ˆın x0 = 0. S˘a mai observ˘am c˘a, dac˘a schimb˘ am valoarea funct¸iei f ˆın x = 0, de exemplu f (0) = 1, funct¸ia obt¸inut˘ a va fi continu˘ a pe [0, 1] \Q ¸si discontinu˘ a pe [0, 1] ∩ Q. d) ii) Avˆand ˆın vedere c˘a f ([0, 1]) nu este un interval, urmeaz˘a c˘a f nu are proprietatea lui Darboux.

205 5.11 Implicat¸ia direct˘a este evident˘ a. Pentru implicat¸ia reciproc˘a, fie a ∈ A, arbitrar ales ¸si Vn = (a −

1 1 , a + ) ∈ V (a) , n ∈ IN∗ . n n

Din ipotez˘a, exist˘a xn ∈ Vn ∩ A astfel ˆıncˆ at f (xn ) = 0, pentru orice n ∈ IN∗ . Dar xn ∈ Vn ∩A antreneaz˘a |xn −a| < n1 , oricare ar fi n ∈ IN∗ , adic˘ a xn → a. Cum f este continu˘a, rezult˘a f (xn ) → f (a) , deci f (a) = 0. 5.12 Vom presupune c˘a f este continu˘ a ˆın x ∈ IR, x0 ∈ IR, (rn ) ⊂ Q, rn → x − x0 . Faptul c˘a x0 + rn → x antreneaz˘ a lim f (x0 + rn ) = f (x) , iar n→∞

egalit˘a¸tile f (x0 + rn ) = f (x0 ) , n ∈ IN implic˘a lim f (x0 + rn ) = f (x0 ) . n→∞

Prin urmare f (x) = f (x0 ) ¸si cum x a fost ales arbitrar, rezult˘a c˘a f este o funct¸ie constant˘a, contradict¸ie! Funct¸ia lui Dirichlet verific˘ a proprietatea din enunt¸. 5.13 a) Dac˘a f este continu˘a ˆın a ¸si ε > 0, atunci exist˘a V ∈ V(a) astfel ˆıncˆat pentru orice x ∈ V ∩I s˘a rezulte |f (x)−f (a)| < ε/2. Dac˘a x0 , x00 ∈ V ∩I vom avea: |f (x0 ) − f (x00 )| ≤ |f (x0 ) − f (a)| + |f (a) − f (x00 )| < ε ¸si trecˆand la supremum dup˘a x0 , x00 , obt¸inem diam (f (V ∩ I)) ≤ ε. Trecem din nou la infimum, dar ˆın raport cu V ¸si avem ωf (a) ≤ ε, pentru orice ε > 0 ceea ce implic˘a ωf (a) = 0. Demonstrat¸i singuri reciproca! b) Vom demonstra c˘a Aε ⊂ Aε , a¸sa ˆıncˆ at fie x ∈ Aε ¸si V ∈ V (x) . Exist˘a D, mult¸ime deschis˘a, astfel ˆıncˆ at x ∈ D ⊂ V, D ∩ Aε 6= ∅; fie y ∈ D ¸si y ∈ Aε , prin urmare diam(f (D ∩ I)) ≥ ωf (y) ≥ ε. Dar atunci diam(f (V ∩ I)) ≥ diam(f (D ∩ I)) ≥ ε, ∀V © ∈ V(x), deci x ∈ª Aε . c) Df = {x ∈ I|ωf (x) > 0} = ∪ ∗ x ∈ I|ωf (x) ≥ n1 = ∪ ∗ A 1 ¸si, n∈IN

n∈IN

n

conform punctului b), concluzia rezult˘a. S˘a mai observ˘am c˘a dac˘a I este interval deschis, atunci mult¸imea punctelor de continuitate Cf este o mult¸ime T S (I ∩ cA 1 ). A1) = Gδ . ˆIntr-adev˘ar Cf = I\Df = I ∩ c( n∈IN∗

n

n∈IN∗

n

5.14 a) Nu exist˘a, deoarece mult¸imea Q nu este o mult¸ime Gδ (vezi problema 5.13). b) Da, de exemplu funct¸ia lui Riemann, modificat˘a ˆın x = 0 (vezi exercit¸iul 5.10 d)).

206 5.15 Fie x ∈ IR. Atunci exist˘a (yn )n∈IN ⊂ Q astfel ˆıncˆ at yn → x. Cum f ¸si g sunt continue, avem f (yn ) → f (x) ¸si g(yn ) → g(x). Dar f (yn ) = g(yn ), pentru orice n ∈ IN ¸si din unicitatea limitei unui ¸sir rezult˘a f (x) = g(x). 5.16 Mult¸imea A este egal˘a cu f −1 ((α, ∞)) ¸si cum (α, ∞) este o mult¸ime deschis˘a, conform teoremei de caracterizare a continuit˘ a¸tii globale 5.36, rezult˘a c˘a A este deschis˘ a. Mult¸imea D este contraimaginea mult¸imii ˆınchise [α, ∞) , prin urmare ea este o mult¸ime ˆınchis˘ a. Am putea demonstra c˘a D este ˆınchis˘a ¸si astfel: fie (xn ) ⊂ D, xn → x0 ¸si s˘a demonstr˘am c˘a x0 ∈ D. Relat¸iile xn ∈ D, ∀n ∈ IN, antreneaz˘ a f (xn ) ≥ α, ∀n ∈ IN ¸si trecˆand la limit˘a pentru n → ∞, rezult˘a c˘a f (x0 ) ≥ α, adic˘ a x0 ∈ D. 5.17 i) S˘a demonstr˘am a ⇒ b. Vom folosi ˆın mod esent¸ial caracterizarea: f este continu˘a dac˘a ¸si numai dac˘a ∀D ⊂ IR, mult¸ime deschis˘ a, f −1 (D) este deschis˘ a.

(1)

Fie B ⊂ IR. Avem implicat¸iile (2)







B ⊂ B ⇒ f −1 (B) ⊂ f −1 (B) ⇒ intf −1 (B) ⊂ intf −1 (B) ◦



¸si luˆand ˆın (1) D = B (care este deschis˘ a), va rezulta c˘a f −1 (B) deschis˘ a ¸si ◦



deci intf −1 (B) = f −1 (B). Relat¸ia (2) devine astfel: (3)



f −1 (B) ⊂ int f −1 (B).

ii) S˘a demonstr˘am b ⇒ a. Pentru a demonstra relat¸ia (1) fie D ⊂ IR, D ◦ deschis˘a. ˆIn (3) lu˘am B = D = D ¸si avem f −1 (D) ⊂ int f −1 (D) , relat¸ie care antreneaz˘a faptul c˘a f −1 (D) deschis˘ a. iii) Echivalent¸a a ⇔ c se demonstreaz˘a la fel cu echivalent¸a a ⇔ b, dar folosind caracterizarea global˘a a continuit˘ a¸tii cu mult¸imi ˆınchise (teorema 5.38). 5.18 a) a f : IR → IR, ( Nu este adev˘arat: funct¸ia continu˘ sin x x , x 6= 0 ¸ f (x) = a relat¸ia si mult¸imea A = {2nπ + π2 |n ∈ IN} nu verific˘ 1, x=0 1 dat˘a. ˆIntr-adev˘ar f (A) = f (A) = { 2nπ+ π |n ∈ IN}, iar f (A) = f (A) ∪ {0}. 2

b) ”⇒” Presupunem c˘a f este un homeomorfism. Incluziunea f (A) ⊂ f (A) este adevarat˘a conform teoremei 5.36. Funct¸ia f −1 este continu˘ a dac˘a ¸si numai dac˘a (4)

∀B ⊂ IR, f −1 (B) ⊂ f −1 (B).

207 Luˆand ˆın aceast˘a relat¸ie B = f (A), obt¸inem: f −1 (f (A)) ⊂ A ¸si aplicˆand f, rezult˘a f (A) ⊂ f (A). ”⇐” Presupunem acum c˘a ∀A ⊂ IRp , f (A) = f (A). Din f (A) ⊂ f (A) rezult˘a c˘a f este continu˘a. Fie B ⊂ IR; folosit¸i incluziunea invers˘ a, f (A) ⊃ f (A), pentru a demonstra (4). 5.19 Dac˘a D este o mult¸ime deschis˘ a din IR, atunci ϕ−1 a A (D) = IR, dac˘ −1 0, 1 ∈ D; ϕA (D) = IR\A, dac˘a 0 ∈ D, 1 ∈ / D; ϕ−1 (D) = A, dac˘ a 0 ∈ / D, 1 ∈ A D ¸si ϕ−1 (D) = ∅, dac˘ a 0 ∈ / D, 1 ∈ / D. Funct ¸ ia ϕA f va fi continu˘ a dac˘a A ¸si numai dac˘a A ¸si IR\A sunt deschise sau, echivalent, A este simultan ¸si ˆınchis˘a ¸si deschis˘a. Rezult˘a c˘a A = ∅ sau A = IR. 5.20 a) Vom presupune, pentru ˆınceput, c˘a a > 0. S¸tim c˘a exist˘a l ∈ IR astfel ˆıncˆat oricare ar fi ε > 0, exist˘a A ∈ IR cu proprietatea c˘a pentru orice x ≥ A, x ∈ [a, ∞) rezult˘a |f (x) − l| < 2ε . Fie ε > 0 (fixat), x, y ≥ A, |x − y| < δ, δ > 0, ales arbitrar. Atunci: (5)

|f (x) − f (y)| ≤ |f (x) − l| + |f (y) − l| < ε.

Fie x < A, |x − y| < δ ¸si s˘a alegem δ < ε. Avem: y = |y| ≤ |x − y| + |x| < δ+A < ε+A ¸si prin urmare x, y ∈ [a, A + ε] . Funct¸ia f este uniform continu˘ a pe [a, A + ε], prin urmare pentru ε > 0 (fixat anterior) exist˘a δ1 > 0 astfel ˆıncˆat: (6)

∀x, y ∈ [a, A + ε] , |x − y| < δ1 ⇒ |f (x) − f (y)| < ε.

Alegˆand δ < δ1 atunci ∀x, y ∈ [a, ∞) , |x − y| < δ are loc una din relat¸iile (5) sau (6) adic˘a concluzia. Solut¸ia cazului y < A este asem˘an˘ atoare. Rezolvat¸i singuri cazul a ≤ 0! c) Dac˘a presupunem c˘a f este uniform continu˘ a pe (a, ∞) , atunci f va fi uniform continu˘a pe (a, b] , b ∈ IR, a < b. Conform teoremei 5.57 f se poate prelungi prin continuitate ˆın x = a, adic˘a exist˘a x→a lim f (x) ∈ IR, fals! x>a

5.21 Dac˘a T > 0 este perioada principal˘a a funct¸iei, atunci f este uniform continu˘a pe mult¸imea [0, 2T ] , adic˘ a 0

∀ε > 0, ∃δ 0 > 0 : ∀x0 , y 0 ∈ [0, 2T ], |x0 − y 0 | < δ ⇒ |f (x0 ) − f (y 0 )| < ε. Trebuie s˘a demonstr˘am c˘a f este uniform continu˘ a pe IR, adic˘a: ∀ε > 0, ∃δ > 0 : ∀x, y ∈ IR, |x − y| < δ ⇒ |f (x) − f (y)| < ε. S˘a demonstr˘am mai ˆıntˆai urm˘atoarea afirmat¸ie: dac˘a x, y ∈ IR, |x − y| < T, atunci exist˘a

208 n0 ∈ IN astfel ˆıncˆat x − n0 T, y − n0 T ∈ [0, 2T ]. ˆIn primul rˆand exist˘a m0 ∈ IN astfel ˆıncˆat x ∈ [m0 T, (m0 + 1) T ] ¸si s˘a presupunem c˘a x ≤ y. Atunci y ∈ [m0 T, (m0 + 2) T ] ¸si relat¸ia de mai sus este adev˘arat˘ a cu n0 = m0 . Cazul x > y se rezolv˘a similar. Fie ε > 0. S˘ a alegem acum δ > 0, δ < min(T, δ 0 ), x, y ∈ IR : |x − y| < δ, ¸si s˘a not˘am x0 = x − n0 T, y 0 = 0 y − n0 T. Avem |x0 − y 0 | = |x − ¯ y| < δ ceea ¯ ce implic˘a |f (x) − f (y)| = 0 0 ¯ ¯ |f (x − n0 T ) − f (y − n0 T )| = ¯f (x ) − f (y )¯ < ε ¸si, astfel, relat¸ia dorit˘a este demonstrat˘a. 5.22 a) i) Intervalul (3, 4] este m˘arginit, f se prelunge¸ste prin continuitate ˆın x = 3 (exist˘a lim f (x) = 1) ¸si din teorema 5.57 rezult˘a c˘a f este uniform x→3

continu˘a pe (3, 4] . ii) f nu este uniform continu˘ a pe (2, 4] (vezi problema 5.20-c)). iii) [1, 4] ⊃ (2, 4], prin urmare f nu este uniform continu˘ a pe [1, 4]. iv) Vezi problema 5.20-a). c) i) Vom demonstra c˘a funct¸ia f : IR → IR, f (x) = ex nu este uniform continu˘a, folosind negat¸ia definit¸iei. Deoarece f este uniform continu˘ a pe un compact de tipul [0, b] , ∀b > 0, va fi necesar s˘a consider˘am dou˘a ¸siruri cu limita ∞. 1 Varianta I: Fie x0n = n, x00n = n + 2n , ∀n ∈ IN∗ . Evalu˘ am 1

1

|f (x0n ) − f (x00n )| = en+ 2n − en = en (e 2n − 1) > en

1 > 1, ∀n ∈ IN∗ 2n

(pentru ultimele doua relat¸ii am folosit inegalit˘a¸tile: ex > 1 + x, ∀x > 0 ¸si respectiv ex > 2x, ∀x > 1; acestea pot fi demonstrate, de exemplu, folosind derivata). A¸sadar exist˘a ε0 = 1 ¸si (x0n ), (x00n ) astfel ˆıncˆ at pentru orice n ∈ IN∗ , |x0n − x00n | < n1 ¸si |f (x0n ) − f (x00n ) | ≥ ε0 ; prin urmare f nu este uniform continu˘a. Varianta a II-a: se consider˘a ¸sirurile x0n = ln n, x00n = n1 + ln n, etc. Demonstrat¸i c˘a f este uniform continu˘ a pe (−∞,¯b) , ∀b¯∈ IR. ¯ ¯ x−y ¯ ¯ ≤ 2 ¯sin x−y ¯ ≤ |x − y| , iii) Avem: |sin x − sin y| = 2 sin 2 cos x+y 2 2 ∀x, y ∈ IR; funct¸ia f : IR → IR, f (x) = sin x este lipschitzian˘ a ¸si deci uniform continu˘a. Aceast˘a concluzie rezult˘a ¸si pe baza exercit¸iului 5.21. O alt˘a solut¸ie se poate da folosind propozit¸ia 6.16. vii) Funct¸ia f : IR → IR, f (x) = sin x2 nu este uniform continu˘ a (rezolvarea este analoag˘a cu cea de la punctul c) i). viii) Observ˘am c˘a f 0 (x) = 2√2x+1 , f 0 : IR → IR este m˘arginit˘ a, ceea ce x2 +x+1 antreneaz˘a c˘a f este lipschitzian˘ a (conform propozit¸iei 6.16), deci uniform continu˘a.

209 5.23 a) Vom studia dac˘a f se prelunge¸ste prin continuitate ˆın punctele 0, 1, 2, adic˘a la mult¸imea (0, 1) ∪ (1, 2) = [0, 2] (teorema 5.57). Calcul˘am mai ∞·0 −x x ˆıntˆai : lim ln x ln (1 − x) = lim ln(1−x) −x · ln x = − lim ln x = 0 (ultima fiind o x→0 x>0

x→0 x>0

x→0 x>0

limit˘a fundamental˘a). A¸sadar f se prelunge¸ste prin continuitate la dreapta punctului x = 0. Printr-o substitut¸ie, calcul˘am lim ln x ln (1 − x) = 0. S˘a x→1 x1

= lim

y→0 y>0

= lim

y→0 y>0

y>0

y cos y − y + y − sin y cos y − 1 y − sin y = lim + lim = y→0 y→0 y sin y y sin y sin y y>0

y>0

y − sin y cos y − 1 + lim = 0. y→0 y y2 y>0

Deci f se prelunge¸ste prin continuitate ˆın punctul x = 1. Funct¸ia f are aceea¸ ³si proprietate ¸si la´ stˆanga punctului x = 2, deoarece exist˘a 1 = ctg 1 − 1. ˆIn concluzie f este uniform continu˘ a. lim ctg (x − 1) − x−1 x→2 x 0, ∃δε > 0 : ∀x, y ∈ D, |x − y| < δε ⇒ |f (x) − f (y) | < ε.

Fie (xn ) ⊂ D un ¸sir Cauchy ¸si s˘a demonstr˘am c˘a (f (xn )) este un ¸sir Cauchy adic˘a : ∀ε > 0, ∃nε ∈ IN : ∀n, m ≥ nε ⇒ |f (xn ) − f (xm ) | < ε. Fie ε > 0. Pentru acest ε, ∃nε ∈ IN : ∀n, m ≥ nε ⇒ |xn − xm | < ε. Dac˘a ˆın prima relat¸ie lu˘am δε = ε ¸si x = xn , y = xm , atunci pentru n, m ≥ nε va rezult˘a c˘a |f (xn ) − f (xm ) | < ε, adic˘a concluzia. S˘ a analiz˘am urm˘atorul exemplu: funct¸ia f : (0, 1) → IR, f (x) = x1 este continu˘a ¸si transform˘a ¸sirul Cauchy (xn )n∈IN∗ = ( n1 )n∈IN∗ , n ≥ 1 ˆın ¸sirul (f (xn ))n∈IN∗ = (n)n∈IN∗ care nu este ¸sir Cauchy. 5.25 Demonstr˘am, mai ˆıntˆai, c˘a mult¸imea K este m˘arginit˘ a. Funct¸ia f : K → IR, f (x) = |x|, este continu˘ a ¸si deci mult¸imea f (K) = {|x| |x ∈ K} este m˘arginit˘a. Dar asta ˆınseamn˘ a exact m˘arginirea mult¸imii K. Pentru a demonstra c˘a mult¸imea K este ˆınchis˘ a, vom presupune prin absurd c˘a exist˘a 1 a ∈ K, a ∈ / K. Funct¸ia f : K → IR, f (x) = ||x−a|| , este continu˘ a ¸si prin

210 urmare, f (K) este m˘arginit˘ a, adic˘a exist˘a M > 0 astfel ˆıncˆ at

1 |x−a|

≤ M,

oricare ar fi x ∈ K. Cum a ∈ K, exist˘ a (xn ) ⊂ K, astfel ˆıncˆ at xn → a; dar 1 atunci lim |xn −a| = ∞, contradict¸ie. n→∞

5.26 Dac˘a presupunem c˘a f nu admite puncte fixe, atunci, din ipoteza de continuitate rezult˘a c˘a, fie f (x) > x, pentru orice x ∈ IR, fie f (x) < x, pentru orice x ∈ IR. ˆIn prima situat¸ie, trecˆand x ˆın f (x) obt¸inem: f (f (x)) > f (x) > x, pentru orice x ∈ IR, deci (f ◦ f ) (x) > x, pentru orice x ∈ IR; prin urmare, f ◦ f nu admite puncte fixe, fals! La fel se rezolv˘a situat¸ia f (x) < x, pentru orice x ∈ IR.

Capitolul 6 FUNCT ¸ II DERIVABILE

Definit¸ia 6.1. Spunem c˘a funct¸ia f : A → IR, A ⊂ IR are derivat˘ a ˆın (a) punctul a ∈ A0 ∩ A, dac˘a exist˘a ˆın IR limita lim f (x)−f . Vom nota aceast˘ a x−a x→a

limit˘a cu f 0 (a). Dac˘a f 0 (a) ∈ IR vom spune c˘a f este derivabil˘ a ˆın a. Definit¸ia 6.2. Spunem c˘a funct¸ia f : A → IR, A ⊂ IR este derivabil˘ a pe mult¸imea B ⊂ A, dac˘a f este derivabil˘ a ˆın fiecare punct din B. Funct¸ia notat˘a f 0 , f 0 : B → IR, care asociaz˘a fiec˘arui punct x ∈ B derivata f 0 (x) se nume¸ste derivata funct¸iei f pe mult¸imea B. Propozit¸ia 6.3. Dac˘a f : A → IR, A ⊂ IR este derivabil˘ a ˆın punctul a ∈ A0 ∩ A, atunci f este continu˘ a ˆın a. Observat¸ia 6.4. Reciproca propozit¸iei anterioare nu este adevarat˘ a. Astfel, funct¸ia f : IR → IR, f (x) = |x| este continu˘ a ˆın punctul x = 0, dar nu este derivabil˘a ˆın acest punct. Definit¸ia 6.5. Spunem c˘a funct¸ia f : A → IR, A ⊂ IR, are derivat˘ a la (a) stˆanga ˆın punctul a ∈ A0s ∩ A dac˘a exist˘a ˆın IR limita x→a lim f (x)−f . Vom x−a x 0 (respectiv f (x) ≥ 0) pentru orice x ∈ I, atunci f este strict cresc˘atoare (respectiv cresc˘atoare) pe I. (iii) Dac˘a f 0 (x) < 0 (respectiv f 0 (x) ≤ 0), pentru orice x ∈ I, atunci f este strict descresc˘atoare (respectiv descresc˘atoare) pe I.

213 Propozit¸ia 6.15. Fie I ⊂ IR, I interval, a ∈ I, f : I → IR, f continu˘ a. 0 Dac˘a f este derivabil˘a pe I\{a} ¸si exist˘a lim f (x) (finit˘a sau infinit˘a), atunci x→a

exist˘a derivata funct¸iei f ˆın a, f 0 (a) ¸si f 0 (a) = lim f 0 (x). x→a

Propozit¸ia 6.16. Dac˘a f : I → IR, I ⊂ IR, I interval, este o funct¸ie derivabil˘a cu derivata marginit˘a, atunci f este lipschitzian˘ a. Teorema 6.17. (Cauchy.) Dac˘a a, b ∈ IR, a < b, f, g : [a, b] → IR sunt dou˘a funct¸ii continue pe [a, b], derivabile pe (a, b) ¸si g 0 (x) 6= 0, pentru orice x ∈ (a, b), atunci g(b) − g(a) 6= 0 ¸si exist˘a c ∈ (a, b) astfel ˆıncˆ at f (b)−f (a) f 0 (c) g(b)−g(a) = g 0 (c) . Teorema 6.18. (Darboux.) Fie I ⊂ IR, I interval. Dac˘a f : I → IR 0 este o funct¸ie derivabil˘a pe I, atunci derivata sa f are proprietatea lui Darboux. Teorema 6.19. (Cauchy.) Fie f, g : I → IR, I ⊂ IR, I interval, a ∈ I, care verific˘a condit¸iile: (i) f (a) = g(a) = 0; (ii) f, g sunt derivabile ˆın a; (iii) g 0 (a) 6= 0. Atunci exist˘a V ∈ V(a) astfel ˆıncˆ at g(x) 6= 0, pentru orice x ∈ V \{a} 0 (a) (x) ¸si lim fg(x) = fg0 (a) . x→a

Teorema 6.20. (Regula lui L0Hˆ ospital-cazul 00 .) Fie f, g : I\{a} → IR, I ⊂ IR, I interval, a ∈ I 0 , f, g derivabile, care verific˘ a condit¸iile: (i) lim f (x) = lim g(x) = 0; x→a

x→a

(ii) g 0 (x) 6= 0, ∀x ∈ I\{a}; 0 (x) (iii) exist˘a lim fg0 (x) = L ∈ IR. x→a

(x) Atunci exist˘a lim fg(x) = L. x→a

Teorema 6.21. (Regula lui L0Hˆ ospital-cazul ∞ ∞ .) Fie f, g : I\{a} → 0 IR, I ⊂ IR, I interval, a ∈ I , f, g derivabile care verific˘ a condit¸iile: (i) lim |g(x)| = ∞; x→a

(ii) g0(x) 6= 0, ∀x ∈ I\{a}; 0 (x) = L ∈ IR. (iii) exist˘a lim fg0 (x)

214 (x) Atunci exist˘a lim fg(x) = L. x→a

Teorema 6.22. (S ¸ irul lui Rolle.) Fie I ⊂ IR, I interval ¸si f : I → IR, o funct¸ie derivabil˘a. Dac˘a x1 , x2 ∈ I, x1 < x2 sunt r˘ad˘ acini consecutive ale derivatei f 0 (adic˘a f 0 (x1 ) = 0, f 0 (x2 ) = 0 ¸si nu exist˘a x ∈ (x1 , x2 ) astfel ˆıncˆat f 0 (x) = 0) atunci: (i) dac˘a f (x1 )f (x2 ) < 0, ecuat¸ia f (x) = 0 are exact o r˘ad˘ acina ˆın intervalul (x1 , x2 ); acin˘ a ˆın (ii) dac˘a f (x1 )f (x2 ) > 0, ecuat¸ia f (x) = 0 nu are nici o r˘ad˘ intervalul (x1 , x2 ); (iii) dac˘a f (x1 ) = 0 sau f (x2 ) = 0, atunci x1 sau x2 este o r˘ad˘ acin˘ a multipl˘ a a ecuat¸iei f (x) = 0 ¸si ecuat¸ia nu are nici o r˘ad˘ acin˘ a ˆın intervalul (x1 , x2 ). Definit¸ia 6.23. Spunem c˘a funct¸ia f : A → IR, A ⊂ IR, A mult¸ime deschis˘a, este derivabil˘ a de dou˘a ori ˆın punctul a ∈ A, dac˘a f este derivabil˘ a ˆıntr-o vecin˘atate a punctului a ¸si funct¸ia derivat˘ a f 0 este derivabil˘ a ˆın a. ˆIn acest caz derivata lui f 0 ˆın a se nume¸ste derivata a doua a lui f ˆın a ¸si se noteaz˘a f 00 (a). Definit¸ia 6.24. Spunem c˘a funct¸ia f : A → IR, A ⊂ IR, A mult¸ime deschis˘a, este derivabil˘a de n ori ˆın punctul a ∈ A (n ∈ IN, n ≥ 2), dac˘a f (n−1) este derivabil˘a ˆıntr-o vecin˘ atate a punctului a ¸si funct¸ia derivat˘ a f (n−1) este (n−1) derivabil˘a ˆın a. ˆIn acest caz derivata lui f ˆın a se nume¸ste derivata de ordin n a lui f ˆın a ¸si se noteaz˘a f (n) (a). Teorema 6.25. (Formula lui Taylor cu restul lui Peano.) Fie I ⊂ IR, I interval deschis ¸si n ∈ IN∗ . Dac˘a f : I → IR este o funct¸ie de n ori derivabil˘a ˆın a ∈ I, atunci exist˘a o funct¸ie α : I → IR cu proprietatea lim α(x) = α(a) = 0, astfel ˆıncˆ at x→a

f 00 (a) f 0 (a) (x − a) + (x − a)2 + .....+ 1! 2! f (n) (a) (x − a)n + (x − a)n + α(x) , n! n!

f (x) = f (a) +

pentru orice x ∈ I. 0 00 (n) Vom nota: Tn (x) = f (a)+ f 1!(a) (x−a)+ f 2!(a) (x−a)2 +. . .+ f n!(a) (x−a)n , n ∗ Rn (x) = α(x) · (x−a) si le vom numi polinomul Taylor, respectiv n! , n ∈ IN ¸ restul lui Peano, de ordin n ˆın punctul a.

215 Teorema 6.26. (Formula lui Taylor cu restul lui Lagrange.) Fie I ⊂ IR, I interval deschis, a ∈ I ¸si n ∈ IN. Dac˘a f : I → IR este o funct¸ie de (n + 1) ori derivabil˘a pe I, atunci pentru orice x ∈ I, x 6= a exist˘ a c ∈ (x, a) sau c ∈ (a, x) astfel ˆıncˆat f 00 (a) f 0 (a) (x − a) + (x − a)2 + .....+ 1! 2! f (n) (a) f (n+1) (c) + (x − a)n + (x − a)n+1 . n! (n + 1)!

f (x) = f (a) +

Particularizˆand a = 0, se obt¸ine formula lui MacLaurin. Vom nota Rn (x) = f (n+1) (c) (n+1)!

· (x − a)n , n ∈ IN ¸si-l vom numi restul lui Lagrange de ordinul n ˆın punctul a. Teorema 6.27. (Puncte de extrem.) Fie I ⊂ IR, interval deschis f : I → IR, o funct¸ie de n ori derivabil˘ a ˆın a ∈ I, (n ∈ IN, n ≥ 2), astfel 0 00 (n−1) ˆıncˆat f (a) = 0, f (a) = 0, ..., f (a) = 0, f (n) (a) 6= 0. (i) Dac˘a n este par, atunci a este punct de extrem, mai exact: punct de maxim local dac˘a f (n) (a) < 0 ¸si punct de minim local dac˘a f (n) (a) > 0. (ii) Dac˘a n este impar, atunci a nu este punct de extrem. Definit¸ia 6.28. a) Fie f : I → IR, I interval deschis, I ⊂ IR. Spunem c˘a f este diferent¸iabil˘a ˆın a ∈ I dac˘a exist˘a c ∈ IR ¸si α : I → IR continu˘ a ˆın a, lim α(x) = 0, astfel ˆıncˆat x→a

f (x) = f (a) + c(x − a) + α(x)(x − a), pentru orice x ∈ I. b) Spunem c˘a f este diferent¸iabil˘ a pe I dac˘a f este diferent¸iabil˘ a ˆın orice punct a ∈ I. Teorema 6.29. Dac˘a I ⊂ IR, I interval deschis, atunci f : I → IR este diferent¸iabil˘a ˆın a ∈ I dac˘a ¸si numai dac˘a f este derivabil˘ a ˆın a. ˆIn acest 0 caz, c = f (a) (c ∈ IR fiind constanta din definit¸ia 6.28). Definit¸ia 6.30. Fie I ⊂ IR, I interval deschis, f : I → IR derivabil˘ a ˆın a ∈ IR. Funct¸ia notat˘a df (a), df (a) : IR → IR, df (a)(h) = f 0 (a)h, ∀h ∈ IR, se nume¸ste diferent¸iala funct¸iei f ˆın punctul a. Definit¸ia 6.31. Fie f : A → IR, A ⊂ IR, A mult¸ime deschis˘ a, n ∈ IN∗ . n Spunem c˘a f este de clas˘a C pe A, dac˘a f este de n ori derivabil˘ a pe A, iar

216 derivata de ordin n, f (n) , este continu˘ a pe A. Vom nota C n (A) = {f |f : A → IR, f este de clas˘a C n pe A}, n ∈ IN∗ ¸si, prin convent¸ie, C 0 (A) = {f | f : A → IR, f continu˘a pe A}. Definit¸ia 6.32. Spunem c˘a f : A → IR, A ⊂ IR, A mult¸ime deschis˘ a, ∞ este de clas˘a C pe A dac˘a f este derivabil˘ a de orice ordin pe A. Vom nota C ∞ (A) = {f |f : A → IR, f este de clas˘a C ∞ pe A}.

217

Probleme 6.1 Studiat¸i(continuitatea ¸si derivabilitatea funct¸iilor: x3 − x2 , x ∈ Q a) f (x) = . Generalizare. 0, x ∈ IR\Q ( xn cos x1 , x 6= 0 b) f (x) = , n ∈ IN. 0, x = 0. 6.2 Fie f : D → IR, D ⊂ IR, D mult¸ime deschis˘ a, f derivabil˘ a ˆın a ∈ D. Atunci pentru orice dou˘a ¸siruri (xn ), (yn ) ⊂ D cu proprietatea xn < a < (xn ) yn , pentru orice n ∈ IN, xn → a, yn → a, avem lim f (yynn)−f = f 0 (a). −xn n→∞

6.3 Fie f : I → IR, I ⊂ IR, I interval deschis, a ∈ I. a ˆın a a) Dac˘a f este continu˘a ˆın a ¸si f (a) 6= 0, atunci |f | este derivabil˘ dac˘a ¸si numai dac˘a f este derivabil˘ a ˆın a. b) Dac˘a f (a) = 0, atunci |f | este derivabil˘ a ˆın a dac˘ a ¸si numai dac˘a f 0 este derivabil˘a ˆın a ¸si f (a) = 0. c) Aplicat¸ie pentru funct¸ia f : IR → IR, f (x) = |(x − α) sin x|, α ∈ IR. 6.4 a)(Exist˘a valori a, b, c ∈ IR, astfel ˆıncˆ at funct¸iei 2 ax + bx + c, x ∈ [−1, 0] f (x) = arcsin x , s˘ a-i putem aplica teorema lui Rolle? , x ∈ (0, 1] x Dar pentru funct ( ¸iile: arctg x x ∈ [−1, 0) x ,√ ; b) f (x) = ax + b 1 − x2 , x ∈ [0, 1] ( ax2 + b, x ∈ [−1, 0] c) f (x) = ; x2 ln x, x ∈ (0, 1] ( √ | sin x| α2 − x2 , x ∈ [−α, α] − {0} ; a ∈ IR, α ∈ (0, ∞)? d) f (x) = a, x=0 6.5 Dac˘a ak , bk ∈ IR, k = 0, n(n ∈ IN∗ ), atunci ecuat¸ia bk cos kx) = 0 are cel put¸in o solut¸ie ˆın (0, 2π).

n P

(ak sin kx +

k=1

6.6 Dac˘a 1 < a < b, f : [a, b] → IR este derivabil˘ a, f (a) = f (b) = 0, f (x) 6= 0, oricare ar fi x ∈ (a, b) atunci exist˘a un punct c ∈ (a, b) astfel ˆıncˆ at f 0 (c) 1 | f (c) | < a ln a .

218 6.7 Demonstrat¸i inegalit˘a¸tile: b−a b−a < tg a − tg b < , 0 < a < b < π2 ; a) cos2 a r cos2 b b2 −ab ab−a2 b2 + 1 a2 +1 , 1 < a < b; b) e b2 +1 < < e a2 + 1 µ ¶ 1 1 bb b−a c) a < < b, 0 < a < b (Examen de Admitere la Facultatea e aa de Informatic˘a– 1999). 1

1

6.8 a) Calculat¸i limitele laterale ale funct¸iei f (x) = x2 (e x − e x+1 ) ˆın punctele x = 0, x = −1 ¸si lim f (x). x→∞

b) Calculat¸i

lim (nα n→∞

1

1

+ 1)(2 n − 2 n+1 ), α ∈ IR. 1

c) Aplicˆand teorema Lagrange funct¸iei f (x) = e x pe intervalul [t, t + 1], t > 0 se obt¸ine punctul c(t). Calculat¸i lim t12 c2 (t). t→∞

6.9 a) Dac˘a A = {α ∈ IR|arctg x < αx, ∀x > 0}, demonstrat¸i c˘a A 6= ∅ ¸si determinat¸i inf A. b) Demonstrat¸i c˘a oricare ar fi a > 0, exist˘a α > 0, astfel ˆıncˆ at arctg x > αx, ∀x ∈ (0, a). c) Dac˘a h : (0, ∞) → IR, h(a) = sup{α > 0|arctg x > αx, ∀x ∈ (0, a)}, calculat¸i lim h(a) ¸si lim h(a). a→0 a>0

a→∞

d) Studiat¸i convergent¸a ¸sirului an = (arctg ◦ ... ◦ arctg)(x) (compunerea se face de n ori), n ∈ IN∗ , x ∈ IR. 6.10 Determinat¸i punctele de extrem ale urm˘atoarelor funct¸ii: 2 a) f : [−10, 10] → IR, f (x) =  |x − 3x + 2|; · ¸ 3π   x ∈ 0, | sin x|, µ 2 ¸; b) f : [0, 2π] → IR, f (x) = 4 3π   , 2π  2 (x − 2π), x ∈ 2  π  10x , 0≤x≤3 c) f : [0, 4] → IR, f (x) = 3(x2 + 1) ;  ln(x + e − 3), 3 < x ≤ 4 (Prof. dr. Anca Precupanu, Examen Analiz˘a matematic˘a I.) ( x3 , x≤0 d) f : IR → IR, f (x) = ; − 12 3 x +e x , x>0 mx + 1 e) f : IR\{−2, 1} → IR, f (x) = 2 , m ∈ IR; x +x−2

219 f) f : IR → IR, f (x) = e−x (ax2 + bx + b), µ a, b ∈ IR; ¶ 1 2 sin 2x g) f : IR → IR, f (x) = (x + x + 2) − − cos x . 2 4 6.11 a) Determinat (¸i punctele de extrem ale funct¸iei sin x1 , x 6= 0 f : IR → IR, f (x) = . 2, x=0 b) Exist˘a V ∈ V(0) astfel ˆıncˆ at f s˘ a fie monoton˘a pe V ∩ (0, ∞)? 6.12 Fie I ⊂ IR, I interval, f : I → IR, continu˘ a. Demonstrat¸i c˘a dac˘a a, b ∈ I sunt puncte de maxim local, atunci exist˘a c ∈ (a, b), punct de minim local. Proprietatea se p˘astreaz˘a dac˘a I nu este interval? a pe [a, b], derivabil˘ a ˆın punctele a, b astfel 6.13 Fie f : [a, b] → IR, continu˘ ˆıncˆat f 0 (a) < 0 ¸si f 0 (b) > 0. S˘a se arate c˘a f admite un punct de minim local ˆın (a, b). x at ecuat¸ia m arctg (x−1)+arctg x−2 = 6.14 a) Determinat¸i m ∈ IR, astfel ˆıncˆ π − 4 , x ∈ IR, s˘a aib˘a o infinitate de solut¸ii. 2x b) Rezolvat¸i ecuat¸ia 2arctgx + arcsin 1+x 2 = π, x ∈ IR.

6.15 a) Aratat¸i c˘a dac˘a α > 1, atunci xα ≥ 1 + α(x − 1), ∀x > 0 (inegalitatea lui Bernoulli), iar dac˘a α ∈ (0, 1), atunci xα ≤ 1 + α(x − 1), ∀x > 0. p q b) Deducet¸i inegalitatea lui Young: ap + bq ≥ ab, oricare ar fi a, b ≥ 0 ¸si p, q > 1, p1 + 1q = 1. at 6.16 Fie f (x) = 3x +ax − 4x − 6x , x ∈ IR. Determinat¸i a > 0, astfel ˆıncˆ f (x) ≥ 0, oricare ar fi x ∈ IR. 6.17 Dac˘a a > 0, atunci ax ≥ xa , pentru orice x > 0, dac˘a ¸si numai dac˘a a = e. 6.18 Fie a1 , a2 , ..., an > 0. Demonstrat¸i c˘a ax1 + ax2 + .... + axn ≥ n, pentru orice x ∈ IR, dac˘a ¸si numai dac˘a a1 a2 ...an = 1. π 6.19 Demonstrat¸i inegalitatea (x + 1) cos x+1 − x cos πx > 1, pentru orice x ≥ 2.

6.20 a) Demonstrat¸i : x − x ≥ 0.

x2 2

≤ ln(1 + x) ≤ x −

x2 2

+

x3 3 ,

pentru orice

220 b) Calculat¸i lim

x→0 x>0

x−ln(x+1) x2

prin dou˘a metode.

c) Definim ¸sirurile un+1 = ln(1 + un ), n ≥ 0, u0 = 1 ¸si vn = Determinat¸i: i) lim un ; ii) lim(vn+1 − vn ) n→∞

1 un , n

≥ 0.

n→∞

d) Demonstrat¸i c˘a oricare ar fi x ∈ (0, 1] are loc inegalitatea: 1 3 1 1 1 − x≤ − ≤ 2 16 ln(1 + x) x 2 2 4 e) Deducet¸i c˘a 41 ≤ vn+1 − vn ≤ 12 , ∀n ∈ IN ¸si n+2 ≤ un ≤ n+4 , ∀n ∈ IN; 1 1 1 f) Demonstrat¸i c˘a oricare ar fi n ∈ IN, 4 + 5 + ... + n+3 < ln(n + 3) ¸si calculat¸i apoi lim nun . n→∞

6.21 Fie funct¸ia f : IR → IR, f (x) = x3 + 3x2 + 3mx + 5, m ∈ IR. a) Determinat¸i m astfel ˆıncˆ at funct¸ia s˘a fie cresc˘atoare pe: i) IR; ii) (0, ∞). b) Dac˘a m = 0 ¸si I = (−∞, −2) demonstrat¸i c˘a restrict¸ia f : I → f (I) este inversabil˘a, iar f −1 este derivabil˘ a de dou˘a ori pe f (I). Calculat¸i −1 0 −1 00 (f ) (5), (f ) (5). at˘ a¸ti ale lui 0, c) Determinat¸i m ∈ IR cu proprietatea c˘a exist˘a I, J vecin˘ respectiv 5 astfel ˆıncˆ at f : I → J s˘ a fie bijectiv˘a. In acest caz calculat¸i (f −1 )0 (5), (f −1 )00 (5). 6.22 a) Demonstrat¸i c˘a funct¸ia f : IR → IR, f (x) = x3 + x + 1 + π4 arctg x, este inversabil˘a, cu inversa derivabil˘ a de dou˘a ori pe IR. b) Calculat¸i: (f −1 )0 (4) ¸si (f −1 )00 (4). −1 (x)]3 √ c) Calculat¸i lim x−[f . x−f −1 (x) x→∞

a, avˆ and derivata f 0 strict 6.23 Fie f : [a, b] → IR, o funct¸ie derivabil˘ cresc˘atoare. a) Demonstrat¸i c˘a ∀x ∈ (a, b], exist˘ a ¸si este unic determinat un num˘ ar cx ∈ (a, x) astfel ˆıncˆ at f (x) − f (a) = (x − a)f 0 (cx ). (a) b) Definim funct¸ia g : [a, b] → IR, g(t) = f (t)−f , pentru orice t ∈ (a, b] ¸si t−a 0 g(a) = f (a). S˘a se arate ca g este continu˘ a pe [a, b] ¸si strict cresc˘atoare pe (a, b]. c) Definim funct¸ia ϕ : [a, b] → IR, ϕ(a) = a, ϕ(x) = cx , unde cx este num˘arul definit la punctul a). S˘a se demonstreze c˘a ϕ este strict cresc˘atoare pe [a, b], iar mult¸imea valorilor funct¸iei este [a, ϕ(b)]. d) Daca ˆın plus, presupunem c˘a f este de dou˘a ori derivabil˘ a pe [a, b], f 00 00 continu˘a ¸si f (x) 6= 0, ∀x ∈ [a, b], atunci ϕ este derivabil˘ a cu derivata continu˘a.

221 6.24 Determinat¸i a ∈ (0, ∞) astfel ˆıncˆ at asimptotele funct¸iei f : IR\{− ln a} → IR, f (x) = 3x − 3 ln |aex − 1|, s˘ a fie concurente. 6.25 Fie f : [a, b] → IR, derivabil˘ a astfel ˆıncˆ at f (a) ≤ f (x) ≤ f (b), pentru orice x ∈ (a, b). Atunci exist˘a o funct¸ie fe : IR → [f (a), f (b)] derivabil˘ a pe IR 0 0 e ¸si f (x) = f (x), pentru orice x ∈ [a, b] dac˘a ¸si numai dac˘a f (a) = f (b) = 0. 6.26 Dac˘a f : (−1, 1) → IR, f derivabil˘ a ˆın 0 ¸si g : (−1, 1) → IR, g(x) = f (| sin x|), atunci g este derivabil˘ a ˆın 0 dac˘a ¸si numai dac˘a f 0 (0) = 0. 6.27 Determinat¸i ( funct¸ia polinomial˘a P astfel ˆıncˆ at funct¸ia 1 2 −1 x e , |x| < 1 f : IR → IR, f (x) = s˘a fie de clas˘a C ∞ pe IR. P (x), |x| ≥ 1 (Examen Titularizare Profesori-2000). a1 a2 6.28 Fie funct¸ia f : IR \ {α1 , α2 , ..., αn } → IR, f (x) = x−α + x−α +.... + 1 2 an x−αn − b, α1 , α2 , ..., αn , b ∈ IR, b 6= 0, a1 , a2 , .., an > 0, αi 6= αj , ∀i, j ∈ 1, n, i 6= j. Demonstrat¸i c˘a ecuat¸ia f (x) = 0 are toate solut¸iile reale.

a, f (0) = 0, 6.29 Fie I ⊂ IR, I interval deschis, 0 ∈ I, f : I → IR, f continu˘ n P 1 0 k f derivabil˘a la dreapta ˆın x = 0. Atunci lim f ( n2 ) = 2 fd (0). n→∞ k=1

a. Presupunem c˘a f 0 (x) > f (x), 6.30 Fie f : IR → IR o funct¸ie derivabil˘ pentru orice x ∈ IR ¸si f (0) = 0. Ar˘atat¸i c˘a f (x) > 0, pentru orice x > 0. 6.31 S˘a se determine funct¸iile f : IR → IR, derivabil˘ a ¸si g : IR → IR, m˘arginit˘a, care verific˘a relat¸ia f (x + y) − f (x − y) = y 2 g (xy) + y, oricare ar fi x, y ∈ IR. 6.32 Fie f : IR → IR o funct¸ie continu˘ a, derivabil˘ a pe IR∗ , cu propriet˘a¸tile 0 tf (t) > 0, pentru orice t ∈ IR∗ ¸si lim f (t) = lim f (t) = +∞. t→∞

t→−∞

at a) Ar˘atat¸i c˘a pentru orice x ∈ IR∗ , exist˘a y ∈ IR∗ , y 6= x, astfel ˆıncˆ f (x) = f (y). Not˘am elementul y prin ϕ(x) ¸si consider˘am ϕ(0) = 0. b) Ar˘atat¸i c˘a ϕ : IR → IR este bijectiv˘a ¸si ϕ−1 = ϕ. c) Ar˘atat¸i c˘a ϕ este continu˘a pe IR. d) Ar˘atat¸i c˘a ϕ este derivabil˘a pe IR∗ . (Examen de Licent¸˘a-iunie, 1996). 6.33 Aplicˆand reguli de tip L0Hˆ ospital, calculat¸i:

222 a) lim xα ln x, α ∈ IR; x→0 x>0

b) lim xα (− ln x)β ; α, β > 0; x→0 x>0

(− ln x)β ; α, β > 0; x→0 xα

c) lim

x>0

xx − x ; x→1 ln x − x + 1 P (x) e) lim ax , P polinom, a ∈ IR; x→∞ e 1 f) lim ( − xα ), α ∈ IR; x→0 sin x

d) lim

x>0

g) lim sin x ln x; x→0 x>0

h) lim xtgx ; x→0 x>0

1

i) lim (cos x) x2 ; x→0 1 x ) x2 ; j) lim ( x→0 arctgx sin 3x − sin x k) lim ; x→0 x − sin x e3x − 3ex + 2 l) lim ; x→0 x2 (x + 1) 1 1+x m) lim 2 ln ; x→0 x 1−x √ 3 1 + x4 − 1 n) lim ; x→0 x4 e x x −e o) lim ; x→e (x − e)2 ( f (x) p) lim , f (x) = x→0 g(x)

1

e− x2 sin2 x, x < 0 ¸si g(x) = ln cos x; x3 , x≥0

r) limπ (tgx)tg2x ; x→ 4

xn − sinn x ; x→0 p xn+2 p n n t) lim ( xn + axn−1 − xn − axn−1 ).

s) lim

x→∞

6.34 Demonstrat¸i c˘a pentru orice α ≥ 0, exist˘a m ≥ 0 astfel ˆıncˆ at x4 e−x < pentru orice x ≥ m.

1 xα ,

223 x

sin x

−e 6.35 Calculat¸i lim ex−sin x : x→0

a) aplicˆand regula lui L0Hˆospital de 3 ori; b) folosind limite fundamentale. tgx−arctgx Acela¸si exercit¸iu pentru lim sin x−x cos x . x→0

6.36 Ar˘atat¸i c˘a reciproca teoremei lui L0Hˆ ospital nu este adev˘arat˘ a pentru funct¸iile: ( x2 sin x1 , x 6= 0 ¸si g(x) = ln(1 + x) ˆın x = 0; a) f (x) = 0, x=0 ( x2 cos x1 , x 6= 0 b) f (x) = ¸si g(x) = sin x ˆın x = 0. 0, x=0 6.37 S˘a se arate c˘a nu se poate aplica regula lui L0Hˆospital (cazul funct¸iilor: a) f (x) = e−2x (cos x + 2 sin x), g(x) = e−x (cos x + sin x); 2 2 b) f (x) = e−2x (cos x + 2 sin x) + e−x sin2 x, g(x) = e−x ; c) f (x) = x − sin x, g(x) = x + sin x.

∞ ∞)

6.38 Se poate aplica teorema lui Cauchy funct¸iilor de mai jos ˆın punctul x=0? ( x3 , x ∈ Q a) f : IR → IR, f (x) = ¸si g : [−1, 1] → IR, g(x) = 0, x ∈ IR\Q arcsin x; ( x2 sin x1 , x 6= 0 b) f , g : IR → IR, f (x) = ¸si g(x) = sin x; 0, x=0 ( sin2 x, x < 0 c) f : IR → IR, f (x) = ¸si g : (− π2 , π2 ) → IR, g(x) = 2 x , x≥0 ln cos x. 6.39 Calculat¸i urm˘atoarele limite: a) lim tgn ( π4 + n1 ); n→∞

b) lim ( π2 arctgn)n ; n→∞

2

c) lim [cos(sin n1 )]n . n→∞

d) lim

n→∞

£ (n + 1)

√ √ ¤ n+1−nnn .

n+1

6.40 Scriet¸i formula lui Taylor de ordinul n(n ∈ IN∗ ) pentru funct¸iile:

224 a) f (x) = (1 + x)α , f : (−1, ∞) → IR, α ∈ IR, ˆın x = 0. Particularizat¸i ˆın cazul√α = 12 . b) f (x) = 2 + x ˆın punctul x = −1; c) f (x) = x1 , ˆın x = 1; d) f (x) = arcsin x, f : [−1, 1] → IR ˆın x = 0; e) f (x) = sin x ˆın x = π2 . 6.41 A) Fie f : IR → IR, f (x) = ex . a) Scriet¸i formula lui Taylor cu rest Lagrange de ordin n pentru funct¸ia f , ˆın punctele x = 0, x = 1. b) Determinat¸i n ∈ IN∗ , astfel ˆıncˆ at polinomul Taylor Tn asociat funct¸iei f ˆın punctul x = 0 s˘a aproximeze f ˆın intervalul [−1, 1] cu o precizie de 3 zecimale exacte. c) Dac˘a n = 4 s˘a se determine p ∈ IR+ , astfel ˆıncˆ at T4 s˘a aproximeze f ˆın intervalul [−p, p] cu o precizie de 0, 001. √ d) Calculat¸i 3 e cu o precizie de 0, 001. B) Aceea¸si problem˘a pentru funct¸iile f : (−1, ∞) → IR, f (x) = ln(1+x); √ g : (−1, ∞) → IR, g(x) = 1 + x, h : IR → IR, h(x) = sin x. 6.42 S˘a se calculeze limitele de la exercit¸iul 6.33, punctele k, l, m, n, t, folosind formula lui Taylor. 6.43 a) S˘a se determine cel mai mare n ∈ IN∗ , astfel ˆıncˆ at funct¸iei f (x) = |x| ln(1 + |x|), f : IR → IR s˘a i se poat˘a aplica formula lui Taylor cu rest Lagrange (Peano) de ordin n ˆın punctul x = 0. Scriet¸i formula ˆın acest caz. (Prof. dr. Anca Precupanu, Examen Analiz˘a matematic˘a I). Aceea¸si problem˘a pentru funct¸iile: b) g : IR → IR, g(x) = ln(2 − sin x) + |x|3 (x + 1); (Examen Analiz˘a matematic˘a I). c) h : IR → IR, h(x) = x3 e1+2|x| . Exist˘a valori a, b, c ∈ IR, astfel ˆıncˆ at funct¸iei 6.44 ( 2 e−x +x , x 0 atunci exist˘a V ∈ V(a), astfel ˆıncˆ at pentru orice x ∈ V s˘a avem f (x) > 0 (teorema 5.12). Presupunˆand c˘a f este derivabil˘a ˆın x = a putem scrie: lim

x→a

|f (x)| − |f (a)| |f (x)| − |f (a)| f (x) − f (a) = x→a lim = x→a lim = f 0 (a) x−a x − a x − a x∈V x∈V

¸si concluzion˘am c˘a |f | este derivabil˘ a ˆın x = a. Implicat¸ia invers˘ a rezult˘a analog. b) i) Dac˘a f este derivabil˘a in x = a ¸si f 0 (a) = 0 vom avea: lim

x→a x>a

|f (x)| − |f (a)| f (x) f (x) = x→a lim | | = | x→a lim | = |f 0 (a)| = 0 x−a x − a x − a x>a x>a

¸si astfel, |f | este derivabil˘a la dreapta ˆın x = a, cu (|f |)0d (a) = |f 0 (a)| = (a)| (x) 0. Analog x→a lim |f (x)|−|f = − x→a lim | fx−a | = −|f 0 (a)| = 0 ¸si astfel, |f | este x−a xa

x>a

f (x) f (x) − f (a) = 0 ⇔ x→a lim = 0, x−a x−a x>a

prin urmare f este derivabil˘a la dreapta ˆın x = a ¸si fd0 (a) = 0. Analog se demonstreaz˘a c˘a exist˘a fs0 (a) = 0. 6.4 a) Pe fiecare din intervalele [−1, 0) ¸si (0, 1], f este continu˘ a, fiind o funct¸ie elementar˘a. Pentru ca f s˘a fie continu˘ a ˆın x = 0 trebuie ca c = 1; condit¸ia f (−1) = f (1) antreneaz˘ a c˘a a − b = π2 − 1. Funct¸ia f este derivabil˘ a ( 2ax + b, x ∈ [−1, 0) pe [−1, 1] \ {0} ¸si f 0 (x) = x−√1−x2 arcsin x ; √ , x ∈ (0, 1] x2 1−x2

228 Studiem acum derivabilitatea funct¸iei ˆın punctul x = 0. Avem: 0

fs (0) =

ax2 + bx + c − c = b; x→0 x−0 lim

x0

lim

x→0 x>0

2x

x>0

x>0

=

√ 1 1−x2

−1

=

x2

³ ´ = 0, √ 2x 1 + 1 − x2

prin urmare f este derivabil˘ a ˆın x = 0 dac˘a ¸si numai dac˘a b = 0. ˆIn concluzie, dac˘a a = π − 1, b = 0, c = 1, vom putea aplica teorema lui 2 Rolle. Determinat¸i ξ ∈ [−1, 1] astfel ˆıncˆ at f 0 (ξ) = 0. d) Studiem continuitatea lui f ˆın punctul x = 0 : √ lim f (x) = lim | sin x| α2 − x2 = 0; condit¸ia f (0) = lim f (x) implic˘a a = 0. x→0

x→0

x→0

Pentru a studia derivabilitatea funct¸iei f avem dou˘a variante: Varianta I. ¯ ¯ √ ¯ ¯ Scrierea funct¸iei f ˆın forma f (x) = ¯sin x α2 − x2 ¯ , x ∈ [−1, 1] , ne su√ 2 2 α] gereaz˘a s˘a aplic˘am exercit¸iul 6.3. Funct¸ia g (x) = sin x √ α − x , x ∈ [−α, x 0 2 2 este derivabil˘a pe intervalul (−α, α) , g (x) = cos x α − x − sin x √α2 −x2 . Funct¸ia f este derivabil˘ a ˆın x = 0, dac˘a exist˘a g 0 (0) = 0, relat¸ie echivalent˘ a cu α = 0, fals! Varianta a II-a. Explicit˘am f ¸si obt¸inem: ( √ − sin x α2 − x2 , x ∈ [−α, 0] √ f (x) = . x ∈ (0, α]. sin x α2 − x2 , Funct¸ia f este derivabil˘ a la stˆanga, ˆın x = 0, deoarece exist˘a limita √ f (x) − f (0) − sin x α2 − x2 = lim = −α ∈ IR(fs0 (0) = −α); lim x→0 x→0 x x x0

x→0 x>0

1

lim f (x) = lim x2 e x = 0 · e−∞ = 0.

x→0 x 0, f (x) < 0. Aceast˘a inegalitate este echivalent˘ a cu arctgx < α, x ˆ oricare ar fi x > 0 ¸si g˘asim o condit¸ie necesar˘a, 1 ≤ α. In aceast˘a situat¸ie, 0 f (x) = x21+1 − α < 0, oricare ar fi x > 0 ¸si constat˘am c˘a f este strict descresc˘atoare pe (0, ∞) , deci pentru orice x > 0, f (x) < 0. Concluzion˘am c˘a A = [1, ∞) ¸si inf A = 1. a a b) Vom demonstra c˘a exist˘a α = arctg astfel ˆıncˆ at arctg x > arctg a a x, pentru orice x ∈ (0, a) ¸si este clar c˘a monotonia funct¸iei g : (0, a] → IR, x g(x) = arctg ne va oferi r˘aspunsul. Constat˘am c˘a g este descresc˘atoare, prin x x a urmare pentru orice x ∈ (0, a), arctg > arctg ceea ce ˆıncheie demonstrat¸ia. x a c) Vom determina mult¸imea B = {α > 0|arctg x > αx, ∀x ∈ (0, a)}. a arctg a Dac˘a 0 < α ≤ arctg a , atunci αx ≤ a x < arctg x, pentru orice x ∈ (0, a), a arctg a ˆ deci (0, a ] ⊂ B. In cazul ˆın care α > arctg a , vom avea f (a) = arctg a − αa < 0 ¸si cu teorema 5.12, va rezulta existent¸a unui interval (a0 , a00 ), 0 < a0 < a < a00 , astfel ˆıncˆ at pentru orice x ∈ (a0 , a00 ), f (x) < 0. Concluzion˘am a ˆ c˘a α ∈ / B ¸si B = (0, arctg a ]. In final lim h(a) = 1, iar lim h(a) = 0. a→0 a>0

a→∞

231 d) Adaug˘am ¸sirului (an ) elementul a0 = x, ceea ce nu schimb˘ a natura lui. Dac˘a x > 0, vom avea a0 > a1 ¸si, prin induct¸ie, se demonstreaz˘a c˘a an > an+1 , ∀n ∈ IN. S¸irul fiind m˘arginit inferior, el va fi convergent; trecˆand la limit˘a ˆın relat¸ia de recurent¸˘ a (an+1 = arctgan , n ∈ IN∗ ) obt¸inem lim an = 0. Cazul x < 0 se rezolv˘a analog.

n→∞

6.10

a) Studiul funct¸iei explicitate ( x2 − 3x + 2, x ∈ [−10, 1] ∪ [2, 10] f (x) = 2 −(x − 3x + 2), x ∈ (1, 2) ne arat˘a c˘a f este derivabil˘a pe [−10, 1) ∪ (1, 2) ∪ (2, 10] ¸si ( 2x − 3, x ∈ [10, 1) ∪ (2, 10] 0 f (x) = −2x + 3, x ∈ (1, 2). Punctele de extrem se determin˘a astfel: i) cele situate ˆın interiorul intervalelor de derivabilitate ale funct¸iei (ˆın cazul nostru (−10, 1), (1, 2), (2, 10)) se g˘asesc printre punctele critice (conform teoremei lui Fermat); ele se determin˘a apoi, fie folosind monotonia funct¸iei, fie cu derivata a doua a funct¸iei. ii) ˆın plus, mai pot fi puncte de extrem extremit˘a¸tile intervalelor de derivabilitate ale funct¸iei f (ˆın cazul nostru x = ±10, x = 1, x = 2). i) Pentru a determina punctele critice vom rezolva, mai ˆıntˆ ai, ecuat¸ia f 0 (x) = 0. Dac˘a x ∈ [−10, 1) ∪ (2, 10], obt¸inem 2x − 3 = 0 ¸si solut¸ia nu convine; dac˘a x ∈ (1, 2), obt¸inem −2x + 3 = 0 ¸si x = 32 este punct critic. Funct¸ia este cresc˘atoare la stˆanga ¸si descresc˘atoare la dreapta punctului x = 32 (facet¸i tabelul de variat¸ie), prin urmare el este punct de maxim. ii) S˘a studiem dac˘a x = −10 este punct de extrem. Funct¸ia f este descresc˘atoare la dreapta punctului x = −10, deci exist˘a V ∈ V(−10) astfel ˆıncˆat pentru orice x ∈ V ∩ [−10, 10], f (x) ≤ f (−10), adic˘ a x = −10 este punct de maxim. Analog x = 10 este punct de maxim. Studiem dac˘a x = 1 este punct de extrem. Pentru x < 1, f 0 (x) < 0 ¸si astfel, f este descresc˘atoare pe [−10, 1); pentru x ∈ (1, 23 ), f 0 (x) > 0 ¸si f este cresc˘atoare pe (1, 23 ). ˆIn concluzie x = 1 este punct de minim. Demonstrat¸i c˘a ¸si x = 2 este punct de minim. b) Se expliciteaz˘a funct¸ia ¸si se studiaz˘a derivabilitatea lui f ˆın punctele x = π, x = 3π a cu cea de la punctul a). ˆIn final, 2 ; rezolvarea este apoi analoag˘ vom obt¸ine c˘a x = 0, x = π sunt puncte de minim, iar x = π2 , x = 3π 2 , x = 2π sunt puncte de maxim (x = π2 este punct critic; x = π, x = 3π sunt puncte 2 ˆın care funct¸ia nu este derivabil˘a). 2) 1 c) Avem f 0 (x) = 10(1−x , pentru , pentru x ∈ [0, 3) ¸si f 0 (x) = x+e−3 2 3(x +1)2 x ∈ (3, 4]. Studiem derivabilitatea ˆın punctul x = 3 : observ˘am c˘a funct¸ia

232 10x admite derivat˘ a ˆın x = 3, deci fs0 (3) 3(x2 +1) 2) 4 g 0 (3) = 10(1−3 = − 15 . Analog fd0 (3) = 1e ¸si concluzion˘am c˘a 3(32 +1)2 derivabil˘a ˆın x = 3. Rezolvˆand ecuat¸ia f 0 (x) = 0 ˆın cazurile

g : IR → IR, g(x) =

= gs0 (3) =

f nu este x ∈ [0, 3) ¸si x ∈ (3, 4] g˘asim punctul critic x = 1, iar studiul monotoniei funct¸iei ne arat˘a c˘a f este descresc˘atoare pe intervalul [1, 3] ¸si cresc˘atoare pe intervalele [0, 1) ¸si (3, 4]. Prin urmare x = 0 este punct de minim, x = 4 este punct de maxim (de¸si ele nu sunt puncte critice!), x = 1 este punct de maxim, el fiind o r˘ad˘acin˘a a derivatei, iar punctul x = 3 este ¸si el punct de extrem, de¸si el nu este punct de derivabilitate al funct¸iei. ( 3x2 , x < 0 1 ¸si studiem derivabilitatea ˆın d) Avem f 0 (x) = 3x2 + x23 e− x2 , x > 0 x = 0, folosind o consecint¸˘ a a teoremei lui Lagrange (propozit¸ia 6.15). 3 Funct¸ia f este continu˘ a; lim f 0 (x) = 0, lim f 0 (x) = 0 + lim uu2 = 0 (u = x1 ). Astfel va exista

x→0 x0

u→∞ e

0 ¸si rezult˘a c˘a f este derivabil˘ a ˆın x = 0,

f 0 (0) = 0. Singurul punct critic este x = 0, dar f este cresc˘atoare ¸si la stˆanga ¸si la dreapta acestuia, prin urmare x = 0 nu este punct de extrem. f) Calcul˘am f 0 (x) = e−x [−ax2 + (2a − b)x] ¸si afl˘am punctele critice x = 0 ¸si x = 2a−b (dac˘a a 6= 0) sau x = 0 (dac˘a a = 0). ˆIn primul caz ambele a puncte critice sunt puncte de extrem (facet¸i tabelul de variat¸ie!). Dac˘a a = 0, atunci x = 0 nu este punct de extrem. g) Calcul˘am f 0 (x) = x + 12 − ( cos22x + sin x) ¸si folosind ¸sirul lui Rolle, se arat˘a c˘a singurul punct critic este x = 0. Calcul˘am: f 00 (x) = 1 + sin 2x − cos x, f 00 (0) = 0; f 000 (x) = 2 cos 2x + sin x, f 000 (0) = 2 6= 0 ¸si, conform teoremei 6.27, rezult˘a c˘a x = 0 nu este punct de extrem. S˘a remarc˘am c˘a ˆın acest caz, studiul monotoniei funct¸iei f ˆıntr-o vecin˘ atate a punctului x = 0 este destul de complicat. 6.11 a) Observ˘am mai ˆıntˆ ai c˘a f nu este derivabil˘ a ˆın x = 0. Rezolvˆand 2 ecuat¸ia f 0 (x) = 0, obt¸inem solut¸iile x = (2k+1)π , k ∈ Z (punctele critice). Analizˆand semnul derivatei const˘at˘ am c˘a f este cresc˘atoare pe fiecare din 2 2 intervalele [ (4k+3)π , (4k+1)π ], k ∈ Z\{0}, ¸si descresc˘atoare pe fiecare din in2 2 2 tervalele [ (4k+5)π , (4k+3)π ], k ∈ Z. Astfel punctele x = (4k+1)π vor fi puncte 2 de maxim, iar x = (4k+3)π puncte de minim. Deoarece f (x) ≤ f (0) pentru orice x ∈ IR, punctul x = 0 este punct de maxim (de¸si f nu este nici m˘acar continu˘a ˆın 0). Demonstrat¸i c˘a dac˘a valoarea funct¸iei f ˆın punctul x = 0 ar fi 0, atunci x = 0 nu ar fi punct de extrem. 2 b) pentru orice V ∈ V(0), exist˘ a k ∈ Z, k > 0 astfel ˆıncˆ at (0, (4k+3)π )⊂

233 V ∩ (0, ∞) ¸si avˆand ˆın vedere punctul a), f nu este monotona pe V ∩ (0, ∞). 6.12 Fie a, b ∈ I, a < b, puncte de maxim local pentru f . Restrict¸ia lui f la intervalul [a, b] (pe care o not˘am tot f ) este o funct¸ie continu˘ a pe un compact, prin urmare ˆı¸si atinge marginea inferioar˘a, adic˘a exist˘a xm ∈ [a, b], astfel ˆıncˆat inf f (x) = f (xm ). Evident, xm este punct de minim local x∈[a,b]

pentru f pe intervalul [a, b]. Dac˘a xm ∈ (a, b) atunci problema e rezolvat˘ a. Dac˘a xm = a, atunci exist˘a V ∈ V(a), astfel ˆıncˆ at f (x) ≥ f (xm ) = f (a) ¸si f (x) ≤ f (a), pentru orice x ∈ V ∩ (a, b). Rezult˘ a c˘a f (x) = f (a) pentru orice x ∈ V ∩ (a, b) ¸si astfel, orice punct x ∈ V ∩ (a, b) este un punct de minim local situat ˆıntre a ¸si b. Cazul xm = b se rezolv˘a analog. Dac˘a I nu este interval, proprietatea nu se p˘astreaz˘ a, un exemplu fiind funct¸ia ( x+1 x , x ∈ [−1, 0) f : [−1, 0) ∪ (0, 1], f (x) = x−1 x , x ∈ (0, 1], care este continu˘a, x = −1, x = 1 sunt puncte de maxim local, dar ea nu admite nici un punct de minim local pe I. 6.13 Relat¸ia f 0 (a) < 0 antreneaz˘ a c˘a exist˘a V ∈ V(a) astfel ˆıncˆ at, pentru f (x)−f (a) orice x ∈ V ∩ (a, b], s˘a avem x−a < 0 sau, echivalent, f (x) < f (a). Deci x = a este punct de maxim local pentru f. Analog se demonstreaz˘a c˘a x = b este tot punct de maxim local pentru f. Concluzia rezult˘a acum conform problemei precedente. 6.14

a) Not˘am x − 1 = t ¸si definim : f (t) = marctgt + arctg

t+1 , ∀t ∈ IR\{1}. t−1

Atunci f 0 (t) = m−1 oricare ar fi t ∈ IR\{1}. Dac˘a m = 1, atunci f (t) = c1 ∈ 1+t2 IR, ∀t ∈ (−∞, 1) ¸si particularizˆand t = 0 obt¸inem c1 = − π4 ; de asemenea f (t) = c2 ∈ IR, ∀t ∈ (1, ∞), c2 = arctg2 + arctg3. Astfel, ecuat¸ia dat˘a va avea o infinitate de solut¸ii, mai exact toate valorile x ∈ (−∞, 2) (corespunz˘atoare valorilor t ∈ (−∞, 1)). Dac˘ a m < 1 vom avea f 0 (t) < 0, deci f este strict descresc˘atoare pe intervalele (−∞, 1) ¸si (1, ∞). Rezult˘ a c˘a ecuat¸ia dat˘a admite cel mult o solut¸ie pe fiecare din aceste intervale, altfel spus cel mult dou˘a solut¸ii pe IR \ {2}. Cazul m > 1 se rezolv˘a analog. 6.15 a) S˘a presupunem c˘a α > 1 ¸si s˘a consider˘am funct¸ia f : [0, ∞) → IR, f (x) = xα −α(x−1)−1. Studiul monotoniei funct¸iei se face astfel: f 0 (x) > 0

234 este echivalent˘a cu α(xα−1 − 1) > 0 sau cu x > 1, prin urmare f este cresc˘atoare pe intervalul (1, ∞) ¸si descresc˘atoare pe intervalul (0, 1). Rezult˘a c˘a f (x) ≥ f (1) = 0, ∀x > 0, adic˘a inegalitatea lui Bernoulli. Cea de-a doua inegalitate se poate demonstra la fel ca mai sus sau astfel: ˆın prima inegalitate not˘am α = β1 , β ∈ (0, 1) ¸si trecem x ˆın xβ . Vom obt¸ine: x≥

1 β (x − 1) + 1 ⇔ xβ ≤ β(x − 1) + 1, ∀x > 0, β ∈ (0, 1) β

¸si a doua inegalitate este demonstrat˘a. b) Varianta I. ˆIn cea de-a doua inegalitate de la punctul a) facem α = p1 , p > 1, x = ab , a > 0, b > 0 ¸si avem: ³a´1

p

b



1 1 1a 1 a b ≤ ⇔ ap bq ≤ + . pb q p q

Trecem apoi a ˆın ap ¸si b ˆın bq ¸si inegalitatea lui Young este demonstrat˘a. Varianta a II-a. Vom da ¸si o demonstrat¸ie direct˘a a inegalit˘a¸tii. Conp q sider˘am funct¸ia f : (0, ∞) → IR, f (a) = ap + bq − ab ( b > 0, fixat) ¸si studiem variat¸ia acestei funct¸ii. Rezolvˆand ecuat¸ia f 0 (a) = 0, obt¸inem 1 solut¸ia a = b p−1 care este punct critic. Studiem apoi semnul derivatei: 1 f 0 (a) > 0 dac˘a ¸si numai dac˘a a > b p−1 , prin urmare f este strict cresc˘atoare 1 1 1 pe intervalul (b p−1 , ∞) ¸si strict descresc˘atoare (0, b p−1 ) ¸si, astfel, b p−1 este punct de minim pentru f . Urmeaz˘a c˘a 1 1 1 p bq 1 bq +1 − b p−1 = bq + − bq = 0, ∀a > 0, f (a) ≥ f (b p−1 ) = b p−1 + p q p q

adic˘a concluzia. 6.16

Ipoteza f (x) ≥ 0, ∀x ∈ IR, implic˘a faptul c˘a x = 0 este punct de ◦

minim al funct¸iei f ¸si cum 0 ∈ IR, urmeaz˘a c˘a f 0 (0) = 0. Aceast˘a relat¸ie antreneaz˘a a = 8, condit¸ie necesar˘a. ˆInlocuind ˆın f obt¸inem: f (x) = 3x + 8x − 4x − 6x = (4x − 3x )(2x − 1) ≥ 0, ∀x ∈ IR, deci condit¸ia este ¸si suficient˘ a. 6.17

Avem: ax ≥ xa , ∀x > 0 ⇔ x ln a ≥ a ln x, ∀x > 0 ⇔

ln a ln x ≤ , ∀x > 0. x a

235 Prin urmare, x = a este punct de maxim pentru funct¸ia f (x) = lnxx pe intervalul (0, ∞) ¸si deci, f 0 (a) = 0. Ultima relat¸ie antreneaz˘ a a = e. Reciproc, dac˘a a = e, va trebui s˘a demonstr˘am c˘a pentru orice x > 0, are loc ex ≥ xe sau echivalent x − e ln x ≥ 0. Funct¸ia g : (0, ∞) → IR, g (x) = x − e ln x, are minimul absolut ˆın x = e, prin urmare g (x) ≥ g (e) = 0, pentru orice x > 0, ceea ce trebuia demonstrat. 6.18 Implicat¸ia “⇒ ” se rezolv˘a analog cu cea de la problema 6.17. Reciproc, presupunˆand c˘a a1 a2 ...an = 1 ¸si scriind inegalitatea mediilor avem: 1 ax1 + ax2 + .... + axn ≥ n (ax1 ax2 ...axn ) n = n, pentru orice x ∈ IR. 6.19 O abordare ˆın maniera problemelor precedente nu va da rezultate ˆın acest caz. Aplicˆand funct¸iei f : [2, ∞) → IR, f (t) = t cos πt , teorema lui Lagrange pe intervalul [x, x + 1], rezult˘a c˘a exist˘a cx ∈ (x, x + 1), astfel ˆıncˆ at π π π f (x + 1) − f (x) = cos cx + cx sin cx . Pentru a evalua aceast˘a expresie, vom studia variat¸ia funct¸iei g(y) = cos y + y sin y, y ∈ (0, π2 ]. Funct¸ia g este strict π cresc˘atoare, deci g(y) > g(0) = 1, ∀y ∈ (0, π2 ]. Luˆ and ˆın particular y = , cx obt¸inem f (x + 1) − f (x) > 1, adic˘ a concluzia. 2

2

x 6.20 a) Not˘am f (x) = ln(x + 1) − x + x2 , x ≥ 0 ¸si avem f 0 (x) = x+1 ≥ 0, pentru orice x ≥ 0. Rezult˘a c˘a f este cresc˘atoare pe [0, ∞), deci f (x) ≥ f (0) = 0, pentru orice x ≥ 0. A doua inegalitate se rezolv˘a analog. b) Conform punctului a) avem:

x − ln(x + 1) 1 x 1 − ≤ ≤ , ∀x ≥ 0 2 2 3 x 2 ¸si trecˆand la limit˘a pentru x → 0, obt¸inem lim

x→0 x>0

x−ln(x+1) x2

= 12 .

Limita se poate calcula ¸si folosind regula lui L0Hˆospital. c) i) Prin induct¸ie rezult˘a imediat c˘a un > 0, pentru orice n ∈ IN. Avˆand ˆın vedere inegalitatea ln(1 + x) < x, pentru orice x > 0, urmeaz˘a c˘a un+1 = ln(1 + un ) < un , pentru orice n ∈ IN, adic˘ a (un ) este strict descresc˘ator. Concluzion˘am c˘a (un ) este convergent ¸si fie lim un = l ≥ 0. n→∞

Trecˆand la limit˘a ˆın relat¸ia de recurent¸˘ a obt¸inem l = ln(1 + l) cu solut¸ia unic˘a l = 0.

236 c) ii) Avem un − ln(1 + un ) n→∞ un ln(1 + un ) un − ln(1 + un ) un = lim lim n→∞ n→∞ ln(1 + un ) u2n x x − ln(x + 1) 1 lim = lim = . 2 x→0 ln(1 + x) x→0 x 2

lim (vn+1 − vn ) = lim

n→∞

d) Inegalitatea din stˆanga este echivalent˘ a cu : ln(x + 1) ≤

−3x2

16x , x ∈ (0, 1]. + 8x + 16

Considerˆand funct¸ia h : [0, 1] → IR, h(x) = ln(1 + x) − −3x216x se demon+8x+16 streaz˘a h0 (x) < 0, ∀x ∈ [0, 1] ceea ce implic˘a h(x) < 0, ∀x ∈ [0, 1]. Inegalitatea din dreapta rezult˘a analog. e) i) ˆIn inegalitatea de la punctul d) lu˘am x = un ∈ (0, 1], ∀n ∈ IN ¸si 1 n obt¸inem 21 − 3u 16 ≤ vn+1 − vn ≤ 2 , ∀n ∈ IN; pentru a demonstra partea 1 n stˆang˘a a inegalit˘a¸tii din enunt¸, r˘amˆ ane s˘a ar˘at˘ am c˘a 12 − 3u 16 ≥ 4 , ∀n ∈ IN relat¸ie echivalent˘a cu un ≤ 43 , ∀n ∈ IN, adev˘ arat˘ a! ii) Scriem inegalit˘a¸tile anterioare pentru n = 0, 1, ..., p − 1, sum˘am de la 0 la p − 1 ¸si vom avea: p p 2 4 ≤ vp − v0 ≤ , ∀p ∈ IN ⇔ ≤ up ≤ , ∀p ∈ IN. 4 2 p+2 p+4 f) Prima afirmat¸ie rezult˘a din exercit¸iul 2.5-d). Pentru a g˘asi lim nun , n→∞ vom determina o ˆıncadrare a ¸sirului, iar apoi vom aplica teorema cle¸stelui 1 k (propozit¸ia 2.26-vi). ˆIn inegalit˘a¸tile 21 − 3u 16 ≤ vk+1 − vk ≤ 2 , k ∈ IN facem k = 0, n − 1 ¸si sumˆand vom obt¸ine: n 3 n − (u0 + u1 + .. + un−1 ) ≤ vn − v0 ≤ , ∀n ∈ IN∗ . 2 16 2 Dar conform celei de a doua inegalit˘a¸ti de la punctul e) avem: 1 1 1 u0 + u1 + .. + un−1 ≤ 4( + + .. + ) ≤ 4 ln(n + 3), ∀n ∈ IN 4 5 n+3 ¸si rezult˘a c˘a: n2 − 34 ln(n + 3) ≤ vn − v0 ≤ n2 , oricare ar fi n ∈ IN. ˆInlocuind vn = u1n , v0 = 1 vom g˘asi ˆıncadrarea pentru ¸sirul (nun ) : 1 3 ln(n + 3) 1 1 1 1 − + ≤ ≤ + , ∀n ∈ IN. 2 4 n n nun 2 n

237 La limit˘a, pentru n → ∞, rezult˘ a c˘a lim nu1 n = 12 ¸si lim nun = 2. S˘ a n→∞ n→∞ observ˘am c˘a, folosind criteriul Stolz-Ces`aro, putem calcula mult mai simplu limita ¸sirului ( nu1 n ) : 1

1 lim = lim un = lim n→∞ nun n→∞ n n→∞

µ

1 un+1

1 − un



1 = lim (vn+1 − vn ) = . n→∞ 2

6.21 a) Funct¸ia f este cresc˘atoare pe IR dac˘a ¸si numai dac˘a f 0 (x) ≥ 0, pentru orice x ∈ IR, ceea ce antreneaz˘ a c˘a ∆ ≤ 0, adic˘ a m ≥ 1. Pentru ca f s˘a fie cresc˘atoare pe (0, ∞), ˆın plus fat¸˘ a de situat¸ia anterioar˘ a, vom considera √ ¸si cazul cˆand ∆ > 0 (m < 1). R˘ad˘ acinile derivatei sunt x1,2 = −1 ± 1 − m ¸si impunˆand condit¸ia (0, ∞) ⊂ (x2 , ∞), obt¸inem ¸si solut¸ia m ∈ [0, 1). ˆIn final, m ∈ [0, 1) ∪ [1, ∞) = [0, ∞). b) Scriet¸i tabelul de variat¸ie al funct¸iei ¸si observat¸i c˘a f (I)=(−∞, f (−2)) = (−∞, 9). Funct¸ia este surjectiv˘a ¸si, fiind strict cresc˘atoare, este ¸si injectiv˘a. Ecuat¸ia f 0 (x) = 0 are solut¸iile x = 0 ¸si x = −2, prin urmare f −1 este 1 derivabil˘a pe mult¸imea (−∞, −2) (teorema 6.9). Avem (f −1 )0 (5) = f 0 (−3) = 1 9 , iar (f −1 )00 (y) = ( =−

1 f 0 (f −1 (y))

1 [f 0 (f −1 (y))]3

deci (f −1 )00 (5) =

12 53

=

)0 = −

1 [f 0 (f −1 (y))]2

f 00 (f −1 (y)) = −

f 00 (f −1 (y))[f −1 (y)]0

f 00 (x) , ∀x ∈ (−∞, −2), y = f (x), f 3 (x)

12 125 .

6.22 a) Funct¸ia f este strict cresc˘atoare pe IR, fiind suma a trei funct¸ii 1 strict cresc˘atoare pe IR (sau deoarece f 0 (x) = 3x2 + 1 + π4 1+x 2 > 0, pentru orice x ∈ IR). Ea este de asemenea continu˘ a ¸si lim f (x) = −∞, x→−∞

lim f (x) = ∞. A¸sadar f este surjectiv˘a ¸si cum f 0 (x) 6= 0, oricare ar fi

x→∞

x ∈ IR, rezult˘a c˘a f −1 este derivabil˘ a pe IR (teorema 6.9). La fel ca ˆın exercit¸iul precedent se demonstreaz˘a c˘a f −1 este derivabil˘ a de dou˘a ori pe IR. b) Se rezolv˘a analog cu problema 6.21-b). c) Notˆand f −1 (x) = u, observ˘am c˘a, lim f −1 (x) = ∞ ¸si avem: x→∞

£

¤3 x − f −1 (x) f (u) − u3 p lim √ = lim = x→∞ x→∞ x − f −1 (x) f (u) − u

238

³ lim

u→∞

arctgu u

u + 1 + π4 arctgu = lim q = x→∞ u3 + u + 1 + π4 arctgu − u ¡ ¢ u 1 + u1 + π4 arctgu u µq = lim u→∞ 3 − u2 1 + u12 + u13 + π4 arctgu u3 ´ π 2 =∞ = 0! .

¶ =0 √1 u

6.23 a) Existent¸a num˘ arului cx este asigurat˘a de teorema lui Lagrange. Dac˘a am presupune c˘a c1x , c2x ∈ (a, x), c1x < c2x , sunt dou˘a numere care verific˘a teorema atunci ar rezulta c˘a f 0 (c1x ) = f 0 (c2x ). Dar cum f 0 este strict cresc˘atoare, vom avea f 0 (cx1 ) < f 0 (c2x ), contradict¸ie! b) i) Funct¸ia g este continu˘ a pe intervalul (a, b] ca o compunere de funct¸ii (a) continue; egalitatea lim g(t) = lim f (t)−f = f 0 (a) = g(a) ne asigur˘a c˘a g t−a t→a

t→a

este continu˘a ¸si ˆın t = a. b) ii) Este suficient s˘a demonstr˘am c˘a g 0 (t) > 0, ∀t ∈ (a, b]. Avem: f 0 (t)(t − a) − (f (t) − f (a)) > 0, ∀t ∈ (a, b] ⇔ (t − a)2 f (t) − f (a) , ∀t ∈ (a, b] ⇔ f 0 (t) > f 0 (ct ), ∀t ∈ (a, b], ⇔ f 0 (t) > t−a ultima relat¸ie fiind adev˘arat˘ a deoarece ct < t ¸si f 0 strict cresc˘atoare. 0 0 c) i) Funct¸ia f : [a, b] → [f (a), f 0 (b)] este inversabil˘ a, prin urmare egalitatea f 0 (ϕ(x)) = f 0 (cx ) = g(x), ∀x ∈ [a, b] implic˘a ϕ(x) = (f 0 )−1 (g(x)), ∀x ∈ [a, b]. Dar f 0 ¸si g sunt strict cresc˘atoare ¸si astfel rezult˘a concluzia. c) ii) Faptul c˘a ϕ este strict cresc˘atoare antreneaz˘ a c˘a ϕ([a, b]) ⊂ [ϕ(a), ϕ(b)] = [a, ϕ(b)]. Reciproc s˘a demonstr˘am c˘a ∀y ∈ [a, ϕ(b)], exist˘a x ∈ [a, b] astfel ˆıncˆ at y = ϕ(x). Aceast˘ a ultim˘a egalitate fiind echivalent˘a cu f 0 (y) = f 0 (ϕ(x)) = g(x), dac˘a am demonstra c˘a f 0 (y) ∈ [g(a), g(b)], atunci num˘arul c˘autat va fi x = g −1 (f 0 (y)). ˆIntr-adev˘ ar constat˘am c˘a g(a) ≤ f 0 (a) ≤ f 0 (y) ≤ f 0 (ϕ(b)) = f 0 (cb ) = g(b), ceea ce ˆıncheie demonstrat¸ia. d) Cum ϕ = (f 0 )−1 ◦ g, vom studia dac˘a (f 0 )−1 ¸si g sunt derivabile cu derivata continu˘a. Funct¸ia g este evident derivabil˘ a, cu derivata continu˘ a pe (a, b]. Relat¸iile g 0 (t) =

lim x→a x>a

g(t) − g(a) f (t) − f (a) − f 0 (a)(t − a) = x→a lim = t−a (t − a)2 x>a = x→a lim x>a

f 0 (t) − f 0 (a) 1 = f 00 (a) 2(t − a) 2

239 ne dovedesc derivabilitatea ˆın x = a, iar lim g 0 (t) = x→a lim x→a x>a

x>a

f 0 (t)(t − a) − f (t) + f (a) 1 = f 00 (a) = g 0 (a) (t − a)2 2

continuitatea lui g 0 ˆın x = a. S˘a observ˘am acum c˘a f 0 este derivabil˘ a, (f 0 )0 (x) 6= 0, ∀x ∈ [a, b], prin urmare, conform teoremei 6.9, (f 0 )−1 este derivabil˘a; ˆın plus, deoarece f 00 este continu˘ a, derivata este chiar continu˘ a. ˆIn concluzie, ϕ este derivabil˘a cu derivata continu˘ a. 6.24

Studiem existent¸a asimptotelor: lim f (x) = lim (3x − 3 ln |aex − 1|) = −∞,

x→−∞

x→−∞

prin urmare nu avem asimptote orizontale la −∞. Calcul˘am:

lim (f (x) −

x→−∞

3x) = lim (−3 ln |aex − 1|) = 0 ¸si, conform definit¸iei 5.23, concluzion˘am x→−∞

c˘a dreapta y = 3x este asimptot˘a oblic˘a la −∞. Egalit˘ a¸tile lim f (x) = lim (3x − 3 ln ex |a −

x→∞

x→∞

1 1 |) = lim (−3 ln |a − x |) = −3 ln a, x x→∞ e e

dovedesc c˘a dreapta y = −3 ln a este asimptot˘a orizontal˘ a la +∞. Observ˘am c˘a f este definit˘a atˆat la stˆanga cˆat ¸si la dreapta punctului x = − ln a, deci putem calcula limita: lim

x→− ln a

= (3x − 3 ln |aex − 1|) = −3 ln a − 3 ln(0+ ) = +∞.

Prin urmare dreapta x = − ln a, este asimptot˘a vertical˘ a pentru f. Observ˘am c˘a cele trei asimptote sunt concurente pentru orice a > 0. 6.25 Pentru implicat¸ia direct˘a, presupunem c˘a exist˘a fe : IR → [f (a), f (b)], cu propriet˘a¸tile ment¸ionate. ˆIn acest caz, fe(x) ≥ f (a) = fe(a), pentru orice x ∈ IR, deci x = a este punct de minim pentru fe. Rezult˘ a c˘a fe0 (a) = 0 ¸si cum fe0 (a + 0) = f 0 (a + 0) = f 0 (a), ˆın final f 0 (a) = 0. Analog f 0 (b) = 0. Invers, definim fe : IR → [f (a), f (b)],   f (a), x < a fe(x) = f (x), x ∈ [a, b]   f (b), x > b ¸si se demonstreaz˘a c˘a fe este derivabil˘ a pe IR ¸si are propriet˘a¸tile cerute.

240

6.26

( f (− sin x), x ∈ [−1, 0] ¸si calcul˘am: Explicit˘am funct¸ia g : g(x) = f (sin x), x ∈ (0, 1] gs0 (0) = lim

x→0 x 0, f (αn +0)f (+∞) < 0; prin urmare a n− a r˘ad˘ acin˘ a a lui f exist˘ a ˆın intervalul (−∞, α1 ) sau ˆın intervalul (αn , +∞), unde s-a notat f (−∞) = lim f (x) x→−∞

¸si analog pentru f (+∞). 6.29

Din definit¸ia derivabilit˘a¸tii la dreapta ˆın x = 0, rezult˘ a c˘a lim (

n→∞

k −1 k ) · [f ( 2 ) − f (0)] = fd0 (0), k ∈ 1, n. 2 n n

Rezult˘a c˘a oricare ar fi ε > 0, exist˘a nε ∈ IN, astfel ˆıncˆ at pentru orice n ≥ nε ¸si orice k ∈ 1, n s˘a avem: µ ¶ k 0 k k (fd (0) − ε) < f < 2 (fd0 (0) + ε) 2 2 n n n (demonstrat¸i c˘a nε nu depinde de k).

242 Prin sumare rezult˘a

fd0 (0)−ε n2

·

n(n+1) 2


0, g(x) > g(0) = 0 ¸si rezult˘a concluzia. 6.31

Pentru y 6= 0, avem f (x + y) − f (x) f (x − y) − f (x) + = yg (xy) + 1, y −y 0

oricare ar fi x, y ∈ IR; trecˆand la limit˘a pentru y → 0, vom obt¸ine f (x) + 0 f (x) = 1 ¸si, ˆın final, f (x) = 12 x + c, c ∈ IR. ˆInlocuind acum f ˆın relat¸ia init¸ial˘a ¸si luˆand y = 1, rezult˘ a g (x) = 0, oricare ar fi x ∈ IR. 6.32 Relat¸ia tf 0 (t) > 0 antreneaz˘ a faptul c˘a f este strict descresc˘atoare pe (−∞, 0) ¸si strict cresc˘atoare pe (0, ∞). S˘a demonstr˘am c˘a f este strict descresc˘atoare chiar pe (−∞, 0]. Este suficient s˘a ar˘at˘ am c˘a pentru orice x < 0, f (x) > f (0). Fie ¸sirul (yn ), strict cresc˘ator, yn → 0, x < yn < 0, ∀n ∈ IN; avem f (x) > f (yn ) ¸si, la limit˘a, f (x) > f (yn ) ≥ lim f (yn ) = n→∞

f (0), ceea ce trebuia demonstrat. Analog se demonstreaz˘a c˘a f este strict cresc˘atoare pe [0, ∞). Funct¸ia f1 , restrict¸ia lui f la (−∞, 0], este inversabil˘ a, −1 f1 fiind continu˘a pe (−∞, 0] ¸si derivabil˘ a pe (−∞, 0) (justificat¸i). Analog f2 , restrict¸ia lui f la [0, ∞), este inversabil˘ a, f2−1 fiind continu˘ a (pe [0, ∞)) ¸si derivabil˘a pe (0, ∞). S˘a observ˘am c˘a f1 ([0, ∞)) = [f1 (0), ∞) = [f (0), ∞) = [f2 (0), ∞) = f2 ([0, ∞)). a), b) Fie x ∈ (−∞, 0); cum f (x) ∈ (f (0), ∞) = f2 ((0, ∞)), urmeaz˘ a c˘a exist˘a y > 0 (unic), astfel ˆıncˆ at f2 (y) = f (y) = f (x), ceea ce trebuia

243 demonstrat. De fapt, y = f2−1 (f1 (x)). Analog, dac˘a x > 0, exist˘a y < 0 at f (y) = f (x). Funct¸ia (unic), mai exact y = f1−1 (f2 (x)) astfel ˆıncˆ  −1  f2 (f1 (x)), x < 0 ϕ : IR → IR, ϕ(x) = f1−1 (f2 (x)), x > 0   0, x = 0 este bijectiv˘a. Pentru y < 0, ϕ−1 (y) = x, astfel ˆıncˆ at ϕ(x) = y, x > 0. Dar ultima relat¸ie este echivalent˘a cu (f1−1 ◦ f2 )(x) = y ⇔ f2 (x) = f1 (y) ⇔ x = (f2−1 ◦ f1 )(y) = ϕ(y) ¸si rezult˘a c˘a ϕ−1 (y) = ϕ(y). Egalitatea se demonstreaz˘a analog dac˘a y > 0. c), d) Funct¸ia ϕ este evident derivabil˘ a pe (−∞, 0) ¸si pe (0, ∞) fiind o compunere de funct¸ii derivabile. Studiem continuitatea ˆın x = 0 : lim ϕ(x) = f2−1 (f1 (0)) = f2−1 (f (0)) = 0;

x→0 x0

6.33 a α > 0, atunci a) Dac˘a α ≤ 0, atunci lim xα ln x = −∞. Dac˘ x→0 x>0

lim xα ln x = lim

x→0 x>0

x→0 x>0

ln x x−α

0 ospital, cazul ∞ ¸si avem o nedeterminare ∞ ∞ . Vom aplica regula lui L Hˆ ∞: 1 funct¸iile f, g : (0, ∞) → IR, f (x) = ln x, g(x) = xα , sunt derivabile, lim |g(x)| = ∞, g 0 (x) 6= 0, ∀x ∈ (0, ∞). Existent¸a limitei x→0 x>0

1 f 0 (x) 1 x = lim = lim =0 0 −α−1 x→0 g (x) x→0 −αx x→0 −αx−α

lim

x>0

x>0

x>0

f (x) x→0 g(x) x>0

antreneaz˘a faptul c˘a exist˘a ¸si limita lim

α

= 0;

b) Avem: lim xα (− ln x)β = lim (−x β ln x)β = 0β = 0 (conform punctului a)!).

x→0 x>0

x→0 x>0

244 d) Aplic˘am regula lui L0Hˆ ospital pentru cazul 00 : funct¸iile f, g : (0, ∞) → IR, f (x) = xx −x, g(x) = ln x−x+1 sunt derivabile, lim f (x) = lim g(x) = 0, g 0 (x) =

1 x

x→1

x→1

− 1 6= 0, ∀x ∈ (0, ∞) \ {1}. Calcul˘ am f 0 (x) xx ln x + xx − 1 ln x = lim = lim xx+1 1 0 x→1 g (x) x→1 x→1 1−x x −1 lim

ln x xx − 1 x(xx − 1) = lim − lim . x→1 1 − x x→1 x − 1 x→1 x − 1

− lim

Limitele obt¸inute se calculeaz˘a tot cu regula lui L0Hˆ ospital, cazul lim ln x x→1 1−x

1 x

= lim −1 = −1; lim x→1

init¸ial˘a, deoarece exist˘a

xx −1

x→1 x−1 0 (x) lim fg0 (x) = x→1

= lim

xx

x→1

∞ ∞

1

:

= 1. Revenind la limita

(x) −2, va rezulta c˘a exist˘a ¸si lim fg(x) = −2.

xn ax , x→∞ e

e) Vom calcula mai ˆıntˆ ai l = lim regula lui L0Hˆospital (cazul

ln x+xx

0 0

x→1

n ∈ IN∗ . Dac˘ a a > 0, aplicˆ and

) de n ori obt¸inem c˘a:

xn nxn−1 n! = lim = ... = lim n ax = 0. ax ax x→∞ e x→∞ ae x→∞ a e p P Dac˘a a ≤ 0 atunci l = ∞. Fie acum P (x) = ap−k xk , p ∈ IN∗ ; lim

Dac˘a a > Dac˘a a ≤ f) Fie l =

k=0

p P

k 0, = ap−k lim exax = 0. x→∞ k=0 P (x) 0, atunci lim eax = (sgn a0 ) · ∞. x→∞ lim ( sin1 x − xα ), α ∈ IR. Dac˘a α > 0, atunci x→0 x>0

lim P (x) ax x→∞ e

l=

1 0+

− 0 = +∞;

dac˘a α = 1, l = ∞. Dac˘ a α < 0, atunci avem o nedeterminare ∞ − ∞. Not˘am 1 xβ 1 − 1) xα = β , β > 0; l = lim β ( x→0 x x sin x x>0

1 x β−1 = lim β ( x − 1). x→0 x sin x x>0

Dac˘a 0 < β < 1, atunci l = ∞(∞ − 1) = ∞; dac˘a β > 1, l = −∞; dac˘a β = 1, obt¸inem o nedeterminare ∞ · 0 : 1 x x − sin x x − sin x ( − 1) = lim = lim = x→0 x sin x x→0 x sin x x→0 x2 lim

x>0

x>0

x>0

1 − cos x sin x = lim = lim = 0. x→0 x→0 2 2x x>0

x>0

245 j) lim (

x→0

arctg x x−arctg x 1 1 x x − arctg x x−arctg · 2 x ] arctg x x ) x2 = lim [(1 + ) x→0 arctg x arctg x

lim

=e

x→0

x−arctg x 1 · 2 arctg x x

=e

x−arctg x x3 x→0

lim

lim

1 1+x2 3x2

1−

= ex→0

1

= e3 .

3x−sin x 3x−cos x 3x+sin x k) lim sinx−sin = lim 3 cos1−cos = lim −9 sinsin = −26. x x x x→0

l) lim e x→0

3x −3ex +2 x3 +x2

x→0

3x

x→0

x

3x

x

−3e = lim 3e3x2−3e = lim 9e6x+2 = 3. +2x x→0

x→0

m) ˆIn acest caz nu putem aplica regula lui L0Hˆospital pentru c˘a limita 0

(x) lim fg0 (x) = lim

2 1−x2

x→0

x→0 2x

lim

Vom avea:

f (x) ). x→0 g(x)

= lim x1 nu exist˘a (atent¸ie! nu rezult˘a c˘a nu exist˘a x→0

2x

ln(1 + 1−x ) 2x 1 1 1 1+x = lim lim · 2 = 2 lim = +∞; ln 2x 2 x→0 1 − x x x→0 x x→0 x 1 − x x→0 1−x lim

x>0

x>0

x>0

1 1+x si, ˆın concluzie, 2 ln 1−x = −∞ ¸ x→0 x x0

limita nu exist˘a.

p) Vom calcula limitele laterale astfel:

1

1

e− x2 x2 e− x2 sin2 x = lim ls (0) = lim x→0 ln(cos x) x→0 ln(cos x) x0

x→0 x>0

α2 (x)x2 , ∀x ∈ (−1, 1). Egalit˘ a¸tile

1 2 [2x + α1 (x)x2 − α2 (x)x2 ] = lim = +∞ x→0 x x2 x>0

¸si lim f (x) = −∞ implic˘a faptul c˘a limita dat˘a nu exist˘a. x→0 x0 ( Analog se arat˘a c˘a exist˘a f 00 (0) = 2 ¸si f 00 (x) = derivabilitatea de ordinul 3 ˆın punctul x = 0 :

2−x , (1−x)2 2+x , (1+x)2

x≤0 x>0

. Studiem

1 2−x [ − 2] = 3; x→0 x − 0 (1 − x)2

fs000 (0) = lim

x

1 3

¸si divergent˘ a pentru α ≤ 13 .

n P

( n1 )3α ,

k=0

6.47 Vom ˆıncerca s˘a g˘asim o ˆıncadrare a funct¸iei sin ˆıntre dou˘a polinoame. Scriind formula lui Taylor cu rest Lagrange de ordinul 3, ˆın punctul x = 0, pentru funct¸ia sin vom avea: sin x = x −

x3 x4 + sin ξ, ξ ∈ (0, x) sau(x, 0). 3! 4!

Dac˘a x ∈ (0, π), atunci sin ξ > 0 ¸si obt¸inem c˘a x − 3!1 x3 ≤ sin x, ∀x ∈ [0, π]. Relat¸ia sin x ≤ x, ∀x > 0 este cunoscut˘a. Luˆand x = nk2 ˆın aceste dou˘a inegalit˘a¸ti ¸si sumˆand pentru k ∈ 1, n obt¸inem n(n + 1) 1 [n(n + 1)]2 n(n + 1) − ≤ an ≤ , ∀n ∈ IN∗ 2n2 3! 4n6 2n2 ¸si, ˆın final, lim an = 21 . S˘a mai observ˘am c˘a o alt˘a solut¸ie se poate obt¸ine n→∞ folosind exercit¸iul 6.29. 6.48 i) Dac˘a α < 0, atunci f admite ˆın x = 0 asimptot˘a vertical˘ a la stˆanga, deci, conform exercit¸iului 5.42, ea nu este uniform continu˘ a. ii) Dac˘a α > 1, vom demonstra c˘a f nu este uniform continu˘ a. Varianta I. Pentru y > x > 0, avem c˘a y α − xα = αξ α−1 (y − x) ≥ ¡ ¢ 1 αxα−1 (y − x) , unde ξ ∈ (x, y) . Luˆand xn = n, yn = n + n1 1−α , avem c˘a yn − xn → 0 ¸si f (yn ) − f (xn ) ≥ α. Deci, f nu este uniform continu˘ a. Varianta a II-a. Considerˆand β ∈ IR, 0 < β < α − 1, xn = n + n1β , yn = n, n ∈ IN∗ , egalitatea ¯µ ¯ ¶α ¯ ¯ 1 α¯ lim |f (xn ) − f (yn )| = lim n ¯ 1 + 1+β − 1¯¯ n→∞ n→∞ n ne sugereaz˘a scrierea dezvolt˘arii binomiale de ordin m ∈ IN∗ cu rest Peano, 1 trecˆand x ˆın n1+β : µ 1+

1 n1+β

¶α

α 1 α (α − 1) =1+ + 1+β 1! n 2!

α (α − 1) .. (α − m + 1) + m!

µ

1 n1+β

µ

¶2

1

+ ...+

n1+β

¶m

µ + Rm

1 n1+β

¶ ,

254 ¡ 1 ¢ ¡ 1 ¢ ¡ 1 ¢m ¡ 1 ¢ unde Rm n1+β = g n1+β , lim g n1+β = 0. Astfel limita anten1+β n→∞ rioar˘a devine: ¯ µ ¶µ ¶m ¯ ¯ ¯ α (α − 1) 1 1 1 α−β−1 ¯ α ¯ = ∞, lim n + + ... + g ¯ ¯ 1! n→∞ 2! n1+β n1+β n1+β ¸si rezult˘a concluzia. iii) Dac˘a α ∈ [0, 1] , atunci f este uniform continu˘ a pe (0, 1]; pe intervalul [1, ∞) f este lipschitzian˘ a , fiind derivabil˘ a cu derivata m˘arginit˘ a ¯ ¯ ´ ³¯ 0 ¯ ¯ ¯ α−1 ¯¯ ≤ α, ∀x ∈ [1, ∞) , prin urmare f este ¸si aici uniform ¯f (x)¯ = αx continu˘a. Vom demonstra acum c˘a f este uniform continu˘ a pe (0, ∞) adic˘a oricare ar fi ε > 0, exist˘a δε > 0, astfel ˆıncˆ at oricare ar fi x, y ∈ (0, ∞) , |x − y| < δε , s˘a rezulte |f (x) − f (y)| < ε. Pentru c˘a f este uniform continu˘ a pe (0, 1] , oricare ar fi ε > 0, exist˘ a δε0 > 0, astfel ˆıncˆ at oricare ar fi x, y ∈ (0, 1] , |x − y| < δε0 , s˘a avem |f (x) − f (y)| < ε. ˆIn mod analog oricare ar fi ε > 0, exist˘a δε00 > 0, astfel ˆıncˆ at oricare ar fi x, y ∈ [1, ∞) , |x − y| < δε00 , s˘ a avem |f (x) − f (y)| < ε. Fie acum δε = min (δε0 , δε00 ) , x ∈ (0, 1] , y ∈ [1, ∞) , astfel ˆıncˆat |x − y| < δε . ˆIn primul rˆand s˘a observ˘am c˘a vom avea ¸si |x − 1| < δε ≤ δε0 , |y − 1| < δε ≤ δε00 , ceea ce, ˆın virtutea relat¸iilor anterioare, va antrena |f (x) − f (y)| ≤ |f (x) − f (1)| + |f (1) − f (y)| < 2ε ¸si concluzia este imediat˘a. 6.49 Vom demonstra, mai ˆıntˆ ai, prin induct¸ie c˘a f (p) (0) = 0, pentru orice 1 a rezult˘a c˘a f (0) = 0. p ∈ IN. Din f ( n ) = 0, oricare ar fi n ∈ IN∗ ¸si f continu˘ 0 (p) Presupunem c˘a f (0) = 0, f (0) = 0, ..., f (0) = 0 ¸si s˘a demonstr˘am c˘a f (p+1) (0) = 0. Pentru aceasta vom scrie formula lui Taylor cu rest Lagrange de ordin (p + 1) : pentru orice x ∈ IR, exist˘a ξp ∈ (0, x) sau ξp ∈ (x, 0) astfel ˆıncˆat f (p+1) (0) p+1 f (p+2) (ξp ) p+2 f (x) = x + x . (p + 1)! (p + 2)! 1 2 1 Luˆand x = n1 obt¸inem |f (p+1) (0)| = | p+2 f (p+2) (ξp ) n1 | ≤ p+2 n , pentru orice ∗ n ∈ IN ¸si, trecˆand la limita pentru n → ∞, rezult˘ a f (p+1) (0) = 0. A¸sadar (p) f (0) = 0, pentru orice p ∈ IN. Rescriem prima relat¸ie:

0 = |f (x)| = |

f (p+2) (ξp ) p+2 2 x |≤| xp+2 |, ∀p ∈ IN (p + 2)! (p + 2)!

¸si pentru p → ∞, vom avea f (x) = 0, pentru orice x ∈ IR, adic˘a concluzia.

255 6.50 Consider˘am un ¸sir (an ) avˆ and limita ∞, astfel ˆıncˆ at an > a+1, ∀n ∈ IN ¸si scriem formula lui Taylor cu rest Lagrange de ordin 3, ˆın punctul an : f 0 (an ) f 00 (an ) f 000 (ξn ) + + , 1! 2! 3! ξn ∈ (an , an + 1) , n ∈ IN

f (an + 1) = f (an ) +

(1)

f 0 (an ) f 00 (an ) f 000 (ηn ) + − , 1! 2! 3! ηn ∈ (an − 1, an ) , n ∈ IN .

f (an − 1) = f (an ) −

(2) Egalitatea

f (an + 1) − f (an − 1) = 2f 0 (an ) +

f 000 (ξn ) f 000 (ηn ) + , 3! 3!

obt¸inut˘a prin sc˘aderea relat¸iilor (1) ¸si (2) justific˘a existent¸a limitei lim f 0 (an ) n→∞

¸si, trecˆand la limit˘a, obt¸inem c˘a lim f 0 (an ) = 0. Cum ¸sirul (an ) a fost ales n→∞ oarecare, rezult˘a c˘a exist˘a lim f 0 (x) = 0. ˆIn mod analog se demonstreaz˘a x→∞

c˘a lim f 00 (x) = 0. x→∞

6.51

a) Avem: √ √ √ lim An = lim ( n a1 + n a2 + .. + n as ) = n→∞ √ √ √ = lim n a1 + lim n a2 + ..+ lim n as = s.

n→∞

n→∞

n→∞

n→∞

b) ˆIn mod analog: ³p p p ´ n b1 + n b2 + .. + n bt = n→∞ p p p = lim n b1 + lim n b2 + ..+ lim n bt = t,

lim Bn = lim

n→∞

n→∞

n→∞

n→∞

¸si avˆand ˆın vedere egalitat¸ile Ak = Bk , oricare ar fi k ∈ IN, k ≥ 2, rezult˘a c˘a s = t. c) Folosind metoda induct¸iei matematice, rezolvarea este imediat˘a. d) Limita cerut˘a este f 0 (0) = ln a. e) Scriind formula lui Taylor cu rest Lagrange de ordin n, ˆın punctul x = 0, pentru funct¸ia f , rezult˘a c˘a oricare ar fi x ∈ IR, exist˘a ξx ∈ (0, x) 0

00

sau ξx ∈ (x, 0) astfel ˆıncˆat f (x) = f (0) + f 1!(0) x + f 2!(0) x2 + ... + f

(n) (0)

n!

xn +

256 f (n+1) (ξx ) n+1 . Observˆand c˘a, dac˘a x (n+1)! x µ ¶ 0 f (0) f 00 (0) f (n) (0) f (x)− f (0)+ 1! x+ 21 x2 +...+ n! xn xn+1

→ 0, atunci ξx → 0, vom avea: lim

x→0

f (n+1) (ξx ) (n+1)! x→0

= lim

=

f (n+1) (0) (n+1)!

oricare ar

fi n ∈ IN (ultima egalitate fiind justificat˘a de continuitatea funct¸iei f (n+1) ). x x x x f) Notˆand g (x) = ax1 + ax2 + .. ¡ 1+¢ as − b1 − b2 − .. − bs , x ∈ IR, g este indefinit derivabil˘a, g (0) = 0, g k = 0, oricare ar fi k ∈ IN, k ≥ 2, ¸si trebuie s˘a demonstr˘am c˘a g (n) (0) = 0, oricare ar fi n ∈ IN∗ . Varianta I. Folosind punctul e) se arat˘a c˘a ³ ´ 0 00 (n) g (x) − g (0) + g 1!(0) x + g 21(0) x2 + ... + g n!(0) xn g (n+1) (0) lim = , x→0 xn+1 (n + 1)! oricare ar fi n ∈ IN. S˘ a presupunem acum c˘a g (k) (0) = 0, oricare ar fi (n+1) g ( k1 ) (0) k ∈ IN∗ . Avem: g (n+1)! = lim xg(x) n+1 = lim 0 = 0, ceea ce n+1 = lim x→0 k→∞ k→∞ ( k1 ) (n+1) implic˘a g (0) = 0. Prin urmare, conform metodei induct¸iei matematice, rezult˘a concluzia. ¯ ¯ Varianta a II-a. Exist˘a dou˘a constante c, M ∈ IR, astfel ˆıncˆ at ¯g (n) (x)¯ ≤ cM n , oricare ar fi x ∈ [−α, α] , α ∈ IR, deci g este dezvoltabil˘ a ˆın serie Taylor pe IR, ˆın jurul punctului x = 0. Mult¸imea zerourilor lui g are un punct de acumulare, x = 0, ceea ce antreneaz˘ a c˘a g este identic zero. g) S˘a presupunem, mai ˆıntˆ ai, c˘a as < bs , deci vom avea ¸si ln as < ln bs . ˆImp˘art¸im relat¸ia de la punctul f) prin (ln bs )n ¸si trecem la limit˘a pentru n→∞: ·µ ¶ µ ¶ µ ¶ ¸ ln a1 n ln a2 n ln as n lim + + ... + = n→∞ ln bs ln bs ln bs ·µ ¶ ¶ ¶ µ µ ¸ ln b1 n ln b2 n ln bs−1 n = lim + + ... + +1 . n→∞ ln bs ln bs ln bs Dar limita din stˆanga este 0, iar cea din dreapta, mai mare sau egal˘a cu 1, contradict¸ie! Daca as > bs , obt¸inem de asemenea o contradict¸ie ¸si astfel, as = bs . Egalitatea de la punctul f) devine (ln a1 )n + (ln a2 )n + ... + (ln as−1 )n = (ln b1 )n + (ln b2 )n + ... + (ln bs−1 )n , oricare ar fi n ∈ IN ¸si analog, se arat˘a c˘a as−1 = bs−1 . ˆIn final, vom obt¸ine concluzia.

Capitolul 7 S ¸ IRURI DE FUNCT ¸ II

Definit¸ia 7.1. Fie D ⊂ R ¸si fn : D → R, n ∈ N ∗ , un ¸sir de funct¸ii. Spunem c˘a punctul x este punct de convergent¸˘ a pentru ¸sirul de funct¸ii ∗ ∗ (fn )n∈N , dac˘a ¸sirul numeric (fn (x))n∈N , este convergent. Mult¸imea punctelor de convergent¸˘a ale ¸sirului (fn )n∈N ∗ o vom numi mult¸ime de convergent¸˘ a ¸si o vom nota cu C. Fiec˘arui x ∈ C, putem s˘a-i asociem num˘ arul real notat f (x) = lim fn (x). Aceast˘a asociere este o funct¸ie f : C → R, pe care o vom n→∞

numi limita punctual˘a a ¸sirului de funct¸ii (fn ). A¸sadar, vom spune c˘a ¸sirul de funct¸ii (fn ) converge punctual la funct¸ia f pe mult¸imea C, dac˘a pentru orice x ∈ C, ¸sirul numeric (fn (x))n∈N ∗ , este convergent la f (x), sau, ˆıntr-o scriere explicit˘a, dac˘a pentru orice x ∈ C ¸si pentru orice ε > 0, exist˘a un rang nε,x ∈ N, astfel ˆıncˆat pentru orice n ≥ nε,x , s˘ a avem |fn (x) − f (x)| < ε. p In acest caz vom scrie fn → f (pe mult¸imea C). Remarca 7.2. Scriind condit¸ia de uniform˘a convergent¸˘ a ˆın forma: pen∗ tru orice ε > 0, exist˘a nε ∈ N , astfel ˆıncˆ at pentru orice n ≥ nε ¸si pentru orice x ∈ A, s˘a avem f (x) − ε < fn (x) < f (x) + ε,obt¸inem interpretarea geometric˘a a convergent¸ei uniforme ¸si anume: ¸sirul (fn )converge uniform la f dac˘a ¸si numai dac˘a, pentru orice ε > 0, exist˘a nε ∈ N ∗ , astfel ˆıncˆ at pentru orice n ≥ nε , graficul funct¸iilor fn este situat ˆın banda determinat˘a de graficele funct¸iilor f − ε si f + ε. p

Remarca 7.3. Dac˘a fn → f (pe mult¸imea C) si A ⊂ C,atunci, ˆın mod p evident, rezult˘a c˘a fn → f (pe mult¸imea A). Definit¸ia 7.4. Fie A ⊂ D ⊂ R, f : D → R ¸si fn : D → R, n ∈ N ∗ , un ¸sir de funct¸ii. Spunem c˘a ¸sirul de funct¸ii (fn ) converge uniform la funct¸ia f pe mult¸imea A dac˘a pentru orice ε > 0, exist˘a un rang nε ∈ N ∗ , astfel ˆıncˆ at pentru orice n ≥ nε ¸si pentru orice x ∈ A, s˘a avem |fn (x) − f (x)| < ε. ˆIn u acest caz vom scrie fn → f (pe mult¸imea A). 257

258 u

Remarca 7.5. Dac˘a fn → f (pe mult¸imea A) si B ⊂ A,atunci, ˆın mod u evident, rezult˘a c˘a fn → f (pe mult¸imea B). Propozit¸ia 7.6. Fie A ⊂ D ⊂ R , f : D → R ¸si fn : D → R, n ∈ N ∗ , un sir de functii convergent uniform la f pe A. Atunci (fn ) converge punctual la f, pe A. Teorema 7.7. Fie A ⊂ D ⊂ R, f : D → R ¸si fn : D → R, n ∈ N ∗ , un ¸sir de funct¸ii. Atunci ¸sirul (fn ) converge uniform la f, pe A dac˘a ¸si numai dac˘a lim (sup|fn (x) − f (x)|) = 0. n→∞ x∈A

Remarca 7.8. Dac˘ a ˆın propozit¸ia anterioar˘ a consider˘am A = [a, b] si notam C[a, b], mult¸imea funct¸iilor continue pe [a, b],este binecunoscut faptul c˘a aplicat¸ia k k∞ : C[a, b] → R, definit˘ a prin ||f ||∞ = sup |f (x)|, f ∈ x∈[a,b]

C[a, b] este o norm˘a. S˘a remarc˘am c˘a un ¸sir (fn ) converge uniform la f, pe [a, b], dac˘a ¸si numai dac˘a (fn ) converge ˆın norma la f (din acest motiv ||||∞ se mai nume¸ste ¸si norma convergent¸ei uniforme). Remarca 7.9. Pentru a putea aplica teorema anterioar˘ a, trebuie, ˆın primul rand, determinat˘a funct¸ia f, limita punctual˘a a ¸sirului de funct¸ii (fn ), f (x) = lim fn (x), x ∈ C (C mult¸imea de convergent¸˘ a a ¸sirului (fn )). n→∞

Definit¸ia 7.10. Fie A ⊂ D ⊂ R, f : D → R ¸si fn : D → R, n ∈ N ∗ , un ¸sir de funct¸ii. Spunem c˘a ¸sirul de funct¸ii (fn ) este sir uniform Cauchy pe multimea A, daca pentru orice ε > 0, exist˘a un rang nε ∈ N ∗ , astfel ˆıncˆ at pentru orice m, n ≥ nε ¸si pentru orice x ∈ A, avem |fm (x) − fn (x)| < ε. Teorema 7.11. (Criteriul lui Cauchy) Fie D ⊂ R, f : D → R si fn : D → R, n ∈ N ∗ , un ¸sir de funct¸ii. S¸irul de funct¸ii (fn ) este uniform convergent pe D dac˘a ¸si numai dac˘a (fn ) este ¸sir uniform Cauchy pe mult¸imea D. Teorema 7.12. (Criteriul majorarii) Fie A ⊂ D ⊂ R, f : D → R si fn : D → R, n ∈ N ∗ , un ¸sir de funct¸ii. Dac˘a exist˘a un ¸sir (αn ) ⊂ R, convergent la 0, astfel ˆıncˆ at, pentru orice n ∈ N ∗ ¸si pentru orice x ∈ A, avem |fn (x) − f (x)| < αn , atunci ¸sirul (fn ) converge uniform la f , pe A. Teorema 7.13. (Criteriul lui Dinii) Fie f : [a, b] → R,continua si fn : [a, b] → R, n ∈ N ∗ , un ¸sir de funct¸ii continue, astfel ˆıncˆ at f1 (x) ≤ f2 (x) ≤ ... ≤ fn (x) ≤ fn+1 (x) ≤ ..., ∀x ∈ [a, b],

259 iar (fn )converge punctual la f. Atunci (fn ) converge uniform la f. Teorema 7.14. (Transfer de m˘ arginire) Fie D ⊂ R, f : D → R si fn : D → R, n ∈ N ∗ , un ¸sir de funct¸ii m˘arginite. Dac˘a (fn ) converge uniform la f pe D, atunci f este m˘arginit˘ a. Teorema 7.15. (Transfer de existent¸˘ a a limitei ˆın punct) Fie D ⊂ R, f : D → R ¸si fn : D → R, n ∈ N ∗ , un ¸sir de funct¸ii uniform convergent la f. Dac˘a x0 ∈ D0 ¸si exista lim fn (x), pentru orice n ∈ N ∗ , x→x0

atunci exist˘a lim f (x) ¸si, ˆın plus, x→x0

lim ( lim fn (x)) = lim ( lim f (x))

x→x0 n→∞

n→∞ x→x0

Teorema 7.16. (Transfer de continuitate) Fie D ⊂ R, f : D → R si fn : D → R, n ∈ N ∗ , un ¸sir de funct¸ii continue ˆıntr-un punct x0 ∈ D. Dac˘ a (fn ) converge uniform la f pe D, atunci f este continu˘ a ˆın x0 . Corolarul 7.17. Fie D ⊂ R, f : D → R ¸si fn : D → R, n ∈ N ∗ , un ¸sir de funct¸ii continue (pe D). Dac˘a (fn ) converge uniform la f pe D, atunci f este continu˘a (pe D). Teorema 7.18. (Transfer de integrabilitate) Dac˘a fn : [a, b] → R, n ∈ N ∗ este un ¸sir de funct¸ii integrabile, convergent uniform la funct¸ia f : [a, b] → R, atunci f este integrabil˘ a ¸si, ˆın plus: Z lim

n→∞ a

Z

b

fn (x)dx =

b

f (x)dx. a

Teorema 7.19. (Transfer de derivabilitate) Fie I ⊂ R un interval marginit si fn : I → R, n ∈ N ∗ un ¸sir de funct¸ii derivabile. Dac˘a exist˘a un punct x0 ∈ I, astfel ˆıncˆat ¸sirul (fn (x0 ))n∈N ∗ este convergent si o functie u g : I → R, astfel ˆıncˆat fn0 → g, pe I, atunci : u 1) exist˘a o funct¸ie f : I → R, astfel ˆıncˆ at fn → f, pe I ¸si 2) f este derivabil˘a pe I ¸si f 0 = g, ceea ce se mai poate scrie:( lim fn )0 = lim fn0 .

n→∞

n→∞

260

Probleme 7.1 a) Precizat¸i care din urm˘atoarele numere sunt puncte de convergent¸˘ a 1 2 n pentru ¸sirul de funct¸ii fn : R → R, fn (x) = x : x = 2 ; x = − 3 ; x = 3, 2; x = −1, iar apoi determinat¸i mult¸imea de convergent¸˘ a a ¸sirului. b) Demonstrat¸i, folosind definit¸ia, iar apoi teorema ca, dac˘a α, β ∈ (0, 1), atunci sirul (fn ) este uniform convergent pe intervalele [0, β] si [α, β]. c) Demonstrat¸i, folosind definit¸ia, iar apoi teorema c˘a ¸sirul (fn ) nu este uniform convergent pe intervalele [0, 1), [0, 1], (−1, 1). d) Justificat¸i din nou ca (fn ) nu este uniform convergent pe intervalul [0, 1], de data aceasta cu ajutorul teoremei. e) Demonstrat¸i c˘a (fn ) verific˘ a concluzia teoremei, de¸si (fn ) nu este uniform convergent. 7.2 a) determinat¸i mult¸imea de convergent¸˘ a ¸si f , funct¸ia limita punctual˘a a ¸sirului de funct¸ii fn (x) = nx , pentru orice n ∈ N ∗ , x ∈ [0, ∞). b) Dac˘a ε = 12 pentru fiecare din valorile x = 5, x = 50, x = 500, at dac˘a n ≥ n 1 ,x , s˘a avem |fn (x) − f (x)| < determinat¸i n 1 ,x ∈ N ∗ , astfel ˆıncˆ 2 2 ε. Ce observat¸i c) demonstrat¸i c˘a ¸sirul (fn ) este uniform convergent pe intervalul [0, 500], folosind definit¸ia, iar apoi folosind teorema... d) demonstrat¸i c˘a ¸sirul (fn ) nu este uniform convergent pe intervalul [0, ∞), folosind din nou definit¸ia, iar apoi teorema. Solut¸ie Dac˘a x ∈ [0, ∞) este fixat, ¸sirul numeric (fn (x))n∈N ∗ = ( nx )n∈N ∗ este convergent la 0, ceea ce se mai poate scrie lim fn (x) = 0. Funct¸ia limita n→∞

punctual˘a a ¸sirului de funct¸ii (fn ) este a¸sadar f : [0, ∞) → R, f (x) = 0.

261

Solut¸ii 7.1

Capitolul 8 SERII DE FUNCT ¸ II. SERII DE PUTERI

Definit¸ia 8.1. Fie (an ) un ¸sir de numere reale ¸si x0 ∈ R. O serie de funct¸ii de forma ∞ X

an (x − x0 )n = a0 + a1 (x − x0 ) + a2 (x − x0 )2 + ... + an (x − x0 )n + . . .

n=0

se nume¸ste serie Taylor. ˆIn cazul ˆın care x0 = 0, seria devine ∞ X

an xn = a0 + a1 x + a2 x2 + ... + an xn + ..

n=0

¸si se nume¸ste serie de puteri. Num˘arul an se nume¸ste coeficientul termenului de rang n. Observat¸ia 8.2. Toate propriet˘a¸tile demonstrate pentru serii de funct¸ii r˘amˆan adev˘arate ¸si pentru serii Taylor (serii de puteri). ˆIn plus, apar ¸si alte rezultate importante. Teorema 8.3. (razei de convergent¸˘ a, Abel) Fie

∞ P n=0

an xn o serie

de puteri. Atunci exist˘a ¸si este unic determinat un num˘ ar real R ∈ [0, ∞], astfel ˆıncˆat: a (chiar absolut) pe intervalul (−R, R); a) daca R > 0, seria este convergent˘ mai mult ea este uniform convergent˘ a pe orice interval ˆınchis ¸si m˘arginit [α, β] ⊂ (−R, R), α, β ∈ R. b) dac˘a 0 < R < ∞, atunci pentru orice x, astfel ˆıncaˆ at |x| > R, seria diverge. 262

263 c) dac˘a R = 0, atunci seria converge doar pentru x = 0. Num˘arul R se nume¸ste raza de convergent¸˘ a a seriei, iar (−R, R) intervalul de convergent¸˘a al seriei. Observat¸ia 8.4. Teorema nu precizeaz˘a nimic ˆın ce prive¸ste convergent¸a ˆın extremit˘a¸tile intervalului de integrare; studiul ˆın aceast˘a situat¸ie se face de la caz la caz. Teorema 8.5. Fie

∞ P n=0

an xn o serie de puteri ¸si R raza de convergent¸˘ a.

Dac˘a seria converge ˆın x = R( x = −R), atunci ea este uniform convergent˘a pe orice interval ˆınchis ¸si m˘arginit [α, β] ⊂ (−R, R] (respectiv [α, β] ⊂ [−R, R)), α, β ∈ R. ∞ P Teorema 8.6. (Cauchy-Hadamard) Fie an xn o serie de puteri, R n=0 p raza de convergent¸˘a ¸si s˘a not˘am l = lim n |an |. Atunci R = 0, dac˘ a l = ∞, n→∞

R = 1l , dac˘a l ∈ (0, ∞) ¸si R = ∞, dac˘a l = 0. Teorema 8.7. Fie

∞ P n=0

an xn o serie de puteri, ¸si s˘a presupunem c˘a

exist˘a un indice n0 ∈ N, astfel ˆıncˆ at an 6= 0, pentru orice n ≥ n0 . Dac˘ a | 1 exist˘a lim |a|an+1 . ∈ R, atunci raza de convergent ¸ a ˘ este R = |an+1 | n| n→∞

lim

n→∞ |an |

Teorema 8.8. Suma unei serii de puteri este o funct¸ie continu˘ a ˆın orice punct interior intervalului de convergent¸˘ a. Teorema 8.9. (Abel) Fie

∞ P n=0

an xn o serie de puteri, R raza de con-

vergent¸˘a ¸si f suma sa. Dac˘a seria este convergent˘ a ˆın x = R( respectiv ˆın x = −R), atunci suma ei este o funct¸ie continu˘ a ˆın x = R( respectiv ˆın x = −R), ceea ce ˆınseamn˘a lim f (x) = f (R) (respectiv lim f (x) = f (−R)). x→R, xR

Teorema 8.10. (de derivare a seriilor de puteri) Fie

∞ P n=0

serie de puteri, R > 0 raza de convergent¸˘ a, iar f suma sa. Atunci: a) seria derivatelor,

∞ P n=0

nan xn , are acea¸si raz˘a de convergent¸˘ a R.

an x n o

264 b) suma seriei este o funct¸ie derivabil˘ a pe intervalul (−R, R), iar derivata ∞ P sa este egal˘a cu suma seriei derivatelor, adic˘a f 0 (x) = nan xn . n=0

Teorema 8.11. (de derivare a seriilor de puteri) Fie

∞ P n=0

serie de puteri, R > 0 raza de convergent¸˘ a ¸si n ∈ N. Atunci:

an xn o

a) seria derivatelor de ordinul n, are acea¸si raz˘a de convergent¸˘ a R. b) suma seriei este o funct¸ie indefinit derivabil˘ a pe intervalul (−R, R), iar derivata sa de ordinul n este egal˘a cu suma seriei derivatelor de ordinul n. Teorema 8.12. (de integrare a seriilor de puteri) Fie

∞ P n=0

an xn o

serie de puteri, R > 0 raza de convergent¸˘ a, iar f suma sa. Atunci: a) seria obt¸inut˘a prin integrarea termen cu termen, acea¸si raz˘a de convergent¸a R.

∞ P n=0

an n+1 , n+1 x

are

b) suma seriei este o funct a pe orice interval [a, b] ⊂ (−R, R), R x¸ie integrabil˘ iar funct¸ia F (x) = 0 f (t)dt, x ∈ (−R, R) este egal˘a cu suma seriei ∞ P an n+1 obt¸inut˘a prin integrarea termen cu termen, F (x) = . n+1 x n=0

265

Probleme 8.1 S˘a se determine mult¸imea de convergent¸˘ a pentru urm˘atoarele serii de puteri: ∞ P a) nα xn ; b)

n=1 ∞ P

n=1 ∞ P

c) d)

n=1 ∞ P n=1 ∞ P

g) h)

2n+1 xn ; n3 +n+1 nxn 2n +1 ;

n (−1)n 3n+2 5n x ;

e) f)

n (−1)n nn+1 2 +2 x ;

n=1 ∞ P n=1 ∞ P

3n +1 n a ; n2 +3 n

(−1)n 2nx+3n ;

n=1 ∞ P

n!xn ;

n=1

i)

∞ P

[2n + (−3)n ]xn ;

n=1

j) (1 + n1 )n xn ; ∞ P 1 1 2n n k) en (1 + n ) x ; n=1

l)

∞ P

2n n n! x ;

n=1 ∞ P

3·8·...·(5n−2) n (−1)n 6·11·...·(5n+1) a ; n=1 ∞ P (n!)2 xn (2n)! ; n=1 ∞ P a(a+1)(a+2).....(a+n) n b(b+1)(b+2)....(b+n) x , a, b n=1 ∞ P (−1)n lnnn xn ; n=1 ∞ √ √ P

m) n) o) p) r) s)

( n+1−

n=1 ∞ P

(x+2)n √ ; n3 n=1

> 0, x > 0;

n − 1)(x − 1)n ;

266 ∞ P

t) u)

n=1 ∞ P

v) w) x)

1·3·...·(2n−1) 2·5·...·(3n−1) (2x 2n

(−1)n xn! ;

n=1 ∞ P n=1 ∞ P

1+(−1)n n x ; n

(−1)n n+1 xn ;

n=1 ∞ P n=1

+ 1)n ;

nn 1−x n en ·n! ( 2x+1 ) ;

8.2 S˘a se demonstreze c˘a dac˘a o serie de puteri este convergent˘ a ˆın punctul x = R, R fiind raza de convergenta, atunci seria converge uniform pe orice interval de forma [α, R], α ∈ (−R, R). 8.3 a) Precizat¸i coeficient¸ii an , n ∈ N, ai seriei de puteri

∞ P

2n+1

(−1)n x2n+1 ,

n=0

iar apoi determinat¸i mult¸imea de convergent¸˘ a. b) Demonstrat¸i c˘a seria dat˘a este uniform convergent˘ a pe intervalul (− 12 , 21 ). Dar pe intervalele [−1, 0], [0, 1]? c) Calculat¸i lim f (x), lim f (x) ar˘ a a determina funct¸ia f. x f˘ x→0

x→0

d) Notˆand suma seriei date cu f (x), justific˘ ati faptul c˘a f este derivabil˘ a 0 pe intervalul (−1, 1], determinat¸i f (x), iar apoi f (x). ∞ ∞ P P 1 1 e) Calculat¸i suma seriilor numerice (−1)n 2n+1 . , (−1)n 22n (2n+1) n=1

8.4 Fie funct¸ia f : D → R,f (x) =

∞ P

n=1

n

(−1)n−1 xn .

n=1

a) Determinat¸i D, domeniul de definit¸ie al funct¸iei f. b) Demonstrat¸i c˘a f este o funct¸ie derivabil˘ a cu derivata continu˘ a. 0 c) Determinat¸i f (x), iar apoi f (x). ∞ P d) Justificat¸i egalitatea (−1)n−1 n1 = ln 2. Cˆ a¸ti termeni ai seriei tren=1

buie ˆınsumat¸i pantru a calcula ln 2 cu o precizie de 10−4 . a a seriei 8.5 a) Determinat¸i mult¸imea de convergent¸˘

∞ P n=1

xn n2

¸si precizat¸i

mult¸imea maxim˘a pe care convergent¸a este uniform˘a. b) Precizat¸i mult¸imea de convergent¸˘ a a seriei derivatelor; explicat¸i de ce seria init¸ial˘a converge ˆın x = 1, iar seria derivatelor nu converge ˆın acest punct.

267 ∞ P

c) Aflat¸i suma seriei date, iar apoi suma seriei numerice

n=1

1 . n2

8.6 Determinat¸i mult¸imea de convergent¸˘ a ¸si suma seriilor: ∞ ∞ P P xn xn n (−1) n+2 ;b) a) n(n+1) . n=0

n=0

8.7 G˘asit¸i D, mult¸imea de convergent¸˘ a, a seriei

∞ P

n

(−1)n (x+4) ; 3 n n2

n=1

b) Justificat¸i ca f (x), suma seriei, verific˘ a relat¸ia f 0 (x) + (x + 4)f 00 (x) = 3 ¸i f (x). (x+4)(x+7) , pentru orice x ∈ D{−7, −4}, iar apoi aflat c) Calculat¸i lim

f (x)

x→−4 x+4

, ˆın dou˘a moduri.

8.8 a) Studiat¸i convergent¸a seriei 1 +

∞ P n=1

α(α−1)...(α−n+1) n x , n!

α ∈ R.

b) Dac˘a suma seriei este f (x), g˘ asit¸i o relat¸ie ˆıntre f ¸si f 0 , iar apoi determinat¸i f. 8.9 a) Demonstrat¸i c˘a seria

∞ P

2n+1

x (−1)n (2n+1)! este absolut convergent˘ a pe

n=0

intervalul (−1, 1) folosind, mai ˆıntˆ ai, criteriul raportului, iar apoi cu teorema ...lui Abel. Care este mult¸imea de convergent¸˘ a a seriei? b) Notˆand cu Sn suma part¸ial˘ a a seriei, justificat¸i inegalitatea | sin x − S2n+1 | ≤

|x|2n+3 , pentru orice x ∈ (−1, 1). (2n + 3)!

Deducet¸i c˘a suma seriei este sin x. a a seriei 8.10 Comparat¸i mult¸imea de convergent¸˘

∞ P

(n+1)xn cu mult¸imea

n=0

de convergent¸˘a a seriei obt¸inute prin integrarea termen cu termen. G˘asit¸i apoi suma seriei init¸iale. 8.11 Determinat¸i mult¸imea de convergent¸˘ a ¸si suma seriilor: ∞ ∞ P P a) (−1)n (n + 1)(n + 2)xn ;b) n2 (x − 1)n . n=1

n=1

8.12 S˘a se arate c˘a urm˘atoarele serii sunt convergente ¸si s˘a se calculeze suma lor cu ajutorul seriilor de puteri: ∞ ∞ ∞ ∞ P P P P (n+1)(n+2) 1 1 n π 2n ; d) a) (−1)n 3n+1 ; b) ;c) (−1) n 2n n! 3 (n+1) 6 (2n)! n=0

n=0

n=1

n=0

268 8.13 S˘a se demonstreze c˘a urm˘atoarele funct¸ii sunt dezvoltabile ˆın serii de puteri ˆın jurul originii, s˘a se g˘aseasca aceast˘a dezvoltare ¸si mult¸imea pe care este valabil˘a: a) f (x) = ex ;b) g(x) = sin x;c) h(x) = cos x. 8.14 a) S˘a se demonstreze c˘a funct¸ia f : R → R, f (x) = (1 + x)α , x > −1, α ∈ RN este dezvoltabil˘ a ˆın serie de puteri ¸si determinat¸i apoi aceast˘a dezvoltare. b) Determinat¸i dezvoltarea ˆın serie de puteri a funct¸iei g : R → R,g(t) = √ 1 , apoi prin integrare termen cu termen pe intervalul [0, x], |x| < 1, 1−t2 deducet¸i dezvoltarea pentru funct¸ia h : R → R,h(x) = arcsin x. G˘asit¸i mult¸imea de convergent¸˘a pentru seria obt¸inut˘ a, iar apoi o formula pentru calculul π num˘arului 2 . 8.15 a) Folosind dezvoltarea ˆın serie geometric˘a, aflat¸i dezvoltarea ˆın serie 1 a funct¸iei f (x) = x+2 ˆın jurul punctelor i) x = 0; ii) x = 1. 1 x b) Aceasˆı problem˘a pentru funct¸iile g(x) = x+3 , h(x) = x2 +5x+6 . 8.16 Determinat¸i ¸sirul (an ) ⊂ R ¸si num˘ arul R ∈ R astfel ˆıncˆ at s˘a aib˘a loc egalitatea: ∞ P a) x1 = an (x − 1)n , |x − 1| < R. n=0 ∞ P

b) tgx =

n=0

an xn , |x| < R.

8.17 a) Justificat¸i, ˆın dou˘a moduri, egalitatea ln(1 − x + x2 ) =

∞ X (−1)n−1 n=1

n

(x3n − xn ), |x| < 1.

b) Verificat¸i c˘a raza de convergent¸˘ a a seriei din stˆanga este 1. 8.18 Determinat¸i dezvolt˘ arile ˆın serie de puteri ale funct¸iilor: 1 1 2 a) f1 (x) = 1+x ; b) f (x) =arctgx;c) f3 (x) = (1−x) 2 2 2 ;d) f4 (x) =sin x;e) f5 (x) =e2x in x = 1;f) f6 (x) = { √ 1+x

1−cos x ,x x2 1 2, x =

6 0 = .; g) f7 (x) = ; h) f8 (x) = 0

8.19 a) S˘a se determine dezvoltarea ˆın serie de puteri a funct¸iei f (x) = ln x, dup˘a puterile funcitie g(x) = 1−x and intervalul pe care are loc 1+x , precizˆ dezvoltarea.

269 b) G˘asit¸i un n ∈ N, astfel ˆıncˆ at suma part¸ial˘ a a seriei obt¸inute s˘a aprox−3 imeze ln x cu precizie de 10 , pentru orice x ∈ ( 12 , 1). c) Calculat¸i ln 2 cu o precizie de 10−5 . 1

− 2 x , x 6= 0 8.20 S˘a se demonstreze c˘a funct¸ia f : R → R, f (x) = { e ., 0, x = 0 este de clasa C ∞ , dar nu este analitic˘a.

8.21 Determinat¸i dezvoltarea MacLaurin a funct¸iei f (x) =

Rx

sin t t dt,

0

R → R, iar apoi determinat¸i, cu o precizie de 10−4 , valoarea integralei 8.22 G˘asit¸i cu o precizie de 10−3 integralele: a) R1 1 0 1+x4 dx.

R

1 2

0

arctgt t dt;b)

R

1 2

0

R

1 2

0

f : sin t t dt.

2

et dt; c)

8.23 Folosind dezvoltarea ˆın serie geometric˘a aflat¸i dezvoltarea pentru 1 funct¸ia f (x) = 1+x ın jurul punctului x = 0. 6 ˆ b) Determinat¸i f (100) (0). Rx 1 c) G˘asit¸i o reprezentare ˆın serie de puteri pentru funct¸ia g(x) = 0 1+t 4 dt ¸si determinat¸i mult¸imea de convergent¸˘ a pentru obt¸inut˘ a. R 1 seria 1 d) G˘asit¸i cu o precizie de 10−3 integrala 0 1+x dx. 4 8.24 Demonstrat¸i egalitatea

R1

ln x 0 1−x dx

=

∞ P n=1

1 . n2

8.25 S˘a se rezolve ecuat¸iile diferent¸iale: a) y 00 − y = 0, y(0) = 0, y 0 (0) = 1, b) y 00 − xy − y = 1, y(0) = 0, y 0 (0) = 1, cautˆand o solut¸ie de forma ∞ P y(x) = an xn . n=0

8.26 Fie funct¸ia f (x) =

∞ P n=0

an xn , dezvoltarea MacLaurin avˆ and o raz˘a

de convergent¸˘a strict pozitiv˘a. Demonstrat¸i c˘a dac˘a f este para(impara) atunci a2k = 0( a2k+1 = 0), pentru orice k ∈ N.

Note istorice

Prezent˘am ˆın continuare un scurt istoric al cˆatorva not¸iuni utilizate ˆın acest volum. Ment¸ion˘am c˘a sursa acestor note o constituie c˘art¸ile scrise de Bourbaki [3], Florica Cˆampan [5], [6], Solomon Marcus [12], Dan I. Papuc [13] ¸si Isaac Schoenberg [16]. Este firesc s˘a preciz˘am pentru ˆınceput semnificat¸ia acestui domeniu fundamental al Matematicii, cel al Analizei matematice. Cuvˆantul Matematic˘ a provine din grecescul matima care ˆınseamn˘ a cunoa¸stere, ¸stiint¸˘ a, iar termenul de Analiz˘ a a fost introdus ˆın Matematic˘a de Vi`ete (1540-1603), cu sensul s˘au obi¸snuit ¸si anume: cercetarea unui fenomen prin determinarea ¸si analizarea fiec˘arui element al s˘au. 1. Mult¸imea numerelor reale Probabil c˘a primele elemente ¸stiint¸ifice de astronomie, biologie sau medicin˘a au ap˘arut ˆınc˘a din preistorie, acum aproximativ trei milioane de ani. Sunt consemnate primele desene rupestre, multe cont¸inˆ and figuri geometrice. Aspectele practice ale viet¸ii primitive, nevoia de a evalua ¸si compara au condus la introducerea conceptului de num˘ ar. Au fost descoperite t˘ablit¸e datate acum cinci mii de ani (ce constituiau adev˘arate dict¸ionare) care cont¸in clasific˘ari ¸si cuno¸stint¸e despre animale, plante, elemente de medicin˘a, astronomie dar ¸si elemente de matematic˘a, de exemplu: tabele numerice sau probleme ¸si solut¸ii concrete, particulare ale acestora, neexistˆand ˆıns˘ a demonstrat¸ii globale. Exist˘a documente vechi de peste dou˘a mii de ani despre existent¸a numerelor rat¸ionale pozitive care erau cunoscute de c˘atre egipteni ¸si babilonieni. Grecii antici foloseau denumirea de num˘ ar pentru num˘ar natural ¸si m˘ arime pentru num˘ ar rat¸ional pozitiv. ˆIn matematica greac˘a era admis˘a ideea c˘a lungimea unui segment poate fi reprezentat˘ a 270

271 doar prin numere sau m˘ arimi. Eudoxos din Cnid (408-355 ˆı.H.) a scris o teorie complet˘a a numerelor rat¸ionale. Un rezultat remarcabil este obt¸inut de matematicienii din S¸coala lui Pitagora (sec. 6-5 ˆı.H.): existent¸a segmentelor a c˘aror lungime nu este nici num˘ ar (num˘ ar natural), nici m˘ arime (num˘ar rat¸ional pozitiv), pe care le-au numit segmente incomensurabile, iar lungimile acestora m˘ arimi incomensurabile (de fapt numere irat¸ionale). S-a demonstrat prin reducere la absurd c˘a lungimea diagonalei unui p˘atrat nu poate fi nici num˘ar natural, nici num˘ ar rat¸ional pozitiv. ˆIn Elementele lui Euclid se scrie c˘a: ”m˘arimile incomensurabile nu sunt rapoarte de numere”. Mai tˆarziu, odat˘a cu descoperirea a dou˘a sensuri diferite pentru anumite m˘arimi, au fost introduse ¸si numerele negative. Aceste numere apar ˆın lucr˘arile lui Diofante din Alexandria (sec. 3 d.H.). Prin descoperirea numerelor irat¸ionale ¸si a numerelor negative s-a ajuns la mult¸imea numerelor reale (denumite a¸sa probabil ˆın sec. 16-17, odat˘a cu descoperirea numerelor imaginare ˆın analiza complex˘a). Un num˘ ar real era definit ca lungime a unui segment de dreapt˘a m˘asurat cu un segment unitate fixat. Aceast˘a definit¸ie geometric˘a (num˘ar real = lungimea unui segment) avea s˘a fie utilizat˘a pˆan˘a ˆın secolul 19 cˆand s-a realizat o teorie riguroas˘a a numerelor reale prin construct¸iile1 lui Weierstrass (1815-1897), Cantor (1845-1918) ¸si Dedekind (1831-1916). ˆIn 1900, Hilbert (1862-1943) a dat o definit¸ie axiomatic˘a a structurii numerelor reale (f˘ar˘ a a utiliza mult¸imea numerelor rat¸ionale), definit¸ie care este folosit˘a ˆın mod curent (definit¸ia 1.22). La mijlocul secolului 19 s-au introdus numerele reale algebrice ¸si numerele reale transcendente (termenul transcendent a fost definit de Leibniz ˆın 1679). ˆIn 1844, Liouville (1809-1882) a demonstrat existent¸a numerelor transcendente, construind efectiv astfel de numere, numite numere Liouville. ˆIn 1937, Mahler a ar˘atat c˘a exist˘a numere transcendente care nu sunt numere Liouville. Gelfond ˆın 1934 ¸si Schneider ˆın 1935 au demonstrat c˘a orice num˘ar de forma ab (unde a este un num˘ ar algebric diferit de 0 ¸si 1, iar b este un num˘ar algebric irat¸ional) este un num˘ ar transcendent. Existent¸a num˘arului π a fost b˘anuit˘ a ˆınc˘ a din preistorie, cˆand s-a constatat c˘a raportul dintre lungimea unui cerc ¸si diametrul s˘au era apropiat de 3. Astfel, pe t˘ablit¸ele de lut ars descoperite ˆın Mesopotamia apare valoarea π = 3. Egiptenii, aproximˆand¡ aria ¢3 cercului cu aria octogonului regulat ˆınscris ˆın cerc, au g˘asit π = 4 89 = 3, 16 valoare care, de¸si nu oferea o aproximat¸ie satisf˘ac˘atoare, reprezenta un mare progres. Arhimede (287212 ˆı.H.) a determinat primele dou˘a zecimale corecte ¸si anume π = 3, 14. ˆIn secolul 5 d.H., Aryabhatta folosea valoarea π = 3, 1416 ca s˘a constru1

vezi Precupanu A.M. [14] ¸si Macovei E. [11]

272 iasc˘a un tabel de sinusuri. Matematicianul chinez Tzu Ciun-Ciji (430-501) a stabilit primele ¸sase zecimale exacte ale num˘ arului π, prin inegalitatea 3, 1415926 < π < 3, 1415927. ˆIn 1882, Lindemann (1852-1939) a demonstrat c˘a π este transcendent, ar˘atˆ and astfel imposibilitatea realiz˘arii cuadraturii cercului ¸si anume, construirea unui p˘atrat cu arie egal˘a cu aria unui cerc dat. Num˘arul e a fost introdus pentru prima dat˘a ˆın 1748 de c˘atre Euler (1707-1783) ca baz˘a a logaritmilor naturali. Convergent¸a ¸sirului xn = ¢n ¡ a fost pus˘a ˆın evident¸˘ a ˆın 1728 de c˘atre Daniel Benoulli (17001 + n1 1782), rezultat generalizat ˆın 1743 de c˘atre Euler pentru ex . Irat¸ionalitatea lui e a fost demonstrat˘a ˆın 1815 de c˘atre Fourier (1768-1830). Mai mult, Hermite (1822-1901) a demonstrat ˆın 1873 c˘a num˘ arul e este transcendent. 1 1 1 Faptul c˘a ¸sirul un = 1 + 2 + 3 + . . . + n este divergent a fost stabilit ˆın 1689 de c˘atre Iacob Bernoulli (1654-1705). Euler a fost cel care a demonstrat convergent¸a ¸sirului vn = 1 + 12 + . . . + n1 − ln n ˆın 1734 ¸si a notat limita sa cu c, constanta care ˆıi poart˘a numele; tot el a obt¸inut valoarea aproximativ˘ aa lui c cu 16 zecimale exacte: 0,5772156649015329. Nu se cunoa¸ste ˆınc˘ a dac˘a c este un num˘ar rat¸ional sau nu. √ Num˘arul irat¸ional Φ = 1+2 5 = 1, 6180339887... este numit num˘ arul de aur pentru c˘a simbolizeaz˘a armonia ¸si frumuset¸ea. Num˘ arul de aur apare ˆın construct¸ia laturii pentagonului regulat. Propriet˘a¸tile acestui poligon, studiate de S¸coala lui Pitagora, au contribuit probabil la alegerea pentagonului regulat stelat ca semn de recunoa¸stere pentru membrii acestei ¸scoli. ˆIn studiul rapoartelor ¸si proport¸iilor, Eudoxos din Cnid a numit acest raport a sect¸iunea de aur (sectio aurea): a+b a = b = Φ. a

b

La grecii antici, un om era considerat frumos dac˘a anumite distant¸e dintre p˘art¸ile corpului se aflau ˆın raportul de aur. Pictorul olandez Mondrian a folosit ˆın tablourile sale dreptunghiul de aur cu raportul de 1,6 ˆıntre lungimea ¸si l˘a¸timea laturilor tabloului. Num˘ arul de aur se reg˘ase¸ste ¸si ˆın ¸sirul (xn ) = Φ (vezi problema 2.36-a)). definit de Fibonacci (1170-1240): lim xxn+1 n n→∞

Regulile de calcul cu ∞ au fost stabilite de Wallis (1616-1703). Cercetarea riguroas˘a a infinitului a ˆınceput ˆın secolul 19, odat˘a cu lucrarea ”Paradoxurile infinitului” a lui Bolzano (1781-1848) publicat˘a postum ˆın 1850. O contribut¸ie esent¸ial˘ a la dezvoltarea teoriei mult¸imilor a fost adus˘a de matematicianul german Cantor (1845-1918). ˆIn 1873 el a publicat lucrarea ”Grundlegen allgemeinen Mannigfoltigkeitslehre” (Fundamentele unei

273 teorii generale a mult¸imilor) ˆın care introduce not¸iunile de relat¸ie, element, mult¸ime, relat¸ia ”a apart¸ine” ˆıntre element ¸si mult¸ime, operat¸ii cu mult¸imi ¸si define¸ste not¸iunile de relat¸ie de echipotent¸˘ a ¸si cardinal, punˆand bazele Teoriei mult¸imilor. Cu ajutorul not¸iunii de cardinal a putut fi posibil˘a cercetarea ¸si cunoa¸sterea infinitului. Cantor a obt¸inut rezultate uluitoare despre mult¸imile infinite, de exemplu: mult¸imea numerelor reale cuprinse ˆıntre 0 ¸si 1 este echivalent˘a cu mult¸imea punctelor planului ¸si echivalent˘ a de asemenea cu mult¸imea punctelor spat¸iului. Sau mult¸imea numerelor algebrice este mai mic˘a decˆat mult¸imea numerelor transcendente, afirmat¸ie care a provocat o polemic˘a aprins˘a pentru c˘a ˆın acea vreme nu se cuno¸steau prea multe exemple de numere transcendente. Lumea matematic˘a a fost ˆımp˘ art¸it˘ a ˆın dou˘a tabere: cei care admiteau noua teorie a lui Cantor ¸si cei care o contestau. Cantor a enunt¸at ˆın 1878 celebra ipoteza a continuumului: ˆıntre ℵ0 ¸si c nu exist˘a alte cardinale (cu alte cuvinte, nu exist˘a nici o mult¸ime A cu proprietatea ℵ0 < card A < c). Ipoteza continuumului va deveni o axiom˘a independent˘a ˆın sistemul de axiome ce stau la baza teoriei mult¸imilor. Astfel, G¨odel ˆın 1940 ¸si Cohen ˆın 1963 au ar˘atat c˘a ipoteza continuumului nu este nici adev˘arat˘a nici fals˘a. Pot fi construite dou˘a structuri ale mult¸imilor: una ˆın care propozit¸ia este adev˘arat˘ a ¸si alta ˆın care aceeast˘a propozit¸ie este fals˘a. 2. S ¸ iruri numerice. Serii numerice Studiul ¸sirurilor ¸si al seriilor numerice dateaz˘a din secolul 17, de¸si not¸iunile de convergent¸˘a ¸si divergent¸˘ a nu erau clar precizate. Erau considerate mai ales cazuri particulare de ¸siruri sau serii, stabilindu-se prin metode concrete convergent¸a sau divergent¸a acestora. Convergent¸a ¸sirurilor ¸si a seriilor a fost riguros definit˘a ¸si studiat˘a ˆın 1812 de c˘atre Gauss (17771855) ¸si ˆın 1821 de c˘atre Cauchy (1789-1857). Convergent¸a seriilor alternate cu termen general tinzˆand la zero a fost stabilit˘a de Leibniz (16461716). Cauchy a definit not¸iunile de ¸sir convergent ¸si de ¸sir fundamental ˆın IR. A fost studiat˘a problema aproxim˘ arii numerelor reale cu ¸siruri de numere rat¸ionale de c˘atre Dirichlet (1805-1859), Liouville, Cebˆı¸sev (18211894), Kronecker (1823-1891), Minkowski (1864-1909). Au fost stabilite criterii de convergent¸˘a pentru serii de c˘atre Abel (1802-1829), Dirichlet, Raabe (1801-1859), Kummer (1810-1893). Simpla convergent¸˘ a ¸si absoluta convergent¸˘a a seriilor numerice au fost studiate de Cauchy, Dirichlet, Riemann (1826-1866). Spre sfˆar¸situl secolului 19, Poincar´e (1854-1912), Borel (1871-1956), Stieltjes (1856-1894), Hadamard (1865-1963) ¸si-au ˆındreptat atent¸ia ¸si spre studiul seriilor divergente care i-au intrigat atˆat de mult pe matematicieni.

274 3. Topologie Termenul de topologie a fost introdus prima dat˘a ˆın 1847 de c˘atre Listing (1808-1882). Dar primele elemente de topologie apar ˆın 1640 la Descartes (1596-1650), apoi la Euler ˆın 1752, la Leibniz ¸si Gauss. Cantor a definit not¸iunile de punct de acumulare, mult¸ime deschis˘ a ¸si mult¸ime ˆınchis˘ a. Toate cercet˘arile erau ˆıns˘a f˘acute ˆın cadrul spat¸iilor euclidiene. Cel care a trecut de la spat¸ii euclidiene la spat¸ii abstracte a fost Riemann care ˆın 1857 a definit ¸si studiat ˆın mod riguros topologia ˆın spat¸ii abstracte ca teorie matematic˘a. Contribut¸ii esent¸iale ˆın teoria spat¸iilor topologice au avut Fr´echet (18781973) ˆın 1906, Riesz (1880-1956) ˆın 1907, Hausdorff (1868-1942) ˆın 1914, Moore (1862-1932) ˆın 1916 ¸si Kuratowski (1896-1980) ˆın 1922. 4. Funct¸ii Funct¸iile erau cunoscute ˆınc˘ a ˆınainte de Hristos sub forma corespondent¸elor ˆıntre m˘arimi definite prin tabele. Egiptenii ¸si grecii au folosit expresia ariei unui cerc ca funct¸ie de diametru ¸si expresia unei progresii aritmetice ca funct¸ie de primul termen, rat¸ie ¸si num˘ arul termenilor. ˆIn anul 499, matematicianul indian Aryabhatta (476-550) elaboreaz˘a un manuscris cu tabele de sinusuri ¸si consinusuri. La sfˆar¸situl secolului 15 s-a observat c˘a relat¸ia n 7→ an stabile¸ste o leg˘atur˘ a ˆıntre adunarea numerelor ˆıntregi ¸si ˆınmult¸irea a dou˘a puteri cu exponent ˆıntreg. Aceast˘a observat¸ie a dus la definirea logaritmilor de c˘atre Stiffel ˆın 1544. ˆIn 1614, Napier (sau Neper) (1550-1617) define¸ste logaritmii naturali (numit¸i ¸si logaritmi neperieni), alc˘atuind primul tabel de logaritmi. ˆIn timpul lui Descartes (15961650) ¸si Fermat (1601-1665) s-au considerat funct¸ii algebrice definite prin polinoame, fract¸ii rat¸ionale, funct¸ii trigonometrice ¸si inversele lor, funct¸ia logaritmic˘a ¸si cea exponent¸ial˘ a. ˆIn terminologia utilizat˘a de Newton (16421727), funct¸ia era o fluent˘ a (adic˘a o cantitate care se scurge cu timpul), fiind numit˘a fluxiune. Leibniz a introdus termenul de ”funct¸ie” ˆın 1694. ˆIn 1714, Johann Bernoulli (1667-1748) a introdus notat¸ia f x care ˆın 1734 va fi transformat˘a ˆın f (x) de c˘atre Euler ¸si Clairaut (1713-1765). Definit¸ia uzual˘a a unei funct¸ii, folosit˘a ˆın prezent (definit¸ia 1.7) a fost dat˘a de c˘atre Dirichlet ˆın 1837 pentru funct¸ii definite pe mult¸imi de numere reale, cu valori ˆın IR. Forma general˘a a definit¸iei a fost posibil˘a abia dup˘a aparit¸ia teoriei mult¸imilor a lui Cantor. De¸si erau b˘anuite cu mult ˆınainte, not¸iunile de limit˘a ¸si continuitate au fost definite ˆın mod riguros mult mai tˆarziu, ˆın secolul 19. ˆInc˘a din antichitate, ˆın secolul 5 ˆı.H., s-a f˘acut un prim pas ˆın operat¸ia de trecere la limit˘a, prin metoda exhaustiei, atribuit˘a lui Democrit din Abdera, Hipocrate

275 din Chios, Eudoxos din Cnid ¸si Antifon. Aceast˘a metod˘a era utilizat˘a pentru a demonstra c˘a dou˘a m˘arimi sunt egale, ar˘atˆ and c˘a diferent¸a lor poate fi f˘acut˘a oricˆat de mic˘a. Teorema lui L0 Hˆ ospital (1661-1764) despre calculul limitelor a fost enunt¸at˘a ¸si demonstrat˘a de c˘atre acesta ˆın 1696. Not¸iunile de limit˘a a unei funct¸ii ˆın limbaj ε − δ ¸si de funct¸ie continu˘ a ˆın IR au fost definite de c˘atre Cauchy ˆın 1821. Contribut¸ii importante la definirea acestor concepte au mai avut Lacroix (1765-1843), Bolzano (1781-1848) ¸si Dirichlet. Calculul diferent¸ial ¸si integral apare ca domeniu nou ¸si distinct al matematicii ˆın a doua jum˘atate a secolului 17. O serie de probleme din geometrie (de exemplu: calculul ariilor determinate de curbe plane, problema existent¸ei tangentei la o curb˘a arbitrar˘a) sau din mecanic˘a (¸si anume: studiul traiectoriei, vitezei ¸si accelerat¸iei ˆın mi¸scarea punctului material) au condus la aparit¸ia teoriei calculului diferent¸ial ¸si integral. Un instrument util ˆın edificarea acestei teorii l-a constituit metoda utiliz˘arii coordonatelor unui punct din plan, cu toate c˘a metoda trecerii la limit˘a, vital˘a pentru not¸iunea de derivat˘a, nu era ˆınc˘a cristalizat˘a. Cei care au pus bazele calculului diferent¸ial ¸si integral pot fi socotit¸i, f˘ar˘ a ˆındoial˘ a, Leibniz ¸si Newton. Printre fondatori merit˘a amintit ¸si Fermat care a studiat unele probleme de extremum. Leibniz a definit conceptele de derivat˘ a, diferent¸ial˘ a ¸si inteR gral˘a, utilizˆand notat¸iile dx, d2 x, f (x)dx. Atˆ at Leibniz, cˆat ¸si Newton au fost condu¸si la definirea derivatei de o serie de probleme practice. Astfel, Leibniz a pornit de la problema existent¸ei tangentei ˆıntr-un punct la graficul unei funct¸ii ¸si derivata reprezenta panta unei curbe, ˆın timp ce Newton pleca de la diverse probleme de reprezent˘ ari mecanice iar derivata reprezenta viteza unui mobil. Newton a observat relat¸iile dintre derivate ¸si integrale ¸si a enunt¸at ¸si demonstrat la sfˆar¸situl secolului 17 teorema fundamental˘ aa calculului diferent¸ial ¸si integral. Acest rezultat a fost obt¸inut ˆın mod independent ¸si de Leibniz, ceea ce a generat o serie de discut¸ii pentru prioritate ˆıntre cei doi (celebre ˆın epoc˘a) ¸si care a ˆımp˘ art¸it lumea matematic˘a ˆın dou˘a tabere. Matematicianul francez Rolle (1652-1719) a obt¸inut o serie de rezultate privind separarea r˘ad˘acinilor unei ecuat¸ii algebrice, descoperind ¸sirul care ˆıi poart˘a numele. ˆIn 1712, Taylor (1685-1731) a prezentat rezultate despre dezvoltarea ˆın serie a funct¸iilor. ˆIn 1742, scot¸ianul MacLaurin (1698-1746) a stabilit dezvoltarea ˆın serie de puteri a funct¸iilor, dezvoltare ce-i poart˘a numele. ˆIn 1741, Clairaut a introdus not¸iunea de diferent¸ial˘ a total˘a a unei funct¸ii de mai multe variabile. Odat˘a cu definirea funct¸iilor continue ¸si a funct¸iilor derivabile, a ap˘arut

276 problema stabilirii relat¸iilor dintre ele. Astfel, ˆın 1872, Weierstrass a prezentat un exemplu de funct¸ie continu˘ a care nu este derivabil˘ a ˆın nici un punct ∞ P n din domeniul s˘au de definit¸ie ¸si anume, f (x) = b cos(an πx), unde b este n=0

un num˘ar ˆıntreg impar ¸si a ∈ (0, 1) astfel ˆıncˆ at ab > 1 + 3π 2 . ˆIn secolul 16 a fost introdus logaritmul. Logaritmul este o funct¸ie ce stabile¸ste un izomorfism ˆıntre grupul multiplicativ al numerelor reale pozitive ¸si grupul aditiv al numerelor reale, cu ajutorul c˘areia operat¸ia de ˆınmult¸ire a numerelor reale pozitive se poate ˆınlocui cu adunarea numerelor reale. Astfel, logaritmii s-au dovedit deosebit de utili ˆın astronomie, la efectuarea unor calcule complicate ¸si probabil aceasta a fost una din rat¸iunile care au condus la definirea logaritmilor. ˆIn 1614, Napier a definit logaritmul natural, alegˆand num˘arul e ca baz˘a a logaritmului. Fiind date dou˘a progresii, prima aritmetic˘a ¸si a doua geometric˘a: (1) 0, r, 2r, . . . , nr, . . . cu rat¸ia r > 0, (2) 1, q, q 2 , . . . , q n , . . . cu rat¸ia q > 1, se observ˘a c˘a produsul a doi termeni din seria (2) se poate ˆınlocui cu suma termenilor corespunz˘atori din seria (1): q n · q m = q n+m 7→ nr + mr = (n + m)r. S˘a consider˘am progresiile (ak )k∈IN ¸si (bk )k∈IN (prima aritmetic˘a ¸si a doua geometric˘a): (10 ) 0,

1 n,

2 n,

...,

¡ ¢2 ¡ (20 ) 1, 1 + n1 , 1 + n1 , . . . , 1

n−1 n , ¢n−1 + n1 ,

1, ¡ ¢n ¡ 1 + n1 , 1

n+1 n ¢ 1 n+1 +n ,

,... ....

cu rat¸iile r = n1 ¸si respectiv q = 1 + n1 . Napier nume¸ste logaritm al unui termen din seria (20 ) ca fiind termenul de ¡ ¢k acela¸si rang din seria (10 ) deci log bk+1 = ak+1 , adic˘a log 1 + n1 = nk . El nume¸ste baz˘a a logaritmului acel termen bk+1 cu proprietatea c˘a ak+1 = 1, echivalent cu log b¡k+1 = ¢1. Dar acest lucru se ˆıntˆ ampl˘ a pentru k = n deci n baza ar fi bn+1 = 1 + n1 . Ideea este de a reduce operat¸ia de ˆınmult¸ire a doi termeni din (20 ) la operat¸ia de adunare a doi termeni din (10 ). Pentru a face acest lucru cu cˆat mai mult¸i termeni, ar trebui ca n s˘a fie cˆat mai mare¡(adic˘a¢ q s˘a fie cˆat mai apropiat de 1). Se ajunge ˆın felul acesta la n lim 1 + n1 , adic˘a la e. n→∞

Teste tip examen

ˆIn final lans˘am c˘atre cititor provocarea (dar ¸si pl˘acerea, sper˘am) de a rezolva cˆateva teste tip (pentru examenul de Analiz˘a matematic˘a din anul I al Facult˘a¸tii de Matematic˘a din Universitatea ”Al.I. Cuza” Ia¸si).

Test A 1. Fie A ⊂ IRp , A nevid˘a. Ar˘atat¸i c˘a A0 este mult¸ime ˆınchis˘ a. 2. Studiat¸i uniforma continuitate a funct¸iei µ ¶cos πx 2x+1  π   , x ∈ (0, 1) 2x + 1 f (x) =   (−x)ln(1+x) , x ∈ (−1, 0). r 3. Trasat¸i graficul funct¸iei f : IR → IR, f (x) = lim

n→∞

x2n+2 . 22n + x2n

Test B 1. Fie a ∈ IRp ¸si r ∈ (0, +∞). Ar˘atat¸i c˘a B(a, r) = D(a, r). µ ¶ ∞ X an x + 1 n 2. Cercetat¸i natura seriei , a ∈ IR, x ∈ IR\{0}. n2 x n=1

1

3. Fie f : A → IR, ∅ 6= A ⊂ IR, f (x) = (x + 1) ln x . a) Determinat¸i A, domeniul maxim de definit¸ie pentru f . 277

278 b) Calculat¸i limitele funct¸iei f ˆın punctele din A0 \A.

Test C 1. Fie a ∈ IRP ¸si r ∈ (0, +∞). Ar˘atat¸i c˘a int D(a, r) = B(a, r). ∞ X (−1)n−1 xn 2. Cercetat¸i natura seriei · , α ∈ IR, x ∈ IR\{−1}. nα 1 + xn n=1 3. Studiat¸i uniforma continuitate a funct¸iei  arcsin 2x − arcsin x  , x ∈ [− 12 , 0),   x3 µ ¶ f (x) = , α ∈ IR.  1 x  α + ln , x ∈ (0, 1). x

Test D p p √ √ 1. Studiat¸i convergent¸a ¸sirului xn = nα ( 4 n + 4 n − 4 n − 4 n), α ∈ IR ¸si ˆın caz de convergent¸˘a determinat¸i lim xn . n→∞

2. Fie A = {(x, y) ∈ IR2 |x2 + y 2 < 3, x ≥ 0} ∪ {(−1, 0)}. Determinat¸i ◦

A, A, A0 , F r A ¸si mult¸imea puntelor izolate pentru A. Este A m˘ arginit˘ a? Este A compact˘a? Este A compact˘a? 3. Studiat¸i continuitatea funct¸iei f : IR2 → IR,  2 2  ln(1 + x + y ) , (x, y) 6= (0, 0),  |x| + |y| , a ∈ IR. f (x, y) =   a, (x, y) = (0, 0)

Test E 1. Determinat¸i inf xn , sup xn , lim inf xn , lim sup xn pentru ¸sirul n

nπ 1 + (−1)[ 3 ] + cos , n ∈ IN. xn = 2 2 2. Fie funct¸ia f : IR2 → IR, p  7 − x2 − y 2 , x2 + y 2 ≤ 7, f (x, y) = 0, x2 + y 2 > 7

279 a) Studiat¸i continuitatea funct¸iei f . b) Ar˘atat¸i c˘a f este uniform continu˘ a pe mult¸imea {(x, y) ∈ IR2 | |x| + |y| ≤ 10}. c) Este f uniform continu˘a pe IR2 ? cos x − e− 3. Calculat¸i, cu ajutorul formulei lui Taylor, limita lim x→0 x4

x2 2

.

Test F 1. Precizat¸i dac˘a mult¸imea A = {(x, y) ∈ IR2 |x3 + y 2 ≤ 1} ∪ {(x, y) ∈ IR2 |x < y} este ˆınchis˘a ˆın IR2 .

 ln2 (1 − x),

2. Fie f : D → IR, f (x) =

x ∈ D ∩ (−∞, 0),

. Determinat¸i x ∈ D ∩ [0, +∞). D, domeniul maxim de definit¸ie pentru f ¸si cercetat¸i dac˘a f este de clas˘a C 2 pe D. ∞ X x 3. Studiat¸i natura seriei cos nx sin , x ∈ IR. n n=1

tg2 x,

Bibliografie

˘ L., Morozan T. – Culegere de probleme de calcul diferent¸ial 1. Arama ¸si integral, vol. I, Editura Tehnic˘ a, Bucure¸sti, 1964. ˘ tinet 2. Ba ¸ u D.M. – S ¸ iruri, Editura Albatros, Bucure¸sti, 1979. 3. Bourbaki N. – El´ements de math´ematique, Paris, 1954-1961. ˆmpu E., Ga ˘ ina ˘ S. – Culegere de probleme de calcul 4. Bucur Gh., Ca diferent¸ial ¸si integral, vol. II, Editura Tehnic˘ a, Bucure¸sti, 1966. ˆ mpan F. – Din istoria cˆ 5. Ca atorva numere de seam˘ a, Editura Albatros,Bucure¸sti, 1973. ˆ mpan F. – Povestiri despre probleme celebre, Editura Albatros, Bu6. Ca cure¸sti, 1987. 7. Costinescu O. – Elemente de topologie general˘ a, Editura Tehnic˘ a, Bucure¸sti, 1969. ˘ esei C., Bˆırsan T. – Topologie general˘ 8. Costinescu O., Amiha a. Probleme, Editura Didactic˘a ¸si Pedagogic˘ a, Bucure¸sti, 1974. 9. Gelbaum B.R., Olmsted J.M.H. – Contraexemple ˆın analiz˘ a, Editura Stiint¸ific˘a, Bucure¸sti, 1973. 10. Iaglom A.M., Iaglom J.M. – Probleme neelementare tratate elementar, Bucure¸sti, 1962. 11. Macovei E. – Metode de construct¸ie pentru corpul numerelor reale, Editura Universit˘ a¸tii ”Al.I. Cuza”, Ia¸si, 1982. 280

281 12. Marcus S. – Not¸iuni de analiz˘ a matematic˘ a, Editura Stiint¸ific˘ a, Bucure¸sti, 1967. 13. Papuc D.I. – Universul matematic al civilizat¸iei umane, Editura Marineasa, Timi¸soara, 2003. 14. Precupanu A.M. – Bazele Analizei Matematice, Editura Universit˘ a¸tii ”Al.I. Cuza”, Ia¸si, 1993. 15. Precupanu A.M., Florescu L., Blendea Gh., Cuciureanu M. – Spat¸ii metrice. Probleme, Editura Universit˘ a¸tii ”Al.I. Cuza”, Ia¸si, 1990. 16. Schoenberg I.J. – Priveli¸sti Matematice, Editura Tehnic˘ a, Bucure¸sti, 1989. 17. Siret ¸ chi Gh. – Calcul diferent¸ial ¸si integral, vol. I, II, Editura Stiint¸ific˘a ¸si Enciclopedic˘a, Bucure¸sti, 1985. a, Matrix Rom, Bu18. Tamas¸ V., Leoreanu V. – Curs de Aritmetic˘ cure¸sti, 2002. 19. Vasilache S. – Elemente de teoria mult¸imilor ¸si a structurilor algebrice, Editura Academiei Romˆane, 1956.

Index

A

derivat˘ a la dreapta, 243 derivat˘ a la stˆ anga, 243 diametrul unei mult¸imi, 202 diferent¸ial˘ a, 247 distant¸a ˘, 173

acoperire, 177 aderent¸a ˘, 175 aderent¸a ˆın topologia indus˘ a, 203 asimptot˘ a oblic˘ a, 199 asimptot˘ a orizontal˘ a, 199 asimptot˘ a vertical˘ a la dreapta, 200 asimptot˘ a vertical˘ a la stˆ anga, 200

E element maximal, 7 element minimal, 7

B bil˘ a deschis˘ a, 174 bil˘ a ˆınchis˘ a, 174

F formula lui Taylor cu restul lui Lagrange, 247 cu restul lui Peano, 246 frontiera unei mult¸imi, 176 funct¸ie bijectiv˘ a, 5 continu˘ a, 201 continu˘ a la dreapta, 202 continu˘ a la stˆ anga, 202 cresc˘ atoare, 11 cu proprietatea lui Darboux, 204 de clas˘ a C n , 247 de clasa C ∞ , 248 derivabil˘ a, 243 derivabil˘ a la dreapta, 243 derivabil˘ a la stˆ anga, 243 descresc˘ atoare, 11 diferent¸iabil˘ a, 247 injectiv˘ a, 5 inversabil˘ a, 5 lipschitzian˘ a, 205 monoton˘ a, 11 periodic˘ a, 12 strict cresc˘ atoare, 11 strict descresc˘ atoare, 11 strict monoton˘ a, 11

C cardinal, 13 clas˘ a de echivalent¸a ˘, 6 contraimagine, 4 criteriul de comparat¸ie cu limit˘ a, 119 de comparat¸ie de specia a II-a, 118 de comparat¸ie de specia I, 118 lui Abel, 121 lui Cauchy de condensare, 119 lui Dirichlet, 121 lui Gauss, 120 lui Leibniz, 122 lui Raabe-Duhamel, 120 major˘ arii, 197 raportului cu limit˘ a, 120 raportului cu limite extreme, 120 raportului cu m˘ arginire, 120 r˘ ad˘ acinii cu limit˘ a, 119 r˘ ad˘ acinii cu limit˘ a superioar˘ a, 119 r˘ ad˘ acinii cu m˘ arginire, 119

D derivat˘ a, 243

282

283 surjectiv˘ a, 5 uniform continu˘ a, 204

H

mult¸imi cardinal echivalente, 13

N norm˘ a, 173

homeomorfism, 203

I

O oscilat¸ie, 202

infimum, 7 interiorul unei mult¸imi, 175

ˆ I ˆınchidere, 175

L limita inferioar˘ a a ¸sirului, 64 superioar˘ a a ¸sirului, 64 la dreapta a unei funct¸ii, 197 la stˆ anga a unei funct¸ii, 197 unei funct¸ii, 196 unui ¸sir, 63

M majorant, 7 margine inferioar˘ a, 7 margine superioar˘ a, 7 maximum, 7 metric˘ a, 173 minimum, 7 minorant, 7 mult¸ime a numerelor reale, 8 cel mult num˘ arabil˘ a, 14 compact˘ a, 177 conex˘ a, 178 conex˘ a prin arce, 178 convex˘ a, 178 derivat˘ a, 176 deschis˘ a, 174 finit˘ a, 13 infinit˘ a, 14 ˆınchis˘ a, 175 majorat˘ a, 7 m˘ arginit˘ a, 7 minorat˘ a, 7 num˘ arabil˘ a, 14 secvent¸ial compact˘ a, 177 mult¸imi echipotente, 13

P perioad˘ a a unei funct¸ii, 12 polinom Taylor, 246 prelungirea prin continuitate, 203 produsul Cauchy al dou˘ a serii, 122 proprietatea lui Arhimede, 10 punct aderent, 175 de acumulare, 176 de acumulare la dreapta, 197 de acumulare la stˆ anga, 196 de discontinuitate de specia a II-a, 203 de discontinuitate de specia I, 203 de maxim absolut (sau global), 244 de maxim local, 244 de minim absolut (sau global), 244 de minim local, 244 puterea continuului, 15

R

regula lui L0Hˆ ospital, 245 relat¸ie antisimetric˘ a, 6 de echipotent¸˘ a, 13 de echivalent¸a ˘, 6 de ordine, 7 reflexiv˘ a, 6 simetric˘ a, 6 total˘ a, 6 tranzitiv˘ a, 6 restul lui Lagrange, 247 restul lui Peano, 246

S serie absolut convergent˘ a, 122 alternat˘ a, 121 armonic˘ a, 116 armonic˘ a generalizat˘ a, 116

284 convergent˘ a, 115 de numere reale (sau serie numeric˘ a), 115 divergent˘ a, 116 geometric˘ a, 116 semiconvergent˘ a (sau condit¸ionat convergent˘ a), 122 telescopic˘ a, 116 sistem fundamental de vecin˘ at˘ a¸ti, 174 sub¸sir, 62 supremum, 7

S ¸ ¸sir al sumelor part¸iale, 115 Cauchy sau fundamental, 63 convergent, 62 cresc˘ ator, 62 de numere reale, 62 descresc˘ ator, 62 divergent, 64 majorat, 62 monoton, 62 Rolle, 246 strict cresc˘ ator, 62 strict descresc˘ ator, 62 strict monoton, 62

T teorema cle¸stelui, 66 de caracterizare a continuit˘ a¸tii globale, 202 de existent¸a ˘ ¸si unicitate a p˘ art¸ii ˆıntregi, 10 de separat¸ie Hausdorff, 174 lui Cauchy (pentru produsul Cauchy al dou˘ a serii), 123 lui Cauchy de caracterizare pentru serii, 118 lui Cantor, 205 lui Cauchy pentru funct¸ii, 245 lui Ces` aro, 65 lui Darboux, 245 lui Fermat, 244 lui Lagrange, 244 lui Mertens, 123 lui Rolle, 244 lui Stolz-Ces` aro, 66

lui Weierstrass pentru ¸siruri, 65 lui Weierstrass pentru funct¸ii, 204

V vecin˘ atate, 62, 63